Вы находитесь на странице: 1из 1368

1/52

Chapter 1. Basic Interest Theory.


Manual for SOA Exam FM/CAS Exam 2.
Chapter 1. Basic Interest Theory.
Section 1.1. Amount and accumulation functions.
c 2009. Miguel A. Arcones. All rights reserved.
Extract from:
Arcones Manual for the SOA Exam FM/CAS Exam 2,
Financial Mathematics. Fall 2009 Edition,
available at http://www.actexmadriver.com/
c 2009. Miguel A. Arcones. All rights reserved. Manual for SOA Exam FM/CAS Exam 2.
2/52
Chapter 1. Basic Interest Theory. Section 1.1. Amount and accumulation functions.
Interest

When money is invested or loaned the amount of money


returned is dierent from the initial one.

The amount of money invested (or loaned) is called the


principal or principle.

The amount of interest earned during a period of time is


I = nal balance invested amount.

The eective rate of interest earned in the period [s, t] is


nal balance invested amount
invested amount
.
c 2009. Miguel A. Arcones. All rights reserved. Manual for SOA Exam FM/CAS Exam 2.
3/52
Chapter 1. Basic Interest Theory. Section 1.1. Amount and accumulation functions.
Interest

When money is invested or loaned the amount of money


returned is dierent from the initial one.

The amount of money invested (or loaned) is called the


principal or principle.

The amount of interest earned during a period of time is


I = nal balance invested amount.

The eective rate of interest earned in the period [s, t] is


nal balance invested amount
invested amount
.
c 2009. Miguel A. Arcones. All rights reserved. Manual for SOA Exam FM/CAS Exam 2.
4/52
Chapter 1. Basic Interest Theory. Section 1.1. Amount and accumulation functions.
Interest

When money is invested or loaned the amount of money


returned is dierent from the initial one.

The amount of money invested (or loaned) is called the


principal or principle.

The amount of interest earned during a period of time is


I = nal balance invested amount.

The eective rate of interest earned in the period [s, t] is


nal balance invested amount
invested amount
.
c 2009. Miguel A. Arcones. All rights reserved. Manual for SOA Exam FM/CAS Exam 2.
5/52
Chapter 1. Basic Interest Theory. Section 1.1. Amount and accumulation functions.
Interest

When money is invested or loaned the amount of money


returned is dierent from the initial one.

The amount of money invested (or loaned) is called the


principal or principle.

The amount of interest earned during a period of time is


I = nal balance invested amount.

The eective rate of interest earned in the period [s, t] is


nal balance invested amount
invested amount
.
c 2009. Miguel A. Arcones. All rights reserved. Manual for SOA Exam FM/CAS Exam 2.
6/52
Chapter 1. Basic Interest Theory. Section 1.1. Amount and accumulation functions.
Example 1
Simon invests $1000 in a bank account. Six months later, the
amount in his bank account is $1049.23.
(i) Find the amount of interest earned by Simon in those 6 months.
(ii) Find the (semiannual) eective rate of interest earned in those
6 months.
Solution: (i) The amount of interest earned by Simon in those 6
months is I = 1049.23 1000 = 49.23.
(ii) The (semiannual) eective rate of interest earned is
1049.23 1000
1000
= 0.004923 = 0.4923%.
c 2009. Miguel A. Arcones. All rights reserved. Manual for SOA Exam FM/CAS Exam 2.
7/52
Chapter 1. Basic Interest Theory. Section 1.1. Amount and accumulation functions.
Example 1
Simon invests $1000 in a bank account. Six months later, the
amount in his bank account is $1049.23.
(i) Find the amount of interest earned by Simon in those 6 months.
(ii) Find the (semiannual) eective rate of interest earned in those
6 months.
Solution: (i) The amount of interest earned by Simon in those 6
months is I = 1049.23 1000 = 49.23.
(ii) The (semiannual) eective rate of interest earned is
1049.23 1000
1000
= 0.004923 = 0.4923%.
c 2009. Miguel A. Arcones. All rights reserved. Manual for SOA Exam FM/CAS Exam 2.
8/52
Chapter 1. Basic Interest Theory. Section 1.1. Amount and accumulation functions.
Example 1
Simon invests $1000 in a bank account. Six months later, the
amount in his bank account is $1049.23.
(i) Find the amount of interest earned by Simon in those 6 months.
(ii) Find the (semiannual) eective rate of interest earned in those
6 months.
Solution: (i) The amount of interest earned by Simon in those 6
months is I = 1049.23 1000 = 49.23.
(ii) The (semiannual) eective rate of interest earned is
1049.23 1000
1000
= 0.004923 = 0.4923%.
c 2009. Miguel A. Arcones. All rights reserved. Manual for SOA Exam FM/CAS Exam 2.
9/52
Chapter 1. Basic Interest Theory. Section 1.1. Amount and accumulation functions.
Amount function
Suppose that an amount A(0) of money is invested at time 0.
A(0) is the principal. Let A(t) denote the value at time t of the
initial investment A(0). The function A(t), t 0, is called the
amount function. Usually, we assume that the amount function
satises the following properties:
(i) For each t 0, A(t) > 0.
(ii) A is nondecreasing.
In this situation,

The amount of interest earned over the period [s, t] is


A(t) A(s).

The eective rate of interest earned in the period [s, t] is


A(t) A(s)
A(s)
.
c 2009. Miguel A. Arcones. All rights reserved. Manual for SOA Exam FM/CAS Exam 2.
10/52
Chapter 1. Basic Interest Theory. Section 1.1. Amount and accumulation functions.
Amount function
Suppose that an amount A(0) of money is invested at time 0.
A(0) is the principal. Let A(t) denote the value at time t of the
initial investment A(0). The function A(t), t 0, is called the
amount function. Usually, we assume that the amount function
satises the following properties:
(i) For each t 0, A(t) > 0.
(ii) A is nondecreasing.
In this situation,

The amount of interest earned over the period [s, t] is


A(t) A(s).

The eective rate of interest earned in the period [s, t] is


A(t) A(s)
A(s)
.
c 2009. Miguel A. Arcones. All rights reserved. Manual for SOA Exam FM/CAS Exam 2.
11/52
Chapter 1. Basic Interest Theory. Section 1.1. Amount and accumulation functions.
Amount function
Suppose that an amount A(0) of money is invested at time 0.
A(0) is the principal. Let A(t) denote the value at time t of the
initial investment A(0). The function A(t), t 0, is called the
amount function. Usually, we assume that the amount function
satises the following properties:
(i) For each t 0, A(t) > 0.
(ii) A is nondecreasing.
In this situation,

The amount of interest earned over the period [s, t] is


A(t) A(s).

The eective rate of interest earned in the period [s, t] is


A(t) A(s)
A(s)
.
c 2009. Miguel A. Arcones. All rights reserved. Manual for SOA Exam FM/CAS Exam 2.
12/52
Chapter 1. Basic Interest Theory. Section 1.1. Amount and accumulation functions.
Example 2
Jessica invests $5000 on March 1, 2008, in a fund which follows
the accumulation function A(t) = (5000)

1 +
t
40

, where t is the
number of years after March 1, 2008.
(i) Find the balance in Jessicas account on October 1, 2008.
(ii) Find the amount of interest earned in those 7 months.
(iii) Find the eective rate of interest earned in that period.
Solution: (i) The balance of Jessicas account on 1012008 is
A(7/12) = (5000)

1 +
7/12
40

= 5072.917.
(ii) The amount of interest earned in those 7 months is
A(7/12) A(0) = 5072.917 5000 = 72.917.
(iii) The eective rate of interest earned in that period is
A(7/12) A(0)
A(0)
=
72.917
5000
= 0.0145834 = 1.45834%.
c 2009. Miguel A. Arcones. All rights reserved. Manual for SOA Exam FM/CAS Exam 2.
13/52
Chapter 1. Basic Interest Theory. Section 1.1. Amount and accumulation functions.
Example 2
Jessica invests $5000 on March 1, 2008, in a fund which follows
the accumulation function A(t) = (5000)

1 +
t
40

, where t is the
number of years after March 1, 2008.
(i) Find the balance in Jessicas account on October 1, 2008.
(ii) Find the amount of interest earned in those 7 months.
(iii) Find the eective rate of interest earned in that period.
Solution: (i) The balance of Jessicas account on 1012008 is
A(7/12) = (5000)

1 +
7/12
40

= 5072.917.
(ii) The amount of interest earned in those 7 months is
A(7/12) A(0) = 5072.917 5000 = 72.917.
(iii) The eective rate of interest earned in that period is
A(7/12) A(0)
A(0)
=
72.917
5000
= 0.0145834 = 1.45834%.
c 2009. Miguel A. Arcones. All rights reserved. Manual for SOA Exam FM/CAS Exam 2.
14/52
Chapter 1. Basic Interest Theory. Section 1.1. Amount and accumulation functions.
Example 2
Jessica invests $5000 on March 1, 2008, in a fund which follows
the accumulation function A(t) = (5000)

1 +
t
40

, where t is the
number of years after March 1, 2008.
(i) Find the balance in Jessicas account on October 1, 2008.
(ii) Find the amount of interest earned in those 7 months.
(iii) Find the eective rate of interest earned in that period.
Solution: (i) The balance of Jessicas account on 1012008 is
A(7/12) = (5000)

1 +
7/12
40

= 5072.917.
(ii) The amount of interest earned in those 7 months is
A(7/12) A(0) = 5072.917 5000 = 72.917.
(iii) The eective rate of interest earned in that period is
A(7/12) A(0)
A(0)
=
72.917
5000
= 0.0145834 = 1.45834%.
c 2009. Miguel A. Arcones. All rights reserved. Manual for SOA Exam FM/CAS Exam 2.
15/52
Chapter 1. Basic Interest Theory. Section 1.1. Amount and accumulation functions.
Example 2
Jessica invests $5000 on March 1, 2008, in a fund which follows
the accumulation function A(t) = (5000)

1 +
t
40

, where t is the
number of years after March 1, 2008.
(i) Find the balance in Jessicas account on October 1, 2008.
(ii) Find the amount of interest earned in those 7 months.
(iii) Find the eective rate of interest earned in that period.
Solution: (i) The balance of Jessicas account on 1012008 is
A(7/12) = (5000)

1 +
7/12
40

= 5072.917.
(ii) The amount of interest earned in those 7 months is
A(7/12) A(0) = 5072.917 5000 = 72.917.
(iii) The eective rate of interest earned in that period is
A(7/12) A(0)
A(0)
=
72.917
5000
= 0.0145834 = 1.45834%.
c 2009. Miguel A. Arcones. All rights reserved. Manual for SOA Exam FM/CAS Exam 2.
16/52
Chapter 1. Basic Interest Theory. Section 1.1. Amount and accumulation functions.
Cashows
Often, we consider the case when several deposits/withdrawals are
made into an account following certain amount function. A series
of (deposits/withdrawals) payments made at dierent times is
called a cashow. The payments can be either made by the
individual or to the individual. An inow is payment to the
individual. An outow is a payment by the individual. We
represent inows by positive numbers and outows by negative
numbers. In a cashow, we have a contribution of C
j
at time t
j
,
for each j = 1, . . . , n. C
j
can be either positive or negative. We
can represent a cashow in a table:
Investments C
1
C
2
C
n
Time (in years) t
1
t
2
t
n
c 2009. Miguel A. Arcones. All rights reserved. Manual for SOA Exam FM/CAS Exam 2.
17/52
Chapter 1. Basic Interest Theory. Section 1.1. Amount and accumulation functions.
Cashow rules
Rule 1: Proportionality. If an investment strategy follows the
amount function A(t), t > 0, an investment of $k made at time 0
with the previous investment strategy, has a value of $
kA(t)
A(0)
at time
t.
Using the amount function A() and proportionality:

Investing A(0) at time zero, we get A(t) at time t.

Investing 1 at time zero, we get


A(t)
A(0)
at time t.

Investing k at time zero, we get


kA(t)
A(0)
at time t.
c 2009. Miguel A. Arcones. All rights reserved. Manual for SOA Exam FM/CAS Exam 2.
18/52
Chapter 1. Basic Interest Theory. Section 1.1. Amount and accumulation functions.
Cashow rules
Rule 1: Proportionality. If an investment strategy follows the
amount function A(t), t > 0, an investment of $k made at time 0
with the previous investment strategy, has a value of $
kA(t)
A(0)
at time
t.
Using the amount function A() and proportionality:

Investing A(0) at time zero, we get A(t) at time t.

Investing 1 at time zero, we get


A(t)
A(0)
at time t.

Investing k at time zero, we get


kA(t)
A(0)
at time t.
c 2009. Miguel A. Arcones. All rights reserved. Manual for SOA Exam FM/CAS Exam 2.
19/52
Chapter 1. Basic Interest Theory. Section 1.1. Amount and accumulation functions.
Cashow rules
Rule 1: Proportionality. If an investment strategy follows the
amount function A(t), t > 0, an investment of $k made at time 0
with the previous investment strategy, has a value of $
kA(t)
A(0)
at time
t.
Using the amount function A() and proportionality:

Investing A(0) at time zero, we get A(t) at time t.

Investing 1 at time zero, we get


A(t)
A(0)
at time t.

Investing k at time zero, we get


kA(t)
A(0)
at time t.
c 2009. Miguel A. Arcones. All rights reserved. Manual for SOA Exam FM/CAS Exam 2.
20/52
Chapter 1. Basic Interest Theory. Section 1.1. Amount and accumulation functions.
Cashow rules
Rule 1: Proportionality. If an investment strategy follows the
amount function A(t), t > 0, an investment of $k made at time 0
with the previous investment strategy, has a value of $
kA(t)
A(0)
at time
t.
Using the amount function A() and proportionality:

Investing A(0) at time zero, we get A(t) at time t.

Investing 1 at time zero, we get


A(t)
A(0)
at time t.

Investing k at time zero, we get


kA(t)
A(0)
at time t.
c 2009. Miguel A. Arcones. All rights reserved. Manual for SOA Exam FM/CAS Exam 2.
21/52
Chapter 1. Basic Interest Theory. Section 1.1. Amount and accumulation functions.
Cashow rules
Rule 1: Proportionality. If an investment strategy follows the
amount function A(t), t > 0, an investment of $k made at time 0
with the previous investment strategy, has a value of $
kA(t)
A(0)
at time
t.
Using the amount function A() and proportionality:

Investing A(0) at time zero, we get A(t) at time t.

Investing 1 at time zero, we get


A(t)
A(0)
at time t.

Investing k at time zero, we get


kA(t)
A(0)
at time t.
c 2009. Miguel A. Arcones. All rights reserved. Manual for SOA Exam FM/CAS Exam 2.
22/52
Chapter 1. Basic Interest Theory. Section 1.1. Amount and accumulation functions.
Present value
Denition 1
The present value at certain time of a cashow is the amount of
the money which need to invest at certain time in other to get the
same balance as that obtained from a cashow.
Since investing k at time zero, we get
kA(t)
A(0)
at time t, we have
that: the present value at time t of a deposit of k made at time
zero is
kA(t)
A(0)
.
Let x be the amount which need to invest at time zero to get a
balance of k at time t. We have that k =
xA(t)
A(0)
. So, x =
kA(0)
A(t)
.
Hence, the present value at time 0 of a balance of k had at time t
is
kA(0)
A(t)
.
c 2009. Miguel A. Arcones. All rights reserved. Manual for SOA Exam FM/CAS Exam 2.
23/52
Chapter 1. Basic Interest Theory. Section 1.1. Amount and accumulation functions.
Present value
Denition 1
The present value at certain time of a cashow is the amount of
the money which need to invest at certain time in other to get the
same balance as that obtained from a cashow.
Since investing k at time zero, we get
kA(t)
A(0)
at time t, we have
that: the present value at time t of a deposit of k made at time
zero is
kA(t)
A(0)
.
Let x be the amount which need to invest at time zero to get a
balance of k at time t. We have that k =
xA(t)
A(0)
. So, x =
kA(0)
A(t)
.
Hence, the present value at time 0 of a balance of k had at time t
is
kA(0)
A(t)
.
c 2009. Miguel A. Arcones. All rights reserved. Manual for SOA Exam FM/CAS Exam 2.
24/52
Chapter 1. Basic Interest Theory. Section 1.1. Amount and accumulation functions.
Present value
Denition 1
The present value at certain time of a cashow is the amount of
the money which need to invest at certain time in other to get the
same balance as that obtained from a cashow.
Since investing k at time zero, we get
kA(t)
A(0)
at time t, we have
that: the present value at time t of a deposit of k made at time
zero is
kA(t)
A(0)
.
Let x be the amount which need to invest at time zero to get a
balance of k at time t. We have that k =
xA(t)
A(0)
. So, x =
kA(0)
A(t)
.
Hence, the present value at time 0 of a balance of k had at time t
is
kA(0)
A(t)
.
c 2009. Miguel A. Arcones. All rights reserved. Manual for SOA Exam FM/CAS Exam 2.
25/52
Chapter 1. Basic Interest Theory. Section 1.1. Amount and accumulation functions.
To know how the value of money changes over time we need to see
how the value of $1 varies over time. The accumulation function
a(t), t 0, is dened as the value at time t of $1 invested at time
0.
By proportionality, a(t) =
A(t)
A(0)
. Observe that a(0) = 1.
Knowing the value function a(t) and the principal A(0), we can
nd the amount function A(t) using the formula A(t) = A(0)a(t).
Using the accumulation function a(t), t 0, we have:

The present value at time t of a deposit of k made at time


zero is ka(t) (=
kA(t)
A(0)
).

The present value at time 0 of a balance of k had at time t is


k
a(t)
(=
kA(0)
A(t)
).
c 2009. Miguel A. Arcones. All rights reserved. Manual for SOA Exam FM/CAS Exam 2.
26/52
Chapter 1. Basic Interest Theory. Section 1.1. Amount and accumulation functions.
To know how the value of money changes over time we need to see
how the value of $1 varies over time. The accumulation function
a(t), t 0, is dened as the value at time t of $1 invested at time
0.
By proportionality, a(t) =
A(t)
A(0)
. Observe that a(0) = 1.
Knowing the value function a(t) and the principal A(0), we can
nd the amount function A(t) using the formula A(t) = A(0)a(t).
Using the accumulation function a(t), t 0, we have:

The present value at time t of a deposit of k made at time


zero is ka(t) (=
kA(t)
A(0)
).

The present value at time 0 of a balance of k had at time t is


k
a(t)
(=
kA(0)
A(t)
).
c 2009. Miguel A. Arcones. All rights reserved. Manual for SOA Exam FM/CAS Exam 2.
27/52
Chapter 1. Basic Interest Theory. Section 1.1. Amount and accumulation functions.
To know how the value of money changes over time we need to see
how the value of $1 varies over time. The accumulation function
a(t), t 0, is dened as the value at time t of $1 invested at time
0.
By proportionality, a(t) =
A(t)
A(0)
. Observe that a(0) = 1.
Knowing the value function a(t) and the principal A(0), we can
nd the amount function A(t) using the formula A(t) = A(0)a(t).
Using the accumulation function a(t), t 0, we have:

The present value at time t of a deposit of k made at time


zero is ka(t) (=
kA(t)
A(0)
).

The present value at time 0 of a balance of k had at time t is


k
a(t)
(=
kA(0)
A(t)
).
c 2009. Miguel A. Arcones. All rights reserved. Manual for SOA Exam FM/CAS Exam 2.
28/52
Chapter 1. Basic Interest Theory. Section 1.1. Amount and accumulation functions.
To know how the value of money changes over time we need to see
how the value of $1 varies over time. The accumulation function
a(t), t 0, is dened as the value at time t of $1 invested at time
0.
By proportionality, a(t) =
A(t)
A(0)
. Observe that a(0) = 1.
Knowing the value function a(t) and the principal A(0), we can
nd the amount function A(t) using the formula A(t) = A(0)a(t).
Using the accumulation function a(t), t 0, we have:

The present value at time t of a deposit of k made at time


zero is ka(t) (=
kA(t)
A(0)
).

The present value at time 0 of a balance of k had at time t is


k
a(t)
(=
kA(0)
A(t)
).
c 2009. Miguel A. Arcones. All rights reserved. Manual for SOA Exam FM/CAS Exam 2.
29/52
Chapter 1. Basic Interest Theory. Section 1.1. Amount and accumulation functions.
To know how the value of money changes over time we need to see
how the value of $1 varies over time. The accumulation function
a(t), t 0, is dened as the value at time t of $1 invested at time
0.
By proportionality, a(t) =
A(t)
A(0)
. Observe that a(0) = 1.
Knowing the value function a(t) and the principal A(0), we can
nd the amount function A(t) using the formula A(t) = A(0)a(t).
Using the accumulation function a(t), t 0, we have:

The present value at time t of a deposit of k made at time


zero is ka(t) (=
kA(t)
A(0)
).

The present value at time 0 of a balance of k had at time t is


k
a(t)
(=
kA(0)
A(t)
).
c 2009. Miguel A. Arcones. All rights reserved. Manual for SOA Exam FM/CAS Exam 2.
30/52
Chapter 1. Basic Interest Theory. Section 1.1. Amount and accumulation functions.
Example 1
The accumulation function of a fund is a(t) = (1.03)
2t
, t 0.
(i) Amanda invests $5000 at time zero in this fund. Find the
balance into Amandas fund at time 2.5 years.
(ii) How much money does Kevin need to invest into the fund at
time 0 to accumulate $10000 at time 3?
Solution: (i) The balance into Amandas fund at time 2.5 years is
ka(2.5) = (5000)(1.03)
2(2.5)
= 5796.370371.
(ii) The amount which Kevin needs to invest at time 0 to
accumulate $10000 at time 3 is
10000
a(3)
=
10000
(1.03)
2(3)
= 8374.842567.
c 2009. Miguel A. Arcones. All rights reserved. Manual for SOA Exam FM/CAS Exam 2.
31/52
Chapter 1. Basic Interest Theory. Section 1.1. Amount and accumulation functions.
Example 1
The accumulation function of a fund is a(t) = (1.03)
2t
, t 0.
(i) Amanda invests $5000 at time zero in this fund. Find the
balance into Amandas fund at time 2.5 years.
(ii) How much money does Kevin need to invest into the fund at
time 0 to accumulate $10000 at time 3?
Solution: (i) The balance into Amandas fund at time 2.5 years is
ka(2.5) = (5000)(1.03)
2(2.5)
= 5796.370371.
(ii) The amount which Kevin needs to invest at time 0 to
accumulate $10000 at time 3 is
10000
a(3)
=
10000
(1.03)
2(3)
= 8374.842567.
c 2009. Miguel A. Arcones. All rights reserved. Manual for SOA Exam FM/CAS Exam 2.
32/52
Chapter 1. Basic Interest Theory. Section 1.1. Amount and accumulation functions.
Example 1
The accumulation function of a fund is a(t) = (1.03)
2t
, t 0.
(i) Amanda invests $5000 at time zero in this fund. Find the
balance into Amandas fund at time 2.5 years.
(ii) How much money does Kevin need to invest into the fund at
time 0 to accumulate $10000 at time 3?
Solution: (i) The balance into Amandas fund at time 2.5 years is
ka(2.5) = (5000)(1.03)
2(2.5)
= 5796.370371.
(ii) The amount which Kevin needs to invest at time 0 to
accumulate $10000 at time 3 is
10000
a(3)
=
10000
(1.03)
2(3)
= 8374.842567.
c 2009. Miguel A. Arcones. All rights reserved. Manual for SOA Exam FM/CAS Exam 2.
33/52
Chapter 1. Basic Interest Theory. Section 1.1. Amount and accumulation functions.
Cashow rules
Rule 2. Growsdependsonbalance rule. If an investment
follows the amount function A(t), t 0, the growth during certain
period where no deposits/withdrawals are made depends on the
balance on the account at the beginning of the period.
If an account has a balance of k at time t and no
deposits/withdrawals are made in the future, then the future
balance in this account does not depend on how the balance of k
at time t was attained.
In particular, the following two accounts have the same balance for
times bigger than t:
1. An account where a unique deposit of k is made at time t.
2. An account where a unique deposit of
k
A(t)
is made at time zero.
c 2009. Miguel A. Arcones. All rights reserved. Manual for SOA Exam FM/CAS Exam 2.
34/52
Chapter 1. Basic Interest Theory. Section 1.1. Amount and accumulation functions.
Cashow rules
Rule 2. Growsdependsonbalance rule. If an investment
follows the amount function A(t), t 0, the growth during certain
period where no deposits/withdrawals are made depends on the
balance on the account at the beginning of the period.
If an account has a balance of k at time t and no
deposits/withdrawals are made in the future, then the future
balance in this account does not depend on how the balance of k
at time t was attained.
In particular, the following two accounts have the same balance for
times bigger than t:
1. An account where a unique deposit of k is made at time t.
2. An account where a unique deposit of
k
A(t)
is made at time zero.
c 2009. Miguel A. Arcones. All rights reserved. Manual for SOA Exam FM/CAS Exam 2.
35/52
Chapter 1. Basic Interest Theory. Section 1.1. Amount and accumulation functions.
Cashow rules
Rule 2. Growsdependsonbalance rule. If an investment
follows the amount function A(t), t 0, the growth during certain
period where no deposits/withdrawals are made depends on the
balance on the account at the beginning of the period.
If an account has a balance of k at time t and no
deposits/withdrawals are made in the future, then the future
balance in this account does not depend on how the balance of k
at time t was attained.
In particular, the following two accounts have the same balance for
times bigger than t:
1. An account where a unique deposit of k is made at time t.
2. An account where a unique deposit of
k
A(t)
is made at time zero.
c 2009. Miguel A. Arcones. All rights reserved. Manual for SOA Exam FM/CAS Exam 2.
36/52
Chapter 1. Basic Interest Theory. Section 1.1. Amount and accumulation functions.
Theorem 1
If an investment follows the amount function A(), the present
value at time t of a deposit of $k made at time s is $
kA(t)
A(s)
=
ka(t)
a(s)
.
Proof. We need to invest
k
A(s)
at time 0 to get a balance of k at
time s. So, investing k at time s is equivalent to investing
k
A(s)
at
time 0. The future value at time t of an investment of
k
A(s)
at time
0 is
kA(t)
A(s)
. Hence, investing k at time s is equivalent to investing
kA(t)
A(s)
at time t.
c 2009. Miguel A. Arcones. All rights reserved. Manual for SOA Exam FM/CAS Exam 2.
37/52
Chapter 1. Basic Interest Theory. Section 1.1. Amount and accumulation functions.
Theorem 1
If an investment follows the amount function A(), the present
value at time t of a deposit of $k made at time s is $
kA(t)
A(s)
=
ka(t)
a(s)
.
Proof. We need to invest
k
A(s)
at time 0 to get a balance of k at
time s. So, investing k at time s is equivalent to investing
k
A(s)
at
time 0. The future value at time t of an investment of
k
A(s)
at time
0 is
kA(t)
A(s)
. Hence, investing k at time s is equivalent to investing
kA(t)
A(s)
at time t.
c 2009. Miguel A. Arcones. All rights reserved. Manual for SOA Exam FM/CAS Exam 2.
38/52
Chapter 1. Basic Interest Theory. Section 1.1. Amount and accumulation functions.
Another way to see the previous theorem is as follows. The
following three accounts have the same balance at any time bigger
than t:
1. An account where a unique deposit of A(0) is made at time
zero.
2. An account where a unique deposit of A(s) is made at time s.
3. An account where a unique deposit of A(t) is made at time t.
Hence,
The present value at time t of an investment of A(s) made at time
s is A(t).
This means that:

If t > s, an investment of A(s) made at time s has an


accumulation value of A(t) at time t.

If t < s, to get an accumulation of A(s) at time s, we need to


invest A(t) at time t.
c 2009. Miguel A. Arcones. All rights reserved. Manual for SOA Exam FM/CAS Exam 2.
39/52
Chapter 1. Basic Interest Theory. Section 1.1. Amount and accumulation functions.
Another way to see the previous theorem is as follows. The
following three accounts have the same balance at any time bigger
than t:
1. An account where a unique deposit of A(0) is made at time
zero.
2. An account where a unique deposit of A(s) is made at time s.
3. An account where a unique deposit of A(t) is made at time t.
Hence,
The present value at time t of an investment of A(s) made at time
s is A(t).
This means that:

If t > s, an investment of A(s) made at time s has an


accumulation value of A(t) at time t.

If t < s, to get an accumulation of A(s) at time s, we need to


invest A(t) at time t.
c 2009. Miguel A. Arcones. All rights reserved. Manual for SOA Exam FM/CAS Exam 2.
40/52
Chapter 1. Basic Interest Theory. Section 1.1. Amount and accumulation functions.
Another way to see the previous theorem is as follows. The
following three accounts have the same balance at any time bigger
than t:
1. An account where a unique deposit of A(0) is made at time
zero.
2. An account where a unique deposit of A(s) is made at time s.
3. An account where a unique deposit of A(t) is made at time t.
Hence,
The present value at time t of an investment of A(s) made at time
s is A(t).
This means that:

If t > s, an investment of A(s) made at time s has an


accumulation value of A(t) at time t.

If t < s, to get an accumulation of A(s) at time s, we need to


invest A(t) at time t.
c 2009. Miguel A. Arcones. All rights reserved. Manual for SOA Exam FM/CAS Exam 2.
41/52
Chapter 1. Basic Interest Theory. Section 1.1. Amount and accumulation functions.
We know that:
The present value at time t of an investment of A(s) made at time
s is A(t), i.e.
A(s) at time s is equivalent to A(t) at time t.
By proportionality,

The present value at time t of an investment of 1 made at


time s is
A(t)
A(s)
, i.e.
1 at time s is equivalent to
A(t)
A(s)
at time t.

The present value at time t of an investment of k made at


time s is
kA(t)
A(s)
, i.e.
k at time s is equivalent to
kA(t)
A(s)
at time t.
c 2009. Miguel A. Arcones. All rights reserved. Manual for SOA Exam FM/CAS Exam 2.
42/52
Chapter 1. Basic Interest Theory. Section 1.1. Amount and accumulation functions.
We know that:
The present value at time t of an investment of A(s) made at time
s is A(t), i.e.
A(s) at time s is equivalent to A(t) at time t.
By proportionality,

The present value at time t of an investment of 1 made at


time s is
A(t)
A(s)
, i.e.
1 at time s is equivalent to
A(t)
A(s)
at time t.

The present value at time t of an investment of k made at


time s is
kA(t)
A(s)
, i.e.
k at time s is equivalent to
kA(t)
A(s)
at time t.
c 2009. Miguel A. Arcones. All rights reserved. Manual for SOA Exam FM/CAS Exam 2.
43/52
Chapter 1. Basic Interest Theory. Section 1.1. Amount and accumulation functions.
We know that:
The present value at time t of an investment of A(s) made at time
s is A(t), i.e.
A(s) at time s is equivalent to A(t) at time t.
By proportionality,

The present value at time t of an investment of 1 made at


time s is
A(t)
A(s)
, i.e.
1 at time s is equivalent to
A(t)
A(s)
at time t.

The present value at time t of an investment of k made at


time s is
kA(t)
A(s)
, i.e.
k at time s is equivalent to
kA(t)
A(s)
at time t.
c 2009. Miguel A. Arcones. All rights reserved. Manual for SOA Exam FM/CAS Exam 2.
44/52
Chapter 1. Basic Interest Theory. Section 1.1. Amount and accumulation functions.
Example 2
The accumulation function of a fund follows the function
a(t) = 1 +
t
20
, t > 0.
(i) Michael invests $3500 into the fund at time 1. Find the value
of Michaels fund account at time 4.
(ii) How much money needs Jason to invest at time 2 to
accumulate $700 at time 4.
Solution: (i) The value of Michaels account at time 4 is
3500
a(4)
a(1)
= (3500)
1+
4
20
1+
1
20
= (3500)
1.20
1.05
= 4000.
(ii) To accumulate $700 at time 4, Jason needs to invest at time 2,
700
a(2)
a(4)
= 700
1.1
1.2
= 641.67.
c 2009. Miguel A. Arcones. All rights reserved. Manual for SOA Exam FM/CAS Exam 2.
45/52
Chapter 1. Basic Interest Theory. Section 1.1. Amount and accumulation functions.
Example 2
The accumulation function of a fund follows the function
a(t) = 1 +
t
20
, t > 0.
(i) Michael invests $3500 into the fund at time 1. Find the value
of Michaels fund account at time 4.
(ii) How much money needs Jason to invest at time 2 to
accumulate $700 at time 4.
Solution: (i) The value of Michaels account at time 4 is
3500
a(4)
a(1)
= (3500)
1+
4
20
1+
1
20
= (3500)
1.20
1.05
= 4000.
(ii) To accumulate $700 at time 4, Jason needs to invest at time 2,
700
a(2)
a(4)
= 700
1.1
1.2
= 641.67.
c 2009. Miguel A. Arcones. All rights reserved. Manual for SOA Exam FM/CAS Exam 2.
46/52
Chapter 1. Basic Interest Theory. Section 1.1. Amount and accumulation functions.
Example 2
The accumulation function of a fund follows the function
a(t) = 1 +
t
20
, t > 0.
(i) Michael invests $3500 into the fund at time 1. Find the value
of Michaels fund account at time 4.
(ii) How much money needs Jason to invest at time 2 to
accumulate $700 at time 4.
Solution: (i) The value of Michaels account at time 4 is
3500
a(4)
a(1)
= (3500)
1+
4
20
1+
1
20
= (3500)
1.20
1.05
= 4000.
(ii) To accumulate $700 at time 4, Jason needs to invest at time 2,
700
a(2)
a(4)
= 700
1.1
1.2
= 641.67.
c 2009. Miguel A. Arcones. All rights reserved. Manual for SOA Exam FM/CAS Exam 2.
47/52
Chapter 1. Basic Interest Theory. Section 1.1. Amount and accumulation functions.
Theorem 3
Present value of a cashow. If an investment account follows
the amount function A(t), t > 0, the (equation of value) present
value at time t of the cashow
Deposits C
1
C
2
C
n
Time t
1
t
2
t
n
where 0 t
1
< t
2
< < t
n
, is
V(t) =
n

j =1
C
j
A(t)
A(t
j
)
.
c 2009. Miguel A. Arcones. All rights reserved. Manual for SOA Exam FM/CAS Exam 2.
48/52
Chapter 1. Basic Interest Theory. Section 1.1. Amount and accumulation functions.
Proof. Let s > t
n
.
Time Balance before deposit Balance after deposit
t
1
0 C
1
t
2
C
1
a(t
2
)
a(t
1
)
=

1
j =1
C
j
a(t
2
)
a(t
j
)
C
1
a(t
2
)
a(t
1
)
+ C
2
=

2
j =1
C
j
a(t
2
)
a(t
j
)
t
3

2
j =1
C
j
a(t
3
)
a(t
j
)

3
j =1
C
j
a(t
3
)
a(t
j
)
t
4

3
j =1
C
j
a(t
4
)
a(t
j
)

4
j =1
C
j
a(t
4
)
a(t
j
)

t
n

n1
j =1
C
j
a(t
n
)
a(t
j
)

n
j =1
C
j
a(t
n
)
a(t
j
)
Since the balance after deposit at time t
1
is C
1
, the balance before
deposit at time t
2
is
a(t
2
)
a(t
1
)
C
1
.
Since the balance after deposit at time t
2
is

2
j =1
C
j
a(t
2
)
a(t
j
)
, the
balance before deposit at time t
3
is
a(t
3
)
a(t
2
)

2
j =1
C
j
a(t
2
)
a(t
j
)
=

2
j =1
C
j
a(t
3
)
a(t
j
)
.
c 2009. Miguel A. Arcones. All rights reserved. Manual for SOA Exam FM/CAS Exam 2.
49/52
Chapter 1. Basic Interest Theory. Section 1.1. Amount and accumulation functions.
Hence, the balance at time s is
a(s)
a(t
n
)
n

j =1
C
j
a(t
n
)
a(t
j
)
=
n

j =1
C
j
a(s)
a(t
j
)
.
The future value of the cashow at time t is
a(t)
a(s)
n

j =1
C
j
a(s)
a(t
j
)
=
n

j =1
C
j
a(t)
a(t
j
)
.
end of proof.
c 2009. Miguel A. Arcones. All rights reserved. Manual for SOA Exam FM/CAS Exam 2.
50/52
Chapter 1. Basic Interest Theory. Section 1.1. Amount and accumulation functions.
Notice that the present value at time t of the cashow
Deposits C
1
C
2
C
n
Time t
1
t
2
t
n
is the same as the sum of the present values at time t of n
separated investment accounts each following the amount function
A, with the j the account having a unique deposit of C
j
at time t
j
.
c 2009. Miguel A. Arcones. All rights reserved. Manual for SOA Exam FM/CAS Exam 2.
51/52
Chapter 1. Basic Interest Theory. Section 1.1. Amount and accumulation functions.
Example 4
The accumulation function of a fund follows the function
a(t) = 1 +
t
20
, t > 0. Jared invests $1000 into the fund at time 1
and he withdraws $500 at time 3. Find the value of Jareds fund
account at time 5.
Solution: The cashow is
deposit/withdrawal 1000 500
Time (in years) 1 3
.
The value of Jareds account at time 5 is
1000
a(5)
a(1)
500
a(5)
a(3)
= 1000
1 +
5
20
1 +
1
20
500
1 +
5
20
1 +
3
20
=(1000)
1.25
1.05
(500)
1.25
1.15
= 1190.48 543.48 = 647.00.
c 2009. Miguel A. Arcones. All rights reserved. Manual for SOA Exam FM/CAS Exam 2.
52/52
Chapter 1. Basic Interest Theory. Section 1.1. Amount and accumulation functions.
Example 4
The accumulation function of a fund follows the function
a(t) = 1 +
t
20
, t > 0. Jared invests $1000 into the fund at time 1
and he withdraws $500 at time 3. Find the value of Jareds fund
account at time 5.
Solution: The cashow is
deposit/withdrawal 1000 500
Time (in years) 1 3
.
The value of Jareds account at time 5 is
1000
a(5)
a(1)
500
a(5)
a(3)
= 1000
1 +
5
20
1 +
1
20
500
1 +
5
20
1 +
3
20
=(1000)
1.25
1.05
(500)
1.25
1.15
= 1190.48 543.48 = 647.00.
c 2009. Miguel A. Arcones. All rights reserved. Manual for SOA Exam FM/CAS Exam 2.
1/21
Chapter 1. Basic Interest Theory.
Manual for SOA Exam FM/CAS Exam 2.
Chapter 1. Basic Interest Theory.
Section 1.2. Simple interest.
c 2009. Miguel A. Arcones. All rights reserved.
Extract from:
Arcones Manual for the SOA Exam FM/CAS Exam 2,
Financial Mathematics. Fall 2009 Edition,
available at http://www.actexmadriver.com/
c 2009. Miguel A. Arcones. All rights reserved. Manual for SOA Exam FM/CAS Exam 2.
2/21
Chapter 1. Basic Interest Theory. Section 1.2. Simple interest.
Simple interest
Under simple interest:

the interest paid over certain period of time is proportional to


the length of this period of time and the principal.

if i is the eective annual rate of simple interest, the amount


of interest earned by a deposit of k held for t years is kit. The
balance at time t years is k + kit = k(1 + it).

interest is found using the principal not the earned interest.


To nd the earned interest, we need to know the amount of
principal, not the balance.

balances under simple interest follow the proportionality rule


and rule about the addition of several deposits/withdrawals.
However, the rule growsdependsonbalance does not
hold.
c 2009. Miguel A. Arcones. All rights reserved. Manual for SOA Exam FM/CAS Exam 2.
3/21
Chapter 1. Basic Interest Theory. Section 1.2. Simple interest.
Simple interest
Under simple interest:

the interest paid over certain period of time is proportional to


the length of this period of time and the principal.

if i is the eective annual rate of simple interest, the amount


of interest earned by a deposit of k held for t years is kit. The
balance at time t years is k + kit = k(1 + it).

interest is found using the principal not the earned interest.


To nd the earned interest, we need to know the amount of
principal, not the balance.

balances under simple interest follow the proportionality rule


and rule about the addition of several deposits/withdrawals.
However, the rule growsdependsonbalance does not
hold.
c 2009. Miguel A. Arcones. All rights reserved. Manual for SOA Exam FM/CAS Exam 2.
4/21
Chapter 1. Basic Interest Theory. Section 1.2. Simple interest.
Simple interest
Under simple interest:

the interest paid over certain period of time is proportional to


the length of this period of time and the principal.

if i is the eective annual rate of simple interest, the amount


of interest earned by a deposit of k held for t years is kit. The
balance at time t years is k + kit = k(1 + it).

interest is found using the principal not the earned interest.


To nd the earned interest, we need to know the amount of
principal, not the balance.

balances under simple interest follow the proportionality rule


and rule about the addition of several deposits/withdrawals.
However, the rule growsdependsonbalance does not
hold.
c 2009. Miguel A. Arcones. All rights reserved. Manual for SOA Exam FM/CAS Exam 2.
5/21
Chapter 1. Basic Interest Theory. Section 1.2. Simple interest.
Simple interest
Under simple interest:

the interest paid over certain period of time is proportional to


the length of this period of time and the principal.

if i is the eective annual rate of simple interest, the amount


of interest earned by a deposit of k held for t years is kit. The
balance at time t years is k + kit = k(1 + it).

interest is found using the principal not the earned interest.


To nd the earned interest, we need to know the amount of
principal, not the balance.

balances under simple interest follow the proportionality rule


and rule about the addition of several deposits/withdrawals.
However, the rule growsdependsonbalance does not
hold.
c 2009. Miguel A. Arcones. All rights reserved. Manual for SOA Exam FM/CAS Exam 2.
6/21
Chapter 1. Basic Interest Theory. Section 1.2. Simple interest.
Suppose that an account earns simple interest with annual
eective rate of i .

If an investment of 1 is made at time zero, then the balance


in this account at time t years is a(t) = 1 + it .

If an investment of k is made at time zero, then the balance


in this account at time t years is k(1 + it).

If an investment of k is made at time s years, then the


balance in this account at time t years, t > s, is
k(1 + i (t s)). Notice that the investment is held for t s
years, and the earned interest is ki (t s).
c 2009. Miguel A. Arcones. All rights reserved. Manual for SOA Exam FM/CAS Exam 2.
7/21
Chapter 1. Basic Interest Theory. Section 1.2. Simple interest.
Suppose that an account earns simple interest with annual
eective rate of i .

If an investment of 1 is made at time zero, then the balance


in this account at time t years is a(t) = 1 + it .

If an investment of k is made at time zero, then the balance


in this account at time t years is k(1 + it).

If an investment of k is made at time s years, then the


balance in this account at time t years, t > s, is
k(1 + i (t s)). Notice that the investment is held for t s
years, and the earned interest is ki (t s).
c 2009. Miguel A. Arcones. All rights reserved. Manual for SOA Exam FM/CAS Exam 2.
8/21
Chapter 1. Basic Interest Theory. Section 1.2. Simple interest.
Suppose that an account earns simple interest with annual
eective rate of i .

If an investment of 1 is made at time zero, then the balance


in this account at time t years is a(t) = 1 + it .

If an investment of k is made at time zero, then the balance


in this account at time t years is k(1 + it).

If an investment of k is made at time s years, then the


balance in this account at time t years, t > s, is
k(1 + i (t s)). Notice that the investment is held for t s
years, and the earned interest is ki (t s).
c 2009. Miguel A. Arcones. All rights reserved. Manual for SOA Exam FM/CAS Exam 2.
9/21
Chapter 1. Basic Interest Theory. Section 1.2. Simple interest.
Notice that the amount k(1 + i (t s)) is not
ka(t)
a(s)
=
k(1+it)
(1+is)
.
Making an investment of
k
(1+is)
at time zero, we have a balance of
k
(1+is)
(1 + is) = k at time s. Making an investment of
k
(1+is)
at
time zero, we have a balance of
k
(1+is)
(1 + it) at time t. This is
not the balance at time t years in an account with an investment
of k made at time s years.

Making an investment of
k
(1+is)
at time zero, we have a
balance of
k
(1+is)
(1 + is) = k at time s. But since interest
does not earn interest, the amount of interest earned in the
period [s, t] is
k
(1+is)
i (t s). Hence, the balance at time t is
k +
k
(1 + is)
i (t s) =
k(1 + is) + ki (t s)
(1 + is)
=
k(1 + it)
(1 + is)
.

Making an investment of k at time s years, we have a balance


of k(1 + i (t s)) at time t.
c 2009. Miguel A. Arcones. All rights reserved. Manual for SOA Exam FM/CAS Exam 2.
10/21
Chapter 1. Basic Interest Theory. Section 1.2. Simple interest.
Present value for simple interest

A deposit of k made at time s has a future value of


k(1 + i (t s)) at time t, if t > s.

To get a balance of k time s, we need to make a deposit of


k
1
1+i (st)
at time t, if t < s.
c 2009. Miguel A. Arcones. All rights reserved. Manual for SOA Exam FM/CAS Exam 2.
11/21
Chapter 1. Basic Interest Theory. Section 1.2. Simple interest.
Present value for simple interest

A deposit of k made at time s has a future value of


k(1 + i (t s)) at time t, if t > s.

To get a balance of k time s, we need to make a deposit of


k
1
1+i (st)
at time t, if t < s.
c 2009. Miguel A. Arcones. All rights reserved. Manual for SOA Exam FM/CAS Exam 2.
12/21
Chapter 1. Basic Interest Theory. Section 1.2. Simple interest.
Present value for simple interest
Theorem 1
If deposits/withdrawals are make according with the table,
Deposits C
1
C
2
C
n
Time t
1
t
2
t
n
where 0 t
1
< t
2
< < t
n
to an account earning simple interest
with annual eective rate of i , then the balance at time t years,
where t > t
n
, is given by
B =
n

j =1
C
j
(1 + i (t t
j
)) =
n

j =1
C
j
+
n

j =1
C
j
i (t t
j
).
c 2009. Miguel A. Arcones. All rights reserved. Manual for SOA Exam FM/CAS Exam 2.
13/21
Chapter 1. Basic Interest Theory. Section 1.2. Simple interest.
Proof.
Time Deposit/withdr. Principal Amount of interest
at that time after the deposit earned up
to that time
t
1
C
1
C
1
0
t
2
C
2
C
1
+ C
2
C
1
i (t
2
t
1
)
t
3
C
3

3
j =1
C
j

2
j =1
C
j
i (t
3
t
j
)

t
k
C
k

k
j =1
C
j

k1
j =1
C
j
i (t
k
t
j
)

t
n
C
n

n
j =1
C
j

n1
j =1
C
j
i (t
n
t
j
)
t 0

n
j =1
C
j

n
j =1
C
j
i (t t
j
)
c 2009. Miguel A. Arcones. All rights reserved. Manual for SOA Exam FM/CAS Exam 2.
14/21
Chapter 1. Basic Interest Theory. Section 1.2. Simple interest.
The amount of interest earned up to time t
3
is
C
1
i (t
2
t
1
) + (C
1
+ C
2
)i (t
3
t
2
) = C
1
i (t
3
t
1
) + C
2
i (t
3
t
2
)
=
2

j =1
C
j
i (t
3
t
j
).
The amount of interest earned up to time t
k
is the amount of
interest earned up to time t
k1
plus the amount of interest earned
in the period [t
k1
, t
k
], which is
k2

j =1
C
j
i (t
k1
t
j
) +
k1

j =1
C
j
i (t
k
t
k1
)
=
k1

j =1
C
j
i (t
k1
t
j
) +
k1

j =1
C
j
i (t
k
t
k1
)
=
k1

j =1
C
j
i (t
k
t
j
).
c 2009. Miguel A. Arcones. All rights reserved. Manual for SOA Exam FM/CAS Exam 2.
15/21
Chapter 1. Basic Interest Theory. Section 1.2. Simple interest.
Theorem 2
If deposits/withdrawals are make according with the table,
Deposits C
1
C
2
C
n
Time t
1
t
2
t
n
where 0 t
1
< t
2
< < t
n
to an account earning simple interest
and the balance at time t years, where t > t
n
, is B, then the
annual eective rate of i
i =
B

n
j =1
C
j

n
j =1
C
j
(t t
j
)
.
Proof. Solving for i in B =

n
j =1
C
j
+

n
j =1
C
j
i (t t
j
), we get the
value of i .
c 2009. Miguel A. Arcones. All rights reserved. Manual for SOA Exam FM/CAS Exam 2.
16/21
Chapter 1. Basic Interest Theory. Section 1.2. Simple interest.
In the formula,
i =
B

n
j =1
C
j

n
j =1
C
j
(t t
j
)
,
B

n
j =1
C
j
is the total amount of interest earned,

n
j =1
C
j
(t t
j
) is the sum of the balances times the amount
balances are in the account.
c 2009. Miguel A. Arcones. All rights reserved. Manual for SOA Exam FM/CAS Exam 2.
17/21
Chapter 1. Basic Interest Theory. Section 1.2. Simple interest.
Example 3
Jeremy invests $1000 into a bank account which pays simple
interest with an annual rate of 7%. Nine months later, Jeremy
withdraws $600 from the account. Find the balance in Jeremys
account one year after the rst deposit was made.
Solution: The cashow of deposits is
deposit/withdrawal 1000 600
Time (in years) 0 0.75
.
The balance one year after the rst deposit was made is
n

j =1
C
j
(1 + i (t t
j
))
=(1000)(1 + (1 0)(0.07)) + (600)(1 + (1 0.75)(0.07))
=459.5.
c 2009. Miguel A. Arcones. All rights reserved. Manual for SOA Exam FM/CAS Exam 2.
18/21
Chapter 1. Basic Interest Theory. Section 1.2. Simple interest.
Example 3
Jeremy invests $1000 into a bank account which pays simple
interest with an annual rate of 7%. Nine months later, Jeremy
withdraws $600 from the account. Find the balance in Jeremys
account one year after the rst deposit was made.
Solution: The cashow of deposits is
deposit/withdrawal 1000 600
Time (in years) 0 0.75
.
The balance one year after the rst deposit was made is
n

j =1
C
j
(1 + i (t t
j
))
=(1000)(1 + (1 0)(0.07)) + (600)(1 + (1 0.75)(0.07))
=459.5.
c 2009. Miguel A. Arcones. All rights reserved. Manual for SOA Exam FM/CAS Exam 2.
19/21
Chapter 1. Basic Interest Theory. Section 1.2. Simple interest.
Time Deposit Principal Amount
made after deposit of interest
at this time earned in
the last period
0 1000 1000 0
0.75 600 400 (1000)(0.07)(0.75) = 52.5
1 0 400 (400)(0.07)(1 0.75) = 7
The balance one year after the rst deposit was made is
400 + 52.5 + 7 = 459.5.
c 2009. Miguel A. Arcones. All rights reserved. Manual for SOA Exam FM/CAS Exam 2.
20/21
Chapter 1. Basic Interest Theory. Section 1.2. Simple interest.
Example 4
On September 1, 2006, John invested $25000 into a bank account
which pays simple interest. On March 1, 2007, Johns wife made a
withdrawal of 5000. The accumulated value of the bank account
on July 1, 2007 was $20575. Calculate the annual eective rate of
interest earned by this account.
Solution: Let September 1, 2006 be time 0. Then, March 1, 2007
is time
6
12
years; and July 1, 2007 is time
10
12
years. The annual
eective rate of interest earned by this account is
i =
B

n
j =1
C
j

n
j =1
C
j
(t t
j
)
=
20575 25000 + 5000
25000

10
12

5000

10
12

6
12

=
575
19166.66667
= 3%.
c 2009. Miguel A. Arcones. All rights reserved. Manual for SOA Exam FM/CAS Exam 2.
21/21
Chapter 1. Basic Interest Theory. Section 1.2. Simple interest.
Example 4
On September 1, 2006, John invested $25000 into a bank account
which pays simple interest. On March 1, 2007, Johns wife made a
withdrawal of 5000. The accumulated value of the bank account
on July 1, 2007 was $20575. Calculate the annual eective rate of
interest earned by this account.
Solution: Let September 1, 2006 be time 0. Then, March 1, 2007
is time
6
12
years; and July 1, 2007 is time
10
12
years. The annual
eective rate of interest earned by this account is
i =
B

n
j =1
C
j

n
j =1
C
j
(t t
j
)
=
20575 25000 + 5000
25000

10
12

5000

10
12

6
12

=
575
19166.66667
= 3%.
c 2009. Miguel A. Arcones. All rights reserved. Manual for SOA Exam FM/CAS Exam 2.
1/27
Chapter 1. Basic Interest Theory.
Manual for SOA Exam FM/CAS Exam 2.
Chapter 1. Basic Interest Theory.
Section 1.3. Compounded interest.
c 2009. Miguel A. Arcones. All rights reserved.
Extract from:
Arcones Manual for the SOA Exam FM/CAS Exam 2,
Financial Mathematics. Fall 2009 Edition,
available at http://www.actexmadriver.com/
c 2009. Miguel A. Arcones. All rights reserved. Manual for SOA Exam FM/CAS Exam 2.
2/27
Chapter 1. Basic Interest Theory. Section 1.3. Compounded interest.
Compound interest
Under compound interest the amount function is
A(t) = A(0)(1 + i )
t
, t 0,
where i is the eective annual rate of interest.
Under compound interest, the eective rate of interest over a
certain period of time depends only on the length of this period, i.e.
for each 0 s < t,
A(t) A(s)
A(s)
=
A(t s) A(0)
A(0)
.
Notice that
A(t) A(s)
A(s)
=
A(0)(1 + i )
t
A(0)(1 + i )
s
A(0)(1 + i )
s
= (1 + i )
ts
1.
The eective rate of interest earned in the nth year is
i
n
=
A(n) A(n 1)
A(n 1)
=
A(0)(1 + i )
n
A(0)(1 + i )
n1
A(0)(1 + i )
n1
= i .
c 2009. Miguel A. Arcones. All rights reserved. Manual for SOA Exam FM/CAS Exam 2.
3/27
Chapter 1. Basic Interest Theory. Section 1.3. Compounded interest.
Compound interest
Under compound interest the amount function is
A(t) = A(0)(1 + i )
t
, t 0,
where i is the eective annual rate of interest.
Under compound interest, the eective rate of interest over a
certain period of time depends only on the length of this period, i.e.
for each 0 s < t,
A(t) A(s)
A(s)
=
A(t s) A(0)
A(0)
.
Notice that
A(t) A(s)
A(s)
=
A(0)(1 + i )
t
A(0)(1 + i )
s
A(0)(1 + i )
s
= (1 + i )
ts
1.
The eective rate of interest earned in the nth year is
i
n
=
A(n) A(n 1)
A(n 1)
=
A(0)(1 + i )
n
A(0)(1 + i )
n1
A(0)(1 + i )
n1
= i .
c 2009. Miguel A. Arcones. All rights reserved. Manual for SOA Exam FM/CAS Exam 2.
4/27
Chapter 1. Basic Interest Theory. Section 1.3. Compounded interest.
Under compound interest, the present value at time t of a deposit
of k made at time s is
kA(t)
A(s)
=
kA(0)(1 + i )
t
A(0)(1 + i )
s
= k(1 + i )
ts
.
If deposits/withdrawals are made according with the table
Deposits C
1
C
2
C
n
Time (in years) t
1
t
2
t
n
where 0 t
1
< t
2
< < t
n
, into an account earning compound
interest with an annual eective rate of interest of i , then the
present value at time t of the cashow is
V(t) =
n

j =1
C
j
(1 + i )
tt
j
.
In particular, the present value of the considered cashow at time
zero is

n
j =1
C
j
(1 + i )
t
j
.
c 2009. Miguel A. Arcones. All rights reserved. Manual for SOA Exam FM/CAS Exam 2.
5/27
Chapter 1. Basic Interest Theory. Section 1.3. Compounded interest.
Under compound interest, the present value at time t of a deposit
of k made at time s is
kA(t)
A(s)
=
kA(0)(1 + i )
t
A(0)(1 + i )
s
= k(1 + i )
ts
.
If deposits/withdrawals are made according with the table
Deposits C
1
C
2
C
n
Time (in years) t
1
t
2
t
n
where 0 t
1
< t
2
< < t
n
, into an account earning compound
interest with an annual eective rate of interest of i , then the
present value at time t of the cashow is
V(t) =
n

j =1
C
j
(1 + i )
tt
j
.
In particular, the present value of the considered cashow at time
zero is

n
j =1
C
j
(1 + i )
t
j
.
c 2009. Miguel A. Arcones. All rights reserved. Manual for SOA Exam FM/CAS Exam 2.
6/27
Chapter 1. Basic Interest Theory. Section 1.3. Compounded interest.
Example 1
A loan with an eective annual interest rate of 5.5% is to be
repaid with the following payments:
(i) 1000 at the end of the rst year.
(ii) 2000 at the end of the second year.
(iii) 5000 at the end of the third year.
Calculate the loaned amount at time 0.
Solution: The cashow of payments to the loan is
Payments 1000 2000 5000
Time 1 2 3
The loaned amount at time zero is the present value at time zero
of the cashow of payments, which is
(1000)(1.055)
1
+ (2000)(1.055)
2
+ (5000)(1.055)
3
=947.8672986 + 1796.904831 + 4258.068321 = 7002.840451.
c 2009. Miguel A. Arcones. All rights reserved. Manual for SOA Exam FM/CAS Exam 2.
7/27
Chapter 1. Basic Interest Theory. Section 1.3. Compounded interest.
Example 1
A loan with an eective annual interest rate of 5.5% is to be
repaid with the following payments:
(i) 1000 at the end of the rst year.
(ii) 2000 at the end of the second year.
(iii) 5000 at the end of the third year.
Calculate the loaned amount at time 0.
Solution: The cashow of payments to the loan is
Payments 1000 2000 5000
Time 1 2 3
The loaned amount at time zero is the present value at time zero
of the cashow of payments, which is
(1000)(1.055)
1
+ (2000)(1.055)
2
+ (5000)(1.055)
3
=947.8672986 + 1796.904831 + 4258.068321 = 7002.840451.
c 2009. Miguel A. Arcones. All rights reserved. Manual for SOA Exam FM/CAS Exam 2.
8/27
Chapter 1. Basic Interest Theory. Section 1.3. Compounded interest.
The accumulation function for simple interest is a(t) = 1 + it,
which is a linear function.
The accumulation function for compound interest is
a(t) = (1 + i )
t
, which is an increasing convex function.
We have that
(i) If 0 < t < 1, then (1 + i )
t
< 1 + it.
(ii) If 1 < t, then 1 + it < (1 + i )
t
.
c 2009. Miguel A. Arcones. All rights reserved. Manual for SOA Exam FM/CAS Exam 2.
9/27
Chapter 1. Basic Interest Theory. Section 1.3. Compounded interest.
Figure 1: comparison of simple and compound accumulation functions
c 2009. Miguel A. Arcones. All rights reserved. Manual for SOA Exam FM/CAS Exam 2.
10/27
Chapter 1. Basic Interest Theory. Section 1.3. Compounded interest.
Usually, we solve for variables in the formula, A(t) = A(0)(1 + i )
t
,
using the TIBAII-Plus calculator.
To turn on the calculator press ON/OFF .
To clear errors press CE/C . It clears the current displays
(including error messages) and tentative operations.
When entering a number, you realized that you make a mistake
you can clear the whole display by pressing CE/C .
When entering numbers, if you would like to save some of the
entered digits, you can press as many times as digits you
would like to remove. Digits are deleted starting from the last
entered digit.
It is recommended to setup the TI-BAIIPlus calculator to 9
decimals. You can do that doing
2nd , FORMAT , 9 , ENTER , 2nd , QUIT .
c 2009. Miguel A. Arcones. All rights reserved. Manual for SOA Exam FM/CAS Exam 2.
11/27
Chapter 1. Basic Interest Theory. Section 1.3. Compounded interest.
We often will use the time value of the money worksheet of the
calculator. There are 5 main nancial variables in this worksheet:

The number of periods N .

The nominal interest for year I/Y .

The present value PV .

The payment per period PMT .

The future value FV .


You can use the calculator to nd one of these nancial variables,
by entering the rest of the variables in the memory of the
calculator and then pressing CPT nancial key , where nancial
key is either N , % i , PV , PMT or FV .
c 2009. Miguel A. Arcones. All rights reserved. Manual for SOA Exam FM/CAS Exam 2.
12/27
Chapter 1. Basic Interest Theory. Section 1.3. Compounded interest.
Here, nancial key is either N , % i , PV , PMT or FV .

You can recall the entries in the time value of the money
worksheet, by pressing RCL nancial key .

To enter a variable in the entry nancial key , type the entry


and press nancial key . The entry of variables can be done
in any order.

To nd the value of any of the ve variables (after entering


the rest of the variables in the memory) press CPT
nancial key .

When computing a variable, a formula using all ve variables


and two auxiliary variables is used
c 2009. Miguel A. Arcones. All rights reserved. Manual for SOA Exam FM/CAS Exam 2.
13/27
Chapter 1. Basic Interest Theory. Section 1.3. Compounded interest.
To setup C/Y =1 and P/Y =1, do
2nd , P/Y , 1 , ENTER , , 1 , ENTER , 2nd , QUIT .
To check that this is so, do
2nd P/Y 2nd QUIT .
If PMT equals zero, C/Y =1 and P/Y =1, you have the
formula,
PV + FV

1 +
I/Y
100

N
= 0. (1)
You can use this to solve for any element of the four elements in
the formula A(t) = A(0)(1 + i )
t
. Unless it is said otherwise, we
will assume that the entries for C/Y and P/Y are both 1
and PMT is 0.
c 2009. Miguel A. Arcones. All rights reserved. Manual for SOA Exam FM/CAS Exam 2.
14/27
Chapter 1. Basic Interest Theory. Section 1.3. Compounded interest.
Example 2
Mary invested $12000 on January 1, 1995. Assuming composite
interest at 5 % per year, nd the accumulated value on January 1,
2002.
Solution: A(t) = 12000(1 + 0.05)
7
= 16885.21. You can do this
in the calculator by entering:
0 PMT 7 N 5 I/Y 12000 PV CPT FV .
Note that since the calculator, uses the formula
PV + FV

1 +
I/Y
100

N
= 0.
the display in your calculator is negative.
c 2009. Miguel A. Arcones. All rights reserved. Manual for SOA Exam FM/CAS Exam 2.
15/27
Chapter 1. Basic Interest Theory. Section 1.3. Compounded interest.
Example 2
Mary invested $12000 on January 1, 1995. Assuming composite
interest at 5 % per year, nd the accumulated value on January 1,
2002.
Solution: A(t) = 12000(1 + 0.05)
7
= 16885.21. You can do this
in the calculator by entering:
0 PMT 7 N 5 I/Y 12000 PV CPT FV .
Note that since the calculator, uses the formula
PV + FV

1 +
I/Y
100

N
= 0.
the display in your calculator is negative.
c 2009. Miguel A. Arcones. All rights reserved. Manual for SOA Exam FM/CAS Exam 2.
16/27
Chapter 1. Basic Interest Theory. Section 1.3. Compounded interest.
Example 3
At what annual rate of compound interest will $200 grow to $275
in 5 years?
Solution: We solve for i in 275 = 200(1 + i )
5
and get
i = 6.5763%. In the calculator, you do
275 FV 5 N 200 PV CPT I/Y .
Since the calculator, uses the formula (1), either the present value
or the future value has to be entered as negative number (and the
other one as a positive number). If you enter both the present
value and the future value as positive values, you get the error
message Error 5 . To clear this error message press CE/C .
c 2009. Miguel A. Arcones. All rights reserved. Manual for SOA Exam FM/CAS Exam 2.
17/27
Chapter 1. Basic Interest Theory. Section 1.3. Compounded interest.
Example 3
At what annual rate of compound interest will $200 grow to $275
in 5 years?
Solution: We solve for i in 275 = 200(1 + i )
5
and get
i = 6.5763%. In the calculator, you do
275 FV 5 N 200 PV CPT I/Y .
Since the calculator, uses the formula (1), either the present value
or the future value has to be entered as negative number (and the
other one as a positive number). If you enter both the present
value and the future value as positive values, you get the error
message Error 5 . To clear this error message press CE/C .
c 2009. Miguel A. Arcones. All rights reserved. Manual for SOA Exam FM/CAS Exam 2.
18/27
Chapter 1. Basic Interest Theory. Section 1.3. Compounded interest.
Example 4
How many years does it take $200 grow to $275 at an eective
annual rate of 5%?
Solution: We solve for t in 275 = 200(1 + 0.05)
t
and get that
t = 6.5270 years. In the calculator, you do
275 FV 5 I/Y 200 PV CPT N .
c 2009. Miguel A. Arcones. All rights reserved. Manual for SOA Exam FM/CAS Exam 2.
19/27
Chapter 1. Basic Interest Theory. Section 1.3. Compounded interest.
Example 4
How many years does it take $200 grow to $275 at an eective
annual rate of 5%?
Solution: We solve for t in 275 = 200(1 + 0.05)
t
and get that
t = 6.5270 years. In the calculator, you do
275 FV 5 I/Y 200 PV CPT N .
c 2009. Miguel A. Arcones. All rights reserved. Manual for SOA Exam FM/CAS Exam 2.
20/27
Chapter 1. Basic Interest Theory. Section 1.3. Compounded interest.
Example 5
At an annual eective rate of interest of 8% how long would it
take to triple your money?
Solution: We solve for t in 3 = (1 + 0.08)
t
and get t = 14.2749
years. In the calculator, you do
3 FV 8 I/Y 1 PV CPT N .
c 2009. Miguel A. Arcones. All rights reserved. Manual for SOA Exam FM/CAS Exam 2.
21/27
Chapter 1. Basic Interest Theory. Section 1.3. Compounded interest.
Example 5
At an annual eective rate of interest of 8% how long would it
take to triple your money?
Solution: We solve for t in 3 = (1 + 0.08)
t
and get t = 14.2749
years. In the calculator, you do
3 FV 8 I/Y 1 PV CPT N .
c 2009. Miguel A. Arcones. All rights reserved. Manual for SOA Exam FM/CAS Exam 2.
22/27
Chapter 1. Basic Interest Theory. Section 1.3. Compounded interest.
Example 6
How much money was needed to invest 10 years in the past to
accumulate $ 10000 at an eective annual rate of 5%?
Solution: We solve for A(0) in 10000 = A(0)(1 + 0.05)
10
and get
that A(0) = 6139.13. In the calculator, you do
10000 FV 5 I/Y 10 N CPT PV .
c 2009. Miguel A. Arcones. All rights reserved. Manual for SOA Exam FM/CAS Exam 2.
23/27
Chapter 1. Basic Interest Theory. Section 1.3. Compounded interest.
Example 6
How much money was needed to invest 10 years in the past to
accumulate $ 10000 at an eective annual rate of 5%?
Solution: We solve for A(0) in 10000 = A(0)(1 + 0.05)
10
and get
that A(0) = 6139.13. In the calculator, you do
10000 FV 5 I/Y 10 N CPT PV .
c 2009. Miguel A. Arcones. All rights reserved. Manual for SOA Exam FM/CAS Exam 2.
24/27
Chapter 1. Basic Interest Theory. Section 1.3. Compounded interest.
The calculator has a memory worksheet with values in the memory,
which stores ten numbers. These ten numbers are called: M0 ,
, M9 .To enter the number in the display into the i th entry of
the memory, press STO i , where i is an integer from 0 to 9. To
recall the number in the memory entry i , press RCL i , where i is
an integer from 0 to 9. The command STO + i adds the
value in display to the entry i in the memory. You can see all the
numbers in the memory by accessing the memory worksheet. To
enter this worksheet press 2nd MEM . Use the arrows , to
move from entry to another. To entry a new value in one entry,
type the number and press ENTER .
c 2009. Miguel A. Arcones. All rights reserved. Manual for SOA Exam FM/CAS Exam 2.
25/27
Chapter 1. Basic Interest Theory. Section 1.3. Compounded interest.
Example 7
A loan with an eective annual interest rate of 5.5% is to be
repaid with the following payments:
(i) 1000 at the end of the rst year.
(ii) 2000 at the end of the second year.
(iii) 5000 at the end of the third year.
Calculate the loaned amount at time 0.
Solution: The cashow of payments to the loan is
Payments 1000 2000 5000
Time 1 2 3
The loaned amount at time zero is the present value at time zero
of the cashow of payments, which is
(1000)(1.055)
1
+ (2000)(1.055)
2
+ (5000)(1.055)
3
=947.8672986 + 1796.904831 + 4258.068321 = 7002.840451.
c 2009. Miguel A. Arcones. All rights reserved. Manual for SOA Exam FM/CAS Exam 2.
26/27
Chapter 1. Basic Interest Theory. Section 1.3. Compounded interest.
Example 7
A loan with an eective annual interest rate of 5.5% is to be
repaid with the following payments:
(i) 1000 at the end of the rst year.
(ii) 2000 at the end of the second year.
(iii) 5000 at the end of the third year.
Calculate the loaned amount at time 0.
Solution: The cashow of payments to the loan is
Payments 1000 2000 5000
Time 1 2 3
The loaned amount at time zero is the present value at time zero
of the cashow of payments, which is
(1000)(1.055)
1
+ (2000)(1.055)
2
+ (5000)(1.055)
3
=947.8672986 + 1796.904831 + 4258.068321 = 7002.840451.
c 2009. Miguel A. Arcones. All rights reserved. Manual for SOA Exam FM/CAS Exam 2.
27/27
Chapter 1. Basic Interest Theory. Section 1.3. Compounded interest.
Using the calculator, you do
1000 FV 1 N 5.5 I/Y CPT PV
and get (1000)(1.055)
1
= 947.8672986. You enter this number in
the memory of the calculator doing STO 1
Next doing
2000 FV 2 N CPT PV
you nd (2000)(1.055)
2
= 1796.904831. Notice that you do not
have to reenter the percentage interest rate. You enter this
number in the memory of the calculator doing STO 2
Next doing
5000 FV 3 N CPT PV
you get (5000)(1.055)
3
= 4258.068321. You enter this number in
the memory of the calculator doing STO 3 .
You can recall and add the three numbers doing
CRCL 1 + CRCL 2 + CRCL 3 =
and get 7002.840451.
c 2009. Miguel A. Arcones. All rights reserved. Manual for SOA Exam FM/CAS Exam 2.
1/19
Chapter 1. Basic Interest Theory.
Manual for SOA Exam FM/CAS Exam 2.
Chapter 1. Basic Interest Theory.
Section 1.4. Present value and discount.
c 2008. Miguel A. Arcones. All rights reserved.
Extract from:
Arcones Manual for the SOA Exam FM/CAS Exam 2,
Financial Mathematics. Spring 2009 Edition,
available at http://www.actexmadriver.com/
c 2008. Miguel A. Arcones. All rights reserved. Manual for SOA Exam FM/CAS Exam 2.
2/19
Chapter 1. Basic Interest Theory. Section 1.4. Present value and discount.
Present value and discount
Suppose that we make an investment of $k in an account earning
compound interest with eective annual rate of interest i . t years
later the balance in this account is k(1 + i )
t
. Here, k(1 + i )
t
is the
future value of the investment t years in the future. Under
compound interest, balances multiply by (1 + i )
t
every t years. $k
at time s is worth $k(1 + i )
t
at time s + t.
The quantity (1 + i )
t
is called the tyear interest factor.
The quantity (1 + i ) is called the interest factor. $k at time s is
worth $k(1 + i ) at time s + 1.
c 2008. Miguel A. Arcones. All rights reserved. Manual for SOA Exam FM/CAS Exam 2.
3/19
Chapter 1. Basic Interest Theory. Section 1.4. Present value and discount.
Present value and discount
Suppose that we make an investment of $k in an account earning
compound interest with eective annual rate of interest i . t years
later the balance in this account is k(1 + i )
t
. Here, k(1 + i )
t
is the
future value of the investment t years in the future. Under
compound interest, balances multiply by (1 + i )
t
every t years. $k
at time s is worth $k(1 + i )
t
at time s + t.
The quantity (1 + i )
t
is called the tyear interest factor.
The quantity (1 + i ) is called the interest factor. $k at time s is
worth $k(1 + i ) at time s + 1.
c 2008. Miguel A. Arcones. All rights reserved. Manual for SOA Exam FM/CAS Exam 2.
4/19
Chapter 1. Basic Interest Theory. Section 1.4. Present value and discount.
Present value and discount
Suppose that we make an investment of $k in an account earning
compound interest with eective annual rate of interest i . t years
later the balance in this account is k(1 + i )
t
. Here, k(1 + i )
t
is the
future value of the investment t years in the future. Under
compound interest, balances multiply by (1 + i )
t
every t years. $k
at time s is worth $k(1 + i )
t
at time s + t.
The quantity (1 + i )
t
is called the tyear interest factor.
The quantity (1 + i ) is called the interest factor. $k at time s is
worth $k(1 + i ) at time s + 1.
c 2008. Miguel A. Arcones. All rights reserved. Manual for SOA Exam FM/CAS Exam 2.
5/19
Chapter 1. Basic Interest Theory. Section 1.4. Present value and discount.
Often, we need to nd the amount of money t years in the past
needed to accumulate certain principal. The present value t years
in the past is the amount of money which will accumulate to the
principal over t years.
In the case of compound interest with eective annual rate of
interest i , the present value of $1 t years in the past is
1
(1+i )
t
. If we
invested
1
(1+i )
t
t years ago in account earning compound interest,
then the current balance is $1. The quantity
1
(1+i )
t
is called the t
year discount. $k at time s is worth $k
t
at time s t.
The quantity =
1
1+i
is called the discount factor. In order to
accumulate $1, we need $ one year in the past.
c 2008. Miguel A. Arcones. All rights reserved. Manual for SOA Exam FM/CAS Exam 2.
6/19
Chapter 1. Basic Interest Theory. Section 1.4. Present value and discount.
Under the accumulation function a(t),

The nth year interest factor is


a(n)
a(n1)
.

The eective rate of interest in the nth year is


i
n
=
a(n)a(n1)
a(n1)
.

The n year discount factor is


n
=
a(n1)
a(n)
.

The eective rate of discount in the nth year is


d
n
=
a(n)a(n1)
a(n)
.
i
n
and d
n
are both proportions of interest over amount values, but
i
n
uses the amount value in the past and d
n
uses the amount value
in the future. Since the amount value in the future is bigger than
the amount value in the past, d
n
< i
n
.
c 2008. Miguel A. Arcones. All rights reserved. Manual for SOA Exam FM/CAS Exam 2.
7/19
Chapter 1. Basic Interest Theory. Section 1.4. Present value and discount.
Notice that

the nth year interest factor is equal to 1 + i


n
.


n
= 1 d
n
.

1 = (1 + i
n
)
n
= (1 + i
n
)(1 d
n
)

{1 unit at time n 1} {1 + i
n
units at time n}. So,
d
n
=
i
n
1+i
n

{1 d
n
unit at time n 1} {1 units at time n}. So,
i
n
=
d
n
1d
n
.
c 2008. Miguel A. Arcones. All rights reserved. Manual for SOA Exam FM/CAS Exam 2.
8/19
Chapter 1. Basic Interest Theory. Section 1.4. Present value and discount.
Under compound interest, the eective rate of discount d
n
is
constant d
n
=
a(n)a(n1)
a(n)
=
(1+i )
n
(1+i )
n1
(1+i )
n
=
i
1+i
. Under
compound interest,
=
1
1 + i
, d = 1 , d =
i
i + 1
and (1 d)(1 + i ) = 1.
c 2008. Miguel A. Arcones. All rights reserved. Manual for SOA Exam FM/CAS Exam 2.
9/19
Chapter 1. Basic Interest Theory. Section 1.4. Present value and discount.
Example 1
Peter invests $738 in a bank account. One year later, his bank
account is $765.
(i) Find the eective annual interest rate earned by Peter in that
year.
(ii) Find the eective annual discount rate earned by Peter in that
year.
Solution: (i)Peter earns an interest amount of 765 738 = 27.
The eective annual interest rate earned by Peter is
27
738
= 3.658537%.
(ii) The eective annual discount rate earned by Peter is
27
765
= 3.529412%.
c 2008. Miguel A. Arcones. All rights reserved. Manual for SOA Exam FM/CAS Exam 2.
10/19
Chapter 1. Basic Interest Theory. Section 1.4. Present value and discount.
Example 1
Peter invests $738 in a bank account. One year later, his bank
account is $765.
(i) Find the eective annual interest rate earned by Peter in that
year.
(ii) Find the eective annual discount rate earned by Peter in that
year.
Solution: (i)Peter earns an interest amount of 765 738 = 27.
The eective annual interest rate earned by Peter is
27
738
= 3.658537%.
(ii) The eective annual discount rate earned by Peter is
27
765
= 3.529412%.
c 2008. Miguel A. Arcones. All rights reserved. Manual for SOA Exam FM/CAS Exam 2.
11/19
Chapter 1. Basic Interest Theory. Section 1.4. Present value and discount.
Example 1
Peter invests $738 in a bank account. One year later, his bank
account is $765.
(i) Find the eective annual interest rate earned by Peter in that
year.
(ii) Find the eective annual discount rate earned by Peter in that
year.
Solution: (i)Peter earns an interest amount of 765 738 = 27.
The eective annual interest rate earned by Peter is
27
738
= 3.658537%.
(ii) The eective annual discount rate earned by Peter is
27
765
= 3.529412%.
c 2008. Miguel A. Arcones. All rights reserved. Manual for SOA Exam FM/CAS Exam 2.
12/19
Chapter 1. Basic Interest Theory. Section 1.4. Present value and discount.
Example 2
If i = 7%, what are d and ?
Solution: We have that d =
i
1+i
=
0.07
1+0.07
= 6.5421% and
=
1
1+i
=
1
1+0.07
= 0.934579.
c 2008. Miguel A. Arcones. All rights reserved. Manual for SOA Exam FM/CAS Exam 2.
13/19
Chapter 1. Basic Interest Theory. Section 1.4. Present value and discount.
Example 2
If i = 7%, what are d and ?
Solution: We have that d =
i
1+i
=
0.07
1+0.07
= 6.5421% and
=
1
1+i
=
1
1+0.07
= 0.934579.
c 2008. Miguel A. Arcones. All rights reserved. Manual for SOA Exam FM/CAS Exam 2.
14/19
Chapter 1. Basic Interest Theory. Section 1.4. Present value and discount.
Example 3
If = 0.95, what are d and i ?
Solution: We have that d = 1 = 1 0.95 = 0.05 and
i =
1

1 =
10.95
0.95
= 5.2632%.
c 2008. Miguel A. Arcones. All rights reserved. Manual for SOA Exam FM/CAS Exam 2.
15/19
Chapter 1. Basic Interest Theory. Section 1.4. Present value and discount.
Example 3
If = 0.95, what are d and i ?
Solution: We have that d = 1 = 1 0.95 = 0.05 and
i =
1

1 =
10.95
0.95
= 5.2632%.
c 2008. Miguel A. Arcones. All rights reserved. Manual for SOA Exam FM/CAS Exam 2.
16/19
Chapter 1. Basic Interest Theory. Section 1.4. Present value and discount.
Example 4
What is the present value of $5,000 to be received in 7 years at an
annual eective rate of discount of 7%?
Solution: The value is (5000)(1 0.07)
7
= 3008.504353.
c 2008. Miguel A. Arcones. All rights reserved. Manual for SOA Exam FM/CAS Exam 2.
17/19
Chapter 1. Basic Interest Theory. Section 1.4. Present value and discount.
Example 4
What is the present value of $5,000 to be received in 7 years at an
annual eective rate of discount of 7%?
Solution: The value is (5000)(1 0.07)
7
= 3008.504353.
c 2008. Miguel A. Arcones. All rights reserved. Manual for SOA Exam FM/CAS Exam 2.
18/19
Chapter 1. Basic Interest Theory. Section 1.4. Present value and discount.
Example 5
At time t = 0, Paul deposits $3500 into a fund crediting interest
with an annual discount factor of 0.96. Find the fund value at time
2.5.
Solution: (3500)(0.96)
2.5
= 3876.055.
c 2008. Miguel A. Arcones. All rights reserved. Manual for SOA Exam FM/CAS Exam 2.
19/19
Chapter 1. Basic Interest Theory. Section 1.4. Present value and discount.
Example 5
At time t = 0, Paul deposits $3500 into a fund crediting interest
with an annual discount factor of 0.96. Find the fund value at time
2.5.
Solution: (3500)(0.96)
2.5
= 3876.055.
c 2008. Miguel A. Arcones. All rights reserved. Manual for SOA Exam FM/CAS Exam 2.
1/24
Chapter 1. Basic Interest Theory.
Manual for SOA Exam FM/CAS Exam 2.
Chapter 1. Basic Interest Theory.
Section 1.5. Nominal rates of interest and discount.
c 2008. Miguel A. Arcones. All rights reserved.
Extract from:
Arcones Manual for the SOA Exam FM/CAS Exam 2,
Financial Mathematics. Spring 2009 Edition,
available at http://www.actexmadriver.com/
c 2008. Miguel A. Arcones. All rights reserved. Manual for SOA Exam FM/CAS Exam 2.
2/24
Chapter 1. Basic Interest Theory. Section 1.5. Nominal rates of interest and discount.
Nominal rate of interest
When dealing with compound interest, often we will rates dierent
from the annual eective interest rate. Suppose that an account
follows compound interest with an annual nominal rate of
interest compounded m times a year of i
(m)
, then

$1 at time zero accrues to $(1 +


i
(m)
m
) at time
1
m
years.

The
1
m
year interest factor is (1 +
i
(m)
m
).

The (
1
m
-year ) mthly eective interest rate is
i
(m)
m
.

$1 at time zero grows to $


_
1 +
i
(m)
m
_
m
in one year.

$1 at time zero grows to $


_
1 +
i
(m)
m
_
mt
in t years.

The accumulation function is a(t) =


_
1 +
i
(m)
m
_
mt
.
c 2008. Miguel A. Arcones. All rights reserved. Manual for SOA Exam FM/CAS Exam 2.
3/24
Chapter 1. Basic Interest Theory. Section 1.5. Nominal rates of interest and discount.
Nominal rate of interest
When dealing with compound interest, often we will rates dierent
from the annual eective interest rate. Suppose that an account
follows compound interest with an annual nominal rate of
interest compounded m times a year of i
(m)
, then

$1 at time zero accrues to $(1 +


i
(m)
m
) at time
1
m
years.

The
1
m
year interest factor is (1 +
i
(m)
m
).

The (
1
m
-year ) mthly eective interest rate is
i
(m)
m
.

$1 at time zero grows to $


_
1 +
i
(m)
m
_
m
in one year.

$1 at time zero grows to $


_
1 +
i
(m)
m
_
mt
in t years.

The accumulation function is a(t) =


_
1 +
i
(m)
m
_
mt
.
c 2008. Miguel A. Arcones. All rights reserved. Manual for SOA Exam FM/CAS Exam 2.
4/24
Chapter 1. Basic Interest Theory. Section 1.5. Nominal rates of interest and discount.
Nominal rate of interest
When dealing with compound interest, often we will rates dierent
from the annual eective interest rate. Suppose that an account
follows compound interest with an annual nominal rate of
interest compounded m times a year of i
(m)
, then

$1 at time zero accrues to $(1 +


i
(m)
m
) at time
1
m
years.

The
1
m
year interest factor is (1 +
i
(m)
m
).

The (
1
m
-year ) mthly eective interest rate is
i
(m)
m
.

$1 at time zero grows to $


_
1 +
i
(m)
m
_
m
in one year.

$1 at time zero grows to $


_
1 +
i
(m)
m
_
mt
in t years.

The accumulation function is a(t) =


_
1 +
i
(m)
m
_
mt
.
c 2008. Miguel A. Arcones. All rights reserved. Manual for SOA Exam FM/CAS Exam 2.
5/24
Chapter 1. Basic Interest Theory. Section 1.5. Nominal rates of interest and discount.
Nominal rate of interest
When dealing with compound interest, often we will rates dierent
from the annual eective interest rate. Suppose that an account
follows compound interest with an annual nominal rate of
interest compounded m times a year of i
(m)
, then

$1 at time zero accrues to $(1 +


i
(m)
m
) at time
1
m
years.

The
1
m
year interest factor is (1 +
i
(m)
m
).

The (
1
m
-year ) mthly eective interest rate is
i
(m)
m
.

$1 at time zero grows to $


_
1 +
i
(m)
m
_
m
in one year.

$1 at time zero grows to $


_
1 +
i
(m)
m
_
mt
in t years.

The accumulation function is a(t) =


_
1 +
i
(m)
m
_
mt
.
c 2008. Miguel A. Arcones. All rights reserved. Manual for SOA Exam FM/CAS Exam 2.
6/24
Chapter 1. Basic Interest Theory. Section 1.5. Nominal rates of interest and discount.
Nominal rate of interest
When dealing with compound interest, often we will rates dierent
from the annual eective interest rate. Suppose that an account
follows compound interest with an annual nominal rate of
interest compounded m times a year of i
(m)
, then

$1 at time zero accrues to $(1 +


i
(m)
m
) at time
1
m
years.

The
1
m
year interest factor is (1 +
i
(m)
m
).

The (
1
m
-year ) mthly eective interest rate is
i
(m)
m
.

$1 at time zero grows to $


_
1 +
i
(m)
m
_
m
in one year.

$1 at time zero grows to $


_
1 +
i
(m)
m
_
mt
in t years.

The accumulation function is a(t) =


_
1 +
i
(m)
m
_
mt
.
c 2008. Miguel A. Arcones. All rights reserved. Manual for SOA Exam FM/CAS Exam 2.
7/24
Chapter 1. Basic Interest Theory. Section 1.5. Nominal rates of interest and discount.
Nominal rate of interest
When dealing with compound interest, often we will rates dierent
from the annual eective interest rate. Suppose that an account
follows compound interest with an annual nominal rate of
interest compounded m times a year of i
(m)
, then

$1 at time zero accrues to $(1 +


i
(m)
m
) at time
1
m
years.

The
1
m
year interest factor is (1 +
i
(m)
m
).

The (
1
m
-year ) mthly eective interest rate is
i
(m)
m
.

$1 at time zero grows to $


_
1 +
i
(m)
m
_
m
in one year.

$1 at time zero grows to $


_
1 +
i
(m)
m
_
mt
in t years.

The accumulation function is a(t) =


_
1 +
i
(m)
m
_
mt
.
c 2008. Miguel A. Arcones. All rights reserved. Manual for SOA Exam FM/CAS Exam 2.
8/24
Chapter 1. Basic Interest Theory. Section 1.5. Nominal rates of interest and discount.
Example 1
Paul takes a loan of $569. Interest in the loan is charged using
compound interest. One month after a loan is taken the balance in
this loan is $581.
(i) Find the monthly eective interest rate, which Paul is charged
in his loan.
(ii) Find the annual nominal interest rate compounded monthly,
which Paul is charged in his loan.
Solution: (i) The monthly eective interest rate, which Paul is
charged in his loan is
i
(12)
12
=
581 569
569
= 2.108963093%.
(ii) The annual nominal interest rate compounded monthly, which
Paul is charged in his loan is
i
(12)
= (12)(0.02108963093) = 25.30755712%.
c 2008. Miguel A. Arcones. All rights reserved. Manual for SOA Exam FM/CAS Exam 2.
9/24
Chapter 1. Basic Interest Theory. Section 1.5. Nominal rates of interest and discount.
Example 1
Paul takes a loan of $569. Interest in the loan is charged using
compound interest. One month after a loan is taken the balance in
this loan is $581.
(i) Find the monthly eective interest rate, which Paul is charged
in his loan.
(ii) Find the annual nominal interest rate compounded monthly,
which Paul is charged in his loan.
Solution: (i) The monthly eective interest rate, which Paul is
charged in his loan is
i
(12)
12
=
581 569
569
= 2.108963093%.
(ii) The annual nominal interest rate compounded monthly, which
Paul is charged in his loan is
i
(12)
= (12)(0.02108963093) = 25.30755712%.
c 2008. Miguel A. Arcones. All rights reserved. Manual for SOA Exam FM/CAS Exam 2.
10/24
Chapter 1. Basic Interest Theory. Section 1.5. Nominal rates of interest and discount.
Example 1
Paul takes a loan of $569. Interest in the loan is charged using
compound interest. One month after a loan is taken the balance in
this loan is $581.
(i) Find the monthly eective interest rate, which Paul is charged
in his loan.
(ii) Find the annual nominal interest rate compounded monthly,
which Paul is charged in his loan.
Solution: (i) The monthly eective interest rate, which Paul is
charged in his loan is
i
(12)
12
=
581 569
569
= 2.108963093%.
(ii) The annual nominal interest rate compounded monthly, which
Paul is charged in his loan is
i
(12)
= (12)(0.02108963093) = 25.30755712%.
c 2008. Miguel A. Arcones. All rights reserved. Manual for SOA Exam FM/CAS Exam 2.
11/24
Chapter 1. Basic Interest Theory. Section 1.5. Nominal rates of interest and discount.
Two rates of interest or discount are said to be equivalent if they
give rise to same accumulation function. Since, the accumulation
function under an annual eective rate of interest i is
a(t) = (1 +i )
t
, we have that a nominal annual rate of interest i
(m)
compounded m times a year is equivalent to an annual eective
rate of interest i , if the rates
a(t) =
_
1 +
i
(m)
m
_
mt
and
a(t) = (1 + i )
t
agree. This happens if and only if
_
1 +
i
(m)
m
_
m
= 1 + i .
c 2008. Miguel A. Arcones. All rights reserved. Manual for SOA Exam FM/CAS Exam 2.
12/24
Chapter 1. Basic Interest Theory. Section 1.5. Nominal rates of interest and discount.
Example 1
John takes a loan of 8,000 at a nominal annual rate of interest of
10% per year convertible quarterly. How much does he owe after
30 months?
Solution: We nd
8000
_
1 +
0.10
4
_30
12
4
= 8000 (1 + 0.025)
10
= 10240.68.
In the calculator, we do:
8000 PV 2.5 I/Y 10 N CPT FV .
c 2008. Miguel A. Arcones. All rights reserved. Manual for SOA Exam FM/CAS Exam 2.
13/24
Chapter 1. Basic Interest Theory. Section 1.5. Nominal rates of interest and discount.
Example 1
John takes a loan of 8,000 at a nominal annual rate of interest of
10% per year convertible quarterly. How much does he owe after
30 months?
Solution: We nd
8000
_
1 +
0.10
4
_30
12
4
= 8000 (1 + 0.025)
10
= 10240.68.
In the calculator, we do:
8000 PV 2.5 I/Y 10 N CPT FV .
c 2008. Miguel A. Arcones. All rights reserved. Manual for SOA Exam FM/CAS Exam 2.
14/24
Chapter 1. Basic Interest Theory. Section 1.5. Nominal rates of interest and discount.
The calculator TIBAIIPlus has a worksheet to convert nominal
rates of interest into eective rates of interest and vice versa. To
enter this worksheet press 2nd ICONV . There are 3 entries in
this worksheet: NOM , EFF and C/Y . C/Y is the number of
times the nominal interest is converted in a year. The relation
between these variables is
1 +
EFF
100
=
_
_
1 +
NOM
100 C/Y
_
_
C/Y
.
You can enter a value in any of these entries by moving to that
entry using the arrows: and . To enter a value in one entry,
type the value and press ENTER . You can compute the
corresponding nominal (eective) rate by moving to the entry
NOM ( EFF ) and pressing the key CPT . It is possible to enter
negative values in the entries NOM and EFF . However, the
value in the entry C/Y has to be positive.
c 2008. Miguel A. Arcones. All rights reserved. Manual for SOA Exam FM/CAS Exam 2.
15/24
Chapter 1. Basic Interest Theory. Section 1.5. Nominal rates of interest and discount.
Example 2
If i
(4)
= 5% nd the equivalent eective annual rate of interest.
Solution: We solve 1 + i =
_
1 +
0.05
4
_
4
and get i = 5.0945%. In
the calculator, you enter the worksheet ICONV and enter: NOM
equal to 5 and C/Y equal to 4. Then, go to EFF and press
CPT . To quit, press 2nd , QUIT .
c 2008. Miguel A. Arcones. All rights reserved. Manual for SOA Exam FM/CAS Exam 2.
16/24
Chapter 1. Basic Interest Theory. Section 1.5. Nominal rates of interest and discount.
Example 2
If i
(4)
= 5% nd the equivalent eective annual rate of interest.
Solution: We solve 1 + i =
_
1 +
0.05
4
_
4
and get i = 5.0945%. In
the calculator, you enter the worksheet ICONV and enter: NOM
equal to 5 and C/Y equal to 4. Then, go to EFF and press
CPT . To quit, press 2nd , QUIT .
c 2008. Miguel A. Arcones. All rights reserved. Manual for SOA Exam FM/CAS Exam 2.
17/24
Chapter 1. Basic Interest Theory. Section 1.5. Nominal rates of interest and discount.
Example 3
If i = 5%, what is the equivalent i
(4)
?
Solution: We solve
_
1 +
i
(4)
4
_
4
= 1 + 0.05 we get that
i
(4)
= 4
_
(1 + 0.05)
1/4
1
_
= 4.9089%. In the calculator, you
enter the worksheet ICONV and enter: EFF equal to 5 and
C/Y equal to 4. Then, go to NOM and press CPT . To quit,
press 2nd , QUIT .
c 2008. Miguel A. Arcones. All rights reserved. Manual for SOA Exam FM/CAS Exam 2.
18/24
Chapter 1. Basic Interest Theory. Section 1.5. Nominal rates of interest and discount.
Example 3
If i = 5%, what is the equivalent i
(4)
?
Solution: We solve
_
1 +
i
(4)
4
_
4
= 1 + 0.05 we get that
i
(4)
= 4
_
(1 + 0.05)
1/4
1
_
= 4.9089%. In the calculator, you
enter the worksheet ICONV and enter: EFF equal to 5 and
C/Y equal to 4. Then, go to NOM and press CPT . To quit,
press 2nd , QUIT .
c 2008. Miguel A. Arcones. All rights reserved. Manual for SOA Exam FM/CAS Exam 2.
19/24
Chapter 1. Basic Interest Theory. Section 1.5. Nominal rates of interest and discount.
The nominal rate of discount d
(m)
is dened as the value such
that 1 unit at the present is equivalent to 1
d
(m)
m
units invested
1
m
years ago, i.e.
{1
d
(m)
m
units at time 0}
_
1 unit at time
1
m
_
.
This implies that
{1 unit at time 0}
_
1
1
d
(m)
m
units at time
1
m
_
.
The accumulation function for compound interest under a the
nominal rate of discount d
(m)
convertible m times a year is
a(t) =
_
1
d
(m)
m
_
mt
. We have that
1 + i =
_
1 +
i
(m)
m
_
m
= (1 d)
1
=
_
1
d
(m)
m
_
m
.
c 2008. Miguel A. Arcones. All rights reserved. Manual for SOA Exam FM/CAS Exam 2.
20/24
Chapter 1. Basic Interest Theory. Section 1.5. Nominal rates of interest and discount.
In the calculator TIBAIIPlus, you may:

given i
(m)
, nd i , by entering
i
(m)
NOM and m C/Y , then in EFF press CPT .

given i , nd i
(m)
, by entering
i EFF and m C/Y , then in NOM press CPT .

given d
(m)
, nd d, by entering
d
(m)
NOM and m C/Y , then in EFF press CPT .
d appears with a negative sign.

given d, nd d
(m)
, by entering
d EFF and m C/Y , then in NOM press CPT .
d
(m)
appears with a negative sign.

given i , nd d, by using the formula i =


1
1d
1.

given d, nd i , by using the formula d = 1


1
1+i
.
c 2008. Miguel A. Arcones. All rights reserved. Manual for SOA Exam FM/CAS Exam 2.
21/24
Chapter 1. Basic Interest Theory. Section 1.5. Nominal rates of interest and discount.
Example 4
If d
(4)
= 5% nd i .
Solution: We solve
_
1
d
(4)
4
_
4
= 1 + i to get d = 4.9070% and
i = 5.1602%. In the calculator, in the ICONV worksheet, we
enter 5 in NOM , 4 in C/Y and we nd that EFF is
4.9070%, then we do
4.9070 % + 1 = 1/x 1 =
to get i = 5.1602%.
c 2008. Miguel A. Arcones. All rights reserved. Manual for SOA Exam FM/CAS Exam 2.
22/24
Chapter 1. Basic Interest Theory. Section 1.5. Nominal rates of interest and discount.
Example 4
If d
(4)
= 5% nd i .
Solution: We solve
_
1
d
(4)
4
_
4
= 1 + i to get d = 4.9070% and
i = 5.1602%. In the calculator, in the ICONV worksheet, we
enter 5 in NOM , 4 in C/Y and we nd that EFF is
4.9070%, then we do
4.9070 % + 1 = 1/x 1 =
to get i = 5.1602%.
c 2008. Miguel A. Arcones. All rights reserved. Manual for SOA Exam FM/CAS Exam 2.
23/24
Chapter 1. Basic Interest Theory. Section 1.5. Nominal rates of interest and discount.
Example 5
If i = 3% nd d
(2)
.
Solution: We solve for d
(2)
in
_
1
d
(2)
2
_
2
= 1 + i . First we nd
that d = 2.9126% doing
3 % + 1 = 1/x 1 =
Then, using the ICONV worksheet, we get that d
(2)
= 2.9341%.
c 2008. Miguel A. Arcones. All rights reserved. Manual for SOA Exam FM/CAS Exam 2.
24/24
Chapter 1. Basic Interest Theory. Section 1.5. Nominal rates of interest and discount.
Example 5
If i = 3% nd d
(2)
.
Solution: We solve for d
(2)
in
_
1
d
(2)
2
_
2
= 1 + i . First we nd
that d = 2.9126% doing
3 % + 1 = 1/x 1 =
Then, using the ICONV worksheet, we get that d
(2)
= 2.9341%.
c 2008. Miguel A. Arcones. All rights reserved. Manual for SOA Exam FM/CAS Exam 2.
1/12
Chapter 1. Basic Interest Theory.
Manual for SOA Exam FM/CAS Exam 2.
Chapter 1. Basic Interest Theory.
Section 1.6. Force of interest.
c 2008. Miguel A. Arcones. All rights reserved.
Extract from:
Arcones Manual for the SOA Exam FM/CAS Exam 2,
Financial Mathematics. Spring 2009 Edition,
available at http://www.actexmadriver.com/
c 2008. Miguel A. Arcones. All rights reserved. Manual for SOA Exam FM/CAS Exam 2.
2/12
Chapter 1. Basic Interest Theory. Section 1.6. Force of interest.
Force of interest
The force of interest
t
of an amount function A(t) is dened by

t
=
d
dt
ln A(t) =
A

(t)
A(t)
.
The force of interest is the fraction of the instantaneous rate of
change of the accumulation function and the accumulation
function.
To nd the force of interest, we may use the accumulation
function,
d
dt
ln A(t) =
d
dt
ln(A(0)a(t)) =
d
dt
ln(A(0)) +
d
dt
ln(a(t))
=
d
dt
ln(a(t)).
c 2008. Miguel A. Arcones. All rights reserved. Manual for SOA Exam FM/CAS Exam 2.
3/12
Chapter 1. Basic Interest Theory. Section 1.6. Force of interest.
Force of interest
The force of interest
t
of an amount function A(t) is dened by

t
=
d
dt
ln A(t) =
A

(t)
A(t)
.
The force of interest is the fraction of the instantaneous rate of
change of the accumulation function and the accumulation
function.
To nd the force of interest, we may use the accumulation
function,
d
dt
ln A(t) =
d
dt
ln(A(0)a(t)) =
d
dt
ln(A(0)) +
d
dt
ln(a(t))
=
d
dt
ln(a(t)).
c 2008. Miguel A. Arcones. All rights reserved. Manual for SOA Exam FM/CAS Exam 2.
4/12
Chapter 1. Basic Interest Theory. Section 1.6. Force of interest.
Force of interest
The force of interest
t
of an amount function A(t) is dened by

t
=
d
dt
ln A(t) =
A

(t)
A(t)
.
The force of interest is the fraction of the instantaneous rate of
change of the accumulation function and the accumulation
function.
To nd the force of interest, we may use the accumulation
function,
d
dt
ln A(t) =
d
dt
ln(A(0)a(t)) =
d
dt
ln(A(0)) +
d
dt
ln(a(t))
=
d
dt
ln(a(t)).
c 2008. Miguel A. Arcones. All rights reserved. Manual for SOA Exam FM/CAS Exam 2.
5/12
Chapter 1. Basic Interest Theory. Section 1.6. Force of interest.
Example 1
Consider the amount function A(t) = 25

1 +
t
4

3
. At what time is
the force of interest equal of 0.5.
Solution: We have that
ln(A(t)) = ln

25

1 +
t
4

= ln 25 + 3 ln

1 +
t
4

.
The force of interest is

t
=
d
dt
ln(A(t)) =
d
dt

ln 25 + 3 ln

1 +
t
4

= 3
1
4
1 +
t
4
=
3
4 + t
.
From the equation,
3
4+t
=
1
2
, we get that t = 2.
c 2008. Miguel A. Arcones. All rights reserved. Manual for SOA Exam FM/CAS Exam 2.
6/12
Chapter 1. Basic Interest Theory. Section 1.6. Force of interest.
Example 1
Consider the amount function A(t) = 25

1 +
t
4

3
. At what time is
the force of interest equal of 0.5.
Solution: We have that
ln(A(t)) = ln

25

1 +
t
4

= ln 25 + 3 ln

1 +
t
4

.
The force of interest is

t
=
d
dt
ln(A(t)) =
d
dt

ln 25 + 3 ln

1 +
t
4

= 3
1
4
1 +
t
4
=
3
4 + t
.
From the equation,
3
4+t
=
1
2
, we get that t = 2.
c 2008. Miguel A. Arcones. All rights reserved. Manual for SOA Exam FM/CAS Exam 2.
7/12
Chapter 1. Basic Interest Theory. Section 1.6. Force of interest.
The force of interest is also called the rate of interest
continuously compounded and the continuous interest rate.
We have that

t
= lim
h0
A(t + h) A(t)
A(t) h
= lim
h0
interest earned over the next h years
investment at time t h
.
The nominal annual rate earned in the next
1
m
years compounded
m times a year at time t is
m(a

t +
1
m

a(t))
a(t)
=
a

t +
1
m

a(t)
a(t)
1
m
.
We have that
lim
m
a

t +
1
m

a(t)
a(t)
1
m
=
t
.
c 2008. Miguel A. Arcones. All rights reserved. Manual for SOA Exam FM/CAS Exam 2.
8/12
Chapter 1. Basic Interest Theory. Section 1.6. Force of interest.
Under compound interest, a(t) = (1 + i )
t
and

t
=
d
dt
ln a(t) =
d
dt
ln(1 + i )
t
=
d
dt
t ln(1 + i ) = ln(1 + i )
Under compound interest, the force of interest is a constant ,
such that = ln(1 + i ) = ln .
Under compound interest,
lim
m
i
(m)
= lim
m
d
(m)
= .
In the case of simple interest, a(t) = 1 + it and

t
=
d
dt
ln(1 +it) =
i
1+it
. The force of interest is decreasing with t.
c 2008. Miguel A. Arcones. All rights reserved. Manual for SOA Exam FM/CAS Exam 2.
9/12
Chapter 1. Basic Interest Theory. Section 1.6. Force of interest.
From the force of interest
t
, we may nd the accumulation
function a(t), using
Theorem 2
For each t 0, a(t) = e
R
t
0

s
ds
.
Proof.
Since
s
=
d
ds
ln a(s) and a(0) = 1,

t
0

s
ds =

t
0
d
ds
ln a(s) ds = ln a(s)

t
0
= ln a(t).
So, a(t) = e
R
t
0

s
ds
.
c 2008. Miguel A. Arcones. All rights reserved. Manual for SOA Exam FM/CAS Exam 2.
10/12
Chapter 1. Basic Interest Theory. Section 1.6. Force of interest.
From the force of interest
t
, we may nd the accumulation
function a(t), using
Theorem 2
For each t 0, a(t) = e
R
t
0

s
ds
.
Proof.
Since
s
=
d
ds
ln a(s) and a(0) = 1,

t
0

s
ds =

t
0
d
ds
ln a(s) ds = ln a(s)

t
0
= ln a(t).
So, a(t) = e
R
t
0

s
ds
.
c 2008. Miguel A. Arcones. All rights reserved. Manual for SOA Exam FM/CAS Exam 2.
11/12
Chapter 1. Basic Interest Theory. Section 1.6. Force of interest.
Example 3
A bank account credits interest using a force of interest
t
=
3t
2
t
3
+2
.
A deposit of 100 is made in the account at time t = 0. Find the
amount of interest earned by the account from the end of the 4th
year until the end of the 8th year.
Solution: First, we nd a(t) = e
R
t
0

s
ds
.

t
0

s
ds =

t
0
3s
2
s
3
+ 2
ds = ln(s
3
+ 2)

t
0
=ln(t
3
+ 2) ln 2 = ln

t
3
+ 2
2

and
a(t) = e
R
t
0

s
ds
= e
ln

t
3
+2
2

=
t
3
+ 2
2
= 1 +
t
3
2
.
The amount of interest earned in the considered period is
100(a(8) a(4)) = (100)

1 +
8
3
2

1 +
4
3
2

= 22400.
c 2008. Miguel A. Arcones. All rights reserved. Manual for SOA Exam FM/CAS Exam 2.
12/12
Chapter 1. Basic Interest Theory. Section 1.6. Force of interest.
Example 3
A bank account credits interest using a force of interest
t
=
3t
2
t
3
+2
.
A deposit of 100 is made in the account at time t = 0. Find the
amount of interest earned by the account from the end of the 4th
year until the end of the 8th year.
Solution: First, we nd a(t) = e
R
t
0

s
ds
.

t
0

s
ds =

t
0
3s
2
s
3
+ 2
ds = ln(s
3
+ 2)

t
0
=ln(t
3
+ 2) ln 2 = ln

t
3
+ 2
2

and
a(t) = e
R
t
0

s
ds
= e
ln

t
3
+2
2

=
t
3
+ 2
2
= 1 +
t
3
2
.
The amount of interest earned in the considered period is
100(a(8) a(4)) = (100)

1 +
8
3
2

1 +
4
3
2

= 22400.
c 2008. Miguel A. Arcones. All rights reserved. Manual for SOA Exam FM/CAS Exam 2.
1/18
Chapter 2. Cashows.
Manual for SOA Exam FM/CAS Exam 2.
Chapter 2. Cashows.
Section 2.1. Cashows.
c 2009. Miguel A. Arcones. All rights reserved.
Extract from:
Arcones Manual for the SOA Exam FM/CAS Exam 2,
Financial Mathematics. Fall 2009 Edition,
available at http://www.actexmadriver.com/
c 2009. Miguel A. Arcones. All rights reserved. Manual for SOA Exam FM/CAS Exam 2.
2/18
Chapter 2. Cashows. Section 2.1. Cashows.
Net present value of cash ows
Recall that a cashow is a series of payments made at dierent
times. We can represent a cashow in a table:
Investments C
1
C
2
C
n
Time (in periods) t
1
t
2
t
n
Assuming compound interest, the present value of a cashow at
time t is
V(t) =
n

j =1
C
j

t
j
t
=
n

j =1
C
j
(1 + i )
tt
j
,
where i is the eective interest per period and is the discount
factor per period. The previous equation is the equation of value.
Under the accumulation function a(), the equation of value is
V(t) =
n

j =1
C
j
a(t)
a(t
j
)
.
c 2009. Miguel A. Arcones. All rights reserved. Manual for SOA Exam FM/CAS Exam 2.
3/18
Chapter 2. Cashows. Section 2.1. Cashows.
Example 1
An investor can invest in a project which requires an investment of
$37400 at time 0. The investment pays $25000 at time 1 and
$15000 at time 2. The investors capital is currently earning an
eective annual rate of interest of 4.5%. Should the investor invest
in the project?
Solution: The net present value of the investment in the project is
25000(1.045)
1
+ 15000(1.045)
2
37400
=23923.445 + 13735.9493 37400 = 259.3943.
Yes, the investor should invest in the project.
c 2009. Miguel A. Arcones. All rights reserved. Manual for SOA Exam FM/CAS Exam 2.
4/18
Chapter 2. Cashows. Section 2.1. Cashows.
Example 1
An investor can invest in a project which requires an investment of
$37400 at time 0. The investment pays $25000 at time 1 and
$15000 at time 2. The investors capital is currently earning an
eective annual rate of interest of 4.5%. Should the investor invest
in the project?
Solution: The net present value of the investment in the project is
25000(1.045)
1
+ 15000(1.045)
2
37400
=23923.445 + 13735.9493 37400 = 259.3943.
Yes, the investor should invest in the project.
c 2009. Miguel A. Arcones. All rights reserved. Manual for SOA Exam FM/CAS Exam 2.
5/18
Chapter 2. Cashows. Section 2.1. Cashows.
A way to analyze the protability of an investment is to nd the
present value at time of the cashow derived from the investment.
The net present value of an investment is the present value of
the inows minus the present value of the outows.
Suppose that a company can select between taking two
investments. Which investment has the biggest net present value
depends on the used interest rate. To valuate investments,
companies use their cost of capital. The cost of capital of a
company is an estimation of how much the company has to pay for
every dollar it borrows. This cost of capital is found using the
whole capital components of the company.
c 2009. Miguel A. Arcones. All rights reserved. Manual for SOA Exam FM/CAS Exam 2.
6/18
Chapter 2. Cashows. Section 2.1. Cashows.
Example 2
A company has cost of capital of 7.5% as an annual eective rate
of interest. Two investment projects have the following forecasted
cash ows:
Project A $20,000 0 0 $25,000 $10,000
Project B $20,000 0 0 $10,000 $26,000
Time in years 0 1 2 3 4
(i) Find the prot made under each investment project.
(ii) Which project has the highest prot?
(iii) Compute the net present value for each project using the
companys cost of capital.
(iv) Which project has the highest net present value?
c 2009. Miguel A. Arcones. All rights reserved. Manual for SOA Exam FM/CAS Exam 2.
7/18
Chapter 2. Cashows. Section 2.1. Cashows.
Solution:(i) The prot for project A is
20000 + (25000) + (10000) = 15000.
The prot for project B is
20000 + (10000) + (26000) = 16000.
(ii) Project B has the highest prot.
(iii) The present value of project A is
20000 + (25000)(1.075)
3
+ (10000)(1.075)
4
= 7612.02.
The present value of project B is
20000 + (10000)(1.075)
3
+ (26000)(1.075)
4
= 7518.419.
(iv) Project A has the highest net present value.
c 2009. Miguel A. Arcones. All rights reserved. Manual for SOA Exam FM/CAS Exam 2.
8/18
Chapter 2. Cashows. Section 2.1. Cashows.
The calculator TIBAIIPlus has a cashow worksheet, which
allows to work with cashows when the deposit times are
nonnegative numbers. After entering the cashow in the
calculator, you can nd either the present value of the cashow
or the internal rate of return. The internal rate of return is the
eective periodic rate of interest. There are 3 keys to enter
dierent parts of this worksheet.

Pressing the key CF , you can enter the cash ow data.

Pressing the key NPV opens a worksheet with two variables


NPV and I . Using this worksheet, you can compute the net
present value.

Pressing the IRR you compute the internal rate of return.


c 2009. Miguel A. Arcones. All rights reserved. Manual for SOA Exam FM/CAS Exam 2.
9/18
Chapter 2. Cashows. Section 2.1. Cashows.
When you press the key CF , you can change the entries: CFo ,
C01 , F01 , C02 , F02 , . . . ., C24 , F24 . CFo is the initial
contribution. C01 is the amount of the rst round of
contributions. F01 is the (number of payments) frequency of the
rst round of contributions. C02 is the amount of the second
round of contributions. F02 is the frequency of the second round
of contributions. C03 , F03 , . . . , C24 , F24 are dened
similarly. To move from one entry to the next, use the arrows
and . To enter a number in one entry, type the number, and
then press ENTER . To clear the values in the worksheet, press
2nd CLR WORK while in the worksheet. To exit the worksheet
type 2nd QUIT . The cashow, which we have is:
Payments CFo C01 C02
Time 0 1 to F01 F01+1 to F01+F02
c 2009. Miguel A. Arcones. All rights reserved. Manual for SOA Exam FM/CAS Exam 2.
10/18
Chapter 2. Cashows. Section 2.1. Cashows.
After you have entered the date you can calculate either the net
present value or the internal rate of return.

To calculate the net present value, press the key NPV , enter
the periodic interest rate in the entry I , then go the entry
NPV using one of the arrows and . Finally press
CPT .

To calculate the internal rate of return, press the keys IRR


and CPT . If the equation does not have a solution, you get
Error 5. If the equation has several solutions, you get the
one with smallest absolute value.
c 2009. Miguel A. Arcones. All rights reserved. Manual for SOA Exam FM/CAS Exam 2.
11/18
Chapter 2. Cashows. Section 2.1. Cashows.
Example 3
Joel wishes to borrow a sum of money. In return, he is prepared to
pay as follows: $100 after 1 year, $200 after 2 years, $300 after 3
years, and $400 after 4 years. If i = 12%, how much can he
borrow?
Solution: The cashow is
contributions 100 200 300 400
Time 1 2 3 4
He can borrow:
(100)(1 + 0.12)
1
+ (200)(1 + 0.12)
2
+(300)(1 + 0.12)
3
+ (400)(1 + 0.12)
4
= 716.4657955
To do this problem using the CF worksheet. Press CF , and enter
CFo =0, C01 =100, F01 =1, C02 =200, F02 =1, C03 =300,
F03 =1, C04 =400, F04 =1, 2nd QUIT . Go to NPV , enter
I =12, and compute NPV and get 716.4657955.
c 2009. Miguel A. Arcones. All rights reserved. Manual for SOA Exam FM/CAS Exam 2.
12/18
Chapter 2. Cashows. Section 2.1. Cashows.
Example 3
Joel wishes to borrow a sum of money. In return, he is prepared to
pay as follows: $100 after 1 year, $200 after 2 years, $300 after 3
years, and $400 after 4 years. If i = 12%, how much can he
borrow?
Solution: The cashow is
contributions 100 200 300 400
Time 1 2 3 4
He can borrow:
(100)(1 + 0.12)
1
+ (200)(1 + 0.12)
2
+(300)(1 + 0.12)
3
+ (400)(1 + 0.12)
4
= 716.4657955
To do this problem using the CF worksheet. Press CF , and enter
CFo =0, C01 =100, F01 =1, C02 =200, F02 =1, C03 =300,
F03 =1, C04 =400, F04 =1, 2nd QUIT . Go to NPV , enter
I =12, and compute NPV and get 716.4657955.
c 2009. Miguel A. Arcones. All rights reserved. Manual for SOA Exam FM/CAS Exam 2.
13/18
Chapter 2. Cashows. Section 2.1. Cashows.
Example 4
A loan with an eective annual interest rate of 5.5% is to be
repaid with the following payments:
(i) 1000 at the end of the rst year.
(ii) 2000 at the end of the second year.
(iii) 5000 at the end of the third year.
Calculate the loaned amount at time 0.
Solution: The cashow of payments to the loan is
Payments 1000 2000 5000
Time 1 2 3
The loaned amount at time zero is the present value at time zero
of the cashow of payments, which is
(1000)(1.055)
1
+ (2000)(1.055)
2
+ (5000)(1.055)
3
= 7002.840451.
Press CF , and enter CFo =0, C01 =1000, F01 =1,
C02 =2000, F02 =1, C03 =5000, F03 =1, 2nd QUIT . Go to
NPV , enter I =5.5, and compute NPV and get 7002.840451.
c 2009. Miguel A. Arcones. All rights reserved. Manual for SOA Exam FM/CAS Exam 2.
14/18
Chapter 2. Cashows. Section 2.1. Cashows.
Example 4
A loan with an eective annual interest rate of 5.5% is to be
repaid with the following payments:
(i) 1000 at the end of the rst year.
(ii) 2000 at the end of the second year.
(iii) 5000 at the end of the third year.
Calculate the loaned amount at time 0.
Solution: The cashow of payments to the loan is
Payments 1000 2000 5000
Time 1 2 3
The loaned amount at time zero is the present value at time zero
of the cashow of payments, which is
(1000)(1.055)
1
+ (2000)(1.055)
2
+ (5000)(1.055)
3
= 7002.840451.
Press CF , and enter CFo =0, C01 =1000, F01 =1,
C02 =2000, F02 =1, C03 =5000, F03 =1, 2nd QUIT . Go to
NPV , enter I =5.5, and compute NPV and get 7002.840451.
c 2009. Miguel A. Arcones. All rights reserved. Manual for SOA Exam FM/CAS Exam 2.
15/18
Chapter 2. Cashows. Section 2.1. Cashows.
Example 5
Helen borrows $5000 from her credit card account at a nominal
annual interest rate of 20% per year convertible monthly. Two
months later, she pays $1000 back. Four months after the
payment she borrows $2000. How much does she owe one year
after the loan is taken out?
Solution: The cashow is
inow/outow 5000 0 1000 0 0 0 2000
Time (in months) 0 1 2 3 4 5 6
c 2009. Miguel A. Arcones. All rights reserved. Manual for SOA Exam FM/CAS Exam 2.
16/18
Chapter 2. Cashows. Section 2.1. Cashows.
Example 5
Helen borrows $5000 from her credit card account at a nominal
annual interest rate of 20% per year convertible monthly. Two
months later, she pays $1000 back. Four months after the
payment she borrows $2000. How much does she owe one year
after the loan is taken out?
Solution: The cashow is
inow/outow 5000 0 1000 0 0 0 2000
Time (in months) 0 1 2 3 4 5 6
c 2009. Miguel A. Arcones. All rights reserved. Manual for SOA Exam FM/CAS Exam 2.
17/18
Chapter 2. Cashows. Section 2.1. Cashows.
Example 5
Helen borrows $5000 from her credit card account at a nominal
annual interest rate of 20% per year convertible monthly. Two
months later, she pays $1000 back. Four months after the
payment she borrows $2000. How much does she owe one year
after the loan is taken out?
Solution: The cashow is
inow/outow 5000 0 1000 0 0 0 2000
Time (in months) 0 1 2 3 4 5 6
The equation of value at time 1 year is
5000

1 +
0.20
12

12
1000

1 +
0.20
12

10
+ 2000

1 +
0.20
12

6
=7125.737519
c 2009. Miguel A. Arcones. All rights reserved. Manual for SOA Exam FM/CAS Exam 2.
18/18
Chapter 2. Cashows. Section 2.1. Cashows.
Example 5
Helen borrows $5000 from her credit card account at a nominal
annual interest rate of 20% per year convertible monthly. Two
months later, she pays $1000 back. Four months after the
payment she borrows $2000. How much does she owe one year
after the loan is taken out?
Solution: The cashow is
inow/outow 5000 0 1000 0 0 0 2000
Time (in months) 0 1 2 3 4 5 6
Press CF , and enter CFo =5000, C01 =0, F01 =1,
C02 =1000, F02 =1, C03 =0, F03 =3, C04 =2000, F04 =1,
2nd QUIT . Go to NPV , enter I =1.66666(=20/12), and
compute NPV = 5843.69. This is the present value at time
0 of the loan. The future value of the loan at time 1 year is
5843.69(1.01666)
12
= 7125.737519.
c 2009. Miguel A. Arcones. All rights reserved. Manual for SOA Exam FM/CAS Exam 2.
1/7
Chapter 2. Cashows.
Manual for SOA Exam FM/CAS Exam 2.
Chapter 2. Cashows.
Section 2.2. Method of equated time.
c 2009. Miguel A. Arcones. All rights reserved.
Extract from:
Arcones Manual for the SOA Exam FM/CAS Exam 2,
Financial Mathematics. Fall 2009 Edition,
available at http://www.actexmadriver.com/
c 2009. Miguel A. Arcones. All rights reserved. Manual for SOA Exam FM/CAS Exam 2.
2/7
Chapter 2. Cashows. Section 2.2. Method of equated time.
Given the cashow
Investments C
1
C
2
C
n
Time t
1
t
2
t
n
we would like to nd a time

t such that a lump sum C =

n
j =1
C
j
invested at time

t is equivalent to the previous cashow. To nd

t,
we solve the equation,
C

t
=
n

j =1
C
j

t
j
, (1)
and get

t =
ln(

n
j =1
C
j

t
j
/C)
ln
(2)
c 2009. Miguel A. Arcones. All rights reserved. Manual for SOA Exam FM/CAS Exam 2.
3/7
Chapter 2. Cashows. Section 2.2. Method of equated time.
Method of equated time
The method of equated time consists on approximating

t by

t =

n
j =1
C
j
t
j
C
=

n
j =1
C
j
t
j

n
j =1
C
j
. (3)
This is the average time of all the times t
j
with the weight
C
j
P
n
k=1
C
k
at t
j
.
c 2009. Miguel A. Arcones. All rights reserved. Manual for SOA Exam FM/CAS Exam 2.
4/7
Chapter 2. Cashows. Section 2.2. Method of equated time.
The rst order Taylor expansion of
t
= (1 + i )
t
on i is 1 ti .
So, Equation (1) is approximately
C(1 ti ) =
n

j =1
C
j
(1 t
j
i ),
whose solution is

t =

n
j =1
C
j
t
j
C
=

n
j =1
C
j
t
j

n
j =1
C
j
.
If the interest were simple, the future value at time t
n
of the
considered cashow would be
C(1 + (t
n
t)i ) =
n

j =1
C
j
(1 + (t
n
t
j
)i ),
whose solution is t =
P
n
j =1
C
j
t
j
C
.
The approximation to t using the method of equating time is the
solution to the considered problem when the accumulation function
follows simple interest.
c 2009. Miguel A. Arcones. All rights reserved. Manual for SOA Exam FM/CAS Exam 2.
5/7
Chapter 2. Cashows. Section 2.2. Method of equated time.
Example 1
Payments of $300, $100 and $200 are due at the ends of years 1,
3, and 5, respectively. Assume an annual eective rate of interest
of 5% per year. (i) Find the point in time at which a payment of
$600 would be equivalent. (ii) Find the approximation to this point
using the method of equated time.
Solution: (i) The time

t solves the equation
600

t
= 300 + 100
3
+ 200
5
=285.71429 + 86.38376 + 156.70523 = 528.80328,
where = (1.05)
1
. Hence,

t =
ln(600/528.80328)
ln(1.05)
= 2.58891.
(ii) The equated time approximation to this point is

t =
(300)(1) + (100)(3) + (200)(5)
600
= 2.666667.
c 2009. Miguel A. Arcones. All rights reserved. Manual for SOA Exam FM/CAS Exam 2.
6/7
Chapter 2. Cashows. Section 2.2. Method of equated time.
Example 1
Payments of $300, $100 and $200 are due at the ends of years 1,
3, and 5, respectively. Assume an annual eective rate of interest
of 5% per year. (i) Find the point in time at which a payment of
$600 would be equivalent. (ii) Find the approximation to this point
using the method of equated time.
Solution: (i) The time

t solves the equation
600

t
= 300 + 100
3
+ 200
5
=285.71429 + 86.38376 + 156.70523 = 528.80328,
where = (1.05)
1
. Hence,

t =
ln(600/528.80328)
ln(1.05)
= 2.58891.
(ii) The equated time approximation to this point is

t =
(300)(1) + (100)(3) + (200)(5)
600
= 2.666667.
c 2009. Miguel A. Arcones. All rights reserved. Manual for SOA Exam FM/CAS Exam 2.
7/7
Chapter 2. Cashows. Section 2.2. Method of equated time.
Example 1
Payments of $300, $100 and $200 are due at the ends of years 1,
3, and 5, respectively. Assume an annual eective rate of interest
of 5% per year. (i) Find the point in time at which a payment of
$600 would be equivalent. (ii) Find the approximation to this point
using the method of equated time.
Solution: (i) The time

t solves the equation
600

t
= 300 + 100
3
+ 200
5
=285.71429 + 86.38376 + 156.70523 = 528.80328,
where = (1.05)
1
. Hence,

t =
ln(600/528.80328)
ln(1.05)
= 2.58891.
(ii) The equated time approximation to this point is

t =
(300)(1) + (100)(3) + (200)(5)
600
= 2.666667.
c 2009. Miguel A. Arcones. All rights reserved. Manual for SOA Exam FM/CAS Exam 2.
1/16
Chapter 2. Cashows.
Manual for SOA Exam FM/CAS Exam 2.
Chapter 2. Cashows.
Section 2.3. Yield rates.
c 2009. Miguel A. Arcones. All rights reserved.
Extract from:
Arcones Manual for the SOA Exam FM/CAS Exam 2,
Financial Mathematics. Fall 2009 Edition,
available at http://www.actexmadriver.com/
c 2009. Miguel A. Arcones. All rights reserved. Manual for SOA Exam FM/CAS Exam 2.
2/16
Chapter 2. Cashows. Section 2.3. Yield rates.
Yield of return
Suppose that the future value at time t of the cashow:
Investments V
0
C
1
C
2
C
n
Time 0 t
1
t
2
t
n
is FV. Then, the rate of return i of the investment satises the
equation,
FV = V
0

t
+
n

j =1
C
j

t
j
t
= V
0
(1 + i )
t
+
n

j =1
C
j
(1 + i )
tt
j
.
The rate of return i , i > 1, solving this equation is called the
yield rate of return or internal rate of return. This equation
can have either no solutions, or one solution, or several solutions.
We are interested in values of i with i > 1. If i < 1, then
(1 + i )
n
> 0 is n is even and (1 + i )
n
< 0 is n is odd. Values of i
with i 1 do not make any sense.
c 2009. Miguel A. Arcones. All rights reserved. Manual for SOA Exam FM/CAS Exam 2.
3/16
Chapter 2. Cashows. Section 2.3. Yield rates.
Example 1
Suppose that John invest $3000 in a business. One year later,
John sells half of this business to a partner for $6000. Two years
after the beginning, the business is in red and John has to pay
$4000 to close this business. What is the rate of interest Johns
got in his investment?
Solution: The cashow is:
Inow 3000 6000 4000
Time 0 1 2
Since John lost money, one expect that i is negative. However,
there is no solution. We have to solve
3000(1 + i )
2
+ 6000(1 + i ) 4000 = 0, or
3(1 + i )
2
6(1 + i ) + 4 = 0. Using the quadratic formula,
1 + i =
6

6
2
4 4 3
2
=
6

12
2
.
There is no solution.
c 2009. Miguel A. Arcones. All rights reserved. Manual for SOA Exam FM/CAS Exam 2.
4/16
Chapter 2. Cashows. Section 2.3. Yield rates.
Example 1
Suppose that John invest $3000 in a business. One year later,
John sells half of this business to a partner for $6000. Two years
after the beginning, the business is in red and John has to pay
$4000 to close this business. What is the rate of interest Johns
got in his investment?
Solution: The cashow is:
Inow 3000 6000 4000
Time 0 1 2
Since John lost money, one expect that i is negative. However,
there is no solution. We have to solve
3000(1 + i )
2
+ 6000(1 + i ) 4000 = 0, or
3(1 + i )
2
6(1 + i ) + 4 = 0. Using the quadratic formula,
1 + i =
6

6
2
4 4 3
2
=
6

12
2
.
There is no solution.
c 2009. Miguel A. Arcones. All rights reserved. Manual for SOA Exam FM/CAS Exam 2.
5/16
Chapter 2. Cashows. Section 2.3. Yield rates.
Example 2
What is the yield rate on a transaction in which a person makes
payments of $100 immediately and $100 at the end of two years,
in exchange for a payment of $201 at the end of one year? Find all
possible solutions.
Solution: The cashow is:
Inow 100 201 100
Time 0 1 2
We have to solve 100 + 201(1 + i )
1
100(1 + i )
2
= 0, or
100(1 + i )
2
201(1 + i )
1
+ 100 = 0. Using the quadratic formula,
1 + i =
201

201
2
4 100 100
200
=
201

201
200
.
The two solutions are i = 10.5124922% and i = 9.512492197%.
c 2009. Miguel A. Arcones. All rights reserved. Manual for SOA Exam FM/CAS Exam 2.
6/16
Chapter 2. Cashows. Section 2.3. Yield rates.
Example 2
What is the yield rate on a transaction in which a person makes
payments of $100 immediately and $100 at the end of two years,
in exchange for a payment of $201 at the end of one year? Find all
possible solutions.
Solution: The cashow is:
Inow 100 201 100
Time 0 1 2
We have to solve 100 + 201(1 + i )
1
100(1 + i )
2
= 0, or
100(1 + i )
2
201(1 + i )
1
+ 100 = 0. Using the quadratic formula,
1 + i =
201

201
2
4 100 100
200
=
201

201
200
.
The two solutions are i = 10.5124922% and i = 9.512492197%.
c 2009. Miguel A. Arcones. All rights reserved. Manual for SOA Exam FM/CAS Exam 2.
7/16
Chapter 2. Cashows. Section 2.3. Yield rates.
Since the internal rate of return could either do not exist or have
several solutions, it is not a good indication of the performance of
general investment strategy. However there exists a unique rate of
return i with i > 1 if either all outows happen before all the
inows, or all inows happen before all the outows.
c 2009. Miguel A. Arcones. All rights reserved. Manual for SOA Exam FM/CAS Exam 2.
8/16
Chapter 2. Cashows. Section 2.3. Yield rates.
Suppose that you an investment strategy consisting of investing
(positive) payments of C
1
, . . . , C
m
at times t
1
< < t
m
. At
times s
1
< < s
n
, we get respective (positive) returns
P
1
, . . . , P
n
, where s
1
> t
m
. The cashow is
Inows C
1
C
2
C
m
P
1
P
2
P
n
Time t
1
t
2
t
m
s
1
s
2
s
n
In this case, there exists a unique solution to the equation
n

k=1
P
k
(1 + i )
s
k

j =1
C
j
(1 + i )
t
j
= 0, i > 1.
Besides,


n
k=1
P
k
>

m
j =1
C
j
, then i > 0.


n
k=1
P
k
<

m
j =1
C
j
, then i < 0.


n
k=1
P
k
=

m
j =1
C
j
, then i = 0.
c 2009. Miguel A. Arcones. All rights reserved. Manual for SOA Exam FM/CAS Exam 2.
9/16
Chapter 2. Cashows. Section 2.3. Yield rates.
Example 3
As the budgeting ocer for Road Kill Motors Inc., you are
evaluating the purchase of a new car factory. The cost of the
factory is $4 million today. It will provide inows of $1.4 million at
the end of each of the rst three years. Find the eective rate of
interest which this investment will provide your company.
Solution: The cashow is
Contributions 4 1.4 1.4 1.4
Time 0 1 2 3
An equation of value for the cashow is
0 = 4 (1.4)(1 + i )
1
(1.4)(1 + i )
2
(1.4)(1 + i )
3
.
In the TIBAIIPlus calculator, we can nd i , by going to CF ,
and enter CFo =4, C01 =1.4, F01 =3, 2nd , QUIT . We can
move between dierent entries using the arrows and . Press
IRR CPT and get IRR = i = 2.47974%.
c 2009. Miguel A. Arcones. All rights reserved. Manual for SOA Exam FM/CAS Exam 2.
10/16
Chapter 2. Cashows. Section 2.3. Yield rates.
Example 3
As the budgeting ocer for Road Kill Motors Inc., you are
evaluating the purchase of a new car factory. The cost of the
factory is $4 million today. It will provide inows of $1.4 million at
the end of each of the rst three years. Find the eective rate of
interest which this investment will provide your company.
Solution: The cashow is
Contributions 4 1.4 1.4 1.4
Time 0 1 2 3
An equation of value for the cashow is
0 = 4 (1.4)(1 + i )
1
(1.4)(1 + i )
2
(1.4)(1 + i )
3
.
In the TIBAIIPlus calculator, we can nd i , by going to CF ,
and enter CFo =4, C01 =1.4, F01 =3, 2nd , QUIT . We can
move between dierent entries using the arrows and . Press
IRR CPT and get IRR = i = 2.47974%.
c 2009. Miguel A. Arcones. All rights reserved. Manual for SOA Exam FM/CAS Exam 2.
11/16
Chapter 2. Cashows. Section 2.3. Yield rates.
Example 4
Find the internal rate of return such that a payment of 400 at the
present, 200 at the end of one year, and 300 at the end of two
years, accumulate to 1000 at the end of 3 years.
Solution: The cashow is
Contributions 400 200 300 1000
Time 0 1 2 3
An equation of value for the cashow is
0 = 400(1 + i )
3
200(1 + i )
2
300(1 + i ) + 1000.
In the TIBAIIPlus calculator, we can nd i , by going to CF ,
and enter CFo =400, C01 =200, F01 =1, C02 =300,
F02 =1, C03 =1000, F03 =1, 2nd , QUIT . We can move
between dierent entries using the arrows and . Press IRR
CPT and get IRR = 5.0709%.
c 2009. Miguel A. Arcones. All rights reserved. Manual for SOA Exam FM/CAS Exam 2.
12/16
Chapter 2. Cashows. Section 2.3. Yield rates.
Example 4
Find the internal rate of return such that a payment of 400 at the
present, 200 at the end of one year, and 300 at the end of two
years, accumulate to 1000 at the end of 3 years.
Solution: The cashow is
Contributions 400 200 300 1000
Time 0 1 2 3
An equation of value for the cashow is
0 = 400(1 + i )
3
200(1 + i )
2
300(1 + i ) + 1000.
In the TIBAIIPlus calculator, we can nd i , by going to CF ,
and enter CFo =400, C01 =200, F01 =1, C02 =300,
F02 =1, C03 =1000, F03 =1, 2nd , QUIT . We can move
between dierent entries using the arrows and . Press IRR
CPT and get IRR = 5.0709%.
c 2009. Miguel A. Arcones. All rights reserved. Manual for SOA Exam FM/CAS Exam 2.
13/16
Chapter 2. Cashows. Section 2.3. Yield rates.
Example 5
An investment fund is established at time 0 with a deposit of
$5000. $6000 is added at the end of 6 months. The fund value,
including interest, is $11500 at the end of 1 year. Find the internal
rate of return as a annual nominal rate convertible semiannually.
Solution: The cashow is
Investments 5000 6000 11500
Time (in half years) 0 1 2
An equation of value for the cashow is
0 = (5000) + (6000)

1 +
i
(2)
2

1
(11500)

1 +
i
(2)
2

2
.
In the TIBAIIPlus calculator, press CF , and enter
CFo =5000, C01 =6000, F01 =1, C02 =11500, F02 =1.
Press IRR , CPT and get IRR =
i
(2)
2
= 3.095064303% and
i
(2)
= 6.190128606%. The sixmonth eective interest rate is
i
(2)
2
.
c 2009. Miguel A. Arcones. All rights reserved. Manual for SOA Exam FM/CAS Exam 2.
14/16
Chapter 2. Cashows. Section 2.3. Yield rates.
Example 5
An investment fund is established at time 0 with a deposit of
$5000. $6000 is added at the end of 6 months. The fund value,
including interest, is $11500 at the end of 1 year. Find the internal
rate of return as a annual nominal rate convertible semiannually.
Solution: The cashow is
Investments 5000 6000 11500
Time (in half years) 0 1 2
An equation of value for the cashow is
0 = (5000) + (6000)

1 +
i
(2)
2

1
(11500)

1 +
i
(2)
2

2
.
In the TIBAIIPlus calculator, press CF , and enter
CFo =5000, C01 =6000, F01 =1, C02 =11500, F02 =1.
Press IRR , CPT and get IRR =
i
(2)
2
= 3.095064303% and
i
(2)
= 6.190128606%. The sixmonth eective interest rate is
i
(2)
2
.
c 2009. Miguel A. Arcones. All rights reserved. Manual for SOA Exam FM/CAS Exam 2.
15/16
Chapter 2. Cashows. Section 2.3. Yield rates.
Example 6
An investment fund is established at time 0 with a deposit of
$5000. $6000 is added at the end of 6 months. The fund value,
including interest, is $11500 at the end of 1 year. Find the internal
rate of return as a annual nominal rate convertible monthly.
Solution: The cashow is
Investments 5000 6000 11500
Time (in months) 0 6 12
An equation of value for the cashow is
0 = (5000) + (6000)

1 +
i
(12)
12

6
(11500)

1 +
i
(12)
12

12
.
In the TIBAIIPlus calculator, press CF , and enter
CFo =5000, C01 =0, F01 =5, C02 =6000, F02 =1, C03 =0,
F03 =5, C04 =11500, F04 =1. Press IRR , CPT and get
IRR =
i
(12)
12
= 0.509314804% and i
(12)
= 6.111777648%.
c 2009. Miguel A. Arcones. All rights reserved. Manual for SOA Exam FM/CAS Exam 2.
16/16
Chapter 2. Cashows. Section 2.3. Yield rates.
Example 6
An investment fund is established at time 0 with a deposit of
$5000. $6000 is added at the end of 6 months. The fund value,
including interest, is $11500 at the end of 1 year. Find the internal
rate of return as a annual nominal rate convertible monthly.
Solution: The cashow is
Investments 5000 6000 11500
Time (in months) 0 6 12
An equation of value for the cashow is
0 = (5000) + (6000)

1 +
i
(12)
12

6
(11500)

1 +
i
(12)
12

12
.
In the TIBAIIPlus calculator, press CF , and enter
CFo =5000, C01 =0, F01 =5, C02 =6000, F02 =1, C03 =0,
F03 =5, C04 =11500, F04 =1. Press IRR , CPT and get
IRR =
i
(12)
12
= 0.509314804% and i
(12)
= 6.111777648%.
c 2009. Miguel A. Arcones. All rights reserved. Manual for SOA Exam FM/CAS Exam 2.
1/9
Chapter 2. Cashows.
Manual for SOA Exam FM/CAS Exam 2.
Chapter 2. Cashows.
Section 2.4. Dollarweighted and timeweighted rates of
return.
c 2009. Miguel A. Arcones. All rights reserved.
Extract from:
Arcones Manual for the SOA Exam FM/CAS Exam 2,
Financial Mathematics. Fall 2009 Edition,
available at http://www.actexmadriver.com/
c 2009. Miguel A. Arcones. All rights reserved. Manual for SOA Exam FM/CAS Exam 2.
2/9
Chapter 2. Cashows. Section 2.4. Dollarweighted and timeweighted rates of return.
Dollarweighted and timeweighted rates of return
If the cashow
Investments V
0
C
1
C
2
C
n
Time 0 t
1
t
2
t
n
has future value FV at time t, then its equation of value is
FV = V
0
(1 + i )
t
+
n

j =1
C
j
(1 + i )
tt
j
.
Using the rst order Taylor expansion 1 + it of (1 + i )
t
, the
previous equation of value is approximately,
FV = V
0
(1 + it) +
n

j =1
C
j
(1 + i (t t
j
)). (1)
Observe Equation (1) represents the future value of the cashow
when simple interest is used.
c 2009. Miguel A. Arcones. All rights reserved. Manual for SOA Exam FM/CAS Exam 2.
3/9
Chapter 2. Cashows. Section 2.4. Dollarweighted and timeweighted rates of return.
The interest rate i which solves
FFV = V
0
(1 + it) +
n

j =1
C
j
(1 + i (t t
j
)).
is called the dollar weighted rate of return, which is
i =
FV V
0

n
j =1
C
j
V
0
t +

n
j =1
(t t
j
)C
j
. (2)
V
0
can be interpreted as the initial balance in an account. C
j
is the
deposit at time t
j
. FV is the nal balance in the account at time
t. Hence, I = FV V
0

n
j =1
C
j
is the interest earned in the
account. V
0
t +

n
j =1
(t t
j
)C
j
is the sum of the deposits
multiplied by the time which the deposits are in the account.
c 2009. Miguel A. Arcones. All rights reserved. Manual for SOA Exam FM/CAS Exam 2.
4/9
Chapter 2. Cashows. Section 2.4. Dollarweighted and timeweighted rates of return.
Example 1
On January 1, 2000, the balance in account is $25200. On April 1,
2000, $500 are deposited in this account and on July 1, 2001, a
withdraw of $1000 is made. The balance in the account on
October 1, 2001 is $25900. What is the annual rate of interest in
this account according with the dollarweighted method?
Solution: The cashow
Investments 25200 500 1000
Time in years 0 3/12 18/12
has a FV at time 21/12 of $25900. So, the annual dollarweighted
rate of interest is
25900 25200 500 + 1000
(25200)(21/12) + (500)(18/12) 1000(3/12)
=
1200
44100 + 750 250
=
1200
44600
= 2.6906%
c 2009. Miguel A. Arcones. All rights reserved. Manual for SOA Exam FM/CAS Exam 2.
5/9
Chapter 2. Cashows. Section 2.4. Dollarweighted and timeweighted rates of return.
Example 1
On January 1, 2000, the balance in account is $25200. On April 1,
2000, $500 are deposited in this account and on July 1, 2001, a
withdraw of $1000 is made. The balance in the account on
October 1, 2001 is $25900. What is the annual rate of interest in
this account according with the dollarweighted method?
Solution: The cashow
Investments 25200 500 1000
Time in years 0 3/12 18/12
has a FV at time 21/12 of $25900. So, the annual dollarweighted
rate of interest is
25900 25200 500 + 1000
(25200)(21/12) + (500)(18/12) 1000(3/12)
=
1200
44100 + 750 250
=
1200
44600
= 2.6906%
c 2009. Miguel A. Arcones. All rights reserved. Manual for SOA Exam FM/CAS Exam 2.
6/9
Chapter 2. Cashows. Section 2.4. Dollarweighted and timeweighted rates of return.
Suppose that we make investments in a fund over time and we
know the outstanding balance before each deposit or withdrawal
occurs. Let B
0
be the initial balance in the fund. Let B
j
be the
balance in the fund immediately before time t
j
, for 1 j n. Let
W
j
be the amount of each deposit or withdrawal at time t
j
.
W
j
> 0 for deposits and W
j
< 0 for withdrawal. In a table, we
have:
Time 0 t
1
t
2
t
n1
t
n
Balance before
depos./withdr.
B
1
B
2
B
n1
B
n
Depos./Withdr. W
1
W
2
W
n1

Balance after
depos./withdr.
B
0
B
1
+ W
1
B
2
+ W
2
B
n1
+ W
n1

The timeweighted annual rate of return i is the solution of
(1 + i )
t
n
=
B
1
B
0

B
2
B
1
+ W
1

B
3
B
2
+ W
2

B
n
B
n1
+ W
n1
.
c 2009. Miguel A. Arcones. All rights reserved. Manual for SOA Exam FM/CAS Exam 2.
7/9
Chapter 2. Cashows. Section 2.4. Dollarweighted and timeweighted rates of return.
In the j th period of time, the balance of the fund has changed
from B
j 1
+ W
j 1
to B
j
. So, the interest factor rate in the j th
period of time is 1 + i
j
=
B
j
B
j 1
+W
j 1
, where i
j
is the eective rate
of return in the period [t
j 1
, t
j
]. Observe that if the investment
followed an annual eective rate of interest of i , the interest factor
from time t
j 1
to time t
j
would be (1 + i )
t
j
t
j 1
. Assuming that
1 + i
j
= (1 + i )
t
j
t
j 1
, we get that
(1+i
1
)(1+i
2
) (1+i
n
) = (1+i )
t
1
(1+i )
t
2
t
1
(1+i )
t
n
t
n1
= (1+i )
t
n
.
The timeweighted annual rate of return i is the solution of
(1 + i )
t
n
=
B
1
B
0

B
2
B
1
+ W
1

B
3
B
2
+ W
2

B
n
B
n1
+ W
n1
.
Usually, the account balance does not follow compound interest
with a xed eective rate i . Usually, 1 + i
j
and (1 + i )
t
j
t
j 1
may
be dierent.
c 2009. Miguel A. Arcones. All rights reserved. Manual for SOA Exam FM/CAS Exam 2.
8/9
Chapter 2. Cashows. Section 2.4. Dollarweighted and timeweighted rates of return.
Example 2
For an investment account, you are given:
Date 11/1/04 3/1/05 8/1/05 2/1/06 4/1/06
Account Balance
(before deposit
or withdrawal)
14,516 14,547 18,351 16,969 18,542
Deposit 3,000 2500
Withdrawal 2,000
Calculate the annual eective yield rate by the time weighted
method.
Solution: The annual eective yield rate i by the time weighted
method satises
(1 + i )
17/12
=
B
1
B
0

B
2
B
1
+W
1

B
3
B
2
+W
2
s
B
n
B
n1
+W
n1
=
14547
14516
18351
14547+3000
16969
183512000
18542
16969+2500
= 1.035877
and i = 2.5193371%.
c 2009. Miguel A. Arcones. All rights reserved. Manual for SOA Exam FM/CAS Exam 2.
9/9
Chapter 2. Cashows. Section 2.4. Dollarweighted and timeweighted rates of return.
Example 2
For an investment account, you are given:
Date 11/1/04 3/1/05 8/1/05 2/1/06 4/1/06
Account Balance
(before deposit
or withdrawal)
14,516 14,547 18,351 16,969 18,542
Deposit 3,000 2500
Withdrawal 2,000
Calculate the annual eective yield rate by the time weighted
method.
Solution: The annual eective yield rate i by the time weighted
method satises
(1 + i )
17/12
=
B
1
B
0

B
2
B
1
+W
1

B
3
B
2
+W
2
s
B
n
B
n1
+W
n1
=
14547
14516
18351
14547+3000
16969
183512000
18542
16969+2500
= 1.035877
and i = 2.5193371%.
c 2009. Miguel A. Arcones. All rights reserved. Manual for SOA Exam FM/CAS Exam 2.
1/8
Chapter 2. Cashows.
Manual for SOA Exam FM/CAS Exam 2.
Chapter 2. Cashows.
Section 2.5. Investment and portfolio methods.
c 2009. Miguel A. Arcones. All rights reserved.
Extract from:
Arcones Manual for the SOA Exam FM/CAS Exam 2,
Financial Mathematics. Fall 2009 Edition,
available at http://www.actexmadriver.com/
c 2009. Miguel A. Arcones. All rights reserved. Manual for SOA Exam FM/CAS Exam 2.
2/8
Chapter 2. Cashows. Section 2.5. Investment and portfolio methods.
Investment and portfolio methods
Suppose that an investment fund pools money from several
identities (individuals or corporations) and makes investments on
behalf of them. Then, the fund faces the question: how to allocate
the returns between the dierent identities? There are two main
ways to allocate interest to the various accounts: the portfolio
method and the investment year method.
The portfolio method is an accounting method that credits all
funds one specied current rate of interest, regardless of when the
money was placed in the account. Usually this rate of interest
changes from year to year. Let i
y
denote the annual interest rate
credited in year y. If x is invested at the beginning of the year y,
then the balance in the account in the year y + t is
x
t1

j =0
(1 + i
y+j
) = x(1 + i
y
)(1 + i
y+1
) (1 + i
y+t1
).
c 2009. Miguel A. Arcones. All rights reserved. Manual for SOA Exam FM/CAS Exam 2.
3/8
Chapter 2. Cashows. Section 2.5. Investment and portfolio methods.
Example 1
Suppose that an investment account credits investors using the
portfolio method with the annual rates in the following table:
Calendar year Portfolio
of portfolio rate rates
y i
y
1999 4.50%
2000 5.50%
2001 4.00%
2002 6.50%
Suppose that 100 was invested on January 1, 1999.
(i) Find the balance on January 1, 2000.
(ii) The balance on January 1, 2001.
(iii) The balance on July 1, 2001.
c 2009. Miguel A. Arcones. All rights reserved. Manual for SOA Exam FM/CAS Exam 2.
4/8
Chapter 2. Cashows. Section 2.5. Investment and portfolio methods.
Solution: (i) The balance on January 1, 2000, is
(100)(1.045) = 104.5.
(ii) The balance on January 1, 2001, is
(100)(1.045)(1.055) = 110.2475.
(iii) The balance on July 1, 2001, is
(100)(1.045)(1.055)(1.04)
1/2
= 112.4308.
c 2009. Miguel A. Arcones. All rights reserved. Manual for SOA Exam FM/CAS Exam 2.
5/8
Chapter 2. Cashows. Section 2.5. Investment and portfolio methods.
The investment year method is an accounting method in which an
investment fund keeps records of the interest rates it earns annually
on funds assigned each year to accounts within the general
account. The investment year method is also called the new
money method. We will assume that accounts are made according
with the year at which the money was invested. For example,
suppose that an investment account credits investors according
with the investment year method using the following table:
Calendar year of Investment year rates
original investment
y i
y
1
i
y
2
i
y
3
i
y
4
i
y
5
1999 4.25% 4.35% 4.47% 4.57% 4.70%
2000 4.56% 4.73% 4.75% 4.98% 4.04%
2001 4.05% 4.04% 4.13% 4.17% 4.24%
2002 4.45% 4.15% 4.23% 4.36% 4.44%
2003 4.25% 4.35% 4.55% 9.55% 5.65%
c 2009. Miguel A. Arcones. All rights reserved. Manual for SOA Exam FM/CAS Exam 2.
6/8
Chapter 2. Cashows. Section 2.5. Investment and portfolio methods.
Calendar year of Investment year rates
original investment
y i
y
1
i
y
2
i
y
3
i
y
4
i
y
5
1999 4.25% 4.35% 4.47% 4.57% 4.70%
2000 4.56% 4.73% 4.75% 4.98% 4.04%
2001 4.05% 4.04% 4.13% 4.17% 4.24%
2002 4.45% 4.15% 4.23% 4.36% 4.44%
2003 4.25% 4.35% 4.55% 9.55% 5.65%
This means that money invested during 2000 earns an eective
rate of interest of 4.56% during 2000, it earns an eective rate of
interest of 4.73% during 2001, and so on. For example, if an
account is open with an investment of x invested on January 1,
2000, then:
the balance on January 1, 2001 is (100)(1.0456);
the balance on January 1, 2002 is (100)(1.0456)(1.0473); and so
on.
c 2009. Miguel A. Arcones. All rights reserved. Manual for SOA Exam FM/CAS Exam 2.
7/8
Chapter 2. Cashows. Section 2.5. Investment and portfolio methods.
Example 2
An investment fund applies the investment year method for the
rst two years, after which a portfolio rate is used. The following
table of interest rates is used:
Calendar year Investment Portfolio
of original investment year rates rates
y i
y
1
i
y
2
i
y+2
2000 5.25% 5.25% 5.40%
2001 5.35% 5.35% 5.65%
2002 5.45% 5.45% 5.10%
2003 5.45% 5.45% 5.34%
2004 5.50% 5.35% 5.55%
2005 5.50% 5.55% 5.65%
c 2009. Miguel A. Arcones. All rights reserved. Manual for SOA Exam FM/CAS Exam 2.
8/8
Chapter 2. Cashows. Section 2.5. Investment and portfolio methods.
(i) Ashley invests $1000 into the fund on January 1, 2000. The
investment year method is applicable for the rst two years, after
which a portfolio rate is used. Calculate Ashleys account
accumulation on January 1, 2006.
(ii) Elizabeth invests $1000 into the fund on January 1, 2000. But,
she redeemed her investment from the fund at the end of every
year and reinvested the money at the new money rate. Calculate
Elizabeths accumulation account on January 1, 2006.
Solution: (i) Ashleys account accumulation on January 1, 2006 is
(1000)(1.0525)(1.0525)(1.0540)(1.0565)(1.0510)(1.0534) = 1365.684.
(ii) Elizabeths investment value on January 1, 2006 is
(1000)(1.0525)(1.0535)(1.0545)(1.0545)(1.055)(1.050) = 1365.814.
c 2009. Miguel A. Arcones. All rights reserved. Manual for SOA Exam FM/CAS Exam 2.
1/10
Chapter 2. Cashows.
Manual for SOA Exam FM/CAS Exam 2.
Chapter 2. Cashows.
Section 2.6. Continuous payments.
c 2009. Miguel A. Arcones. All rights reserved.
Extract from:
Arcones Manual for the SOA Exam FM/CAS Exam 2,
Financial Mathematics. Fall 2009 Edition,
available at http://www.actexmadriver.com/
c 2009. Miguel A. Arcones. All rights reserved. Manual for SOA Exam FM/CAS Exam 2.
2/10
Chapter 2. Cashows. Section 2.6. Continuous payments.
Continuous payments
Suppose that the payments are made very often. Then by
approximation, instead of a sum, we have an integral. It is like the
payments are made continuously. Let V(t) be the outstanding
fund balance at time t of the cashow. Assume that contributions
are made continuously at an instantaneous rate C(t), then the
equation of value is
V(t) = V(0)(1 + i )
t
+

t
0
C(s)(1 + i )
ts
ds. (1)
c 2009. Miguel A. Arcones. All rights reserved. Manual for SOA Exam FM/CAS Exam 2.
3/10
Chapter 2. Cashows. Section 2.6. Continuous payments.
(1) appears as the limit of the equation of value for the cashow:
Inow V(0) C(t
1
)(t
1
0) C(t
2
)(t
2
t
1
) C(t
n
)(t
n
t
n1
)
Time 0 t
1
t
2
t
n
as max
1j n
(t
j
t
j 1
) 0, where
0 = t
0
< t
1
< t
2
< < t
m
= t. The equation of value at time t
for this cashow is
V(t) = V(0)(1 + i )
t
+
n

j =1
C(t
j
)(t
j
t
j 1
)(1 + i )
tt
j
,
which tends to
V(t) = V(0)(1 + i )
t
+

t
0
C(s)(1 + i )
ts
ds
as max
1j n
(t
j
t
j 1
) 0.
Recall that the Riemann integral of a function f is dened as

t
0
f (s) ds = lim
max
1j n
(t
j
t
j 1
)0
n

j =1
f (t
j
)(t
j
t
j 1
).
c 2009. Miguel A. Arcones. All rights reserved. Manual for SOA Exam FM/CAS Exam 2.
4/10
Chapter 2. Cashows. Section 2.6. Continuous payments.
Example 1
A continuousyear annuity pays a constant rate 1 at time t where
0 t n. Interest is compounded with an annual rate of interest
of i .
(i) Find the present value of the annuity at time 0.
(ii) Find the future value of the annuity at time n.
Solution: (i) The present value of this continuous annuity is
PV =

n
0
(1 + i )
t
dt =

n
0
e
t ln(1+i )
dt =
e
t ln(1+i )
ln(1 + i )

n
0
=
1
ln(1 + i )

e
n ln(1+i )
ln(1 + i )
=
1 (1 + i )
n
ln(1 + i )
.
(ii) The future value of the continuous annuity at time n is
FV =

n
0
(1 + i )
nt
dt = (1 + i )
n
PV =
(1 + i )
n
1
ln(1 + i )
.
c 2009. Miguel A. Arcones. All rights reserved. Manual for SOA Exam FM/CAS Exam 2.
5/10
Chapter 2. Cashows. Section 2.6. Continuous payments.
Example 1
A continuousyear annuity pays a constant rate 1 at time t where
0 t n. Interest is compounded with an annual rate of interest
of i .
(i) Find the present value of the annuity at time 0.
(ii) Find the future value of the annuity at time n.
Solution: (i) The present value of this continuous annuity is
PV =

n
0
(1 + i )
t
dt =

n
0
e
t ln(1+i )
dt =
e
t ln(1+i )
ln(1 + i )

n
0
=
1
ln(1 + i )

e
n ln(1+i )
ln(1 + i )
=
1 (1 + i )
n
ln(1 + i )
.
(ii) The future value of the continuous annuity at time n is
FV =

n
0
(1 + i )
nt
dt = (1 + i )
n
PV =
(1 + i )
n
1
ln(1 + i )
.
c 2009. Miguel A. Arcones. All rights reserved. Manual for SOA Exam FM/CAS Exam 2.
6/10
Chapter 2. Cashows. Section 2.6. Continuous payments.
If instead of compound interest, the time value of money follows
the accumulation function a(t), then the future value at time t of
an initial outstanding balance V(0) and continuous payments
C(s), in the interval 0 s t is
V(t) = V(0)a(t) +

t
0
C(s)
a(t)
a(s)
ds.
c 2009. Miguel A. Arcones. All rights reserved. Manual for SOA Exam FM/CAS Exam 2.
7/10
Chapter 2. Cashows. Section 2.6. Continuous payments.
Example 2
The force of interest at time t is
t
=
t
3
10
. Find the present value of
a fouryear continuous annuity which has a rate of payments at
time t of 5t
3
.
Solution: The accumulation function is
a(t) = exp

t
0

s
ds

= exp

t
0
s
3
10
ds

= e
t
4
40
.
The present value of the four-year continuous annuity is

4
0
C(s)
a(s)
ds =

4
0
5s
3
e

s
4
40
ds = 50e
s
4
40

4
0
= 50 50e
6.4
=49.91692.
c 2009. Miguel A. Arcones. All rights reserved. Manual for SOA Exam FM/CAS Exam 2.
8/10
Chapter 2. Cashows. Section 2.6. Continuous payments.
Example 2
The force of interest at time t is
t
=
t
3
10
. Find the present value of
a fouryear continuous annuity which has a rate of payments at
time t of 5t
3
.
Solution: The accumulation function is
a(t) = exp

t
0

s
ds

= exp

t
0
s
3
10
ds

= e
t
4
40
.
The present value of the four-year continuous annuity is

4
0
C(s)
a(s)
ds =

4
0
5s
3
e

s
4
40
ds = 50e
s
4
40

4
0
= 50 50e
6.4
=49.91692.
c 2009. Miguel A. Arcones. All rights reserved. Manual for SOA Exam FM/CAS Exam 2.
9/10
Chapter 2. Cashows. Section 2.6. Continuous payments.
Example 2
The force of interest at time t is
t
=
t
3
10
. Find the present value of
a fouryear continuous annuity which has a rate of payments at
time t of 5t
3
.
Solution: The accumulation function is
a(t) = exp

t
0

s
ds

= exp

t
0
s
3
10
ds

= e
t
4
40
.
The present value of the four-year continuous annuity is

4
0
C(s)
a(s)
ds =

4
0
5s
3
e

s
4
40
ds = 50e
s
4
40

4
0
= 50 50e
6.4
=49.91692.
c 2009. Miguel A. Arcones. All rights reserved. Manual for SOA Exam FM/CAS Exam 2.
10/10
Chapter 2. Cashows. Section 2.6. Continuous payments.
Example 2
The force of interest at time t is
t
=
t
3
10
. Find the present value of
a fouryear continuous annuity which has a rate of payments at
time t of 5t
3
.
Solution: The accumulation function is
a(t) = exp

t
0

s
ds

= exp

t
0
s
3
10
ds

= e
t
4
40
.
The present value of the four-year continuous annuity is

4
0
C(s)
a(s)
ds =

4
0
5s
3
e

s
4
40
ds = 50e
s
4
40

4
0
= 50 50e
6.4
=49.91692.
c 2009. Miguel A. Arcones. All rights reserved. Manual for SOA Exam FM/CAS Exam 2.
1/??
Chapter 3. Annuities.
Manual for SOA Exam FM/CAS Exam 2.
Chapter 3. Annuities.
Section 3.1. Geometric series.
c 2009. Miguel A. Arcones. All rights reserved.
Extract from:
Arcones Manual for the SOA Exam FM/CAS Exam 2,
Financial Mathematics. Fall 2009 Edition,
available at http://www.actexmadriver.com/
c 2009. Miguel A. Arcones. All rights reserved. Manual for SOA Exam FM/CAS Exam 2.
2/??
Chapter 3. Annuities. Section 3.1. Geometric series.
We use the summation notation

n
i =m
x
i
to mean
x
m
+ x
m+1
+ + x
n1
+ x
n
.
Usually arithmetic rules hold. In particular:


n
i =m
(x
i
+ y
i
) =

n
i =m
x
i
+

n
i =m
y
i


n
i =m
ax
i
= a

n
i =m
x
i
.

If m k n,
n

i =m
x
i
=
k

i =m
x
i
+
n

i =k+1
x
i
.


n
i =m
1 = n m + 1.
c 2009. Miguel A. Arcones. All rights reserved. Manual for SOA Exam FM/CAS Exam 2.
3/??
Chapter 3. Annuities. Section 3.1. Geometric series.
We use the summation notation

n
i =m
x
i
to mean
x
m
+ x
m+1
+ + x
n1
+ x
n
.
Usually arithmetic rules hold. In particular:


n
i =m
(x
i
+ y
i
) =

n
i =m
x
i
+

n
i =m
y
i


n
i =m
ax
i
= a

n
i =m
x
i
.

If m k n,
n

i =m
x
i
=
k

i =m
x
i
+
n

i =k+1
x
i
.


n
i =m
1 = n m + 1.
c 2009. Miguel A. Arcones. All rights reserved. Manual for SOA Exam FM/CAS Exam 2.
4/??
Chapter 3. Annuities. Section 3.1. Geometric series.
We use the summation notation

n
i =m
x
i
to mean
x
m
+ x
m+1
+ + x
n1
+ x
n
.
Usually arithmetic rules hold. In particular:


n
i =m
(x
i
+ y
i
) =

n
i =m
x
i
+

n
i =m
y
i


n
i =m
ax
i
= a

n
i =m
x
i
.

If m k n,
n

i =m
x
i
=
k

i =m
x
i
+
n

i =k+1
x
i
.


n
i =m
1 = n m + 1.
c 2009. Miguel A. Arcones. All rights reserved. Manual for SOA Exam FM/CAS Exam 2.
5/??
Chapter 3. Annuities. Section 3.1. Geometric series.
We use the summation notation

n
i =m
x
i
to mean
x
m
+ x
m+1
+ + x
n1
+ x
n
.
Usually arithmetic rules hold. In particular:


n
i =m
(x
i
+ y
i
) =

n
i =m
x
i
+

n
i =m
y
i


n
i =m
ax
i
= a

n
i =m
x
i
.

If m k n,
n

i =m
x
i
=
k

i =m
x
i
+
n

i =k+1
x
i
.


n
i =m
1 = n m + 1.
c 2009. Miguel A. Arcones. All rights reserved. Manual for SOA Exam FM/CAS Exam 2.
6/??
Chapter 3. Annuities. Section 3.1. Geometric series.
Denition 1
The sequence of real numbers

n=0
= {a + nd}

n=0
is called an
arithmetic sequence.
The sequence {x
n
}

n=0
= {a +nd}

n=0
satises that for each n 1,
x
n1
+ d = x
n
. Notice that
x
n1
+ d = a + (n 1)d + d = a + nd = x
n
.
Theorem 1
If a sequence {x
n
}

n=0
of real numbers satises x
n
= x
n1
+ d, for
each n 1, then x
n
= x
0
+ nd for each n 1.
Proof.
The proof is by induction on n. The case n = 0 is obvious. Assume
that the case n holds. Then,
x
n+1
= x
n
+ d = x
0
+ nd + d = x
0
+ (n + 1)d.
c 2009. Miguel A. Arcones. All rights reserved. Manual for SOA Exam FM/CAS Exam 2.
7/??
Chapter 3. Annuities. Section 3.1. Geometric series.
Denition 1
The sequence of real numbers

n=0
= {a + nd}

n=0
is called an
arithmetic sequence.
The sequence {x
n
}

n=0
= {a +nd}

n=0
satises that for each n 1,
x
n1
+ d = x
n
. Notice that
x
n1
+ d = a + (n 1)d + d = a + nd = x
n
.
Theorem 1
If a sequence {x
n
}

n=0
of real numbers satises x
n
= x
n1
+ d, for
each n 1, then x
n
= x
0
+ nd for each n 1.
Proof.
The proof is by induction on n. The case n = 0 is obvious. Assume
that the case n holds. Then,
x
n+1
= x
n
+ d = x
0
+ nd + d = x
0
+ (n + 1)d.
c 2009. Miguel A. Arcones. All rights reserved. Manual for SOA Exam FM/CAS Exam 2.
8/??
Chapter 3. Annuities. Section 3.1. Geometric series.
Denition 1
The sequence of real numbers

n=0
= {a + nd}

n=0
is called an
arithmetic sequence.
The sequence {x
n
}

n=0
= {a +nd}

n=0
satises that for each n 1,
x
n1
+ d = x
n
. Notice that
x
n1
+ d = a + (n 1)d + d = a + nd = x
n
.
Theorem 1
If a sequence {x
n
}

n=0
of real numbers satises x
n
= x
n1
+ d, for
each n 1, then x
n
= x
0
+ nd for each n 1.
Proof.
The proof is by induction on n. The case n = 0 is obvious. Assume
that the case n holds. Then,
x
n+1
= x
n
+ d = x
0
+ nd + d = x
0
+ (n + 1)d.
c 2009. Miguel A. Arcones. All rights reserved. Manual for SOA Exam FM/CAS Exam 2.
9/??
Chapter 3. Annuities. Section 3.1. Geometric series.
Denition 1
The sequence of real numbers

n=0
= {a + nd}

n=0
is called an
arithmetic sequence.
The sequence {x
n
}

n=0
= {a +nd}

n=0
satises that for each n 1,
x
n1
+ d = x
n
. Notice that
x
n1
+ d = a + (n 1)d + d = a + nd = x
n
.
Theorem 1
If a sequence {x
n
}

n=0
of real numbers satises x
n
= x
n1
+ d, for
each n 1, then x
n
= x
0
+ nd for each n 1.
Proof.
The proof is by induction on n. The case n = 0 is obvious. Assume
that the case n holds. Then,
x
n+1
= x
n
+ d = x
0
+ nd + d = x
0
+ (n + 1)d.
c 2009. Miguel A. Arcones. All rights reserved. Manual for SOA Exam FM/CAS Exam 2.
10/??
Chapter 3. Annuities. Section 3.1. Geometric series.
Theorem 2

n
j =1
j =
n(n+1)
2
.
Proof.
2
n

j =1
j = (1 + 2 + + (n 1) + n) + (1 + 2 + + (n 1) + n)
=(1 + 2 + + (n 1) + n) + (n + (n 1) + + 2 + 1)
=(1 + n) + (2 + n 1) + (3 + n 2) + (n + 1)
=(n + 1) + (n + 1) + (n + 1) + (n + 1) = n(n + 1).
Previous theorem can be proved by induction.
Note that in the summation

n
j =1
j , there are n numbers and the
average of these numbers is
n+1
2
. Hence,

n
j =1
j =
n(n+1)
2
.
c 2009. Miguel A. Arcones. All rights reserved. Manual for SOA Exam FM/CAS Exam 2.
11/??
Chapter 3. Annuities. Section 3.1. Geometric series.
Theorem 2

n
j =1
j =
n(n+1)
2
.
Proof.
2
n

j =1
j = (1 + 2 + + (n 1) + n) + (1 + 2 + + (n 1) + n)
=(1 + 2 + + (n 1) + n) + (n + (n 1) + + 2 + 1)
=(1 + n) + (2 + n 1) + (3 + n 2) + (n + 1)
=(n + 1) + (n + 1) + (n + 1) + (n + 1) = n(n + 1).
Previous theorem can be proved by induction.
Note that in the summation

n
j =1
j , there are n numbers and the
average of these numbers is
n+1
2
. Hence,

n
j =1
j =
n(n+1)
2
.
c 2009. Miguel A. Arcones. All rights reserved. Manual for SOA Exam FM/CAS Exam 2.
12/??
Chapter 3. Annuities. Section 3.1. Geometric series.
Theorem 2

n
j =1
j =
n(n+1)
2
.
Proof.
2
n

j =1
j = (1 + 2 + + (n 1) + n) + (1 + 2 + + (n 1) + n)
=(1 + 2 + + (n 1) + n) + (n + (n 1) + + 2 + 1)
=(1 + n) + (2 + n 1) + (3 + n 2) + (n + 1)
=(n + 1) + (n + 1) + (n + 1) + (n + 1) = n(n + 1).
Previous theorem can be proved by induction.
Note that in the summation

n
j =1
j , there are n numbers and the
average of these numbers is
n+1
2
. Hence,

n
j =1
j =
n(n+1)
2
.
c 2009. Miguel A. Arcones. All rights reserved. Manual for SOA Exam FM/CAS Exam 2.
13/??
Chapter 3. Annuities. Section 3.1. Geometric series.
Theorem 2

n
j =1
j =
n(n+1)
2
.
Proof.
2
n

j =1
j = (1 + 2 + + (n 1) + n) + (1 + 2 + + (n 1) + n)
=(1 + 2 + + (n 1) + n) + (n + (n 1) + + 2 + 1)
=(1 + n) + (2 + n 1) + (3 + n 2) + (n + 1)
=(n + 1) + (n + 1) + (n + 1) + (n + 1) = n(n + 1).
Previous theorem can be proved by induction.
Note that in the summation

n
j =1
j , there are n numbers and the
average of these numbers is
n+1
2
. Hence,

n
j =1
j =
n(n+1)
2
.
c 2009. Miguel A. Arcones. All rights reserved. Manual for SOA Exam FM/CAS Exam 2.
14/??
Chapter 3. Annuities. Section 3.1. Geometric series.
For an arithmetic sequence,
n

j =0
(a + jd) =
n

j =0
a + d
n

j =0
j = (n + 1)a + d
n(n + 1)
2
.
c 2009. Miguel A. Arcones. All rights reserved. Manual for SOA Exam FM/CAS Exam 2.
15/??
Chapter 3. Annuities. Section 3.1. Geometric series.
Example 1
Find

100
k=10
k.
Solution 1:

100
k=10
k =

100
k=1
k

9
k=1
k =
(100)(101)
2

(9)(10)
2
= 5005.
Solution 2: By the change of variables k = j + 9,
100

k=10
k =
91

j =1
(j + 9) =
(91)(92)
2
+ (9)(91) = 5005.
Notice that if k = 10, then j = 9; and if k = 100, then j = 91.
c 2009. Miguel A. Arcones. All rights reserved. Manual for SOA Exam FM/CAS Exam 2.
16/??
Chapter 3. Annuities. Section 3.1. Geometric series.
Example 1
Find

100
k=10
k.
Solution 1:

100
k=10
k =

100
k=1
k

9
k=1
k =
(100)(101)
2

(9)(10)
2
= 5005.
Solution 2: By the change of variables k = j + 9,
100

k=10
k =
91

j =1
(j + 9) =
(91)(92)
2
+ (9)(91) = 5005.
Notice that if k = 10, then j = 9; and if k = 100, then j = 91.
c 2009. Miguel A. Arcones. All rights reserved. Manual for SOA Exam FM/CAS Exam 2.
17/??
Chapter 3. Annuities. Section 3.1. Geometric series.
Example 1
Find

100
k=10
k.
Solution 1:

100
k=10
k =

100
k=1
k

9
k=1
k =
(100)(101)
2

(9)(10)
2
= 5005.
Solution 2: By the change of variables k = j + 9,
100

k=10
k =
91

j =1
(j + 9) =
(91)(92)
2
+ (9)(91) = 5005.
Notice that if k = 10, then j = 9; and if k = 100, then j = 91.
c 2009. Miguel A. Arcones. All rights reserved. Manual for SOA Exam FM/CAS Exam 2.
18/??
Chapter 3. Annuities. Section 3.1. Geometric series.
Denition 2
The sequence {ar
n
}

n=0
is called a geometric sequence.
The geometric sequence {ar
n
}

n=0
satises that for each n 1,
rx
n1
= rar
n1
= ar
n
= x
n
, where x
n
= ar
n
.
c 2009. Miguel A. Arcones. All rights reserved. Manual for SOA Exam FM/CAS Exam 2.
19/??
Chapter 3. Annuities. Section 3.1. Geometric series.
Denition 2
The sequence {ar
n
}

n=0
is called a geometric sequence.
The geometric sequence {ar
n
}

n=0
satises that for each n 1,
rx
n1
= rar
n1
= ar
n
= x
n
, where x
n
= ar
n
.
c 2009. Miguel A. Arcones. All rights reserved. Manual for SOA Exam FM/CAS Exam 2.
20/??
Chapter 3. Annuities. Section 3.1. Geometric series.
Theorem 3
If a sequence satises x
n
= rx
n1
, for each n 1, then x
n
= x
0
r
n
for each n 1.
Proof.
The proof is by induction on n. The case n = 0 is obvious. Assume
that the case n holds. Then,
x
n+1
= rx
n
= rx
0
r
n
= x
0
r
n+1
= x
n+1
.
c 2009. Miguel A. Arcones. All rights reserved. Manual for SOA Exam FM/CAS Exam 2.
21/??
Chapter 3. Annuities. Section 3.1. Geometric series.
Theorem 3
If a sequence satises x
n
= rx
n1
, for each n 1, then x
n
= x
0
r
n
for each n 1.
Proof.
The proof is by induction on n. The case n = 0 is obvious. Assume
that the case n holds. Then,
x
n+1
= rx
n
= rx
0
r
n
= x
0
r
n+1
= x
n+1
.
c 2009. Miguel A. Arcones. All rights reserved. Manual for SOA Exam FM/CAS Exam 2.
22/??
Chapter 3. Annuities. Section 3.1. Geometric series.
Theorem 4
For any r R,
r
n+1
1 = (r 1)
n

j =0
r
j
.
Proof.

n
j =0
r
j
(r 1) = (1 + r + r
2
+ + r
n
)(r 1)
= (r + r
2
+ + r
n+1
) (1 + r + r
2
+ + r
n
) = r
n+1
1.
c 2009. Miguel A. Arcones. All rights reserved. Manual for SOA Exam FM/CAS Exam 2.
23/??
Chapter 3. Annuities. Section 3.1. Geometric series.
In particular, we that
(x
2
1) = (x 1)(1 + x),
(x
3
1) = (x 1)(1 + x + x
2
),
(x
4
1) = (x 1)(1 + x + x
2
+ x
3
),
(x
5
1) = (x 1)(1 + x + x
2
+ x
3
+ x
4
).
c 2009. Miguel A. Arcones. All rights reserved. Manual for SOA Exam FM/CAS Exam 2.
24/??
Chapter 3. Annuities. Section 3.1. Geometric series.
Corollary 1
(i) If r = 1,

n
j =0
r
j
=
r
n+1
1
r 1
.
(ii) If r = 1,

n
j =0
r
j
= n + 1.
Proof.
(i) If r = 1, from r
n+1
1 = (r 1)

n
j =0
r
j
, we get that
n

j =0
r
j
=
r
n+1
1
r 1
.
(ii) If r = 1,
n

j =0
r
j
=
n

j =0
1 = n + 1.
c 2009. Miguel A. Arcones. All rights reserved. Manual for SOA Exam FM/CAS Exam 2.
25/??
Chapter 3. Annuities. Section 3.1. Geometric series.
Corollary 1
(i) If r = 1,

n
j =0
r
j
=
r
n+1
1
r 1
.
(ii) If r = 1,

n
j =0
r
j
= n + 1.
Proof.
(i) If r = 1, from r
n+1
1 = (r 1)

n
j =0
r
j
, we get that
n

j =0
r
j
=
r
n+1
1
r 1
.
(ii) If r = 1,
n

j =0
r
j
=
n

j =0
1 = n + 1.
c 2009. Miguel A. Arcones. All rights reserved. Manual for SOA Exam FM/CAS Exam 2.
26/??
Chapter 3. Annuities. Section 3.1. Geometric series.
Example 2
Find

20
k=5
2
k
.
Solution:
20

k=5
2
k
=
20

k=0
2
k

k=0
2
k
=
2
21
1
2 1

2
5
1
2 1
= 2
21
2
5
= 2097120.
c 2009. Miguel A. Arcones. All rights reserved. Manual for SOA Exam FM/CAS Exam 2.
27/??
Chapter 3. Annuities. Section 3.1. Geometric series.
Example 2
Find

20
k=5
2
k
.
Solution:
20

k=5
2
k
=
20

k=0
2
k

k=0
2
k
=
2
21
1
2 1

2
5
1
2 1
= 2
21
2
5
= 2097120.
c 2009. Miguel A. Arcones. All rights reserved. Manual for SOA Exam FM/CAS Exam 2.
28/??
Chapter 3. Annuities. Section 3.1. Geometric series.
We also have that for 1 m n and r = 1,
n

j =m
r
j
=
n

j =0
r
j

m1

j =0
r
j
=
r
n+1
1
r 1

r
m
1
r 1
=
r
n+1
r
m
r 1
.
Example 3
Find

20
k=5
2
k
.
Solution:

20
k=5
2
k
=
2
21
2
5
21
= 2
21
2
5
= 2097120.
c 2009. Miguel A. Arcones. All rights reserved. Manual for SOA Exam FM/CAS Exam 2.
29/??
Chapter 3. Annuities. Section 3.1. Geometric series.
We also have that for 1 m n and r = 1,
n

j =m
r
j
=
n

j =0
r
j

m1

j =0
r
j
=
r
n+1
1
r 1

r
m
1
r 1
=
r
n+1
r
m
r 1
.
Example 3
Find

20
k=5
2
k
.
Solution:

20
k=5
2
k
=
2
21
2
5
21
= 2
21
2
5
= 2097120.
c 2009. Miguel A. Arcones. All rights reserved. Manual for SOA Exam FM/CAS Exam 2.
30/??
Chapter 3. Annuities. Section 3.1. Geometric series.
We also have that for 1 m n and r = 1,
n

j =m
r
j
=
n

j =0
r
j

m1

j =0
r
j
=
r
n+1
1
r 1

r
m
1
r 1
=
r
n+1
r
m
r 1
.
Example 3
Find

20
k=5
2
k
.
Solution:

20
k=5
2
k
=
2
21
2
5
21
= 2
21
2
5
= 2097120.
c 2009. Miguel A. Arcones. All rights reserved. Manual for SOA Exam FM/CAS Exam 2.
31/??
Chapter 3. Annuities. Section 3.1. Geometric series.
Theorem 5
For |r | < 1,

j =0
r
j
=
1
1r
.
Proof.
If |r | < 1, then ln(|r |) < 0 and |r
n+1
| |r |
n+1
= e
(n+1) ln(|r |)
0,
as n . Hence,

j =0
r
j
= lim
n
n

j =0
r
j
= lim
n
r
n+1
1
r 1
=
1
1 r
.
c 2009. Miguel A. Arcones. All rights reserved. Manual for SOA Exam FM/CAS Exam 2.
32/??
Chapter 3. Annuities. Section 3.1. Geometric series.
Theorem 5
For |r | < 1,

j =0
r
j
=
1
1r
.
Proof.
If |r | < 1, then ln(|r |) < 0 and |r
n+1
| |r |
n+1
= e
(n+1) ln(|r |)
0,
as n . Hence,

j =0
r
j
= lim
n
n

j =0
r
j
= lim
n
r
n+1
1
r 1
=
1
1 r
.
c 2009. Miguel A. Arcones. All rights reserved. Manual for SOA Exam FM/CAS Exam 2.
33/??
Chapter 3. Annuities. Section 3.1. Geometric series.
Corollary 2
For |r | < 1,

j =n
r
j
=
r
n
1 r
.
Proof.
By the change of variables j = k + n,

j =n
r
j
=

k=0
r
k+n
= r
n

k=0
r
k
=
r
n
1 r
.
c 2009. Miguel A. Arcones. All rights reserved. Manual for SOA Exam FM/CAS Exam 2.
34/??
Chapter 3. Annuities. Section 3.1. Geometric series.
Corollary 2
For |r | < 1,

j =n
r
j
=
r
n
1 r
.
Proof.
By the change of variables j = k + n,

j =n
r
j
=

k=0
r
k+n
= r
n

k=0
r
k
=
r
n
1 r
.
c 2009. Miguel A. Arcones. All rights reserved. Manual for SOA Exam FM/CAS Exam 2.
35/??
Chapter 3. Annuities. Section 3.1. Geometric series.
Example 4
Find

k=9
3
k
.
Solution:

k=9
3
k
=
3
9
1 (1/3)
= 0.0000762079.
c 2009. Miguel A. Arcones. All rights reserved. Manual for SOA Exam FM/CAS Exam 2.
36/??
Chapter 3. Annuities. Section 3.1. Geometric series.
Example 4
Find

k=9
3
k
.
Solution:

k=9
3
k
=
3
9
1 (1/3)
= 0.0000762079.
c 2009. Miguel A. Arcones. All rights reserved. Manual for SOA Exam FM/CAS Exam 2.
37/??
Chapter 3. Annuities. Section 3.1. Geometric series.
Theorem 6
For r = 1,
n

j =1
jr
j
=
nr
n+2
(n + 1)r
n+1
+ r
(r 1)
2
.
Proof.
Taking derivatives with respect to r in the inequality

n
j =0
r
j
=
r
n+1
1
r 1
, we get that

n
j =1
jr
j 1
=
(n+1)r
n
(r 1)(r
n+1
1)
(r 1)
2
=
nr
n+1
(n+1)r
n
+1
(r 1)
2
c 2009. Miguel A. Arcones. All rights reserved. Manual for SOA Exam FM/CAS Exam 2.
38/??
Chapter 3. Annuities. Section 3.1. Geometric series.
Theorem 6
For r = 1,
n

j =1
jr
j
=
nr
n+2
(n + 1)r
n+1
+ r
(r 1)
2
.
Proof.
Taking derivatives with respect to r in the inequality

n
j =0
r
j
=
r
n+1
1
r 1
, we get that

n
j =1
jr
j 1
=
(n+1)r
n
(r 1)(r
n+1
1)
(r 1)
2
=
nr
n+1
(n+1)r
n
+1
(r 1)
2
c 2009. Miguel A. Arcones. All rights reserved. Manual for SOA Exam FM/CAS Exam 2.
39/??
Chapter 3. Annuities. Section 3.1. Geometric series.
Corollary 3
For |r | < 1,

j =1
jr
j
=
r
(r 1)
2
.
Proof.
If |r | < 1, then

j =1
jr
j
= lim
n
n

j =1
jr
j
= lim
n
nr
n+2
(n + 1)r
n+1
+ r
(r 1)
2
=
r
(r 1)
2
.
c 2009. Miguel A. Arcones. All rights reserved. Manual for SOA Exam FM/CAS Exam 2.
40/??
Chapter 3. Annuities. Section 3.1. Geometric series.
Corollary 3
For |r | < 1,

j =1
jr
j
=
r
(r 1)
2
.
Proof.
If |r | < 1, then

j =1
jr
j
= lim
n
n

j =1
jr
j
= lim
n
nr
n+2
(n + 1)r
n+1
+ r
(r 1)
2
=
r
(r 1)
2
.
c 2009. Miguel A. Arcones. All rights reserved. Manual for SOA Exam FM/CAS Exam 2.
41/??
Chapter 3. Annuities. Section 3.1. Geometric series.
Example 5
Find

k=1
k4
k
.
Solution:

k=1
k4
k
=
4
1
(1 4
1
)
2
=
4
9
= 0.4444444444.
c 2009. Miguel A. Arcones. All rights reserved. Manual for SOA Exam FM/CAS Exam 2.
42/??
Chapter 3. Annuities. Section 3.1. Geometric series.
Example 5
Find

k=1
k4
k
.
Solution:

k=1
k4
k
=
4
1
(1 4
1
)
2
=
4
9
= 0.4444444444.
c 2009. Miguel A. Arcones. All rights reserved. Manual for SOA Exam FM/CAS Exam 2.
1/82
Chapter 3. Annuities.
Manual for SOA Exam FM/CAS Exam 2.
Chapter 3. Annuities.
Section 3.2. Level payment annuities.
c 2009. Miguel A. Arcones. All rights reserved.
Extract from:
Arcones Manual for the SOA Exam FM/CAS Exam 2,
Financial Mathematics. Fall 2009 Edition,
available at http://www.actexmadriver.com/
c 2009. Miguel A. Arcones. All rights reserved. Manual for SOA Exam FM/CAS Exam 2.
2/82
Chapter 3. Annuities. Section 3.2. Level payment annuities.
An annuity is a sequence of payments made at equal intervals of
time. We have n periods of times
[0, t], [t, 2t], [2t, 3t], . . . [(n 1)t, nt] with the same length. By a
change of units, we will assume that intervals have unit length. So,
the intervals are [0, 1], [1, 2], [2, 3], . . . [n 1, n]. We order the
periods as follows:
Time interval Name
Beginning
of the period
End
of the period
[0, 1] 1st period time 0 time 1
[1, 2] 2nd period time 1 time 2
[2, 3] 3rd period time 2 time 3

[n 2, n 1] (n 1)th period time n 2 time n 1
[n 1, n] nth period time n 1 time n
c 2009. Miguel A. Arcones. All rights reserved. Manual for SOA Exam FM/CAS Exam 2.
3/82
Chapter 3. Annuities. Section 3.2. Level payment annuities.
An annuity is said to have level payments if all payments C
j
are
equal.
An annuity has nonlevel payments if some payments C
j
are
dierent from other ones.
The payments can be made either at the beginning or at the end
of intervals of time.
For an annuityimmediate payments are made at the end of the
intervals of time.
An annuityimmediate is a cashow of the type:
Contributions 0 C
1
C
2
C
n
Time 0 1 2 n
For an annuitydue the payments are made at the beginning of
the intervals of time.
An annuitydue is a cashow of the type:
Contributions C
0
C
1
C
n1
0
Time 0 1 n 1 n
c 2009. Miguel A. Arcones. All rights reserved. Manual for SOA Exam FM/CAS Exam 2.
4/82
Chapter 3. Annuities. Section 3.2. Level payment annuities.
An annuity is said to have level payments if all payments C
j
are
equal.
An annuity has nonlevel payments if some payments C
j
are
dierent from other ones.
The payments can be made either at the beginning or at the end
of intervals of time.
For an annuityimmediate payments are made at the end of the
intervals of time.
An annuityimmediate is a cashow of the type:
Contributions 0 C
1
C
2
C
n
Time 0 1 2 n
For an annuitydue the payments are made at the beginning of
the intervals of time.
An annuitydue is a cashow of the type:
Contributions C
0
C
1
C
n1
0
Time 0 1 n 1 n
c 2009. Miguel A. Arcones. All rights reserved. Manual for SOA Exam FM/CAS Exam 2.
5/82
Chapter 3. Annuities. Section 3.2. Level payment annuities.
An annuity is said to have level payments if all payments C
j
are
equal.
An annuity has nonlevel payments if some payments C
j
are
dierent from other ones.
The payments can be made either at the beginning or at the end
of intervals of time.
For an annuityimmediate payments are made at the end of the
intervals of time.
An annuityimmediate is a cashow of the type:
Contributions 0 C
1
C
2
C
n
Time 0 1 2 n
For an annuitydue the payments are made at the beginning of
the intervals of time.
An annuitydue is a cashow of the type:
Contributions C
0
C
1
C
n1
0
Time 0 1 n 1 n
c 2009. Miguel A. Arcones. All rights reserved. Manual for SOA Exam FM/CAS Exam 2.
6/82
Chapter 3. Annuities. Section 3.2. Level payment annuities.
An annuity is said to have level payments if all payments C
j
are
equal.
An annuity has nonlevel payments if some payments C
j
are
dierent from other ones.
The payments can be made either at the beginning or at the end
of intervals of time.
For an annuityimmediate payments are made at the end of the
intervals of time.
An annuityimmediate is a cashow of the type:
Contributions 0 C
1
C
2
C
n
Time 0 1 2 n
For an annuitydue the payments are made at the beginning of
the intervals of time.
An annuitydue is a cashow of the type:
Contributions C
0
C
1
C
n1
0
Time 0 1 n 1 n
c 2009. Miguel A. Arcones. All rights reserved. Manual for SOA Exam FM/CAS Exam 2.
7/82
Chapter 3. Annuities. Section 3.2. Level payment annuities.
An annuity is said to have level payments if all payments C
j
are
equal.
An annuity has nonlevel payments if some payments C
j
are
dierent from other ones.
The payments can be made either at the beginning or at the end
of intervals of time.
For an annuityimmediate payments are made at the end of the
intervals of time.
An annuityimmediate is a cashow of the type:
Contributions 0 C
1
C
2
C
n
Time 0 1 2 n
For an annuitydue the payments are made at the beginning of
the intervals of time.
An annuitydue is a cashow of the type:
Contributions C
0
C
1
C
n1
0
Time 0 1 n 1 n
c 2009. Miguel A. Arcones. All rights reserved. Manual for SOA Exam FM/CAS Exam 2.
8/82
Chapter 3. Annuities. Section 3.2. Level payment annuities.
The cashow of an annuityimmediate with level payments of one
is
Contributions 0 1 1 1
Time 0 1 2 n
If the time value of the money follows an accumulation function
a(t), then the present value of an annuityimmediate with
level annual payments of one is
a
n|
=
1
a(1)
+
1
a(2)
+ +
1
a(n)
=
n

j =1
1
a(j )
.
The accumulated value of an annuityimmediate with level
annual payments of one is
s
n|
=
a(n)
a(1)
+
a(n)
a(2)
+ +
a(n)
a(n)
=
n

j =1
a(n)
a(j )
.
c 2009. Miguel A. Arcones. All rights reserved. Manual for SOA Exam FM/CAS Exam 2.
9/82
Chapter 3. Annuities. Section 3.2. Level payment annuities.
Example 1
You are given that
t
=
1
8+t
, t 0. Find a
n|
and s
n|
.
Solution: We have that

t
0

s
ds =

t
0
1
8 + s
ds = ln(8 + s)

t
0
= ln

8 + t
8

.
So, a(t) = e
R
t
0

s
=
8+t
8
,
a
n|
=
n

j =1
1
a(j )
=
n

j =1
8
8 + j
and
s
n|
=
n

j =1
a(n)
a(j )
=
n

j =1
8 + n
8 + j
.
c 2009. Miguel A. Arcones. All rights reserved. Manual for SOA Exam FM/CAS Exam 2.
10/82
Chapter 3. Annuities. Section 3.2. Level payment annuities.
Example 1
You are given that
t
=
1
8+t
, t 0. Find a
n|
and s
n|
.
Solution: We have that

t
0

s
ds =

t
0
1
8 + s
ds = ln(8 + s)

t
0
= ln

8 + t
8

.
So, a(t) = e
R
t
0

s
=
8+t
8
,
a
n|
=
n

j =1
1
a(j )
=
n

j =1
8
8 + j
and
s
n|
=
n

j =1
a(n)
a(j )
=
n

j =1
8 + n
8 + j
.
c 2009. Miguel A. Arcones. All rights reserved. Manual for SOA Exam FM/CAS Exam 2.
11/82
Chapter 3. Annuities. Section 3.2. Level payment annuities.
The cashow of an annuitydue with n level payments of one is
Contributions 1 1 1 1 0
Time 0 1 2 n 1 n
The present value of an annuitydue with n level annual
payments of one is
a
n|
= 1 +
1
a(1)
+
1
a(2)
+ +
1
a(n 1)
=
n1

j =0
1
a(j )
.
The future value at time n of an annuitydue with level
annual payments of one is
s
n|
= a(n) +
a(n)
a(1)
+
a(n)
a(2)
+ +
a(n)
a(n 1)
=
n1

j =0
a(n)
a(j )
.
c 2009. Miguel A. Arcones. All rights reserved. Manual for SOA Exam FM/CAS Exam 2.
12/82
Chapter 3. Annuities. Section 3.2. Level payment annuities.
Example 2
Suppose that the annual eective interest rate for year n is
i
n
=
2
n+4
. Find a
n|
and s
n|
.
Solution: Since 1 + i
n
= 1 +
2
n+4
=
n+6
n+4
,
a(n) = (1 + i
1
)(1 + i
1
) (1 + i
n
) =
1 + 6
1 + 4

2 + 6
2 + 4

3 + 6
3 + 4

n + 6
n + 4
=
(n + 6)(n + 5)
30
,
a
n|
=
n

j =1
1
a(j )
=
n1

j =0
30
(j + 6)(j + 5)
=
n1

j =0
(30)

1
j + 5

1
j + 6

=30

1
0 + 5
+ +
1
n 1 + 5

30

1
1 + 5
+ +
1
n + 5

=(30)

1
5

1
n + 5

=
6n
n + 5
, s
n|
= a(n)a
n|
=
n(n + 6)
5
.
c 2009. Miguel A. Arcones. All rights reserved. Manual for SOA Exam FM/CAS Exam 2.
13/82
Chapter 3. Annuities. Section 3.2. Level payment annuities.
Example 2
Suppose that the annual eective interest rate for year n is
i
n
=
2
n+4
. Find a
n|
and s
n|
.
Solution: Since 1 + i
n
= 1 +
2
n+4
=
n+6
n+4
,
a(n) = (1 + i
1
)(1 + i
1
) (1 + i
n
) =
1 + 6
1 + 4

2 + 6
2 + 4

3 + 6
3 + 4

n + 6
n + 4
=
(n + 6)(n + 5)
30
,
a
n|
=
n

j =1
1
a(j )
=
n1

j =0
30
(j + 6)(j + 5)
=
n1

j =0
(30)

1
j + 5

1
j + 6

=30

1
0 + 5
+ +
1
n 1 + 5

30

1
1 + 5
+ +
1
n + 5

=(30)

1
5

1
n + 5

=
6n
n + 5
, s
n|
= a(n)a
n|
=
n(n + 6)
5
.
c 2009. Miguel A. Arcones. All rights reserved. Manual for SOA Exam FM/CAS Exam 2.
14/82
Chapter 3. Annuities. Section 3.2. Level payment annuities.
Remember:
The cashow of an annuityimmediate with n level payments of
one is
Contributions 0 1 1 1
Time 0 1 2 n
The cashow of an annuitydue with n level payments of one is
Contributions 1 1 1 1 0
Time 0 1 2 n 1 n
c 2009. Miguel A. Arcones. All rights reserved. Manual for SOA Exam FM/CAS Exam 2.
15/82
Chapter 3. Annuities. Section 3.2. Level payment annuities.
Remember:
The cashow of an annuityimmediate with n level payments of
one is
Contributions 0 1 1 1
Time 0 1 2 n
The cashow of an annuitydue with n level payments of one is
Contributions 1 1 1 1 0
Time 0 1 2 n 1 n
c 2009. Miguel A. Arcones. All rights reserved. Manual for SOA Exam FM/CAS Exam 2.
16/82
Chapter 3. Annuities. Section 3.2. Level payment annuities.
Theorem 1
If i = 0, the present value of an annuityimmediate with level
payments of one is
a
n|i
= a
n|
= +
2
+ +
n
=
(1
n
)
1
=
1
n
1

1
=
1
n
i
=
1 (1 + i )
n
i
,
where we have used that (1 + i ) = 1.
If i = 0, the future value of an annuityimmediate with level
payments of one at time n is
s
n|i
= s
n|
= (1+i )
n1
+(1+i )
n2
+ +(1+i )+1 =
(1 + i )
n
1
i
.
If i = 0, a
n|i
= s
n|i
= n.
c 2009. Miguel A. Arcones. All rights reserved. Manual for SOA Exam FM/CAS Exam 2.
17/82
Chapter 3. Annuities. Section 3.2. Level payment annuities.
Theorem 1
If i = 0, the present value of an annuityimmediate with level
payments of one is
a
n|i
= a
n|
= +
2
+ +
n
=
(1
n
)
1
=
1
n
1

1
=
1
n
i
=
1 (1 + i )
n
i
,
where we have used that (1 + i ) = 1.
If i = 0, the future value of an annuityimmediate with level
payments of one at time n is
s
n|i
= s
n|
= (1+i )
n1
+(1+i )
n2
+ +(1+i )+1 =
(1 + i )
n
1
i
.
If i = 0, a
n|i
= s
n|i
= n.
c 2009. Miguel A. Arcones. All rights reserved. Manual for SOA Exam FM/CAS Exam 2.
18/82
Chapter 3. Annuities. Section 3.2. Level payment annuities.
Theorem 1
If i = 0, the present value of an annuityimmediate with level
payments of one is
a
n|i
= a
n|
= +
2
+ +
n
=
(1
n
)
1
=
1
n
1

1
=
1
n
i
=
1 (1 + i )
n
i
,
where we have used that (1 + i ) = 1.
If i = 0, the future value of an annuityimmediate with level
payments of one at time n is
s
n|i
= s
n|
= (1+i )
n1
+(1+i )
n2
+ +(1+i )+1 =
(1 + i )
n
1
i
.
If i = 0, a
n|i
= s
n|i
= n.
c 2009. Miguel A. Arcones. All rights reserved. Manual for SOA Exam FM/CAS Exam 2.
19/82
Chapter 3. Annuities. Section 3.2. Level payment annuities.
For example,
a
1|i
= ,
a
2|i
= +
2
,
a
3|i
= +
2
+
3
,
a
4|i
= +
2
+
3
+
4
.
s
1|i
= 1,
s
2|i
= 1 + 1 + i ,
s
3|i
= 1 + 1 + i + (1 + i )
2
,
s
4|i
= 1 + 1 + i + (1 + i )
2
+ (1 + i )
3
.
c 2009. Miguel A. Arcones. All rights reserved. Manual for SOA Exam FM/CAS Exam 2.
20/82
Chapter 3. Annuities. Section 3.2. Level payment annuities.
Example 3
Calculate the present value of $5000 paid at the end of each year
for 15 years using an annual eective interest rate of 7.5%.
Solution: The present value is
(5000)a
15|0.075
= (5000)
1 (1.075)
15
0.075
= 44135.59873.
c 2009. Miguel A. Arcones. All rights reserved. Manual for SOA Exam FM/CAS Exam 2.
21/82
Chapter 3. Annuities. Section 3.2. Level payment annuities.
Example 3
Calculate the present value of $5000 paid at the end of each year
for 15 years using an annual eective interest rate of 7.5%.
Solution: The present value is
(5000)a
15|0.075
= (5000)
1 (1.075)
15
0.075
= 44135.59873.
c 2009. Miguel A. Arcones. All rights reserved. Manual for SOA Exam FM/CAS Exam 2.
22/82
Chapter 3. Annuities. Section 3.2. Level payment annuities.
If every period is exactly one year, then i in the formulas above is
the annual eective rate of interest. If the length of a period is not
a year, i in the formulas above is the eective rate of interest per
period. If each period lasts t years, then the tyear interest factor
is (1 + i )
t
and the tyear eective rate of interest is (1 + i )
t
1.
So, if each period lasts t years, we use the previous formulas with i
replaced by (1 + i )
t
1, where the last i is the annual eective
rate of interest.
c 2009. Miguel A. Arcones. All rights reserved. Manual for SOA Exam FM/CAS Exam 2.
23/82
Chapter 3. Annuities. Section 3.2. Level payment annuities.
For example, suppose that the payments are made each 1/m years.
Then, the interest factor for 1/m years is (1 + i )
1/m
and the 1/m
year interest rate is (1 +i )
1/m
1 =
i
(m)
m
, where i
(m)
is the nominal
annual rate of interest convertible m times a year. For example,
the present value at time 0 of the annuity
Contributions 0 1 1 1
Time (in years) 0 1/m 2/m n/m
is a
n|(1+i )
1/m
1
= a
n|i
(m)
/m
. The future value at time n/m years of
the previous cashow is s
n|(1+i )
1/m
1
= s
n|i
(m)
/m
.
c 2009. Miguel A. Arcones. All rights reserved. Manual for SOA Exam FM/CAS Exam 2.
24/82
Chapter 3. Annuities. Section 3.2. Level payment annuities.
Example 4
John invest $500 into an account at the end of each month for 5
years. The annual eective interest rate is 4.5%. Calculate the
balance of this account at the end of 5 years.
Solution: The number of payments made is (5)(12) = 60. The
cashow is
Contributions 500 500 500 500
Time (in months) 1 2 2 60
The onemonth eective interest rate is (1.045)
1/12
1. Hence,
the balance of this account at the end of 5 years is
(500)s
60|(1.045)
1/12
1
= (500)
((1.045)
1/12
1 + 1)
60
1
(1.045)
1/12
1
=(500)
(1.045)
5
1
(1.045)
1/12
1
= 33495.8784.
c 2009. Miguel A. Arcones. All rights reserved. Manual for SOA Exam FM/CAS Exam 2.
25/82
Chapter 3. Annuities. Section 3.2. Level payment annuities.
Example 4
John invest $500 into an account at the end of each month for 5
years. The annual eective interest rate is 4.5%. Calculate the
balance of this account at the end of 5 years.
Solution: The number of payments made is (5)(12) = 60. The
cashow is
Contributions 500 500 500 500
Time (in months) 1 2 2 60
The onemonth eective interest rate is (1.045)
1/12
1. Hence,
the balance of this account at the end of 5 years is
(500)s
60|(1.045)
1/12
1
= (500)
((1.045)
1/12
1 + 1)
60
1
(1.045)
1/12
1
=(500)
(1.045)
5
1
(1.045)
1/12
1
= 33495.8784.
c 2009. Miguel A. Arcones. All rights reserved. Manual for SOA Exam FM/CAS Exam 2.
26/82
Chapter 3. Annuities. Section 3.2. Level payment annuities.
If payments are made every m years, the interest factor per period
is (1 + i )
m
1. For example, the future value at time nm years of
the annuity
Contributions 0 1 1 1
Time (in years) 0 m 2m nm
is a
n|(1+i )
m
1
, where i is the annual eective rate of interest. The
future value at time nm years of the previous annuity is s
n|(1+i )
m
1
.
c 2009. Miguel A. Arcones. All rights reserved. Manual for SOA Exam FM/CAS Exam 2.
27/82
Chapter 3. Annuities. Section 3.2. Level payment annuities.
Example 5
A cashow pays $8000 at the end of every other year for 16 years.
The rst payment is made in two years. The annual eective
interest rate is 6.5%. Calculate the present value of this cashow.
Solution: The cashow is
Contributions 8000 8000 8000 8000
Time (in years) 2 4 6 16
Notice that eight payments are made. The twoyear eective
interest rate is (1.065)
2
1. Hence, the present value of the
cashow is
(8000)a
8|(1.065)
2
1
= (8000)
1 ((1.065)
2
1 + 1)
8
(1.065)
2
1
=(8000)
1 (1.065)
16
(1.065)
2
1
= 37841.21717.
c 2009. Miguel A. Arcones. All rights reserved. Manual for SOA Exam FM/CAS Exam 2.
28/82
Chapter 3. Annuities. Section 3.2. Level payment annuities.
Example 5
A cashow pays $8000 at the end of every other year for 16 years.
The rst payment is made in two years. The annual eective
interest rate is 6.5%. Calculate the present value of this cashow.
Solution: The cashow is
Contributions 8000 8000 8000 8000
Time (in years) 2 4 6 16
Notice that eight payments are made. The twoyear eective
interest rate is (1.065)
2
1. Hence, the present value of the
cashow is
(8000)a
8|(1.065)
2
1
= (8000)
1 ((1.065)
2
1 + 1)
8
(1.065)
2
1
=(8000)
1 (1.065)
16
(1.065)
2
1
= 37841.21717.
c 2009. Miguel A. Arcones. All rights reserved. Manual for SOA Exam FM/CAS Exam 2.
29/82
Chapter 3. Annuities. Section 3.2. Level payment annuities.
Theorem 2
If i = 0, the present value of an annuitydue with level
payments of one is
a
n|i
= a
n|
= 1 + +
2
+ +
n1
=
1
n
1
=
1
n
d
,
where we have used that = 1 d.
If i = 0, the future value at time n of an annuitydue with
level payments of one is
s
n|i
= s
n|i
= (1 + i )
n
+ (1 + i )
n1
+ + (1 + i )
=
(1 + i )
n+1
(1 + i )
i
=
(1 + i )
n
1
d
,
where we have used that 1 d =
1
1+i
and d =
i
1+i
.
If i = 0, a
n|i
= s
n|i
= n.
c 2009. Miguel A. Arcones. All rights reserved. Manual for SOA Exam FM/CAS Exam 2.
30/82
Chapter 3. Annuities. Section 3.2. Level payment annuities.
The annuities factors which have introduced give the present value
of the cashow
Contributions 1 1 1
Time 1 2 n
at dierent times.
The present value of the cashow at time 0 is a
n|i
.
The present value of the cashow at time 1 is a
n|i
.
The present value of the cashow at time n is s
n|i
.
The present value of the cashow at time n + 1 is s
n|i
.
c 2009. Miguel A. Arcones. All rights reserved. Manual for SOA Exam FM/CAS Exam 2.
31/82
Chapter 3. Annuities. Section 3.2. Level payment annuities.
6 6 6 6
0 1 2 3 n 1 n n + 1
1 1 1 1 1
a
n|i
a
n|i
s
n|i
s
n|i
Figure 1: Present value of an annuity at dierent times
c 2009. Miguel A. Arcones. All rights reserved. Manual for SOA Exam FM/CAS Exam 2.
32/82
Chapter 3. Annuities. Section 3.2. Level payment annuities.
Often, the contributions do not start at time 0. But, a
n|i
is always
the present value of a level unity annuity one period before the rst
payment. a
n|i
is the present value of a level unity annuity at the
time of the rst payment. s
n|i
is the future value of a level unity
annuity at the time of the last payment. s
n|i
is the future value of
a level unity annuity one period after the last payment. For
example, for the cashow
Contributions 1 1 1
Time t + 1 t + 2 t + n
The present value of the cashow at time t is a
n|i
.
The present value of the cashow at time t + 1 is a
n|i
.
The present value of the cashow at time t + n is s
n|i
.
The present value of the cashow at time t + n + 1 is s
n|i
.
c 2009. Miguel A. Arcones. All rights reserved. Manual for SOA Exam FM/CAS Exam 2.
33/82
Chapter 3. Annuities. Section 3.2. Level payment annuities.
6 6 6 6
t t + 1 t + 2 t + 3 t + n 1 t + n t + n + 1
1 1 1 1 1
a
n|i
a
n|i
s
n|i
s
n|i
Figure 2: Present value of an annuity at dierent times
c 2009. Miguel A. Arcones. All rights reserved. Manual for SOA Exam FM/CAS Exam 2.
34/82
Chapter 3. Annuities. Section 3.2. Level payment annuities.
Example 6
Investment contributions of $4500 are made at the beginning of
the year for 20 years into an account. This account pays an annual
eective interest rate is 7%. Calculate the accumulated value at
the end of 25 years.
Solution: The cashow of payments is
Contributions 4500 4500 4500
Time 0 1 19
We can nd the accumulated value at the end of 25 years doing
c 2009. Miguel A. Arcones. All rights reserved. Manual for SOA Exam FM/CAS Exam 2.
35/82
Chapter 3. Annuities. Section 3.2. Level payment annuities.
Example 6
Investment contributions of $4500 are made at the beginning of
the year for 20 years into an account. This account pays an annual
eective interest rate is 7%. Calculate the accumulated value at
the end of 25 years.
Solution: The cashow of payments is
Contributions 4500 4500 4500
Time 0 1 19
We can nd the accumulated value at the end of 25 years doing
either
a
20|0.07
(1 + 0.07)
26
=
1 (1 + 0.07)
20
0.07
(1 + 0.07)
26
=
(1 + 0.07)
26
(1 + 0.07)
6
0.07
= 276854.31,
c 2009. Miguel A. Arcones. All rights reserved. Manual for SOA Exam FM/CAS Exam 2.
36/82
Chapter 3. Annuities. Section 3.2. Level payment annuities.
Example 6
Investment contributions of $4500 are made at the beginning of
the year for 20 years into an account. This account pays an annual
eective interest rate is 7%. Calculate the accumulated value at
the end of 25 years.
Solution: The cashow of payments is
Contributions 4500 4500 4500
Time 0 1 19
We can nd the accumulated value at the end of 25 years doing
or
a
20|0.07
(1 + 0.07)
25
=
1 (1 + 0.07)
20
1 (1 + 0.07)
1
(1 + 0.07)
25
=
(1 + 0.07)
26
(1 + 0.07)
6
0.07
= 276854.31,
c 2009. Miguel A. Arcones. All rights reserved. Manual for SOA Exam FM/CAS Exam 2.
37/82
Chapter 3. Annuities. Section 3.2. Level payment annuities.
Example 6
Investment contributions of $4500 are made at the beginning of
the year for 20 years into an account. This account pays an annual
eective interest rate is 7%. Calculate the accumulated value at
the end of 25 years.
Solution: The cashow of payments is
Contributions 4500 4500 4500
Time 0 1 19
We can nd the accumulated value at the end of 25 years doing
or
s
20|0.07
(1 + 0.07)
6
=
(1 + 0.07)
20
1
0.07
(1 + 0.07)
6
=
(1 + 0.07)
26
(1 + 0.07)
6
0.07
= 276854.31,
c 2009. Miguel A. Arcones. All rights reserved. Manual for SOA Exam FM/CAS Exam 2.
38/82
Chapter 3. Annuities. Section 3.2. Level payment annuities.
Example 6
Investment contributions of $4500 are made at the beginning of
the year for 20 years into an account. This account pays an annual
eective interest rate is 7%. Calculate the accumulated value at
the end of 25 years.
Solution: The cashow of payments is
Contributions 4500 4500 4500
Time 0 1 19
We can nd the accumulated value at the end of 25 years doing
or
s
20|0.07
(1 + 0.07)
5
=
(1 + 0.07)
20
1
1 (1 + 0.07)
1
(1 + 0.07)
5
=
(1 + 0.07)
26
(1 + 0.07)
6
0.07
= 276854.31.
c 2009. Miguel A. Arcones. All rights reserved. Manual for SOA Exam FM/CAS Exam 2.
39/82
Chapter 3. Annuities. Section 3.2. Level payment annuities.
Theorem 3
(Interest factor relations for annuities)
a
n|i
= (1 + i )a
n|i
, s
n|i
= (1 + i )
n
a
n|i
, s
n|i
= (1 + i )
n+1
a
n|i
Proof.
Consider the cashow:
Contributions 0 1 1 1
Time 0 1 2 n
The present value at time 0 of the cashow is a
n|i
.
The present value at time 1 of the cashow is a
n|i
.
The present value at time n of the cashow is s
n|i
.
The present value at time n + 1 of the cashow is s
n|i
.
c 2009. Miguel A. Arcones. All rights reserved. Manual for SOA Exam FM/CAS Exam 2.
40/82
Chapter 3. Annuities. Section 3.2. Level payment annuities.
Theorem 3
(Interest factor relations for annuities)
a
n|i
= (1 + i )a
n|i
, s
n|i
= (1 + i )
n
a
n|i
, s
n|i
= (1 + i )
n+1
a
n|i
Proof.
Consider the cashow:
Contributions 0 1 1 1
Time 0 1 2 n
The present value at time 0 of the cashow is a
n|i
.
The present value at time 1 of the cashow is a
n|i
.
The present value at time n of the cashow is s
n|i
.
The present value at time n + 1 of the cashow is s
n|i
.
c 2009. Miguel A. Arcones. All rights reserved. Manual for SOA Exam FM/CAS Exam 2.
41/82
Chapter 3. Annuities. Section 3.2. Level payment annuities.
An application of
a
n|i
= (1 + i )a
n|i
.
Example 7
If a
n|i
= 11.5174109 and a
n|i
= 11.9205203, nd n.
Solution: We have that a
n|i
= (1 + i )a
n|i
. So,
1 + i =
11.9205203
11.5174109
= 1.035 and i = 3.5%. Solving
11.5174109 = a
n|3.5%
=
1(1.035)
n
0.035
, we get
(1.035)
n
= 1 (11.5174109)(0.035) = 0.5968906185 and
n =
log 0.5968906185
log 1.035
= 15.
c 2009. Miguel A. Arcones. All rights reserved. Manual for SOA Exam FM/CAS Exam 2.
42/82
Chapter 3. Annuities. Section 3.2. Level payment annuities.
An application of
a
n|i
= (1 + i )a
n|i
.
Example 7
If a
n|i
= 11.5174109 and a
n|i
= 11.9205203, nd n.
Solution: We have that a
n|i
= (1 + i )a
n|i
. So,
1 + i =
11.9205203
11.5174109
= 1.035 and i = 3.5%. Solving
11.5174109 = a
n|3.5%
=
1(1.035)
n
0.035
, we get
(1.035)
n
= 1 (11.5174109)(0.035) = 0.5968906185 and
n =
log 0.5968906185
log 1.035
= 15.
c 2009. Miguel A. Arcones. All rights reserved. Manual for SOA Exam FM/CAS Exam 2.
43/82
Chapter 3. Annuities. Section 3.2. Level payment annuities.
An application of
s
n|i
= (1 + i )s
n|i
.
Example 8
If s
n|i
= 21 and s
n|i
= 20, nd i .
Solution: We have that
21 = s
n|i
= (1 + i )s
n|i
= (1 + i )(20).
So, i =
21
20
1 = 5%.
c 2009. Miguel A. Arcones. All rights reserved. Manual for SOA Exam FM/CAS Exam 2.
44/82
Chapter 3. Annuities. Section 3.2. Level payment annuities.
An application of
s
n|i
= (1 + i )s
n|i
.
Example 8
If s
n|i
= 21 and s
n|i
= 20, nd i .
Solution: We have that
21 = s
n|i
= (1 + i )s
n|i
= (1 + i )(20).
So, i =
21
20
1 = 5%.
c 2009. Miguel A. Arcones. All rights reserved. Manual for SOA Exam FM/CAS Exam 2.
45/82
Chapter 3. Annuities. Section 3.2. Level payment annuities.
Theorem 4
(Induction relations)
a
n|i
= 1 + a
n1|i
and s
n|i
= 1 + s
n1|i
.
Proof.
Consider the cashows
Cashow 1 Contributions 1 0 0 0 0
Cashow 2 Contributions 0 1 1 1 0
Cashow 3 Contributions 1 1 1 1 0
Time 0 1 2 n 1 n
We have that the third cashow is the sum of the rst two. The
present value of the previous cashows are 1 and a
n1|i
and a
n|i
,
respectively. This implies the rst relation. The second relation
follows similarly.
c 2009. Miguel A. Arcones. All rights reserved. Manual for SOA Exam FM/CAS Exam 2.
46/82
Chapter 3. Annuities. Section 3.2. Level payment annuities.
Theorem 4
(Induction relations)
a
n|i
= 1 + a
n1|i
and s
n|i
= 1 + s
n1|i
.
Proof.
Consider the cashows
Cashow 1 Contributions 1 0 0 0 0
Cashow 2 Contributions 0 1 1 1 0
Cashow 3 Contributions 1 1 1 1 0
Time 0 1 2 n 1 n
We have that the third cashow is the sum of the rst two. The
present value of the previous cashows are 1 and a
n1|i
and a
n|i
,
respectively. This implies the rst relation. The second relation
follows similarly.
c 2009. Miguel A. Arcones. All rights reserved. Manual for SOA Exam FM/CAS Exam 2.
47/82
Chapter 3. Annuities. Section 3.2. Level payment annuities.
An application of
a
n|i
= 1 + a
n1|i
.
Example 9
If a
10|i
= 8, nd a
9|i
.
Solution: We have that a
9|i
= a
10|i
1 = 7.
c 2009. Miguel A. Arcones. All rights reserved. Manual for SOA Exam FM/CAS Exam 2.
48/82
Chapter 3. Annuities. Section 3.2. Level payment annuities.
An application of
a
n|i
= 1 + a
n1|i
.
Example 9
If a
10|i
= 8, nd a
9|i
.
Solution: We have that a
9|i
= a
10|i
1 = 7.
c 2009. Miguel A. Arcones. All rights reserved. Manual for SOA Exam FM/CAS Exam 2.
49/82
Chapter 3. Annuities. Section 3.2. Level payment annuities.
An application of
s
n|i
= 1 + s
n1|i
Example 10
If s
10|i
= 15, nd s
11|i
.
Solution: We have that s
11|i
= 1 + s
10|i
= 16.
c 2009. Miguel A. Arcones. All rights reserved. Manual for SOA Exam FM/CAS Exam 2.
50/82
Chapter 3. Annuities. Section 3.2. Level payment annuities.
An application of
s
n|i
= 1 + s
n1|i
Example 10
If s
10|i
= 15, nd s
11|i
.
Solution: We have that s
11|i
= 1 + s
10|i
= 16.
c 2009. Miguel A. Arcones. All rights reserved. Manual for SOA Exam FM/CAS Exam 2.
51/82
Chapter 3. Annuities. Section 3.2. Level payment annuities.
Theorem 5
(Amortization relations)
1
a
n|i
=
1
s
n|i
+ i and
1
a
n|i
=
1
s
n|i
+ d.
c 2009. Miguel A. Arcones. All rights reserved. Manual for SOA Exam FM/CAS Exam 2.
52/82
Chapter 3. Annuities. Section 3.2. Level payment annuities.
Proof of
1
a
n|i
=
1
s
n|i
+ i : Suppose that a loan of $1 is paid in n
payments made at the end of the each period. Then, each
payment should be
1
a
n|i
. Suppose that we pay the loan as follows,
we pay i at the end of each period and put x in an extra account
paying an eective rate of interest i . Since the initial loan is $1 the
interest accrued at the end of the rst period is i . But, we pay i at
the end of the rst period. Hence, immediately after this payment
we owe $1. Proceeding in this way, we deduce that we owe $1
immediately after each payment. The money in the extra account
accumulates to xs
n|i
at time n. In order to pay the loan, we need
x =
1
s
n|i
. Our total payments at the end of the each period are
1
s
n|i
+ i . Since this series of payments repays the loan of $1, we
must have that
1
a
n|i
=
1
s
n|i
+ i . The proof of the second formula is
similar and it is omitted.
c 2009. Miguel A. Arcones. All rights reserved. Manual for SOA Exam FM/CAS Exam 2.
53/82
Chapter 3. Annuities. Section 3.2. Level payment annuities.
An application of
1
a
n|i
=
1
s
n|i
+ i .
Example 11
If s
n|i
= 15.9171265 and a
n|i
= 8.86325164, calculate n.
Solution: Using that
1
a
n|i
=
1
s
n|i
+ i ,we get that
i =
1
8.86325164

1
15.9171265
= 5%. From a
n|5%
= 8.86325164, we
get n = 12.
c 2009. Miguel A. Arcones. All rights reserved. Manual for SOA Exam FM/CAS Exam 2.
54/82
Chapter 3. Annuities. Section 3.2. Level payment annuities.
An application of
1
a
n|i
=
1
s
n|i
+ i .
Example 11
If s
n|i
= 15.9171265 and a
n|i
= 8.86325164, calculate n.
Solution: Using that
1
a
n|i
=
1
s
n|i
+ i ,we get that
i =
1
8.86325164

1
15.9171265
= 5%. From a
n|5%
= 8.86325164, we
get n = 12.
c 2009. Miguel A. Arcones. All rights reserved. Manual for SOA Exam FM/CAS Exam 2.
55/82
Chapter 3. Annuities. Section 3.2. Level payment annuities.
In the calculator TIBAIIPlus we can use the time value of
money worksheet to solve problems with annuities. There are 5
main nancial variables in this worksheet:

The number of periods N .

The nominal interest for year I/Y .

The present value PV .

The payment per period PMT .

The future value FV .


Recall that how to use the money worksheet was explained in
Section 1.3. It is recommended that you setup P/Y =1 and
C/Y =1, by pressing:
2nd , P/Y , 1 , ENTER , , 1 , ENTER , 2nd , QUIT .
Unless it is said otherwise, we will assume that the entries for
C/Y and P/Y are both 1. To check that this is so, do
2nd P/Y 2nd QUIT .
c 2009. Miguel A. Arcones. All rights reserved. Manual for SOA Exam FM/CAS Exam 2.
56/82
Chapter 3. Annuities. Section 3.2. Level payment annuities.
When BGN is setup at END (and C/Y and P/Y have value 1),
the value time of money formula in the calculator is
PV + PMT
1 (1 + i )
N
i
+ FV(1 + i )
N
= 0.
Using the calculator, we can solve for any variable in the equation:
L + Pa
n|i
+ F(1 + i )
n
= 0.
This equation is equivalent to
L(1 + i )
n
+ Ps
n|i
+ F = 0.
In the calculator, we input the variables we know using: L as the
PV , P as the PMT , F as the FV , i as the I/Y , n as the N .
L, P and F can take negative values.
c 2009. Miguel A. Arcones. All rights reserved. Manual for SOA Exam FM/CAS Exam 2.
57/82
Chapter 3. Annuities. Section 3.2. Level payment annuities.
To solve for any variable from the equation
L + Pa
n|i
= 0
we input in the calculator: L as PV , P as PMT , 0 as FV , i as
the I/Y , n as N . If we are solving for either I/Y or N , PV
and PMT must have dierent signs.
To solve for any variable from the equation
Ps
n|i
+ F = 0.
we input: F as the FV , P as the PMT , i as the I/Y , n as the
N and 0 as the PV . If we are solving for either I/Y or N ,
FV and PMT must have dierent signs.
c 2009. Miguel A. Arcones. All rights reserved. Manual for SOA Exam FM/CAS Exam 2.
58/82
Chapter 3. Annuities. Section 3.2. Level payment annuities.
Example 12
What must you deposit at the end of each of the next 10 years in
order to accumulate 20,000 at the end of the 10 years assuming
i = 5%.
Solution: We solve for P in 20000 = Ps
10|0.05
and get
P = 1590.091499. In the calculator TIBAIIPlus, press:
10 N 20000 FV 5 I/Y 0 PV CPT PMT
Note the display in the calculator is negative.
c 2009. Miguel A. Arcones. All rights reserved. Manual for SOA Exam FM/CAS Exam 2.
59/82
Chapter 3. Annuities. Section 3.2. Level payment annuities.
Example 12
What must you deposit at the end of each of the next 10 years in
order to accumulate 20,000 at the end of the 10 years assuming
i = 5%.
Solution: We solve for P in 20000 = Ps
10|0.05
and get
P = 1590.091499. In the calculator TIBAIIPlus, press:
10 N 20000 FV 5 I/Y 0 PV CPT PMT
Note the display in the calculator is negative.
c 2009. Miguel A. Arcones. All rights reserved. Manual for SOA Exam FM/CAS Exam 2.
60/82
Chapter 3. Annuities. Section 3.2. Level payment annuities.
Example 13
If i
(12)
= 9% and $300 is deposited at the end of the each month
for 1 year, what will the accumulated value be in 1 year?
Solution: We solve for FV in FV = 300s
12|0.09/12
and get
FV = 3752.275907. In the calculator, press:
12 N 300 PMT 0.75 I/Y 0 PV CPT FV
where we used that 9/12 = 0.75.
Since the length of a period is a month:

The monthly eective interest rate is


i
(12)
12
=
0.09
12
= 0.75%.

The number of periods is 12 months.


c 2009. Miguel A. Arcones. All rights reserved. Manual for SOA Exam FM/CAS Exam 2.
61/82
Chapter 3. Annuities. Section 3.2. Level payment annuities.
Example 13
If i
(12)
= 9% and $300 is deposited at the end of the each month
for 1 year, what will the accumulated value be in 1 year?
Solution: We solve for FV in FV = 300s
12|0.09/12
and get
FV = 3752.275907. In the calculator, press:
12 N 300 PMT 0.75 I/Y 0 PV CPT FV
where we used that 9/12 = 0.75.
Since the length of a period is a month:

The monthly eective interest rate is


i
(12)
12
=
0.09
12
= 0.75%.

The number of periods is 12 months.


c 2009. Miguel A. Arcones. All rights reserved. Manual for SOA Exam FM/CAS Exam 2.
62/82
Chapter 3. Annuities. Section 3.2. Level payment annuities.
When BGN is setup at BGN, the value time of money formula in
this calculator is
PV + PMT (1 + i )
1 (1 + i )
N
i
+ FV(1 + i )
N
= 0.
To solve for a variable from the equation:
L + Pa
n|i
+ F(1 + i )
n
= 0,
we proceed as before, with payments setup at beginning.
Previous equation is equivalent to
L(1 + i )
n
+ Ps
n|i
+ F = 0,
To change the setting of the payments (either at the beginning of
the period, or at the end of the period), press:
2nd , BGN , 2nd , SET , 2nd , Quit .
If the calculator is setup with payments at the end of the periods,
there is no indicator in the screen. If the calculator is setup with
payments at the beginning of the periods, the indicator BGN
appears in the screen.
c 2009. Miguel A. Arcones. All rights reserved. Manual for SOA Exam FM/CAS Exam 2.
63/82
Chapter 3. Annuities. Section 3.2. Level payment annuities.
Example 14
A company purchases 100 acres of land for $200,000 and agrees to
remit 10 equal annual installments of $27,598 each at the
beginning of the year. What is the annual interest rate on this
loan?
Solution: We solve for i in the equation 200000 = 27598a
10|i
to
get i = 8%. In the calculator, set payments at the beginning of the
period and press:
10 N 27598 PMT 200000 PV 0 FV CPT I/Y
c 2009. Miguel A. Arcones. All rights reserved. Manual for SOA Exam FM/CAS Exam 2.
64/82
Chapter 3. Annuities. Section 3.2. Level payment annuities.
Example 14
A company purchases 100 acres of land for $200,000 and agrees to
remit 10 equal annual installments of $27,598 each at the
beginning of the year. What is the annual interest rate on this
loan?
Solution: We solve for i in the equation 200000 = 27598a
10|i
to
get i = 8%. In the calculator, set payments at the beginning of the
period and press:
10 N 27598 PMT 200000 PV 0 FV CPT I/Y
c 2009. Miguel A. Arcones. All rights reserved. Manual for SOA Exam FM/CAS Exam 2.
65/82
Chapter 3. Annuities. Section 3.2. Level payment annuities.
Example 15
An annuity pays $7000 at the end of the year for 7 years with the
rst payment made 5 years from now. The eective annual rate of
interest is 6.5%. Find the present value of the this annuity.
c 2009. Miguel A. Arcones. All rights reserved. Manual for SOA Exam FM/CAS Exam 2.
66/82
Chapter 3. Annuities. Section 3.2. Level payment annuities.
Example 15
An annuity pays $7000 at the end of the year for 7 years with the
rst payment made 5 years from now. The eective annual rate of
interest is 6.5%. Find the present value of the this annuity.
Solution 1: The cashow of payments is
Payments 7000 7000 7000 7000 7000 7000 7000
Time 5 6 7 8 9 10 11
Using an immediate annuity, (7000)a
7|0.06
is the present value of the
annuity, one period before the rst payment, i.e. (7000)a
7|0.06
is the
present value of the annuity at time 4. So, the present value of the
annuity is
(1.06)
4
(7000)a
7|0.06
= (0.7920937)(7000)(5.582381)
=30952.38.
c 2009. Miguel A. Arcones. All rights reserved. Manual for SOA Exam FM/CAS Exam 2.
67/82
Chapter 3. Annuities. Section 3.2. Level payment annuities.
Example 15
An annuity pays $7000 at the end of the year for 7 years with the
rst payment made 5 years from now. The eective annual rate of
interest is 6.5%. Find the present value of the this annuity.
Solution 2: The cashow of payments is
Payments 7000 7000 7000 7000 7000 7000 7000
Time 5 6 7 8 9 10 11
Using a due annuity, (7000)a
7|0.06
is the present value of the annuity,
at the time of the rst payment, i.e. (7000)a
7|0.06
is the present
value of the annuity at time 5. So, the present value of the annuity
is
(1.06)
5
(7000)a
7|0.06
= (0.7472582)(7000)(5.917324326)
=30952.38.
c 2009. Miguel A. Arcones. All rights reserved. Manual for SOA Exam FM/CAS Exam 2.
68/82
Chapter 3. Annuities. Section 3.2. Level payment annuities.
Example 16
An annuityimmediate pays $7000 at the end of the year for 7
years. The current annual eective rate of interest is 4.5% for the
rst three years and 5.5% thereafter. Find the present value of this
annuity.
Solution: The cashow is
Payments 7000 7000 7000 7000 7000 7000 7000
Time 1 2 3 4 5 6 7
Consider two cashows: one with the rst three payments and
another one with the last four payments. The present value at
time 0 of the rst three payments is (7000)a
3|0.045
. The present
value at time 3 of the last four payments is (7000)a
4|0.055
. The
present value at time 0 of the last four payments is
(7000)(1.045)
3
a
4|0.055
. The present value of the whole annuity is
(7000)a
3|0.045
+ (7000)(1.045)
3
a
4|0.055
=19242.75048 + 21500.85804 = 40743.60852.
c 2009. Miguel A. Arcones. All rights reserved. Manual for SOA Exam FM/CAS Exam 2.
69/82
Chapter 3. Annuities. Section 3.2. Level payment annuities.
Example 16
An annuityimmediate pays $7000 at the end of the year for 7
years. The current annual eective rate of interest is 4.5% for the
rst three years and 5.5% thereafter. Find the present value of this
annuity.
Solution: The cashow is
Payments 7000 7000 7000 7000 7000 7000 7000
Time 1 2 3 4 5 6 7
Consider two cashows: one with the rst three payments and
another one with the last four payments. The present value at
time 0 of the rst three payments is (7000)a
3|0.045
. The present
value at time 3 of the last four payments is (7000)a
4|0.055
. The
present value at time 0 of the last four payments is
(7000)(1.045)
3
a
4|0.055
. The present value of the whole annuity is
(7000)a
3|0.045
+ (7000)(1.045)
3
a
4|0.055
=19242.75048 + 21500.85804 = 40743.60852.
c 2009. Miguel A. Arcones. All rights reserved. Manual for SOA Exam FM/CAS Exam 2.
70/82
Chapter 3. Annuities. Section 3.2. Level payment annuities.
Theorem 6
Consider the cashow
Contributions 0
1
m
1
m

1
m
1
m

1
m
Time (in years) 0
1
m
2
m

m
m
m+1
m

nm
m
The present value of this cashow is
a
(m)
n|i
=
1
n
i
(m)
.
The future value at time n of this cashow is
s
(m)
n|i
=
(1 + i )
n
1
i
(m)
.
Proof: The present value of the cashow is
1
m
a
nm|i
(m)
/m
=
1
m
1

1+
i
(m)
m

mn
i
(m)
m
=
1
n
i
(m)
.
c 2009. Miguel A. Arcones. All rights reserved. Manual for SOA Exam FM/CAS Exam 2.
71/82
Chapter 3. Annuities. Section 3.2. Level payment annuities.
Example 17
Suppose that Arthur takes a mortgage for L at an annual nominal
rate of interest of 7.5% compounded monthly. The loan is paid at
the end of the month with level payments of $1200 for n years.
Suppose at the last minute, Arthur changes the conditions of his
loan so that the payments will biweekly. The duration of the loan
and the eective annual rate remain unchanged. Calculate the
amount of the biweekly payment. Assume that there are 365/7
weeks in a year.
c 2009. Miguel A. Arcones. All rights reserved. Manual for SOA Exam FM/CAS Exam 2.
72/82
Chapter 3. Annuities. Section 3.2. Level payment annuities.
Solution: Let P be the biweekly payment. We have that
L = (12)(1200)a
(12)
n|i
= (12)(1200)
1
n
i
(12)
and
L = (365/14)(P)a
(365/14)
n|i
= (365/14)(P)
1
n
i
(365/14)
Hence,
(12)(1200)
i
(12)
=
(365/14)(P)
i
(365/14)
.
and
P =
(12)(1200)i
(365/14)
i
(12)
(365/14)
=
(12)(1200)(0.07485856348)
(0.075)(365/14)
= 551.2871743,
using that
i
(365/14)
= (365/14)

1 +
0.075
12

12(14/365)
= 7.485856348%.
c 2009. Miguel A. Arcones. All rights reserved. Manual for SOA Exam FM/CAS Exam 2.
73/82
Chapter 3. Annuities. Section 3.2. Level payment annuities.
Theorem 7
Consider the cashow
Contributions
1
m
1
m
1
m

1
m
0
Time (in years) 0
1
m
2
m

nm1
m
nm
m
The present value of this cashow is
a
(m)
n|i
=
1
n
d
(m)
.
The future value at time n of this cashow is
s
(m)
n|i
=
(1 + i )
n
1
d
(m)
.
Proof: The present value of the cashow is
1
m
a
nm|i
(m)
/m
=
1
m
1

1 +
i
(m)
m

mn
d
(m)
m
=
1
n
d
(m)
.
c 2009. Miguel A. Arcones. All rights reserved. Manual for SOA Exam FM/CAS Exam 2.
74/82
Chapter 3. Annuities. Section 3.2. Level payment annuities.
Theorem 8
For integers k, n 1, a
nk|i
= a
n|i
a
k|(1+i )
n
1
.
Proof.
We have that
a
nk|i
a
n|i
=
1
nk
i
1
n
i
= 1 +
n
+
2n
+ +
n(k1)
= a
k|(1+i )
n
1
.
c 2009. Miguel A. Arcones. All rights reserved. Manual for SOA Exam FM/CAS Exam 2.
75/82
Chapter 3. Annuities. Section 3.2. Level payment annuities.
Theorem 8
For integers k, n 1, a
nk|i
= a
n|i
a
k|(1+i )
n
1
.
Proof.
We have that
a
nk|i
a
n|i
=
1
nk
i
1
n
i
= 1 +
n
+
2n
+ +
n(k1)
= a
k|(1+i )
n
1
.
c 2009. Miguel A. Arcones. All rights reserved. Manual for SOA Exam FM/CAS Exam 2.
76/82
Chapter 3. Annuities. Section 3.2. Level payment annuities.
An application of a
nk|i
= a
n|i
a
k|(1+i )
n
1
.
Example 18
Carrie receives 200,000 from a life insurance policy. She uses the
fund to purchase to two dierent annuities, each costing 100,000.
The rst annuity is a 24year annuityimmediate paying k per year
to herself. The second annuity is a 8 year annuityimmediate
paying 2k per year to her boyfriend. Both annuities are based upon
an annual eective interest rate of i , i > 0. Determine i .
Solution: We have that 100000 = ka
24|i
= 2ka
8|i
. So,
2 =
a
24|i
a
8|i
= a
3|(1+i )
8
1
. Using the calculator, we get that
(1 + i )
8
1 = 0.618034 and i = 6.1997%.
c 2009. Miguel A. Arcones. All rights reserved. Manual for SOA Exam FM/CAS Exam 2.
77/82
Chapter 3. Annuities. Section 3.2. Level payment annuities.
An application of a
nk|i
= a
n|i
a
k|(1+i )
n
1
.
Example 18
Carrie receives 200,000 from a life insurance policy. She uses the
fund to purchase to two dierent annuities, each costing 100,000.
The rst annuity is a 24year annuityimmediate paying k per year
to herself. The second annuity is a 8 year annuityimmediate
paying 2k per year to her boyfriend. Both annuities are based upon
an annual eective interest rate of i , i > 0. Determine i .
Solution: We have that 100000 = ka
24|i
= 2ka
8|i
. So,
2 =
a
24|i
a
8|i
= a
3|(1+i )
8
1
. Using the calculator, we get that
(1 + i )
8
1 = 0.618034 and i = 6.1997%.
c 2009. Miguel A. Arcones. All rights reserved. Manual for SOA Exam FM/CAS Exam 2.
78/82
Chapter 3. Annuities. Section 3.2. Level payment annuities.
Theorem 9
For integers k, n 1,
a
nk|i
s
n|i
= a
k|(1+i )
n
1
.
Proof.
By the previous theorem, a
nk|i
= a
n|i
a
k|(1+i )
n
1
. The claim follows
noticing that a
n|i
= (1 + i )
n
s
n|i
and
a
k|(1+i )
n
1
= (1 + i )
n
a
k|(1+i )
n
1
.
c 2009. Miguel A. Arcones. All rights reserved. Manual for SOA Exam FM/CAS Exam 2.
79/82
Chapter 3. Annuities. Section 3.2. Level payment annuities.
Theorem 9
For integers k, n 1,
a
nk|i
s
n|i
= a
k|(1+i )
n
1
.
Proof.
By the previous theorem, a
nk|i
= a
n|i
a
k|(1+i )
n
1
. The claim follows
noticing that a
n|i
= (1 + i )
n
s
n|i
and
a
k|(1+i )
n
1
= (1 + i )
n
a
k|(1+i )
n
1
.
c 2009. Miguel A. Arcones. All rights reserved. Manual for SOA Exam FM/CAS Exam 2.
80/82
Chapter 3. Annuities. Section 3.2. Level payment annuities.
An application of a
nk|i
= s
n|i
a
k|(1+i )
n
1
.
Example 19
The present value of a 4nyear annuityimmediate of 1 at the end
of every year is 16.663. The present value of a 4nyear
annuityimmediate of 1 at the end of every fourth year is 3.924.
Find n and i .
Solution: We know that 16.663 = a
4n|i
and 3.924 = a
n|(1+i )
4
1
.
Dividing the rst equation over the second one, we get that
4.246432 =
16.663
3.924
=
a
4n|i
a
n|(1+i )
4
1
= s
4|i
and i = 4%. From the equation 16.663 = a
4n|4%
, we get n = 7.
c 2009. Miguel A. Arcones. All rights reserved. Manual for SOA Exam FM/CAS Exam 2.
81/82
Chapter 3. Annuities. Section 3.2. Level payment annuities.
An application of a
nk|i
= s
n|i
a
k|(1+i )
n
1
.
Example 19
The present value of a 4nyear annuityimmediate of 1 at the end
of every year is 16.663. The present value of a 4nyear
annuityimmediate of 1 at the end of every fourth year is 3.924.
Find n and i .
Solution: We know that 16.663 = a
4n|i
and 3.924 = a
n|(1+i )
4
1
.
Dividing the rst equation over the second one, we get that
4.246432 =
16.663
3.924
=
a
4n|i
a
n|(1+i )
4
1
= s
4|i
and i = 4%. From the equation 16.663 = a
4n|4%
, we get n = 7.
c 2009. Miguel A. Arcones. All rights reserved. Manual for SOA Exam FM/CAS Exam 2.
82/82
Chapter 3. Annuities. Section 3.2. Level payment annuities.
Since a
n|i
=

n
j =1
(1 +i )
j
as a function of n, a
n|i
increases with n.
As a function of i , i 0, a
n|i
decreases with i . We have that
a
n|0
= n and a
n|
= 0. So, n > a
n|i
> 0 for i > 0; and a
n|i
> n for
0 > i > 1.
It is proved in the manual that
a
n|i
=

j =1
(n + j 1)!
(n 1)!j !
(i )
j 1
.
The rst order Taylor expansion of a
n|i
on i is
a
n|i
n
n(n + 1)
2
i .
The second order Taylor expansion of a
n|i
on i is
a
n|i
n
n(n + 1)
2
i +
n(n + 1)(n + 2)
6
i
2
.
c 2009. Miguel A. Arcones. All rights reserved. Manual for SOA Exam FM/CAS Exam 2.
1/16
Chapter 3. Annuities.
Manual for SOA Exam FM/CAS Exam 2.
Chapter 3. Annuities.
Section 3.3. Level payment perpetuities.
c 2009. Miguel A. Arcones. All rights reserved.
Extract from:
Arcones Manual for the SOA Exam FM/CAS Exam 2,
Financial Mathematics. Fall 2009 Edition,
available at http://www.actexmadriver.com/
c 2009. Miguel A. Arcones. All rights reserved. Manual for SOA Exam FM/CAS Exam 2.
2/16
Chapter 3. Annuities. Section 3.3. Level payment perpetuities.
A perpetuity is a series of payments made forever along equal
intervals of time. By a change of units, we will assume that
intervals have unit length.
The payments can be made either at the beginning or at the end
of the intervals.
A perpetuity has level payments if all payments C
j
, j 0, are
equal.
A perpetuity has nonlevel payments if some payments C
j
are
dierent from other ones.
c 2009. Miguel A. Arcones. All rights reserved. Manual for SOA Exam FM/CAS Exam 2.
3/16
Chapter 3. Annuities. Section 3.3. Level payment perpetuities.
For a perpetuityimmediate the payments are made at the end
of the periods of time, i.e. at the times 1, 2, . . . . So, a
perpetuityimmediate is a cashow of the type:
Payments C
1
C
2
C
3

Time 1 2 3
For a perpetuitydue the payments are made at the beginning of
the intervals of time, i.e. at the times 0, 1, 2, . . . . So, a perpetuity
immediate is a cashow of the type:
Payments C
0
C
1
C
2

Time 0 1 2
c 2009. Miguel A. Arcones. All rights reserved. Manual for SOA Exam FM/CAS Exam 2.
4/16
Chapter 3. Annuities. Section 3.3. Level payment perpetuities.
Theorem 1
The cashow value of a perpetuityimmediate with level payments
of one is
Payments 1 1 1
Time 1 2 3
The present value of a perpetuityimmediate with level payments
of one is
a
|i
= +
2
+ =

1
=
1
i
.
c 2009. Miguel A. Arcones. All rights reserved. Manual for SOA Exam FM/CAS Exam 2.
5/16
Chapter 3. Annuities. Section 3.3. Level payment perpetuities.
Example 1
The present value of a series of payments of 3 at the end of every
eight years, forever, is equal to 9.5. Calculate the eective annual
rate of interest.
Solution: The 8year interest factor is (1 + i )
8
. So, the 8year
eective interest rate is (1 + i )
8
1. We have that
9.5 = (3)a
|(1+i )
8
1
=
3
(1+i )
8
1
and
i =

1 +
3
9.5

1/8
1 = 3.489979511%.
c 2009. Miguel A. Arcones. All rights reserved. Manual for SOA Exam FM/CAS Exam 2.
6/16
Chapter 3. Annuities. Section 3.3. Level payment perpetuities.
Example 1
The present value of a series of payments of 3 at the end of every
eight years, forever, is equal to 9.5. Calculate the eective annual
rate of interest.
Solution: The 8year interest factor is (1 + i )
8
. So, the 8year
eective interest rate is (1 + i )
8
1. We have that
9.5 = (3)a
|(1+i )
8
1
=
3
(1+i )
8
1
and
i =

1 +
3
9.5

1/8
1 = 3.489979511%.
c 2009. Miguel A. Arcones. All rights reserved. Manual for SOA Exam FM/CAS Exam 2.
7/16
Chapter 3. Annuities. Section 3.3. Level payment perpetuities.
Theorem 2
The cashow value of a perpetuitydue with level payments of one
is
Payments 1 1 1
Time 0 1 2
The present value of an perpetuitydue with level payments of one
is
a
|i
= 1 + +
2
+ =
1
1
=
1
d
=
1 + i
i
.
c 2009. Miguel A. Arcones. All rights reserved. Manual for SOA Exam FM/CAS Exam 2.
8/16
Chapter 3. Annuities. Section 3.3. Level payment perpetuities.
Example 2
John uses his retirement fund to buy a perpetuitydue of 20,000
per year based on an annual nominal yield of interest i = 8%
compounded monthly. Find Johns retirement fund.
Solution: Since i
(12)
= 8%, i = 8.299950681%. Johns retirement
fund is worth
(20000)a
|i
= (20000)
1 + i
i
= (20000)
1.08299950681
0.08299950681
= 260963.8554.
c 2009. Miguel A. Arcones. All rights reserved. Manual for SOA Exam FM/CAS Exam 2.
9/16
Chapter 3. Annuities. Section 3.3. Level payment perpetuities.
Example 2
John uses his retirement fund to buy a perpetuitydue of 20,000
per year based on an annual nominal yield of interest i = 8%
compounded monthly. Find Johns retirement fund.
Solution: Since i
(12)
= 8%, i = 8.299950681%. Johns retirement
fund is worth
(20000)a
|i
= (20000)
1 + i
i
= (20000)
1.08299950681
0.08299950681
= 260963.8554.
c 2009. Miguel A. Arcones. All rights reserved. Manual for SOA Exam FM/CAS Exam 2.
10/16
Chapter 3. Annuities. Section 3.3. Level payment perpetuities.
Example 3
A perpetuity pays $1 at the end of every year plus an additional $1
at the end of every second year. The eective rate of interest is
i = 5%. Find the present value of the perpetuity at time 0.
c 2009. Miguel A. Arcones. All rights reserved. Manual for SOA Exam FM/CAS Exam 2.
11/16
Chapter 3. Annuities. Section 3.3. Level payment perpetuities.
Solution: The cashow of the perpetuity is
Payments 1 2 1 2 1 2
Time 1 2 3 4 5 6
This cashow can be decomposed into the cashows:
Payments 1 1 1
Time 1 3 5
and
Payments 2 2 2
Time 2 4 6
The present value at time 0 of the rst part of the cashow is
1+i
(1+i )
2
1
. The present value at time 0 of the second part of the
cashow is
2
(1+i )
2
1
. Hence, the present value at time of the total
cashow is
1 + i
(1 + i )
2
1
+
2
(1 + i )
2
1
=
3 + i
i (2 + i )
=
3 + 0.05
0.05(2 + 0.05)
= 29.7561.
c 2009. Miguel A. Arcones. All rights reserved. Manual for SOA Exam FM/CAS Exam 2.
12/16
Chapter 3. Annuities. Section 3.3. Level payment perpetuities.
Theorem 3
The cashow value of the perpetuityimmediate with level
payments of one per year and m payments per year is
Payments 0
1
m
1
m

Time 0
1
m
2
m

The present value of the previous perpetuityimmediate is
a
(m)
|i
=
1
m

1/m
+
2/m
+

=
1
m

1/m

1
1
1/m
=
1
m(
1/m
1)
=
1
i
(m)
.
c 2009. Miguel A. Arcones. All rights reserved. Manual for SOA Exam FM/CAS Exam 2.
13/16
Chapter 3. Annuities. Section 3.3. Level payment perpetuities.
Example 4
A perpetuity pays x at the end of each month. The nominal
annual rate of interest compounded monthly is i
(12)
. Calculate the
percentage of increase in the value of this perpetuity if the nominal
annual rate of interest compounded monthly decreases by 10%.
Solution: At the rate i
(12)
, the present value of the perpetuity is
x
i
(12)
. At the rate i
(12)
(0.9), the present value of the perpetuity is
x
i
(12)
(0.9)
. The percentage of increase in the value of this perpetuity
is
x
i
(12)
(0.9)

x
i
(12)
x
i
(12)
=
1
0.9
1 = 11.11111111%.
c 2009. Miguel A. Arcones. All rights reserved. Manual for SOA Exam FM/CAS Exam 2.
14/16
Chapter 3. Annuities. Section 3.3. Level payment perpetuities.
Example 4
A perpetuity pays x at the end of each month. The nominal
annual rate of interest compounded monthly is i
(12)
. Calculate the
percentage of increase in the value of this perpetuity if the nominal
annual rate of interest compounded monthly decreases by 10%.
Solution: At the rate i
(12)
, the present value of the perpetuity is
x
i
(12)
. At the rate i
(12)
(0.9), the present value of the perpetuity is
x
i
(12)
(0.9)
. The percentage of increase in the value of this perpetuity
is
x
i
(12)
(0.9)

x
i
(12)
x
i
(12)
=
1
0.9
1 = 11.11111111%.
c 2009. Miguel A. Arcones. All rights reserved. Manual for SOA Exam FM/CAS Exam 2.
15/16
Chapter 3. Annuities. Section 3.3. Level payment perpetuities.
Theorem 4
The cashow value of the perpetuitydue with level payments of
one per year and m payments per year is
Payments
1
m
1
m
1
m

Time 0
1
m
2
m

The present value of an perpetuitydue with level payments of one
per year and m payments per year is
a
(m)
|i
=
1
m

1 +
1/m
+
2/m
+

=
1
m

1
1
1/m
=
1
d
(m)
.
c 2009. Miguel A. Arcones. All rights reserved. Manual for SOA Exam FM/CAS Exam 2.
16/16
Chapter 3. Annuities. Section 3.3. Level payment perpetuities.
Example 5
The present value of a series of payments of $500 at the beginning
of every month years, forever, is equal to $10000. Calculate the
nominal annual discount rate compounded monthly.
Solution: We have that
10000 = 500
1
d
(12)
.
and d
(12)
=
500
10000
= 0.05.
c 2009. Miguel A. Arcones. All rights reserved. Manual for SOA Exam FM/CAS Exam 2.
1/37
Chapter 3. Annuities.
Manual for SOA Exam FM/CAS Exam 2.
Chapter 3. Annuities.
Section 3.4. Non-level payment annuities and perpetuities.
c 2009. Miguel A. Arcones. All rights reserved.
Extract from:
Arcones Manual for the SOA Exam FM/CAS Exam 2,
Financial Mathematics. Fall 2009 Edition,
available at http://www.actexmadriver.com/
c 2009. Miguel A. Arcones. All rights reserved. Manual for SOA Exam FM/CAS Exam 2.
2/37
Chapter 3. Annuities. Section 3.4. Non-level payment annuities and perpetuities.
Theorem 1
(Geometric Annuity) Let i , r > 1. The present value of the
annuity
Payments 1 1 + r (1 + r )
2
(1 + r )
n1
Time 1 2 3 n
is
(Ga)
n|i ,r
=
1
1 + r
a
n|
i r
1+r
=
1
1 + i
a
n|
i r
1+r
.
c 2009. Miguel A. Arcones. All rights reserved. Manual for SOA Exam FM/CAS Exam 2.
3/37
Chapter 3. Annuities. Section 3.4. Non-level payment annuities and perpetuities.
Proof: Let i

=
i r
1+r
. Then,
1+i
1+r
= 1 + i

. The present value of the


cashow at time 0 is
n

j =1
(1 + r )
j 1
(1 + i )
j
=
1
1 + r
n

j =1

1 + i
1 + r

j
=
1
1 + r
n

j =1

1 + i

j
=
1
1 + r
a
n|i
.
Using that a
n|i
= (1 + i )a
n|i
, we get that
1
1 + r
a
n|i
=
1
1 + r
1
1 + i

a
n|i
=
1
1 + i
a
n|i
.
c 2009. Miguel A. Arcones. All rights reserved. Manual for SOA Exam FM/CAS Exam 2.
4/37
Chapter 3. Annuities. Section 3.4. Non-level payment annuities and perpetuities.
Example 1
An annuity provides for 10 annuals payments, the rst payment a
year hence being $2600. The payments increase in such a way that
each payment is 3% greater than the previous one. The annual
eective rate of interest is 4%. Find the present value of this
annuity.
Solution: The cashow is
Payments 2600 2600(1.03) 2600(1.03)
2
2600(1.03)
9
Time 1 2 3 10
The present value at time 0 of the annuity is
(2600) (Ga)
n|i ,r
=
2600
1.03
a
10

|
0.040.03
1.03
= 2524.271845a
10|0.9708737864%
=23945.54454.
c 2009. Miguel A. Arcones. All rights reserved. Manual for SOA Exam FM/CAS Exam 2.
5/37
Chapter 3. Annuities. Section 3.4. Non-level payment annuities and perpetuities.
Example 1
An annuity provides for 10 annuals payments, the rst payment a
year hence being $2600. The payments increase in such a way that
each payment is 3% greater than the previous one. The annual
eective rate of interest is 4%. Find the present value of this
annuity.
Solution: The cashow is
Payments 2600 2600(1.03) 2600(1.03)
2
2600(1.03)
9
Time 1 2 3 10
The present value at time 0 of the annuity is
(2600) (Ga)
n|i ,r
=
2600
1.03
a
10

|
0.040.03
1.03
= 2524.271845a
10|0.9708737864%
=23945.54454.
c 2009. Miguel A. Arcones. All rights reserved. Manual for SOA Exam FM/CAS Exam 2.
6/37
Chapter 3. Annuities. Section 3.4. Non-level payment annuities and perpetuities.
Example 2
An annuity provides for 20 annuals payments, the rst payment a
year hence being $4500. The payments increase in such a way that
each payment is 4.5% greater than the previous one. The annual
eective rate of interest is 4.5%. Find the present value of this
annuity.
Solution: The cashow is
Payments 4500 4500(1.045) 4500(1.045)
2
4500(1.045)
19
Time 1 2 3 20
The present value at time 0 of the annuity is
(4500) (Ga)
n|i ,r
= (4500)
1
1.045
a
20

|
0.0450.045
1.045
= 4500
20
1.045
= 86124.40191.
c 2009. Miguel A. Arcones. All rights reserved. Manual for SOA Exam FM/CAS Exam 2.
7/37
Chapter 3. Annuities. Section 3.4. Non-level payment annuities and perpetuities.
Example 2
An annuity provides for 20 annuals payments, the rst payment a
year hence being $4500. The payments increase in such a way that
each payment is 4.5% greater than the previous one. The annual
eective rate of interest is 4.5%. Find the present value of this
annuity.
Solution: The cashow is
Payments 4500 4500(1.045) 4500(1.045)
2
4500(1.045)
19
Time 1 2 3 20
The present value at time 0 of the annuity is
(4500) (Ga)
n|i ,r
= (4500)
1
1.045
a
20

|
0.0450.045
1.045
= 4500
20
1.045
= 86124.40191.
c 2009. Miguel A. Arcones. All rights reserved. Manual for SOA Exam FM/CAS Exam 2.
8/37
Chapter 3. Annuities. Section 3.4. Non-level payment annuities and perpetuities.
Example 3
Chris makes annual deposits into a bank account at the beginning
of each year for 10 years. Chris initial deposit is equal to 100, with
each subsequent deposit k% greater than the previous year deposit.
The bank credits interest at an annual eective rate of 4.5%. At
the end of 10 years, the accumulated amount in Chris account is
equal to 1657.22. Calculate k.
Solution: The cashow is
Payments 100 100(1 + r ) 100(1 + r )
2
100(1 + r )
9
Time 0 1 2 9
The accumulated amount at the end of 10 years is
1657.22 = (100)
1
1+i
a
n|
i r
1+r
(1 + i )
11
. Hence,
a
10|
0.045r
1+r
=
(1657.22)(1.045)
10
100
= 10.67129833,
0.045r
1+r
= 0.0141511755, r =
0.045+0.0141511755
10.0141511755
= 6% and k = 6.
c 2009. Miguel A. Arcones. All rights reserved. Manual for SOA Exam FM/CAS Exam 2.
9/37
Chapter 3. Annuities. Section 3.4. Non-level payment annuities and perpetuities.
Example 3
Chris makes annual deposits into a bank account at the beginning
of each year for 10 years. Chris initial deposit is equal to 100, with
each subsequent deposit k% greater than the previous year deposit.
The bank credits interest at an annual eective rate of 4.5%. At
the end of 10 years, the accumulated amount in Chris account is
equal to 1657.22. Calculate k.
Solution: The cashow is
Payments 100 100(1 + r ) 100(1 + r )
2
100(1 + r )
9
Time 0 1 2 9
The accumulated amount at the end of 10 years is
1657.22 = (100)
1
1+i
a
n|
i r
1+r
(1 + i )
11
. Hence,
a
10|
0.045r
1+r
=
(1657.22)(1.045)
10
100
= 10.67129833,
0.045r
1+r
= 0.0141511755, r =
0.045+0.0141511755
10.0141511755
= 6% and k = 6.
c 2009. Miguel A. Arcones. All rights reserved. Manual for SOA Exam FM/CAS Exam 2.
10/37
Chapter 3. Annuities. Section 3.4. Non-level payment annuities and perpetuities.
Corollary 1
The present value of the perpetuity
Payments 1 1 + r (1 + r )
2
(1 + r )
n1

Time 1 2 3 n
is
(Ga)
|i ,r
=

1
i r
if i > r ,
if i r .
Proof.
If i > r , (Ga)
|i ,r
=
1
1+r
a
|
i r
1+r
=
1
1+r
1
i r
1+r
=
1
i r
.
If i r , (Ga)
|i ,r
=
1
1+r
a
|
i r
1+r
= .
c 2009. Miguel A. Arcones. All rights reserved. Manual for SOA Exam FM/CAS Exam 2.
11/37
Chapter 3. Annuities. Section 3.4. Non-level payment annuities and perpetuities.
Corollary 1
The present value of the perpetuity
Payments 1 1 + r (1 + r )
2
(1 + r )
n1

Time 1 2 3 n
is
(Ga)
|i ,r
=

1
i r
if i > r ,
if i r .
Proof.
If i > r , (Ga)
|i ,r
=
1
1+r
a
|
i r
1+r
=
1
1+r
1
i r
1+r
=
1
i r
.
If i r , (Ga)
|i ,r
=
1
1+r
a
|
i r
1+r
= .
c 2009. Miguel A. Arcones. All rights reserved. Manual for SOA Exam FM/CAS Exam 2.
12/37
Chapter 3. Annuities. Section 3.4. Non-level payment annuities and perpetuities.
Example 4
An perpetuityimmediate provides annual payments. The rst
payment of 13000 is one year from now. Each subsequent payment
is 3.5% more than the one preceding it. The annual eective rate
of interest is i = 6%. Find the present value of this perpetuity.
Solution: The present value is
(13000) (Ga)
|i ,r
= (13000)
1
i r
= (13000)
1
0.060.035
= 520000.
c 2009. Miguel A. Arcones. All rights reserved. Manual for SOA Exam FM/CAS Exam 2.
13/37
Chapter 3. Annuities. Section 3.4. Non-level payment annuities and perpetuities.
Example 4
An perpetuityimmediate provides annual payments. The rst
payment of 13000 is one year from now. Each subsequent payment
is 3.5% more than the one preceding it. The annual eective rate
of interest is i = 6%. Find the present value of this perpetuity.
Solution: The present value is
(13000) (Ga)
|i ,r
= (13000)
1
i r
= (13000)
1
0.060.035
= 520000.
c 2009. Miguel A. Arcones. All rights reserved. Manual for SOA Exam FM/CAS Exam 2.
14/37
Chapter 3. Annuities. Section 3.4. Non-level payment annuities and perpetuities.
Theorem 2
(Increasing Annuity) The present value of the annuity
Payments 1 2 3 n
Time 1 2 3 n
is
(Ia)
n|i
=
a
n|i
(1 + i ) n
n
i
=
a
n|i
n
n
i
.
The accumulated value of this cashow at time n is
(Is)
n|i
=
s
n|i
n
i
.
c 2009. Miguel A. Arcones. All rights reserved. Manual for SOA Exam FM/CAS Exam 2.
15/37
Chapter 3. Annuities. Section 3.4. Non-level payment annuities and perpetuities.
Proof:
The cashow is the sum of the cashows:
cashow 1 1 1 1 1 1
cashow 2 0 1 1 1 1
cashow 3 0 0 1 1 1


cashow n 0 0 0 0 1
Time 1 2 3 n 1 n
The future value at time n of all these cashows is
(Is)
n|i
=
n

j =1
s
j |i
=
n

j =1
(1 + i )
j
1
i
=
s
n|i
n
i
.
c 2009. Miguel A. Arcones. All rights reserved. Manual for SOA Exam FM/CAS Exam 2.
16/37
Chapter 3. Annuities. Section 3.4. Non-level payment annuities and perpetuities.
In the calculator, it is possible to nd a
n|i
n
n
in one
computation. With payments setup at the beginning, we enter n
in N , i in I/Y , 1 in PMT and n in FV . We ask the
calculator to compute PV .
c 2009. Miguel A. Arcones. All rights reserved. Manual for SOA Exam FM/CAS Exam 2.
17/37
Chapter 3. Annuities. Section 3.4. Non-level payment annuities and perpetuities.
Example 5
Find the present value at time 0 of an annuityimmediate such that
the payments start at 1, each payment thereafter increases by 1
until reaching 10, and they remain at that level until 25 payments
in total are made. The eective annual rate of interest is 4%.
Solution: The cashow is
Payments 1 2 3 10 10 10
Time 1 2 3 10 11 25
The present value at time 0 of the perpetuity is
(Ia)
10|0.04
+ (1 + 0.04)
10
10a
15|0.04
=
a
n|4%
10(1 + 0.04)
10
0.04
+ (1 + 0.04)
10
(10)a
15|0.04
=41.99224806 + 75.11184164 = 117.1040897.
c 2009. Miguel A. Arcones. All rights reserved. Manual for SOA Exam FM/CAS Exam 2.
18/37
Chapter 3. Annuities. Section 3.4. Non-level payment annuities and perpetuities.
Example 5
Find the present value at time 0 of an annuityimmediate such that
the payments start at 1, each payment thereafter increases by 1
until reaching 10, and they remain at that level until 25 payments
in total are made. The eective annual rate of interest is 4%.
Solution: The cashow is
Payments 1 2 3 10 10 10
Time 1 2 3 10 11 25
The present value at time 0 of the perpetuity is
(Ia)
10|0.04
+ (1 + 0.04)
10
10a
15|0.04
=
a
n|4%
10(1 + 0.04)
10
0.04
+ (1 + 0.04)
10
(10)a
15|0.04
=41.99224806 + 75.11184164 = 117.1040897.
c 2009. Miguel A. Arcones. All rights reserved. Manual for SOA Exam FM/CAS Exam 2.
19/37
Chapter 3. Annuities. Section 3.4. Non-level payment annuities and perpetuities.
Theorem 3
(Decreasing Annuity) The present value of the annuity
Payments n n 1 n 2 1
Time 1 2 3 n
is
(Da)
n|i
=
n a
n|i
i
.
The accumulated value of this cashow at time n is
(Ds)
n|i
=
n(1 + i )
n
s
n|i
i
.
c 2009. Miguel A. Arcones. All rights reserved. Manual for SOA Exam FM/CAS Exam 2.
20/37
Chapter 3. Annuities. Section 3.4. Non-level payment annuities and perpetuities.
Proof.
The cashow is the sum of the cashows:
cashow 1 1 1 1 1 1
cashow 2 1 1 1 1 0
cashow 3 1 1 1 0 0


cashow n 1 0 0 0 0
Time 1 2 3 n 1 n
The present value at time 0 of all these cashows is
n

j =1
a
j |i
=
n

j =1
1 (1 + i )
j
i
=
n a
n|i
i
.
c 2009. Miguel A. Arcones. All rights reserved. Manual for SOA Exam FM/CAS Exam 2.
21/37
Chapter 3. Annuities. Section 3.4. Non-level payment annuities and perpetuities.
In the calculator, it is possible to nd n(1 + i )
n
s
n|i
in one
computation. We enter n in N , i in I/Y , 1 in PMT and n in
PV . We ask the calculator to compute FV .
c 2009. Miguel A. Arcones. All rights reserved. Manual for SOA Exam FM/CAS Exam 2.
22/37
Chapter 3. Annuities. Section 3.4. Non-level payment annuities and perpetuities.
Example 6
Find the present value of a 15year decreasing annuityimmediate
paying 150000 the rst year and decreasing by 10000 each year
thereafter. The eective annual interest rate of 4.5%.
Solution: The cashow of payments is
Payments (15)(10000) (14)(10000) (1)(10000)
Time 1 2 15
The present value of this cashow is
(10000) (Da)
15|4.5%
= (10000)
15 a
15|4.5%
0.045
= 946767.616.
c 2009. Miguel A. Arcones. All rights reserved. Manual for SOA Exam FM/CAS Exam 2.
23/37
Chapter 3. Annuities. Section 3.4. Non-level payment annuities and perpetuities.
Example 6
Find the present value of a 15year decreasing annuityimmediate
paying 150000 the rst year and decreasing by 10000 each year
thereafter. The eective annual interest rate of 4.5%.
Solution: The cashow of payments is
Payments (15)(10000) (14)(10000) (1)(10000)
Time 1 2 15
The present value of this cashow is
(10000) (Da)
15|4.5%
= (10000)
15 a
15|4.5%
0.045
= 946767.616.
c 2009. Miguel A. Arcones. All rights reserved. Manual for SOA Exam FM/CAS Exam 2.
24/37
Chapter 3. Annuities. Section 3.4. Non-level payment annuities and perpetuities.
Theorem 4
(Increasing Perpetuity) The present value of the perpetuity
Payments 1 2 3
Time 1 2 3
is (Ia)
|i
=
1+i
i
2
.
c 2009. Miguel A. Arcones. All rights reserved. Manual for SOA Exam FM/CAS Exam 2.
25/37
Chapter 3. Annuities. Section 3.4. Non-level payment annuities and perpetuities.
Proof:
The cashow is the sum of the innitely many cashows:
cashow 1 1 1 1 1 1
cashow 2 0 1 1 1 1
cashow 3 0 0 1 1 1


Time 1 2 3 n 1 n
The present value at time 0 of all these cashows is

j =1
1
i
(1 + i )
(j 1)
=
1
i
1
1
1
1+i
=
1 + i
i
2
.
c 2009. Miguel A. Arcones. All rights reserved. Manual for SOA Exam FM/CAS Exam 2.
26/37
Chapter 3. Annuities. Section 3.4. Non-level payment annuities and perpetuities.
Example 7
An investor is considering the purchase of 500 ordinary shares in a
company. This company pays dividends at the end of each year.
The next payment is one year from now and it is $3 per share. The
investor believes that each subsequent payment per share will
increase by $1 each year forever. Calculate the present value of this
dividend stream at a rate of interest of 6.5% per annum eective.
c 2009. Miguel A. Arcones. All rights reserved. Manual for SOA Exam FM/CAS Exam 2.
27/37
Chapter 3. Annuities. Section 3.4. Non-level payment annuities and perpetuities.
Solution: The cashow of payments is
Payments 3 4 5
Time 1 2 3
The cashow is the sum of the following cashows
Payments 2 2 2
Payments 1 2 3
Time 1 2 3
Hence, the present value of this dividend stream is
(500)(2)a
|6.5%
+ (500)(1) (Ia)
|6.5%
=(500)(2)
1
0.065
+ (500)(1)
1.065
0.065
2
=15384.6154 + 126035.503 = 141420.118.
c 2009. Miguel A. Arcones. All rights reserved. Manual for SOA Exam FM/CAS Exam 2.
28/37
Chapter 3. Annuities. Section 3.4. Non-level payment annuities and perpetuities.
Theorem 5
(Rainbow Immediate) The present value of the annuity
Payments 1 2 n 1 n n 1 2 1
Time 1 2 n 1 n n + 1 2n 2 2n 1
is a
n|i
a
n|i
.
c 2009. Miguel A. Arcones. All rights reserved. Manual for SOA Exam FM/CAS Exam 2.
29/37
Chapter 3. Annuities. Section 3.4. Non-level payment annuities and perpetuities.
Proof: The value of the annuity is
(Ia)
n|i
+
n
(Da)
n1|i
=
a
n|i
n
n
i
+

n
(n1a
n1|i
)
i
=
a
n|i

n
(1+a
n1|i
)
i
=
(1+i )a
n|i

n
(1+i )a
n|i
i
=
(1+i )a
n|i
(1
n
)
i
= (1 + i )a
n

|i
a
n|i
= a
n|i
a
n|i
c 2009. Miguel A. Arcones. All rights reserved. Manual for SOA Exam FM/CAS Exam 2.
30/37
Chapter 3. Annuities. Section 3.4. Non-level payment annuities and perpetuities.
Example 8
A 15 year annuity pays 1000 at the end of year 1 and increases by
1000 each year until the payment is 8000 at the end of year 8.
Payments then decrease by 1000 each year until a payment of 1000
is paid at the end of year 15. The annual eective interest rate of
6.5%. Compute the present value of this annuity.
Solution: The cashow is
Paym. 1000 2000 7000 8000 7000 2000 1000
Time 1 2 7 8 9 14 15
The present value is
1000a
8|i
a
8|i
= (1000)(6.08875096)(6.48451977) = 39482.626.
c 2009. Miguel A. Arcones. All rights reserved. Manual for SOA Exam FM/CAS Exam 2.
31/37
Chapter 3. Annuities. Section 3.4. Non-level payment annuities and perpetuities.
Example 8
A 15 year annuity pays 1000 at the end of year 1 and increases by
1000 each year until the payment is 8000 at the end of year 8.
Payments then decrease by 1000 each year until a payment of 1000
is paid at the end of year 15. The annual eective interest rate of
6.5%. Compute the present value of this annuity.
Solution: The cashow is
Paym. 1000 2000 7000 8000 7000 2000 1000
Time 1 2 7 8 9 14 15
The present value is
1000a
8|i
a
8|i
= (1000)(6.08875096)(6.48451977) = 39482.626.
c 2009. Miguel A. Arcones. All rights reserved. Manual for SOA Exam FM/CAS Exam 2.
32/37
Chapter 3. Annuities. Section 3.4. Non-level payment annuities and perpetuities.
Theorem 6
(Paused Rainbow Immediate) The present value of the annuity
Paym. 1 2 n 1 n n n 1 2 1
Time 1 2 n 1 n n + 1 n + 2 2n 1 2n
is a
n+1|i
a
n+1|i
.
Proof: The present value of the annuity is
(Ia)
n|i
+
n
(Da)
n

|i
=
a
n|i
n
n
i
+

n
(na
n|i
)
i
=
a
n|i

n
a
n|i
i
=
(1+i )a
n|i
(1ia
n|i
)a
n|i
i
= a
n|i
(1 + a
n|i
) = a
n|i
a
n+1|i
.
c 2009. Miguel A. Arcones. All rights reserved. Manual for SOA Exam FM/CAS Exam 2.
33/37
Chapter 3. Annuities. Section 3.4. Non-level payment annuities and perpetuities.
Theorem 6
(Paused Rainbow Immediate) The present value of the annuity
Paym. 1 2 n 1 n n n 1 2 1
Time 1 2 n 1 n n + 1 n + 2 2n 1 2n
is a
n+1|i
a
n+1|i
.
Proof: The present value of the annuity is
(Ia)
n|i
+
n
(Da)
n

|i
=
a
n|i
n
n
i
+

n
(na
n|i
)
i
=
a
n|i

n
a
n|i
i
=
(1+i )a
n|i
(1ia
n|i
)a
n|i
i
= a
n|i
(1 + a
n|i
) = a
n|i
a
n+1|i
.
c 2009. Miguel A. Arcones. All rights reserved. Manual for SOA Exam FM/CAS Exam 2.
34/37
Chapter 3. Annuities. Section 3.4. Non-level payment annuities and perpetuities.
Example 9
A 20 year annuity pays 5000 at the end of year 1 and increases by
5000 each year until the payment is 50000 at the end of year 10.
The payment remains constant for one year. Payments then
decrease by 5000 each year until a payment of 5000 is paid at the
end of year 20. The annual eective interest rate of 4%. Compute
the present value of this annuity.
Solution: The cashow is
Paym. (1)(5000) (10)5000 (10)5000 (1)(5000)
Time 1 10 11 20
The present value is
5000a
11|i
a
10|i
= (5000)(8.11089578)(9.11089578) = 369487.631
c 2009. Miguel A. Arcones. All rights reserved. Manual for SOA Exam FM/CAS Exam 2.
35/37
Chapter 3. Annuities. Section 3.4. Non-level payment annuities and perpetuities.
Example 9
A 20 year annuity pays 5000 at the end of year 1 and increases by
5000 each year until the payment is 50000 at the end of year 10.
The payment remains constant for one year. Payments then
decrease by 5000 each year until a payment of 5000 is paid at the
end of year 20. The annual eective interest rate of 4%. Compute
the present value of this annuity.
Solution: The cashow is
Paym. (1)(5000) (10)5000 (10)5000 (1)(5000)
Time 1 10 11 20
The present value is
5000a
11|i
a
10|i
= (5000)(8.11089578)(9.11089578) = 369487.631
c 2009. Miguel A. Arcones. All rights reserved. Manual for SOA Exam FM/CAS Exam 2.
36/37
Chapter 3. Annuities. Section 3.4. Non-level payment annuities and perpetuities.
Theorem 7
The present value of the annuity
Payments 0
1
m
1
m

1
m
2
m

2
m

n
m
Time 0
1
m
2
m
1 1 +
1
m
2 n
is
(Ia)
(m)
n|i
=
a
n|i
n
n
i
(m)
.
For the proof, see the manual.
c 2009. Miguel A. Arcones. All rights reserved. Manual for SOA Exam FM/CAS Exam 2.
37/37
Chapter 3. Annuities. Section 3.4. Non-level payment annuities and perpetuities.
Theorem 8
The present value of the annuity
Payments 0
1
m
2
2
m
2
3
m
2

n
m
2
Time 0
1
m
2
m
3
m
n
is

I
(m)
a

(m)
n|i
=
a
(m)
n|i
n
n
i
(m)
.
For the proof, see the manual.
c 2009. Miguel A. Arcones. All rights reserved. Manual for SOA Exam FM/CAS Exam 2.
1/15
Chapter 3. Annuities.
Manual for SOA Exam FM/CAS Exam 2.
Chapter 3. Annuities.
Section 3.5. Continuous annuities.
c 2009. Miguel A. Arcones. All rights reserved.
Extract from:
Arcones Manual for the SOA Exam FM/CAS Exam 2,
Financial Mathematics. Fall 2009 Edition,
available at http://www.actexmadriver.com/
c 2009. Miguel A. Arcones. All rights reserved. Manual for SOA Exam FM/CAS Exam 2.
2/15
Chapter 3. Annuities. Section 3.5. Continuous annuities.
Continuous annuities
Annuities with length of period very small are approximately
continuous annuities.
For example, the cashows
Inow 0
1
m
1
m

1
m
1
m

1
m
Time 0
1
m
2
m

m
m
m+1
m

nm
m
and
Inow
1
m
1
m
1
m

1
m
1
m

1
m
0
Time 0
1
m
2
m

m
m
m+1
m

nm1
m
nm
m
tend to a continuous cashow with rate C(t) = 1, 0 t n, as
m .
c 2009. Miguel A. Arcones. All rights reserved. Manual for SOA Exam FM/CAS Exam 2.
3/15
Chapter 3. Annuities. Section 3.5. Continuous annuities.
Theorem 1
The present value of a continuous annuity with rate C(t) = 1,
0 t n, is
a
n

|i
=
1
n

.
The future value at time n of a continuous annuity with rate of
one is
s
n|i
=
(1 + i )
n
1

.
Proof: We have that

n
0

t
dt =
e
t ln
ln

n
0
=
1
n
ln
=
1 e
n

=
1
n

.
c 2009. Miguel A. Arcones. All rights reserved. Manual for SOA Exam FM/CAS Exam 2.
4/15
Chapter 3. Annuities. Section 3.5. Continuous annuities.
Recall
Theorem 2
Consider the cashow
Inow 0
1
m
1
m

1
m
1
m

1
m
Time 0
1
m
2
m

m
m
m+1
m

nm
m
Then, the present value of this cashow is
a
(m)
n|i
=
1
n
i
(m)
,
where i
(m)
is the nominal annual rate of interest convertible m
times at year. The future value at time n of this cashow is
s
(m)
n|i
=
(1 + i )
n
1
i
(m)
.
c 2009. Miguel A. Arcones. All rights reserved. Manual for SOA Exam FM/CAS Exam 2.
5/15
Chapter 3. Annuities. Section 3.5. Continuous annuities.
Recall
Theorem 3
Consider the cashow
Inow
1
m
1
m
1
m

1
m
1
m

1
m
0
Time 0
1
m
2
m

m
m
m+1
m

nm1
m
nm
m
The present value of this cashow is
a
(m)
n|i
=
1
n
d
(m)
,
where d
(m)
is the nominal annual rate of discount convertible m
times at year. The future value at time n of this cashow is
s
(m)
n|i
=
(1 + i )
n
1
d
(m)
.
c 2009. Miguel A. Arcones. All rights reserved. Manual for SOA Exam FM/CAS Exam 2.
6/15
Chapter 3. Annuities. Section 3.5. Continuous annuities.
Theorem 4
a
n|i
= lim
m
a
(m)
n|i
= lim
m
a
(m)
n|i
and
s
n|i
= lim
m
s
(m)
n|i
= lim
m
s
(m)
n|i
.
Proof:
lim
m
a
(m)
n|i
= lim
m
1
n
i
(m)
=
1
n

= a
n|i
.
lim
m
a
(m)
n|i
= lim
m
1
n
d
(m)
=
1
n

= a
n|i
.
c 2009. Miguel A. Arcones. All rights reserved. Manual for SOA Exam FM/CAS Exam 2.
7/15
Chapter 3. Annuities. Section 3.5. Continuous annuities.
Given a real number x, the integer part of x is the largest integer
smaller than or equal to x, i.e. the integer k satisfying
k x < k + 1. The integer part of x is noted by [x]. Next
theorem considers the continuous annuity with rate equal to the
integer part.
Theorem 5
The present value of a continuous annuity with C(t) = [t],
0 t n, is
(I a)
n|i
=
a
n|i
n
n

.
c 2009. Miguel A. Arcones. All rights reserved. Manual for SOA Exam FM/CAS Exam 2.
8/15
Chapter 3. Annuities. Section 3.5. Continuous annuities.
Proof.
The present value of the continuous cashow is
(I a)
n|i
=

n
0
C(s)
s
ds =
n

j =1

j
j 1
j
s
ds =
n

j =1
j (
j

j 1
)
ln
=
n

j =1
j (e
j
e
(j 1)
)

=
n

j =1
j (e
(j 1)
e
j
)

=
1 + e

+ + e
(n1)
ne
n

.
Now, e

= and
1 + e

+ + e
(n1)
= 1 + + +
n1
=
1
n
1
= a
n|i
.
So, (I a)
n|i
=
a
n|i
n
n

.
c 2009. Miguel A. Arcones. All rights reserved. Manual for SOA Exam FM/CAS Exam 2.
9/15
Chapter 3. Annuities. Section 3.5. Continuous annuities.
Recall
Theorem 6
The present value of the annuity
Payments 0
1
m
1
m

1
m
2
m

2
m

n
m
Time 0
1
m
2
m
1 1 +
1
m
2 n
is
(Ia)
(m)
n|i
=
a
n|i
n
n
i
(m)
.
c 2009. Miguel A. Arcones. All rights reserved. Manual for SOA Exam FM/CAS Exam 2.
10/15
Chapter 3. Annuities. Section 3.5. Continuous annuities.
Theorem 7
(I a)
n|i
= lim
m
(Ia)
(m)
n

|i
.
Proof.
We have that
lim
m
(Ia)
(m)
n|i
= lim
m
a
n|i
n
n
i
(m)
=
a
n|i
n
n

= (I a)
n|i
.
c 2009. Miguel A. Arcones. All rights reserved. Manual for SOA Exam FM/CAS Exam 2.
11/15
Chapter 3. Annuities. Section 3.5. Continuous annuities.
Theorem 7
(I a)
n|i
= lim
m
(Ia)
(m)
n

|i
.
Proof.
We have that
lim
m
(Ia)
(m)
n|i
= lim
m
a
n|i
n
n
i
(m)
=
a
n|i
n
n

= (I a)
n|i
.
c 2009. Miguel A. Arcones. All rights reserved. Manual for SOA Exam FM/CAS Exam 2.
12/15
Chapter 3. Annuities. Section 3.5. Continuous annuities.
Theorem 8
The present value of a continuous annuity with C(t) = t,
0 t n, is

I a

n|i
=
a
n|i
n
n

.
Proof.
By the change of variables x = s,

I a

n|i
=

n
0
C(s)
s
ds =

n
0
s
s
ds =

n
0
se
s
ds
=
2

n
0
xe
x
dx =
2
(1 x)e
x

n
0
=
2

2
e
n
(1 + n) =
1 e
n

2

ne
n

=
a
n|i
n
n

.
c 2009. Miguel A. Arcones. All rights reserved. Manual for SOA Exam FM/CAS Exam 2.
13/15
Chapter 3. Annuities. Section 3.5. Continuous annuities.
Recall
Theorem 9
The present value of the annuity
Payments 0
1
m
2
2
m
2
3
m
2

n
m
2
Time 0
1
m
2
m
3
m
n
is

I
(m)
a

(m)
n|i
=
a
(m)
n|i
n
n
i
(m)
.
c 2009. Miguel A. Arcones. All rights reserved. Manual for SOA Exam FM/CAS Exam 2.
14/15
Chapter 3. Annuities. Section 3.5. Continuous annuities.
Theorem 10

I a

n|i
= lim
m

I
(m)
a

(m)
n|i
.
Proof.
lim
m

I
(m)
a

(m)
n|i
= lim
m
a
(m)
n|i
n
n
i
(m)
= lim
m
1
n
i
(m)
d
(m)

n
n
i
(m)
=
1
n

2

n
n

=
a
n|i
n
n

I a

n|i
.
c 2009. Miguel A. Arcones. All rights reserved. Manual for SOA Exam FM/CAS Exam 2.
15/15
Chapter 3. Annuities. Section 3.5. Continuous annuities.
Theorem 10

I a

n|i
= lim
m

I
(m)
a

(m)
n|i
.
Proof.
lim
m

I
(m)
a

(m)
n|i
= lim
m
a
(m)
n|i
n
n
i
(m)
= lim
m
1
n
i
(m)
d
(m)

n
n
i
(m)
=
1
n

2

n
n

=
a
n|i
n
n

I a

n|i
.
c 2009. Miguel A. Arcones. All rights reserved. Manual for SOA Exam FM/CAS Exam 2.
1/52
Chapter 4. Amortization and sinking bonds.
Manual for SOA Exam FM/CAS Exam 2.
Chapter 4. Amortization and sinking bonds.
Section 4.1. Amortization schedules.
c 2009. Miguel A. Arcones. All rights reserved.
Extract from:
Arcones Manual for the SOA Exam FM/CAS Exam 2,
Financial Mathematics. Fall 2009 Edition,
available at http://www.actexmadriver.com/
c 2009. Miguel A. Arcones. All rights reserved. Manual for SOA Exam FM/CAS Exam 2.
2/52
Chapter 4. Amortization and sinking bonds. Section 4.1. Amortization schedules.
In this chapter, we study dierent problems related with the
payment of a loan. Suppose that a borrower (also called debtor)
takes a loan from a lender. The borrower will make payments
which eventually will repay the loan. Payments made by the
borrower can be applied to the outstanding balance or not.
According with the amortization method, all the payments made
by the borrower reduce the outstanding balance of the loan.
When a loan is paid usually, the total amount of loan payments
exceed the loan amount. The nance charge is the total amount
of interest paid (the total payments minus the loan payments).
c 2009. Miguel A. Arcones. All rights reserved. Manual for SOA Exam FM/CAS Exam 2.
3/52
Chapter 4. Amortization and sinking bonds. Section 4.1. Amortization schedules.
The simplest way to pay a loan is by unique payment. Suppose
that a borrower takes a loan with amount L at time zero and the
lender charges an annual eective rate of interest of i . If the
borrower pays the loan with a lump sum P at time n, then
P = L(1 +i )
n
. The nance charge in this situation is L(1 +i )
n
L.
c 2009. Miguel A. Arcones. All rights reserved. Manual for SOA Exam FM/CAS Exam 2.
4/52
Chapter 4. Amortization and sinking bonds. Section 4.1. Amortization schedules.
Example 1
Juan borrows $35,000 for four years at an annual nominal interest
rate of 7.5% convertible monthly. Juan will pay the loan with a
unique payment at the end of four years.
(i) Find the amount of this payment.
(ii) Find the nance charge which Juan is charged in this loan.
Solution: (i) The amount of the loan payment is
(35000)

1 +
0.075
12

(12)(4)
= 47200.97.
(ii) The nance charge which Juan is charged in this loan is
47200.97 35000 = 12200.97.
c 2009. Miguel A. Arcones. All rights reserved. Manual for SOA Exam FM/CAS Exam 2.
5/52
Chapter 4. Amortization and sinking bonds. Section 4.1. Amortization schedules.
Example 1
Juan borrows $35,000 for four years at an annual nominal interest
rate of 7.5% convertible monthly. Juan will pay the loan with a
unique payment at the end of four years.
(i) Find the amount of this payment.
(ii) Find the nance charge which Juan is charged in this loan.
Solution: (i) The amount of the loan payment is
(35000)

1 +
0.075
12

(12)(4)
= 47200.97.
(ii) The nance charge which Juan is charged in this loan is
47200.97 35000 = 12200.97.
c 2009. Miguel A. Arcones. All rights reserved. Manual for SOA Exam FM/CAS Exam 2.
6/52
Chapter 4. Amortization and sinking bonds. Section 4.1. Amortization schedules.
Suppose that a borrower takes a loan of L at time 0 and repays the
loan in a series of payments C
1
, . . . , C
n
at times t
1
, . . . , t
n
, where
0 < t
1
< t
2
< < t
n
. The debtor cashow is
Inows L C
1
C
2
C
n
C
n
Time 0 t
1
t
2
t
3
t
n
Assume that the loan increases with a certain accumulation
function a(t), t 0. Since the loan will be repaid, the present
value a time zero (or any other time) of this cashow is zero.
Hence
L =
n

j =1
C
j
a(t
j
)
.
The nance charge for this loan is
n

j =1
C
j
L =
n

j =1
C
j

n

j =1
C
j
a(t
j
)
=
n

j =1
C
j

1
1
a(t
j
)

.
c 2009. Miguel A. Arcones. All rights reserved. Manual for SOA Exam FM/CAS Exam 2.
7/52
Chapter 4. Amortization and sinking bonds. Section 4.1. Amortization schedules.
According with the retrospective method, the outstanding
balance at certain point is the present value of the loan at that
time minus the present value of the payments made at that time.
For the cashow
Inows L C
1
C
2
C
n
C
n
Time 0 t
1
t
2
t
3
t
n
the outstanding balance immediately after the kth payment, is
B
k
= La(t
k
)
k

j =1
a(t
k
)C
j
a(t
j
)
.
Of course, we have that B
0
= L, B
n
= 0.
c 2009. Miguel A. Arcones. All rights reserved. Manual for SOA Exam FM/CAS Exam 2.
8/52
Chapter 4. Amortization and sinking bonds. Section 4.1. Amortization schedules.
According to the prospective method, the outstanding balance
after the kth payment is equal to the present value of the
remaining payments.
For the cashow
Inows L C
1
C
2
C
n
C
n
Time 0 t
1
t
2
t
3
t
n
the outstanding balance immediately after the kth payment, is
B
k
=
n

j =k+1
a(t
k
)C
j
a(t
j
)
Of course, we have that
La(t
k
)
k

j =1
a(t
k
)C
j
a(t
j
)
=
n

j =k+1
a(t
k
)C
j
a(t
j
)
.
c 2009. Miguel A. Arcones. All rights reserved. Manual for SOA Exam FM/CAS Exam 2.
9/52
Chapter 4. Amortization and sinking bonds. Section 4.1. Amortization schedules.
An inductive relation for the outstanding balance is
B
k
= B
k1
a(t
k
)
a(t
k1
)
C
k
.
Previous relation says that the outstanding balance after the kth
payment is the accumulation of the previous outstanding balance
minus the amount of the payment made.
c 2009. Miguel A. Arcones. All rights reserved. Manual for SOA Exam FM/CAS Exam 2.
10/52
Chapter 4. Amortization and sinking bonds. Section 4.1. Amortization schedules.
During the period [t
k1
, t
k
], the amount of interest accrued is
I
k
= B
k1

a(t
k
)
a(t
k1
)
1

.
Immediately before the kth payment, the outstanding balance is
B
k1
+ I
k
= B
k1
a(t
k
)
a(t
k1
)
. Immediately after the kth payment, the
outstanding balance is B
k
= B
k1
+ I
k
C
k
. The kth payment
C
k
can be split as I
k
plus C
k
I
k
. I
k
is called the interest portion
of the kth payment. C
k
I
k
is called the principal portion of the
kthe payment. If C
k
I
k
< 0, then the outstanding balance
increases during the kth period. Notice that
C
k
I
k
= C
k
B
k1

a(t
k
)
a(t
k1
)
1

= B
k1
B
k
is the reduction on principal made during the the kperiod. The
total amount of reduction on principal is equal to the loan amount:

n
k=1
(B
k
B
k1
) = B
n
B
0
= L.
c 2009. Miguel A. Arcones. All rights reserved. Manual for SOA Exam FM/CAS Exam 2.
11/52
Chapter 4. Amortization and sinking bonds. Section 4.1. Amortization schedules.
Under compound interest,
L =
n

j =1
C
j
(1 + i )
t
j
.
The outstanding balance immediately after the kth payment is
B
k
= L(1 + i )
t
k

j =1
C
j
(1 + i )
t
k
t
j
=
n

j =k+1
C
j
(1 + i )
t
k
t
j
.
The inductive relation for outstanding balances is
B
k
= B
k1
(1 + i )
t
k
t
k1
C
k
.
The amount of interest accrued during the period [t
k1
, t
k
] is
I
k
= B
k1

(1 + i )
t
k
t
k1
1

.
The principal portion of the kthe payment is
C
k
I
k
= C
k
B
k1

(1 + i )
t
k
t
k1
1

= B
k1
B
k
.
The nance charge is

n
j =1
C
j
L =

n
j =1
C
j
(1 (1 + i )
t
j
) .
c 2009. Miguel A. Arcones. All rights reserved. Manual for SOA Exam FM/CAS Exam 2.
12/52
Chapter 4. Amortization and sinking bonds. Section 4.1. Amortization schedules.
Usually, we consider payments made at equally spaced intervals of
time and compound interest. Suppose that a borrower takes a loan
L at time 0 and repays the loan in a series of level payments
C
1
, . . . , C
n
at times t
0
, 2t
0
, . . . , nt
0
. By a change of units, we may
assume that t
0
= 1. Hence, the debtor cashow is
Inows L C
1
C
2
C
n
C
n
Time 0 1 2 3 n
Let i be the eective rate of interest per period. Then, we have
that
L =
n

j =1
C
j
(1 + i )
j
.
c 2009. Miguel A. Arcones. All rights reserved. Manual for SOA Exam FM/CAS Exam 2.
13/52
Chapter 4. Amortization and sinking bonds. Section 4.1. Amortization schedules.
The outstanding balance immediately after the kth payment, is
B
k
= L(1 + i )
k

j =1
C
j
(1 + i )
kj
=
n

j =k+1
C
j
(1 + i )
kj
.
The amount of interest accrued during the kth year is iB
k1
. The
principal portion of the kth payment is C
k
iB
k1
= B
k
B
k1
.
Hence, the outstanding balance after the kth payment is
B
k
= B
k1
(C
k
iB
k1
) = (1 + i )B
k1
C
k
.
The nance charge is
n

j =1
C
j
L =
n

j =1
C
j
(1 (1 + i )
j
) =
n

j =1
C
j
(1
j
).
c 2009. Miguel A. Arcones. All rights reserved. Manual for SOA Exam FM/CAS Exam 2.
14/52
Chapter 4. Amortization and sinking bonds. Section 4.1. Amortization schedules.
Example 2
Roger buys a car for $25,000 by making level payments at the end
of the month for three years. Roger is charged an annual nominal
interest rate of 8.5% compounded monthly in his loan.
(i) Find the amount of each monthly payment.
(ii) Find the total amount of payments made by Roger.
(iii) Find the total interest paid by Roger during the duration of
the loan.
(iv) Calculate the outstanding loan balance immediately after the
12th payment has been made using the retrospective method.
(v) Calculate the outstanding loan balance immediately after the
12th payment has been made using the prospective method.
Solution:
c 2009. Miguel A. Arcones. All rights reserved. Manual for SOA Exam FM/CAS Exam 2.
15/52
Chapter 4. Amortization and sinking bonds. Section 4.1. Amortization schedules.
Example 2
Roger buys a car for $25,000 by making level payments at the end
of the month for three years. Roger is charged an annual nominal
interest rate of 8.5% compounded monthly in his loan.
(i) Find the amount of each monthly payment.
(ii) Find the total amount of payments made by Roger.
(iii) Find the total interest paid by Roger during the duration of
the loan.
(iv) Calculate the outstanding loan balance immediately after the
12th payment has been made using the retrospective method.
(v) Calculate the outstanding loan balance immediately after the
12th payment has been made using the prospective method.
Solution:
(i) Let P be the monthly payment. We have that 25000 =
Pa
36

|0.085/12
and P = 789.1884356.
c 2009. Miguel A. Arcones. All rights reserved. Manual for SOA Exam FM/CAS Exam 2.
16/52
Chapter 4. Amortization and sinking bonds. Section 4.1. Amortization schedules.
Example 2
Roger buys a car for $25,000 by making level payments at the end
of the month for three years. Roger is charged an annual nominal
interest rate of 8.5% compounded monthly in his loan.
(i) Find the amount of each monthly payment.
(ii) Find the total amount of payments made by Roger.
(iii) Find the total interest paid by Roger during the duration of
the loan.
(iv) Calculate the outstanding loan balance immediately after the
12th payment has been made using the retrospective method.
(v) Calculate the outstanding loan balance immediately after the
12th payment has been made using the prospective method.
Solution:
(ii) The total amount of payments made by Roger is
(36)(789.1884356) = 28410.78368.
c 2009. Miguel A. Arcones. All rights reserved. Manual for SOA Exam FM/CAS Exam 2.
17/52
Chapter 4. Amortization and sinking bonds. Section 4.1. Amortization schedules.
Example 2
Roger buys a car for $25,000 by making level payments at the end
of the month for three years. Roger is charged an annual nominal
interest rate of 8.5% compounded monthly in his loan.
(i) Find the amount of each monthly payment.
(ii) Find the total amount of payments made by Roger.
(iii) Find the total interest paid by Roger during the duration of
the loan.
(iv) Calculate the outstanding loan balance immediately after the
12th payment has been made using the retrospective method.
(v) Calculate the outstanding loan balance immediately after the
12th payment has been made using the prospective method.
Solution:
(iii) The total interest paid by Roger during the duration of the loan
is 28410.78368 25000 = 3410.78368.
c 2009. Miguel A. Arcones. All rights reserved. Manual for SOA Exam FM/CAS Exam 2.
18/52
Chapter 4. Amortization and sinking bonds. Section 4.1. Amortization schedules.
Example 2
Roger buys a car for $25,000 by making level payments at the end
of the month for three years. Roger is charged an annual nominal
interest rate of 8.5% compounded monthly in his loan.
(i) Find the amount of each monthly payment.
(ii) Find the total amount of payments made by Roger.
(iii) Find the total interest paid by Roger during the duration of
the loan.
(iv) Calculate the outstanding loan balance immediately after the
12th payment has been made using the retrospective method.
(v) Calculate the outstanding loan balance immediately after the
12th payment has been made using the prospective method.
Solution:
(iv) The outstanding loan balance immediately after the 12th pay-
ment has been made using the retrospective method. is
(25000)(1+0.085/12)
12
(789.1884356)s
12

|0.085/12
= 17361.71419.
c 2009. Miguel A. Arcones. All rights reserved. Manual for SOA Exam FM/CAS Exam 2.
19/52
Chapter 4. Amortization and sinking bonds. Section 4.1. Amortization schedules.
Example 2
Roger buys a car for $25,000 by making level payments at the end
of the month for three years. Roger is charged an annual nominal
interest rate of 8.5% compounded monthly in his loan.
(i) Find the amount of each monthly payment.
(ii) Find the total amount of payments made by Roger.
(iii) Find the total interest paid by Roger during the duration of
the loan.
(iv) Calculate the outstanding loan balance immediately after the
12th payment has been made using the retrospective method.
(v) Calculate the outstanding loan balance immediately after the
12th payment has been made using the prospective method.
Solution:
(v) The outstanding loan balance immediately after the 12th pay-
ment has been made using the prospective method is
(789.1884356)a
24

|0.085/12
= 17361.71419.
c 2009. Miguel A. Arcones. All rights reserved. Manual for SOA Exam FM/CAS Exam 2.
20/52
Chapter 4. Amortization and sinking bonds. Section 4.1. Amortization schedules.
Example 3
A loan is being repaid with 10 payments of $3000 followed by 20
payments of $5000 at the end of each year. The eective annual
rate of interest is 4.5%.
(i) Calculate the amount of the loan.
(ii) Calculate the outstanding loan balance immediately after the
15th payment has been made by both the prospective and the
retrospective method.
(iii) Calculate the amounts of interest and principal paid in the
16th payment.
Solution:
c 2009. Miguel A. Arcones. All rights reserved. Manual for SOA Exam FM/CAS Exam 2.
21/52
Chapter 4. Amortization and sinking bonds. Section 4.1. Amortization schedules.
Example 3
A loan is being repaid with 10 payments of $3000 followed by 20
payments of $5000 at the end of each year. The eective annual
rate of interest is 4.5%.
(i) Calculate the amount of the loan.
(ii) Calculate the outstanding loan balance immediately after the
15th payment has been made by both the prospective and the
retrospective method.
(iii) Calculate the amounts of interest and principal paid in the
16th payment.
Solution:
(i) The cashow of payments is
Inows 3000 3000 3000 5000 5000 5000
Time 1 2 10 11 12 30
The loan amount is 3000a
10

|4.5%
+ (1.045)
10
5000a
20

|4.5%
=
23738.1545 + 41880.8518 = 65619.0063.
c 2009. Miguel A. Arcones. All rights reserved. Manual for SOA Exam FM/CAS Exam 2.
22/52
Chapter 4. Amortization and sinking bonds. Section 4.1. Amortization schedules.
Example 3
A loan is being repaid with 10 payments of $3000 followed by 20
payments of $5000 at the end of each year. The eective annual
rate of interest is 4.5%.
(i) Calculate the amount of the loan.
(ii) Calculate the outstanding loan balance immediately after the
15th payment has been made by both the prospective and the
retrospective method.
(iii) Calculate the amounts of interest and principal paid in the
16th payment.
Solution:
(ii) The outstanding loan balance immediately after the 15th pay-
ment using the retrospective method is
65619.0063(1.045)
15
3000(1.045)
5
s
10

|4.5%
5000s
5

|4.5%
= 126991.311 45940.0337 27353.5486 = 53697.7287.
c 2009. Miguel A. Arcones. All rights reserved. Manual for SOA Exam FM/CAS Exam 2.
23/52
Chapter 4. Amortization and sinking bonds. Section 4.1. Amortization schedules.
Example 3
A loan is being repaid with 10 payments of $3000 followed by 20
payments of $5000 at the end of each year. The eective annual
rate of interest is 4.5%.
(i) Calculate the amount of the loan.
(ii) Calculate the outstanding loan balance immediately after the
15th payment has been made by both the prospective and the
retrospective method.
(iii) Calculate the amounts of interest and principal paid in the
16th payment.
Solution:
The outstanding loan balance immediately after the 15th payment
using the prospective method is 5000a
15

|4.5%
= 53697.7286.
c 2009. Miguel A. Arcones. All rights reserved. Manual for SOA Exam FM/CAS Exam 2.
24/52
Chapter 4. Amortization and sinking bonds. Section 4.1. Amortization schedules.
Example 3
A loan is being repaid with 10 payments of $3000 followed by 20
payments of $5000 at the end of each year. The eective annual
rate of interest is 4.5%.
(i) Calculate the amount of the loan.
(ii) Calculate the outstanding loan balance immediately after the
15th payment has been made by both the prospective and the
retrospective method.
(iii) Calculate the amounts of interest and principal paid in the
16th payment.
Solution:
(iii) The amount of interest paid in the 16th payment is
(53697.7286)(0.045) = 2416.39779. The amount of interest paid in
the 16th payment is 5000 2416.39779 = 2583.60221.
c 2009. Miguel A. Arcones. All rights reserved. Manual for SOA Exam FM/CAS Exam 2.
25/52
Chapter 4. Amortization and sinking bonds. Section 4.1. Amortization schedules.
Next, we consider the amortization method of repaying a loan with
level payments made at the end of periods of the same length. Let
L be the amount borrowed. Let P be the level payment. Let n be
the number of payments. Let i be the eective rate of interest per
payment period. The cashow of payments is
Inows P P P P
Time 1 2 3 n
We have that L = Pa
n

|i
.
The outstanding principal after the kth payment is
B
k
= L(1 + i )
k
Ps
k

|i
= P(a
n

|i
(1 + i )
k
s
k

|i
) = Pa
nk

|i
.
The interest portion of the kth payment is
iB
k1
= iPa
n+1k

|i
= P(1
n+1k
).
The principal reduction of the kth payment is
B
k1
B
k
= P iB
k1
= P
n+1k
.
c 2009. Miguel A. Arcones. All rights reserved. Manual for SOA Exam FM/CAS Exam 2.
26/52
Chapter 4. Amortization and sinking bonds. Section 4.1. Amortization schedules.
Using that B
k1
= Pa
n+1k

|i
and B
k1
B
k
= P
n+1k
, we get
that B
k
= P(a
n+1k

|i

n+1k
). The outstanding principal after
the kth payment can be found using all these formulas
B
k
= L(1 + i )
k
Ps
k

|i
= P(a
n

|i
(1 + i )
k
s
k

|i
)
=Pa
nk

|i
= P(a
n+1k

|i

n+1k
).
c 2009. Miguel A. Arcones. All rights reserved. Manual for SOA Exam FM/CAS Exam 2.
27/52
Chapter 4. Amortization and sinking bonds. Section 4.1. Amortization schedules.
The following is the amortization schedule for a loan of L = Pa
n

|i
with level payments of P.
Period Payment Interest paid Principal repaid Outstanding balance
0 L = Pa
n

|i
1 P P(1
n
) P
n
Pa
n1

|i
2 P P(1
n1
) P
n1
Pa
n2

|i
3 P P(1
n2
) P
n2
Pa
n3

|i


k P P(1
n+1k
) P
n+1k
Pa
nk

|i


n 1 P P(1
2
) P
2
Pa
1

|i
n P P(1 ) P 0
c 2009. Miguel A. Arcones. All rights reserved. Manual for SOA Exam FM/CAS Exam 2.
28/52
Chapter 4. Amortization and sinking bonds. Section 4.1. Amortization schedules.
Example 4
The following is the amortization schedule of a loan of $20,000.00
at an eective interest rate of 8% for 12 years.
Time
Payment
amount
Interest
paid
Principal
reduction
Balance
0 2000.00
1 2653.90 1600.00 1053.90 18946.10
2 2653.90 1515.69 1138.21 17807.89
3 2653.90 1424.63 1229.27 16578.62
4 2653.90 1326.29 1327.61 15251.01
5 2653.90 1220.08 1433.82 13817.19
6 2653.90 1105.38 1548.52 12268.67
7 2653.90 981.49 1672.41 10596.26
8 2653.90 847.70 1806.20 8790.06
9 2653.90 703.20 1950.70 6839.36
10 2653.90 547.15 2106.75 4732.61
11 2653.90 378.61 2275.29 2457.32
12 2653.91 196.59 2457.32 0.00
c 2009. Miguel A. Arcones. All rights reserved. Manual for SOA Exam FM/CAS Exam 2.
29/52
Chapter 4. Amortization and sinking bonds. Section 4.1. Amortization schedules.
Example 5
A loan of 100,000 is being repaid by 15 equal annual installments
made at the end of each year at 6% interest eective annually.
(i) Find the amount of each annual installment.
(ii) Find the nance charge of this loan.
(iii) Find how much interest is accrued in the rst year.
(iv) Find how principal is repaid in the rst payment.
(v) Find the balance in the loan immediately after the rst
payment.
Solution:
c 2009. Miguel A. Arcones. All rights reserved. Manual for SOA Exam FM/CAS Exam 2.
30/52
Chapter 4. Amortization and sinking bonds. Section 4.1. Amortization schedules.
Example 5
A loan of 100,000 is being repaid by 15 equal annual installments
made at the end of each year at 6% interest eective annually.
(i) Find the amount of each annual installment.
(ii) Find the nance charge of this loan.
(iii) Find how much interest is accrued in the rst year.
(iv) Find how principal is repaid in the rst payment.
(v) Find the balance in the loan immediately after the rst
payment.
Solution:
(i)We solve 100000 = Pa
15

|6%
and get P = 10296.2764.
c 2009. Miguel A. Arcones. All rights reserved. Manual for SOA Exam FM/CAS Exam 2.
31/52
Chapter 4. Amortization and sinking bonds. Section 4.1. Amortization schedules.
Example 5
A loan of 100,000 is being repaid by 15 equal annual installments
made at the end of each year at 6% interest eective annually.
(i) Find the amount of each annual installment.
(ii) Find the nance charge of this loan.
(iii) Find how much interest is accrued in the rst year.
(iv) Find how principal is repaid in the rst payment.
(v) Find the balance in the loan immediately after the rst
payment.
Solution:
(ii) The nance charge is (15)(10296.2764) 100000 = 54444.146.
c 2009. Miguel A. Arcones. All rights reserved. Manual for SOA Exam FM/CAS Exam 2.
32/52
Chapter 4. Amortization and sinking bonds. Section 4.1. Amortization schedules.
Example 5
A loan of 100,000 is being repaid by 15 equal annual installments
made at the end of each year at 6% interest eective annually.
(i) Find the amount of each annual installment.
(ii) Find the nance charge of this loan.
(iii) Find how much interest is accrued in the rst year.
(iv) Find how principal is repaid in the rst payment.
(v) Find the balance in the loan immediately after the rst
payment.
Solution:
(iii) The amount of interest accrued in the rst year is
(100000)(0.06) = 6000.
c 2009. Miguel A. Arcones. All rights reserved. Manual for SOA Exam FM/CAS Exam 2.
33/52
Chapter 4. Amortization and sinking bonds. Section 4.1. Amortization schedules.
Example 5
A loan of 100,000 is being repaid by 15 equal annual installments
made at the end of each year at 6% interest eective annually.
(i) Find the amount of each annual installment.
(ii) Find the nance charge of this loan.
(iii) Find how much interest is accrued in the rst year.
(iv) Find how principal is repaid in the rst payment.
(v) Find the balance in the loan immediately after the rst
payment.
Solution:
(iv) The amount of principal repaid in the rst year is 10296.2764
6000 = 4296.2764.
c 2009. Miguel A. Arcones. All rights reserved. Manual for SOA Exam FM/CAS Exam 2.
34/52
Chapter 4. Amortization and sinking bonds. Section 4.1. Amortization schedules.
Example 5
A loan of 100,000 is being repaid by 15 equal annual installments
made at the end of each year at 6% interest eective annually.
(i) Find the amount of each annual installment.
(ii) Find the nance charge of this loan.
(iii) Find how much interest is accrued in the rst year.
(iv) Find how principal is repaid in the rst payment.
(v) Find the balance in the loan immediately after the rst
payment.
Solution:
(v) The balance in the loan immediately after the rst payment is
100000 4296.2764 = 95703.7236.
c 2009. Miguel A. Arcones. All rights reserved. Manual for SOA Exam FM/CAS Exam 2.
35/52
Chapter 4. Amortization and sinking bonds. Section 4.1. Amortization schedules.
Example 6
A loan L is being paid with 20 equal annual payments at the end
of each year. The principal portion of the 8th payment is 827.65
and the interest portion is 873.81. Find L.
Solution: We know that
827.65 = P
n+1k
= P
13
, 873.81 = P(1
n+1k
) = P(1
13
).
Adding the two equations, we get that
P = 827.65 + 873.81 = 1701.46. From the equation
827.65 = 1701.46(1 + i )
13
, we get that i = 5.7%. Hence,
L = 1701.46a
20

|5.7%
= 20000.
c 2009. Miguel A. Arcones. All rights reserved. Manual for SOA Exam FM/CAS Exam 2.
36/52
Chapter 4. Amortization and sinking bonds. Section 4.1. Amortization schedules.
Example 6
A loan L is being paid with 20 equal annual payments at the end
of each year. The principal portion of the 8th payment is 827.65
and the interest portion is 873.81. Find L.
Solution: We know that
827.65 = P
n+1k
= P
13
, 873.81 = P(1
n+1k
) = P(1
13
).
Adding the two equations, we get that
P = 827.65 + 873.81 = 1701.46. From the equation
827.65 = 1701.46(1 + i )
13
, we get that i = 5.7%. Hence,
L = 1701.46a
20

|5.7%
= 20000.
c 2009. Miguel A. Arcones. All rights reserved. Manual for SOA Exam FM/CAS Exam 2.
37/52
Chapter 4. Amortization and sinking bonds. Section 4.1. Amortization schedules.
A way to pay a loan is to pay interest as it accrues and to pay the
principal in level installments. Suppose that a loan of amount L is
paid at the end of each year for n years. At the end of each year
two payments are made: one paying the interest accrued and
another one making a principal payment of
L
n
. At the end of j years
the outstanding balance is
L(nj )
n
. Hence, the interest payment at
the end of j years is i
L(n+1j )
n
. The total payment made at the end
of the j th year is
L
n
+ i
L(n+1j )
n
=
L
n
(1 + i (n + 1 j )).
c 2009. Miguel A. Arcones. All rights reserved. Manual for SOA Exam FM/CAS Exam 2.
38/52
Chapter 4. Amortization and sinking bonds. Section 4.1. Amortization schedules.
Example 7
Samuel takes a loan of $175000. He will pay the loan in 15 years
by paying the interest accrued at the end of each year, and paying
level payments of the principal at the end of each year. The annual
eective rate of interest of the loan is 8.5%.
c 2009. Miguel A. Arcones. All rights reserved. Manual for SOA Exam FM/CAS Exam 2.
39/52
Chapter 4. Amortization and sinking bonds. Section 4.1. Amortization schedules.
Example 7
Samuel takes a loan of $175000. He will pay the loan in 15 years
by paying the interest accrued at the end of each year, and paying
level payments of the principal at the end of each year. The annual
eective rate of interest of the loan is 8.5%.
(i) Find the amount of each payment of principal.
c 2009. Miguel A. Arcones. All rights reserved. Manual for SOA Exam FM/CAS Exam 2.
40/52
Chapter 4. Amortization and sinking bonds. Section 4.1. Amortization schedules.
Example 7
Samuel takes a loan of $175000. He will pay the loan in 15 years
by paying the interest accrued at the end of each year, and paying
level payments of the principal at the end of each year. The annual
eective rate of interest of the loan is 8.5%.
(i) Find the amount of each payment of principal.
Solution: (i) The annual payment of principal is
175000
15
= 11666.67.
c 2009. Miguel A. Arcones. All rights reserved. Manual for SOA Exam FM/CAS Exam 2.
41/52
Chapter 4. Amortization and sinking bonds. Section 4.1. Amortization schedules.
Example 7
Samuel takes a loan of $175000. He will pay the loan in 15 years
by paying the interest accrued at the end of each year, and paying
level payments of the principal at the end of each year. The annual
eective rate of interest of the loan is 8.5%.
(ii) Find the outstanding principal owed at the end of the ninth year.
c 2009. Miguel A. Arcones. All rights reserved. Manual for SOA Exam FM/CAS Exam 2.
42/52
Chapter 4. Amortization and sinking bonds. Section 4.1. Amortization schedules.
Example 7
Samuel takes a loan of $175000. He will pay the loan in 15 years
by paying the interest accrued at the end of each year, and paying
level payments of the principal at the end of each year. The annual
eective rate of interest of the loan is 8.5%.
(ii) Find the outstanding principal owed at the end of the ninth year.
Solution: (ii) The outstanding principal owed at the end of the
ninth year is
(175000)(159)
15
= 70000.
c 2009. Miguel A. Arcones. All rights reserved. Manual for SOA Exam FM/CAS Exam 2.
43/52
Chapter 4. Amortization and sinking bonds. Section 4.1. Amortization schedules.
Example 7
Samuel takes a loan of $175000. He will pay the loan in 15 years
by paying the interest accrued at the end of each year, and paying
level payments of the principal at the end of each year. The annual
eective rate of interest of the loan is 8.5%.
(iii) Find the interest accrued during the tenth year.
c 2009. Miguel A. Arcones. All rights reserved. Manual for SOA Exam FM/CAS Exam 2.
44/52
Chapter 4. Amortization and sinking bonds. Section 4.1. Amortization schedules.
Example 7
Samuel takes a loan of $175000. He will pay the loan in 15 years
by paying the interest accrued at the end of each year, and paying
level payments of the principal at the end of each year. The annual
eective rate of interest of the loan is 8.5%.
(iii) Find the interest accrued during the tenth year.
Solution: (iii) The amount of interest paid at the end of the tenth
year is (0.085)70000 = 5950.
c 2009. Miguel A. Arcones. All rights reserved. Manual for SOA Exam FM/CAS Exam 2.
45/52
Chapter 4. Amortization and sinking bonds. Section 4.1. Amortization schedules.
Example 7
Samuel takes a loan of $175000. He will pay the loan in 15 years
by paying the interest accrued at the end of each year, and paying
level payments of the principal at the end of each year. The annual
eective rate of interest of the loan is 8.5%.
(iv) Find the total amount of payments made at the end of the tenth
year.
c 2009. Miguel A. Arcones. All rights reserved. Manual for SOA Exam FM/CAS Exam 2.
46/52
Chapter 4. Amortization and sinking bonds. Section 4.1. Amortization schedules.
Example 7
Samuel takes a loan of $175000. He will pay the loan in 15 years
by paying the interest accrued at the end of each year, and paying
level payments of the principal at the end of each year. The annual
eective rate of interest of the loan is 8.5%.
(iv) Find the total amount of payments made at the end of the tenth
year.
Solution: (iv) The total amount of payments made at the end of
the tenth year is 11666.67 + 5950 = 17616.67.
c 2009. Miguel A. Arcones. All rights reserved. Manual for SOA Exam FM/CAS Exam 2.
47/52
Chapter 4. Amortization and sinking bonds. Section 4.1. Amortization schedules.
Example 7
Samuel takes a loan of $175000. He will pay the loan in 15 years
by paying the interest accrued at the end of each year, and paying
level payments of the principal at the end of each year. The annual
eective rate of interest of the loan is 8.5%.
(v) Find the total amount of payments which Samuel makes.
c 2009. Miguel A. Arcones. All rights reserved. Manual for SOA Exam FM/CAS Exam 2.
48/52
Chapter 4. Amortization and sinking bonds. Section 4.1. Amortization schedules.
Example 7
Samuel takes a loan of $175000. He will pay the loan in 15 years
by paying the interest accrued at the end of each year, and paying
level payments of the principal at the end of each year. The annual
eective rate of interest of the loan is 8.5%.
(v) Find the total amount of payments which Samuel makes.
Solution: (v) The interest payment at the end of j years is
i
L(n+1j )
n
= (0.085)
(175000)(16j )
15
. Hence, the total interest pay-
ments are
(0.085)
15

j =1
(175000)(16 j )
15
=(0.085)
175000
15

(16)(15)
(16)(15)
2

= 119000.
The total amount of payments which Samuel makes is 119000 +
175000 = 294000.
c 2009. Miguel A. Arcones. All rights reserved. Manual for SOA Exam FM/CAS Exam 2.
49/52
Chapter 4. Amortization and sinking bonds. Section 4.1. Amortization schedules.
Example 8
A loan of $150000 is going to be paid over 20 years with monthly
payments. The rst payment is one month from now. During each
year, the payments are constant. But, they increase by 3% each
year. The annual eective rate of interest is 6%. Calculate the
total amount of the payments made during the rst year. Calculate
the outstanding loan balance on the loan ten years from now after
the payment is made.
c 2009. Miguel A. Arcones. All rights reserved. Manual for SOA Exam FM/CAS Exam 2.
50/52
Chapter 4. Amortization and sinking bonds. Section 4.1. Amortization schedules.
Solution: Let P be the monthly payment during the rst year.
During the kth year, 12 payments of P(1.03)
k1
are made. The
value of these payments at the end of the kth year is
P(1.03)
k1
s
12|i
(12)
/12
= P(1.03)
k1
12.32652834, where we have
used that i
(12)
= 5.84106068%. So, the cashow of payments is
equivalent to
Payments P12.3265 P(1.03)12.3265 P(1.03)
19
12.3265
Time 1 2 20
The present value of this cashow is the loan amount:
150000 =
12.3265P
1 + r
a
n|
i r
1+r
=
12.3265P
1.03
a
20|
0.060.03
1+0.03
= 179.493145P
and P = 835.6865105.
The outstanding loan balance on the loan ten years from now after
the payment is made is
(835.6865105)(1.03)
9
(12.3265)a
10|
0.060.03
1+0.03
= 201586.9934.
c 2009. Miguel A. Arcones. All rights reserved. Manual for SOA Exam FM/CAS Exam 2.
51/52
Chapter 4. Amortization and sinking bonds. Section 4.1. Amortization schedules.
Example 9
Mary takes on a loan of $135,000. The loan is being repaid by a
10year increasing annuityimmediate. The initial payment is
10000, and each subsequent payment is x larger than the preceding
payment. The annual eective interest rate is 6.5%. Determine
the principal outstanding immediately after the 5th payment.
c 2009. Miguel A. Arcones. All rights reserved. Manual for SOA Exam FM/CAS Exam 2.
52/52
Chapter 4. Amortization and sinking bonds. Section 4.1. Amortization schedules.
Solution: The cashow is
Contributions 10000 10000 + x 10000 + 2x 10000 + 9x
Time 1 2 3 10
The present value of the payments is
135000 = (10000 x)a
10|6.5%
+ x(Ia)
10|6.5%
=(10000 x)(7.188830223) + 35.82836665x.
So, x =
135000(7.188830223)(10000)
35.828366657.188830223
= 2203.656401.
The payments to be made after the 5-th payment are
Contributions 10000 + (5)(2203.656401) 10000 + (9)(2203.656401)
Time 6 10
Its present value at time 5 is
(10000 + (4)(2203.656401))a
5|6.5%
+ (2203.656401)(Ia)
5|6.5%
=78187.55276 + 26321.63894 = 104509.1917.
c 2009. Miguel A. Arcones. All rights reserved. Manual for SOA Exam FM/CAS Exam 2.
1/17
Chapter 4. Amortization and sinking bonds.
Manual for SOA Exam FM/CAS Exam 2.
Chapter 4. Amortization and sinking bonds.
Section 4.2. Sinking funds.
c 2008. Miguel A. Arcones. All rights reserved.
Extract from:
Arcones Manual for the SOA Exam FM/CAS Exam 2,
Financial Mathematics. Spring 2009 Edition,
available at http://www.actexmadriver.com/
c 2008. Miguel A. Arcones. All rights reserved. Manual for SOA Exam FM/CAS Exam 2.
2/17
Chapter 4. Amortization and sinking bonds. Section 4.2. Sinking funds.
An alternate way to repay a loan is to make two payments, one
directly to the lender and another to an auxiliary fund. The
auxiliary fund is called a sinking fund. The payments made
directly to the lender apply to the principal. The deposits made
into the sinking fund do not. Usually, the sinking fund accumulates
with a dierent interest rate than the rate charged by the lender.
At the end of the duration of the loan, the borrower withdraws the
total accumulated in the sinking fund and uses this money to pay
the loan to the lender.
c 2008. Miguel A. Arcones. All rights reserved. Manual for SOA Exam FM/CAS Exam 2.
3/17
Chapter 4. Amortization and sinking bonds. Section 4.2. Sinking funds.
Usually, we consider the case of payments made at the end of each
of n periods. The simplest case is the one when all the payments
are equal. Let i be the periodic eective rate charged by the lender
on the loan. At the end of each period, the borrower pays P
directly to lender. The borrower deposits Q into a sinking fund
earning a rate of interest j . Usually, j < i . The cashow of
payments to the principal is
Contributions 0 P P P P + R
Time 0 1 2 n 1 n
where R is the lumpsum payment obtained by withdrawing the
total accumulated in the sinking fund at the end of n periods. The
cashow of deposits in the sinking fund is
Contributions 0 Q Q Q Q
Time 0 1 2 3 n
Hence, the accumulated value in the sinking fund at time n is
R = Qs
n

|j
.
c 2008. Miguel A. Arcones. All rights reserved. Manual for SOA Exam FM/CAS Exam 2.
4/17
Chapter 4. Amortization and sinking bonds. Section 4.2. Sinking funds.
The borrowers total cashow is
Contributions L P Q P Q P Q P Q
Time 0 1 2 3 n
The lender cashow is
Contributions L P P P P + R
Time 0 1 2 3 n
In order to the loan to be repaid:
L = Pa
n

|i
+ R(1 + i )
n
= Pa
n

|i
+ Qs
n

|j
(1 + i )
n
.
c 2008. Miguel A. Arcones. All rights reserved. Manual for SOA Exam FM/CAS Exam 2.
5/17
Chapter 4. Amortization and sinking bonds. Section 4.2. Sinking funds.
Example 1
Dave borrows 150,000 from a trust company at an annual eective
rate of interest of 9.5%. He agrees to pay the interest annually at
the end of the each year, and build up a sinking fund which will
repay the loan at the end of 15 years. The sinking fund
accumulates at annual eective rate of interest of 4.5%. Calculate:
c 2008. Miguel A. Arcones. All rights reserved. Manual for SOA Exam FM/CAS Exam 2.
6/17
Chapter 4. Amortization and sinking bonds. Section 4.2. Sinking funds.
Example 1
Dave borrows 150,000 from a trust company at an annual eective
rate of interest of 9.5%. He agrees to pay the interest annually at
the end of the each year, and build up a sinking fund which will
repay the loan at the end of 15 years. The sinking fund
accumulates at annual eective rate of interest of 4.5%. Calculate:
(i) The annual interest payments.
c 2008. Miguel A. Arcones. All rights reserved. Manual for SOA Exam FM/CAS Exam 2.
7/17
Chapter 4. Amortization and sinking bonds. Section 4.2. Sinking funds.
Example 1
Dave borrows 150,000 from a trust company at an annual eective
rate of interest of 9.5%. He agrees to pay the interest annually at
the end of the each year, and build up a sinking fund which will
repay the loan at the end of 15 years. The sinking fund
accumulates at annual eective rate of interest of 4.5%. Calculate:
(i) The annual interest payments.
Solution: (i) The annual interest payment is 150000(0.095) =
14250.
c 2008. Miguel A. Arcones. All rights reserved. Manual for SOA Exam FM/CAS Exam 2.
8/17
Chapter 4. Amortization and sinking bonds. Section 4.2. Sinking funds.
Example 1
Dave borrows 150,000 from a trust company at an annual eective
rate of interest of 9.5%. He agrees to pay the interest annually at
the end of the each year, and build up a sinking fund which will
repay the loan at the end of 15 years. The sinking fund
accumulates at annual eective rate of interest of 4.5%. Calculate:
(ii) The annual sinking fund payment.
c 2008. Miguel A. Arcones. All rights reserved. Manual for SOA Exam FM/CAS Exam 2.
9/17
Chapter 4. Amortization and sinking bonds. Section 4.2. Sinking funds.
Example 1
Dave borrows 150,000 from a trust company at an annual eective
rate of interest of 9.5%. He agrees to pay the interest annually at
the end of the each year, and build up a sinking fund which will
repay the loan at the end of 15 years. The sinking fund
accumulates at annual eective rate of interest of 4.5%. Calculate:
(ii) The annual sinking fund payment.
Solution: (ii) 150000 = Qs
15

|4.5%
and Q = 7217.071217.
c 2008. Miguel A. Arcones. All rights reserved. Manual for SOA Exam FM/CAS Exam 2.
10/17
Chapter 4. Amortization and sinking bonds. Section 4.2. Sinking funds.
Example 1
Dave borrows 150,000 from a trust company at an annual eective
rate of interest of 9.5%. He agrees to pay the interest annually at
the end of the each year, and build up a sinking fund which will
repay the loan at the end of 15 years. The sinking fund
accumulates at annual eective rate of interest of 4.5%. Calculate:
(iii) Daves total annual outlay.
c 2008. Miguel A. Arcones. All rights reserved. Manual for SOA Exam FM/CAS Exam 2.
11/17
Chapter 4. Amortization and sinking bonds. Section 4.2. Sinking funds.
Example 1
Dave borrows 150,000 from a trust company at an annual eective
rate of interest of 9.5%. He agrees to pay the interest annually at
the end of the each year, and build up a sinking fund which will
repay the loan at the end of 15 years. The sinking fund
accumulates at annual eective rate of interest of 4.5%. Calculate:
(iii) Daves total annual outlay.
Solution: (iii) Daves total annual outlay is 14250+7217.071217 =
21467.07122.
c 2008. Miguel A. Arcones. All rights reserved. Manual for SOA Exam FM/CAS Exam 2.
12/17
Chapter 4. Amortization and sinking bonds. Section 4.2. Sinking funds.
Example 1
Dave borrows 150,000 from a trust company at an annual eective
rate of interest of 9.5%. He agrees to pay the interest annually at
the end of the each year, and build up a sinking fund which will
repay the loan at the end of 15 years. The sinking fund
accumulates at annual eective rate of interest of 4.5%. Calculate:
(iv) The annual eective rate of interest i

for which the payments


made at the end of the year will be equal to Daves total annual
outlay.
c 2008. Miguel A. Arcones. All rights reserved. Manual for SOA Exam FM/CAS Exam 2.
13/17
Chapter 4. Amortization and sinking bonds. Section 4.2. Sinking funds.
Example 1
Dave borrows 150,000 from a trust company at an annual eective
rate of interest of 9.5%. He agrees to pay the interest annually at
the end of the each year, and build up a sinking fund which will
repay the loan at the end of 15 years. The sinking fund
accumulates at annual eective rate of interest of 4.5%. Calculate:
(iv) The annual eective rate of interest i

for which the payments


made at the end of the year will be equal to Daves total annual
outlay.
Solution: (iv) 150000 = (21467.07122)a
15

|i

and i

=
11.52440895%. Notice that although Dave borrows at a rate 9.5%,
by getting only 4.5% in his sinking fund, the actual rate of interest
Dave which is paying is higher.
c 2008. Miguel A. Arcones. All rights reserved. Manual for SOA Exam FM/CAS Exam 2.
14/17
Chapter 4. Amortization and sinking bonds. Section 4.2. Sinking funds.
Example 1
Dave borrows 150,000 from a trust company at an annual eective
rate of interest of 9.5%. He agrees to pay the interest annually at
the end of the each year, and build up a sinking fund which will
repay the loan at the end of 15 years. The sinking fund
accumulates at annual eective rate of interest of 4.5%. Calculate:
(v) Find i +
n1
n+1
(i j ), where i = 9.5%, j = 4.5% and n = 15, and
compare with i

.
c 2008. Miguel A. Arcones. All rights reserved. Manual for SOA Exam FM/CAS Exam 2.
15/17
Chapter 4. Amortization and sinking bonds. Section 4.2. Sinking funds.
Example 1
Dave borrows 150,000 from a trust company at an annual eective
rate of interest of 9.5%. He agrees to pay the interest annually at
the end of the each year, and build up a sinking fund which will
repay the loan at the end of 15 years. The sinking fund
accumulates at annual eective rate of interest of 4.5%. Calculate:
(v) Find i +
n1
n+1
(i j ), where i = 9.5%, j = 4.5% and n = 15, and
compare with i

.
Solution: (v) i +
n1
n+1
(i j ) = 0.095+
(151)(0.950.045)
15+1
= 0.13875.
which is sort of close to 0.1152440895.
c 2008. Miguel A. Arcones. All rights reserved. Manual for SOA Exam FM/CAS Exam 2.
16/17
Chapter 4. Amortization and sinking bonds. Section 4.2. Sinking funds.
Example 2
Steve repays a loan of $18,000 by making interest payment at the
end of the year for 15 years and equal deposits at the end of each
year into a sinking fund for 15 years. At the end of the 15 years,
Steve withdraws the balance from the sinking fund and pays the
loan. The sinking fund earns 6% eective annually. Immediately
after the fourth payment, the yield on the sinking fund increases to
7% eective annually. At that time, Steve adjusts his sinking fund
payment to x so that the sinking fund will accumulate to $18,000,
15 years after the original loan date. Find x.
Solution: Let Q be the initial payment Joe makes to the sinking
fund. Then, 18000 = Qs
15

|6%
. Hence, Q = 773.3297512. The
balance in the sinking fund immediately after the fourth payment is
773.3297512s
4

|6%
= 3383.020703. The nal accumulation in the
sinking fund is 18000. So, 18000 = 3383.020703(1.07)
11
+ xs
11

|7%
and x = 689.2751, which can be found doing:
11 N 7 I/Y 3383.020703 PV 18000 FV CPT PMT
c 2008. Miguel A. Arcones. All rights reserved. Manual for SOA Exam FM/CAS Exam 2.
17/17
Chapter 4. Amortization and sinking bonds. Section 4.2. Sinking funds.
Example 2
Steve repays a loan of $18,000 by making interest payment at the
end of the year for 15 years and equal deposits at the end of each
year into a sinking fund for 15 years. At the end of the 15 years,
Steve withdraws the balance from the sinking fund and pays the
loan. The sinking fund earns 6% eective annually. Immediately
after the fourth payment, the yield on the sinking fund increases to
7% eective annually. At that time, Steve adjusts his sinking fund
payment to x so that the sinking fund will accumulate to $18,000,
15 years after the original loan date. Find x.
Solution: Let Q be the initial payment Joe makes to the sinking
fund. Then, 18000 = Qs
15

|6%
. Hence, Q = 773.3297512. The
balance in the sinking fund immediately after the fourth payment is
773.3297512s
4

|6%
= 3383.020703. The nal accumulation in the
sinking fund is 18000. So, 18000 = 3383.020703(1.07)
11
+ xs
11

|7%
and x = 689.2751, which can be found doing:
11 N 7 I/Y 3383.020703 PV 18000 FV CPT PMT
c 2008. Miguel A. Arcones. All rights reserved. Manual for SOA Exam FM/CAS Exam 2.
1/6
Chapter 4. Amortization and sinking bonds.
Manual for SOA Exam FM/CAS Exam 2.
Chapter 4. Amortization and sinking bonds.
Section 4.3. Reinvestment.
c 2009. Miguel A. Arcones. All rights reserved.
Extract from:
Arcones Manual for the SOA Exam FM/CAS Exam 2,
Financial Mathematics. Fall 2009 Edition,
available at http://www.actexmadriver.com/
c 2009. Miguel A. Arcones. All rights reserved. Manual for SOA Exam FM/CAS Exam 2.
2/6
Chapter 4. Amortization and sinking bonds. Section 4.3. Reinvestment.
Suppose that a bank account pays interest in the original deposit,
but not in the obtained interest. Then, it will wise to withdraw the
interest and invest it in another account. In other situations, it
makes sense to reinvest the earned interest in a dierent
investment. For example, a stock pays dividends and/or capital
gains, which can be invested somewhere else. We obtain a ow of
interest payments which are invested at dierent rate from the one
in the initial investment. Suppose that a mortgage company makes
a loan to a customer. To know the mortgage companys return in
its investment we need to take in account the interest rate charged
to the customer in the loan and the interest rate which the
mortgage company gets in the monthly payments it receives.
c 2009. Miguel A. Arcones. All rights reserved. Manual for SOA Exam FM/CAS Exam 2.
3/6
Chapter 4. Amortization and sinking bonds. Section 4.3. Reinvestment.
Example 1
Payments of $2500 are invested at the end of each quarter for 5
years. The payments earn interest at an annual nominal interest
rate converted quarterly of 14% and the interest payments are
reinvested at an annual nominal interest rate converted quarterly
of 10%. Find the total accumulation for both accounts at the end
of 5 years.
c 2009. Miguel A. Arcones. All rights reserved. Manual for SOA Exam FM/CAS Exam 2.
4/6
Chapter 4. Amortization and sinking bonds. Section 4.3. Reinvestment.
Solution: The balance in the rst account is
Balance in the 1st account 2500 2(2500) (20)(2500)
Time (in quarters) 1 2 20
Since the rate of interest per quarter is 14%/4 = 3.5%, the
interest payments are
Cashow of interest (1)(0.035)(2500) 19(0.035)(2500)
Time (in quarters) 2 20
The total accumulation for both accounts at the end of the 5th
year is
(20)(2500) + (0.035)(2500) (Is)
19

|0.025
= 50000 + (87.5)

(1.025)s
19

|0.025
19
0.025

= 50000 + (87.5)

24.5446576119
0.025

= 69406.30164.
c 2009. Miguel A. Arcones. All rights reserved. Manual for SOA Exam FM/CAS Exam 2.
5/6
Chapter 4. Amortization and sinking bonds. Section 4.3. Reinvestment.
Example 2
John invests 1000 at the beginning of each year for 5 years at an
annual eective interest rate of 10% and reinvests the interest at
an annual eective interest rate of 8%. Calculate the total value of
his investment at the end of 5 years.
c 2009. Miguel A. Arcones. All rights reserved. Manual for SOA Exam FM/CAS Exam 2.
6/6
Chapter 4. Amortization and sinking bonds. Section 4.3. Reinvestment.
Solution: The balance in Johns account is
Balance in Johns account 1000 2000 3000 4000 5000 5000
Time 0 1 2 3 4 5
The interest payments which John gets are
Johns interest payments 0 100 200 300 400 500
Time 0 1 2 3 4 5
The total accumulation of Johns investments at time 5 is
5000 + 100 (Is)
5

|0.08
= 5000 + 100

(1.08)s
5

|0.08
5
0.08

=5000 + 100

6.335929037 5
0.08

= 6669.911296.
c 2009. Miguel A. Arcones. All rights reserved. Manual for SOA Exam FM/CAS Exam 2.
1/??
Chapter 5. Bonds.
Manual for SOA Exam FM/CAS Exam 2.
Chapter 5. Bonds.
Section 5.1. Securities.
c 2009. Miguel A. Arcones. All rights reserved.
Extract from:
Arcones Manual for the SOA Exam FM/CAS Exam 2,
Financial Mathematics. Fall 2009 Edition,
available at http://www.actexmadriver.com/
c 2009. Miguel A. Arcones. All rights reserved. Manual for SOA Exam FM/CAS Exam 2.
2/??
Chapter 5. Bonds. Section 5.1. Securities.
Securities
When a corporation or a public institution needs to raise money, it
arranges contracts with investors. These contracts are called
nancial assets or securities. From the investors view point,
securities are investment instruments, endorsed by a corporation,
government, or other organization. The borrower oers either debt
or ownership (equity). In the case of debt, the borrower agrees to
make a series of payments to the investor. In the case of equity
securities, the borrower gives a part of the ownership of the
corporation to the investor.
c 2009. Miguel A. Arcones. All rights reserved. Manual for SOA Exam FM/CAS Exam 2.
3/??
Chapter 5. Bonds. Section 5.1. Securities.
Stock
For a corporation, claims to its ownership are determined by shares
of stock. The proportion of a company which an investor owns is
the fraction of its shares owned over the total number of shares
outstanding.
There are several types of stocks. Some stocks provide voting
rights to the holder. Some stocks entitle the holder to receive
payments, such as dividends and/or capital appreciation. Dierent
types of stocks have dierent order of preference to the companys
assets in the case of liquidation (the company is forced to sells its
assets, pay outstanding debts, and distribute the remainder to
shareholders).
Common stock entitles the holder to payments of dividends and
capital appreciation. Common stock gives voting rights.
A preferred stock is a stock which provides a xed dividend that
does not uctuate. Preferred stock shareholders do not enjoy
voting rights.
c 2009. Miguel A. Arcones. All rights reserved. Manual for SOA Exam FM/CAS Exam 2.
4/??
Chapter 5. Bonds. Section 5.1. Securities.
Bonds
A way for a government or corporation to raise money is to issue
bonds. A bond is a certicate issued from a borrower to a lender
agreeing to make payment(s) in a loan. The price of a bond P is
the amount that the lender pays (loans) to the government or
corporation for the bond. Types of available bonds are: U.S.
government securities, municipal bonds, corporate bonds,
mortgage and assetbacked securities, federal agency securities and
foreign government bonds.
c 2009. Miguel A. Arcones. All rights reserved. Manual for SOA Exam FM/CAS Exam 2.
1/17
Chapter 5. Bonds.
Manual for SOA Exam FM/CAS Exam 2.
Chapter 5. Bonds.
Section 5.2. Price of a bond.
c 2009. Miguel A. Arcones. All rights reserved.
Extract from:
Arcones Manual for the SOA Exam FM/CAS Exam 2,
Financial Mathematics. Fall 2009 Edition,
available at http://www.actexmadriver.com/
c 2009. Miguel A. Arcones. All rights reserved. Manual for SOA Exam FM/CAS Exam 2.
2/17
Chapter 5. Bonds. Section 5.2. Price of a bond.
Bonds
There are two kind of bonds: accumulation bonds and bonds
with coupons. The time at which the loan is repaid is called the
maturity date (or redemption date).

In the case of accumulation bonds, the borrower agrees to pay


the loan plus interest at a unique date, called the redemption
time. An accumulation bond is also called a zero coupon
bond.

The most common bonds are bonds with coupons. For bonds
with coupons, the borrower agrees to make period payments
(coupons) plus a balloon payment (the redemption value)
C at the maturity date.
c 2009. Miguel A. Arcones. All rights reserved. Manual for SOA Exam FM/CAS Exam 2.
3/17
Chapter 5. Bonds. Section 5.2. Price of a bond.
Cashow of a bond with coupons.
Every bond has a face value (or par value) F. The coupon
payment is Fr . Here, r is the coupon rate per interest period.
Often, the payments are semiannually and 2r is the annual nominal
rate of interest convertibly semiannually. A bond is called
redeemable at par if C = F. Unless said otherwise we assume
that a bond is redeemable at par. Let n be the number of interest
periods until the redemption date. Let i be the yield rate per
interest period. The cashow for the borrower is
Contributions P Fr Fr Fr Fr C
Time 0 1 2 3 n
c 2009. Miguel A. Arcones. All rights reserved. Manual for SOA Exam FM/CAS Exam 2.
4/17
Chapter 5. Bonds. Section 5.2. Price of a bond.
Variables for a bond.

P = the price of a bond.

F = the par or face value of a bond.

C = the redemption value of a bond.

r = the coupon rate of a bond.

Fr = the amount of a coupon.

i = the yield rate of the bond per coupon period.

=
1
1+i
= the discount factor per coupon period.

n = the number of coupon payment periods.

g =
Fr
C
= the modied coupon rate of a bond

G =
Fr
i
= the base amount of a bond.

K = C
n
= the present value, compounded at the yield rate,
of the redemption value of a bond

P C = the premium (if P > C).

C P = the discount (if C > P).

k =
PC
C
premium as a fraction of redemption value.
c 2009. Miguel A. Arcones. All rights reserved. Manual for SOA Exam FM/CAS Exam 2.
5/17
Chapter 5. Bonds. Section 5.2. Price of a bond.
Price of a bond
The basic formula for the price of a bond is
P = Fra
n

|i
+ C(1 + i )
n
= Fra
n

|i
+ K.
The premium/discount formula for the price of a bond is
P = Fra
n

|i
+ C
n
= Fra
n

|i
+ C(1 ia
n

|i
)
=C + (Fr Ci )a
n

|i
= C + C(g i )a
n

|i
.
The base amount formula for the price of a bond is
P = Fra
n

|i
+C
n
= Gia
n

|i
+C
n
= G(1
n
)+C
n
= G+(CG)
n
.
The Makehan formula for the price of a bond is
P = Fra
n

|i
+ C
n
= Cg
1
n
i
+ C
n
=
g
i
(C C
n
) + C
n
=
g
i
(C K) + K.
c 2009. Miguel A. Arcones. All rights reserved. Manual for SOA Exam FM/CAS Exam 2.
6/17
Chapter 5. Bonds. Section 5.2. Price of a bond.
Example 1
Find the price of a 10year bond, redeemable at par, with face
value of $10,000 and coupon rate of 10%, convertible quarterly,
that will yield 8%, convertible quarterly.
Solution: We know that F = C = 10000, n = (10)(4) = 40,
r =
0.10
4
= 0.025, Fr = (10000)(0.025) = 250. and
i =
0.08
4
= 0.02. So, the price of the bond is
P = Fra
n

|i
+ C(1 + i )
n
= 250a
40

|0.02
+ 10000(1.02)
40
=11367.77396.
In the calculator, do:
40 N 2 I/Y 250 PMT 10000 PMT CPT PV .
c 2009. Miguel A. Arcones. All rights reserved. Manual for SOA Exam FM/CAS Exam 2.
7/17
Chapter 5. Bonds. Section 5.2. Price of a bond.
Example 1
Find the price of a 10year bond, redeemable at par, with face
value of $10,000 and coupon rate of 10%, convertible quarterly,
that will yield 8%, convertible quarterly.
Solution: We know that F = C = 10000, n = (10)(4) = 40,
r =
0.10
4
= 0.025, Fr = (10000)(0.025) = 250. and
i =
0.08
4
= 0.02. So, the price of the bond is
P = Fra
n

|i
+ C(1 + i )
n
= 250a
40

|0.02
+ 10000(1.02)
40
=11367.77396.
In the calculator, do:
40 N 2 I/Y 250 PMT 10000 PMT CPT PV .
c 2009. Miguel A. Arcones. All rights reserved. Manual for SOA Exam FM/CAS Exam 2.
8/17
Chapter 5. Bonds. Section 5.2. Price of a bond.
Example 2
A 30 year bond matures at its face value of 10,000. It pays
semiannual coupons of 600. Calculate the price of the bond if the
annual nominal interest rate convertible semiannually is 7.5%.
Solution: We know that F = C = 10000, n = (30)(2) = 60,
Fr = 600, and i =
7.5%
2
= 3.75%. The price of the bond is
(600)a
60

|0.0375
+ (10000)(1 + 0.0375)
60
= 15341.03109.
c 2009. Miguel A. Arcones. All rights reserved. Manual for SOA Exam FM/CAS Exam 2.
9/17
Chapter 5. Bonds. Section 5.2. Price of a bond.
Example 2
A 30 year bond matures at its face value of 10,000. It pays
semiannual coupons of 600. Calculate the price of the bond if the
annual nominal interest rate convertible semiannually is 7.5%.
Solution: We know that F = C = 10000, n = (30)(2) = 60,
Fr = 600, and i =
7.5%
2
= 3.75%. The price of the bond is
(600)a
60

|0.0375
+ (10000)(1 + 0.0375)
60
= 15341.03109.
c 2009. Miguel A. Arcones. All rights reserved. Manual for SOA Exam FM/CAS Exam 2.
10/17
Chapter 5. Bonds. Section 5.2. Price of a bond.
Remember that unless said otherwise a bond is redeemable at par.
Example 3
What is the price of a 5year 100 parvalue bond having quarterly
coupons at a quarter rate of 1.5% that is bought to yield a
nominal annual rate of 12% convertible monthly?
Solution: Solution: We know that F = C = 100,
n = (5)(4) = 20, r = 0.015 and Fr = (100)(0.015) = 1.5. Let j be
the eective yield rate per quarter. We have that i
(12)
= 12%,
i = 12.68250301% and i
(4)
= 12.1204%, j = i
(4)
/4 = 3.0301%.
Hence,
P = 1.5a
20

|3.0301%
+ 100(1.030301)
20
= 77.29919664.
c 2009. Miguel A. Arcones. All rights reserved. Manual for SOA Exam FM/CAS Exam 2.
11/17
Chapter 5. Bonds. Section 5.2. Price of a bond.
Remember that unless said otherwise a bond is redeemable at par.
Example 3
What is the price of a 5year 100 parvalue bond having quarterly
coupons at a quarter rate of 1.5% that is bought to yield a
nominal annual rate of 12% convertible monthly?
Solution: Solution: We know that F = C = 100,
n = (5)(4) = 20, r = 0.015 and Fr = (100)(0.015) = 1.5. Let j be
the eective yield rate per quarter. We have that i
(12)
= 12%,
i = 12.68250301% and i
(4)
= 12.1204%, j = i
(4)
/4 = 3.0301%.
Hence,
P = 1.5a
20

|3.0301%
+ 100(1.030301)
20
= 77.29919664.
c 2009. Miguel A. Arcones. All rights reserved. Manual for SOA Exam FM/CAS Exam 2.
12/17
Chapter 5. Bonds. Section 5.2. Price of a bond.
Example 4
The price of a zero coupon 1000 face value bond is 599.4584. The
yield rate convertible semiannually is 6.5%. Calculate the
maturity date.
Solution: Let n be the maturity date in years. We have that
599.4584 = (1000)

1 +
0.065
2

2n
and n = 8 years.
c 2009. Miguel A. Arcones. All rights reserved. Manual for SOA Exam FM/CAS Exam 2.
13/17
Chapter 5. Bonds. Section 5.2. Price of a bond.
Example 4
The price of a zero coupon 1000 face value bond is 599.4584. The
yield rate convertible semiannually is 6.5%. Calculate the
maturity date.
Solution: Let n be the maturity date in years. We have that
599.4584 = (1000)

1 +
0.065
2

2n
and n = 8 years.
c 2009. Miguel A. Arcones. All rights reserved. Manual for SOA Exam FM/CAS Exam 2.
14/17
Chapter 5. Bonds. Section 5.2. Price of a bond.
Example 5
What is the yield as an annual eective rate of interest on a 100
parvalue 10year bond with coupon rate 6%, convertible monthly,
that is selling for 90?
Solution: We know that P = 90, C = F = 100,
n = (10)(12) = 120, r =
0.06
12
= 0.005 and
Fr = (100)(0.005) = 0.5. Let j = i
(12)
/12. Then,
90 = P = Fra
n

|j
+ C(1 + j )
12n
= (0.5)a
120

|j
+ 100(1 + j )
120
.
Hence, j = 0.618181404%, i
(12)
= 12j = 7.419376846% and
i = 7.676949087%.
c 2009. Miguel A. Arcones. All rights reserved. Manual for SOA Exam FM/CAS Exam 2.
15/17
Chapter 5. Bonds. Section 5.2. Price of a bond.
Example 5
What is the yield as an annual eective rate of interest on a 100
parvalue 10year bond with coupon rate 6%, convertible monthly,
that is selling for 90?
Solution: We know that P = 90, C = F = 100,
n = (10)(12) = 120, r =
0.06
12
= 0.005 and
Fr = (100)(0.005) = 0.5. Let j = i
(12)
/12. Then,
90 = P = Fra
n

|j
+ C(1 + j )
12n
= (0.5)a
120

|j
+ 100(1 + j )
120
.
Hence, j = 0.618181404%, i
(12)
= 12j = 7.419376846% and
i = 7.676949087%.
c 2009. Miguel A. Arcones. All rights reserved. Manual for SOA Exam FM/CAS Exam 2.
16/17
Chapter 5. Bonds. Section 5.2. Price of a bond.
Example 6
A 1000 par value 10year bond with semiannual coupons and
redeemable at 1200 is purchased to yield 8% convertible
semiannually. The rst coupon is 50. Each subsequent coupon is
3% greater than the preceding coupon. Find the price of the bond.
Solution: The cashow of coupons is
Coupons 50 (50)(1.03) (50)(1.03)
19
Time (in halfyears) 1 2 20
The present value of the payments is
(50)
1
1.03
a
20

|
0.040.03
1.03
+ (1200)(1.04)
20
=878.5721 + 547.6643 = 1426.2364.
c 2009. Miguel A. Arcones. All rights reserved. Manual for SOA Exam FM/CAS Exam 2.
17/17
Chapter 5. Bonds. Section 5.2. Price of a bond.
Example 6
A 1000 par value 10year bond with semiannual coupons and
redeemable at 1200 is purchased to yield 8% convertible
semiannually. The rst coupon is 50. Each subsequent coupon is
3% greater than the preceding coupon. Find the price of the bond.
Solution: The cashow of coupons is
Coupons 50 (50)(1.03) (50)(1.03)
19
Time (in halfyears) 1 2 20
The present value of the payments is
(50)
1
1.03
a
20

|
0.040.03
1.03
+ (1200)(1.04)
20
=878.5721 + 547.6643 = 1426.2364.
c 2009. Miguel A. Arcones. All rights reserved. Manual for SOA Exam FM/CAS Exam 2.
1/??
Chapter 5. Bonds.
Manual for SOA Exam FM/CAS Exam 2.
Chapter 5. Bonds.
Section 5.3. Book value and amortization schedules.
c 2009. Miguel A. Arcones. All rights reserved.
Extract from:
Arcones Manual for the SOA Exam FM/CAS Exam 2,
Financial Mathematics. Fall 2009 Edition,
available at http://www.actexmadriver.com/
c 2009. Miguel A. Arcones. All rights reserved. Manual for SOA Exam FM/CAS Exam 2.
2/??
Chapter 5. Bonds. Section 5.3. Book value and amortization schedules.
Book value
The book value B
k
of a bond at time k of a bond is the present
value of the payments to be made, i.e. the present value at time k
of the remaining n k coupons and the redemption value C. This
is also the outstanding balance of the loan at that time. So, B
k
can be found using any of the following expressions:
B
k
= Fra
nk

|i
+C
nk
= C+(Fr Ci )a
nk

|i
= C+C(g i )a
nk

|i
.
Notice that
Fra
nk

|i
+ C
nk
= Fra
nk

|i
+ C(1 ia
nk

|i
)
=C + (Fr Ci )a
nk

|i
= C + C(g i )a
nk

|i
.
c 2009. Miguel A. Arcones. All rights reserved. Manual for SOA Exam FM/CAS Exam 2.
3/??
Chapter 5. Bonds. Section 5.3. Book value and amortization schedules.
Of course, we have that B
0
= P and B
n
= C. The previous
formula for B
k
is that of the outstanding balance of a loan using
the prospective method. Using the retrospective method, the book
value of a bond is
B
k
= P(1 + i )
k
Frs
k

|i
.
The previous formula is equivalent to
P = B
k
(1 + i )
k
+ Fra
k

|i
.
A way to interpret the previous formula is as follow. B
k
is the
balance in the loan after the rst k payments are made. The
present value of B
k
and the payments made until that moment
equals the initial balance.
c 2009. Miguel A. Arcones. All rights reserved. Manual for SOA Exam FM/CAS Exam 2.
4/??
Chapter 5. Bonds. Section 5.3. Book value and amortization schedules.
Example 1
Zack buys a 20 year bond with a par value of 4000 and 10%
semiannual coupons. He attains an annual yield of 5% convertible
semiannually. The redemption value of the bond is 1200. Find the
book value of the bond at the end of the 12th year.
Solution: We have that F = 4000, r = 5%,
Fr = (4000)(0.05) = 200, n = 40, k = 24 and C = 1200. Hence,
B
24
= Fra
nk

|i
+ C
nk
= (200)a
16

|5%
+ (1200)(1.05)
16
=3419.350452.
c 2009. Miguel A. Arcones. All rights reserved. Manual for SOA Exam FM/CAS Exam 2.
5/??
Chapter 5. Bonds. Section 5.3. Book value and amortization schedules.
Example 1
Zack buys a 20 year bond with a par value of 4000 and 10%
semiannual coupons. He attains an annual yield of 5% convertible
semiannually. The redemption value of the bond is 1200. Find the
book value of the bond at the end of the 12th year.
Solution: We have that F = 4000, r = 5%,
Fr = (4000)(0.05) = 200, n = 40, k = 24 and C = 1200. Hence,
B
24
= Fra
nk

|i
+ C
nk
= (200)a
16

|5%
+ (1200)(1.05)
16
=3419.350452.
c 2009. Miguel A. Arcones. All rights reserved. Manual for SOA Exam FM/CAS Exam 2.
6/??
Chapter 5. Bonds. Section 5.3. Book value and amortization schedules.
Example 2
An nyear 5000 par value bond pays 6% annual coupons. At
annual yield of 3%, the book value of the bond at the end of year
7 is 5520. Calculate the price of the bond.
Solution: We have that F = C = 5000, r = 6%,
Fr = (5000)(0.06) = 300, B
7
= 5520. Hence,
P = B
k
(1 +i )
k
+Fra
k

|i
= 5520(1.03)
7
+(300)a
7

|3%
= 6357.35.
c 2009. Miguel A. Arcones. All rights reserved. Manual for SOA Exam FM/CAS Exam 2.
7/??
Chapter 5. Bonds. Section 5.3. Book value and amortization schedules.
Example 2
An nyear 5000 par value bond pays 6% annual coupons. At
annual yield of 3%, the book value of the bond at the end of year
7 is 5520. Calculate the price of the bond.
Solution: We have that F = C = 5000, r = 6%,
Fr = (5000)(0.06) = 300, B
7
= 5520. Hence,
P = B
k
(1 +i )
k
+Fra
k

|i
= 5520(1.03)
7
+(300)a
7

|3%
= 6357.35.
c 2009. Miguel A. Arcones. All rights reserved. Manual for SOA Exam FM/CAS Exam 2.
8/??
Chapter 5. Bonds. Section 5.3. Book value and amortization schedules.
Inductive relation for book value
Theorem 1
B
k+1
= B
k
(1 + i ) Fr .
Proof:
B
k
(1 + i ) Fr = (Fra
nk

|i
+ C
nk
)(1 + i ) Fr
=Fr ((1 + i )a
nk

|i
1) + C
nk1
=Fra
nk1

|i
+ C
nk1
= B
k+1
.
B
k
is the outstanding balance at time k. One period later, the
principal has increased to B
k
(1 + i ), i.e. B
k
(1 + i ) is the
outstanding balance immediately before the (k + 1)th payment is
made. Immediately after the (k + 1)th payment to principal of Fr
is made, the outstanding balance is B
k+1
= B
k
(1 + i ) Fr .
c 2009. Miguel A. Arcones. All rights reserved. Manual for SOA Exam FM/CAS Exam 2.
9/??
Chapter 5. Bonds. Section 5.3. Book value and amortization schedules.
Inductive relation for book value
Theorem 1
B
k+1
= B
k
(1 + i ) Fr .
Proof:
B
k
(1 + i ) Fr = (Fra
nk

|i
+ C
nk
)(1 + i ) Fr
=Fr ((1 + i )a
nk

|i
1) + C
nk1
=Fra
nk1

|i
+ C
nk1
= B
k+1
.
B
k
is the outstanding balance at time k. One period later, the
principal has increased to B
k
(1 + i ), i.e. B
k
(1 + i ) is the
outstanding balance immediately before the (k + 1)th payment is
made. Immediately after the (k + 1)th payment to principal of Fr
is made, the outstanding balance is B
k+1
= B
k
(1 + i ) Fr .
c 2009. Miguel A. Arcones. All rights reserved. Manual for SOA Exam FM/CAS Exam 2.
10/??
Chapter 5. Bonds. Section 5.3. Book value and amortization schedules.
Inductive relation for book value
Theorem 1
B
k+1
= B
k
(1 + i ) Fr .
Proof:
B
k
(1 + i ) Fr = (Fra
nk

|i
+ C
nk
)(1 + i ) Fr
=Fr ((1 + i )a
nk

|i
1) + C
nk1
=Fra
nk1

|i
+ C
nk1
= B
k+1
.
B
k
is the outstanding balance at time k. One period later, the
principal has increased to B
k
(1 + i ), i.e. B
k
(1 + i ) is the
outstanding balance immediately before the (k + 1)th payment is
made. Immediately after the (k + 1)th payment to principal of Fr
is made, the outstanding balance is B
k+1
= B
k
(1 + i ) Fr .
c 2009. Miguel A. Arcones. All rights reserved. Manual for SOA Exam FM/CAS Exam 2.
11/??
Chapter 5. Bonds. Section 5.3. Book value and amortization schedules.
Example 3
Consider a 30year $50,000 parvalue bond with semiannual
coupons, with r = 0.03, and yield rate 10%, convertible
semiannually.
(i) Find the book value of the bond immediately after the 25th
coupon payment.
(ii) Find book price immediately before the 26th coupon payment.
(iii) Find the book value of the bond immediately after the 26th
coupon payment.
Solution: (i) We have that F = C = 50000, i = 0.05,
Fr = 50000(0.03) = 1500 and n = 60. So,
B
25
= Fra
nk

|i
+C
nk
= 1500a
35

|0.05
+50000(1.05)
35
= 33625.80571.
(ii) Just before the next coupon payment the book value is
33625.80571(1.05) = 35307.096.
(iii) The book value of the bond immediately after the 26th
coupon payment is 35307.096 1500 = 33807.096.
c 2009. Miguel A. Arcones. All rights reserved. Manual for SOA Exam FM/CAS Exam 2.
12/??
Chapter 5. Bonds. Section 5.3. Book value and amortization schedules.
Example 3
Consider a 30year $50,000 parvalue bond with semiannual
coupons, with r = 0.03, and yield rate 10%, convertible
semiannually.
(i) Find the book value of the bond immediately after the 25th
coupon payment.
(ii) Find book price immediately before the 26th coupon payment.
(iii) Find the book value of the bond immediately after the 26th
coupon payment.
Solution: (i) We have that F = C = 50000, i = 0.05,
Fr = 50000(0.03) = 1500 and n = 60. So,
B
25
= Fra
nk

|i
+C
nk
= 1500a
35

|0.05
+50000(1.05)
35
= 33625.80571.
(ii) Just before the next coupon payment the book value is
33625.80571(1.05) = 35307.096.
(iii) The book value of the bond immediately after the 26th
coupon payment is 35307.096 1500 = 33807.096.
c 2009. Miguel A. Arcones. All rights reserved. Manual for SOA Exam FM/CAS Exam 2.
13/??
Chapter 5. Bonds. Section 5.3. Book value and amortization schedules.
Example 3
Consider a 30year $50,000 parvalue bond with semiannual
coupons, with r = 0.03, and yield rate 10%, convertible
semiannually.
(i) Find the book value of the bond immediately after the 25th
coupon payment.
(ii) Find book price immediately before the 26th coupon payment.
(iii) Find the book value of the bond immediately after the 26th
coupon payment.
Solution: (i) We have that F = C = 50000, i = 0.05,
Fr = 50000(0.03) = 1500 and n = 60. So,
B
25
= Fra
nk

|i
+C
nk
= 1500a
35

|0.05
+50000(1.05)
35
= 33625.80571.
(ii) Just before the next coupon payment the book value is
33625.80571(1.05) = 35307.096.
(iii) The book value of the bond immediately after the 26th
coupon payment is 35307.096 1500 = 33807.096.
c 2009. Miguel A. Arcones. All rights reserved. Manual for SOA Exam FM/CAS Exam 2.
14/??
Chapter 5. Bonds. Section 5.3. Book value and amortization schedules.
Example 3
Consider a 30year $50,000 parvalue bond with semiannual
coupons, with r = 0.03, and yield rate 10%, convertible
semiannually.
(i) Find the book value of the bond immediately after the 25th
coupon payment.
(ii) Find book price immediately before the 26th coupon payment.
(iii) Find the book value of the bond immediately after the 26th
coupon payment.
Solution: (i) We have that F = C = 50000, i = 0.05,
Fr = 50000(0.03) = 1500 and n = 60. So,
B
25
= Fra
nk

|i
+C
nk
= 1500a
35

|0.05
+50000(1.05)
35
= 33625.80571.
(ii) Just before the next coupon payment the book value is
33625.80571(1.05) = 35307.096.
(iii) The book value of the bond immediately after the 26th
coupon payment is 35307.096 1500 = 33807.096.
c 2009. Miguel A. Arcones. All rights reserved. Manual for SOA Exam FM/CAS Exam 2.
15/??
Chapter 5. Bonds. Section 5.3. Book value and amortization schedules.
General inductive relation for book value
By induction from the previous formula, we get that
B
k+m
= B
k
(1 + i )
m
Frs
m

|i
.
Notice that:

B
k+m
is the outstanding balance immediately after the k + m
payment.

B
k
(1 + i )
m
is the accrued balance at time k + m of the
outstanding balance immediately after the k payment.

Frs
m

|i
is the future value at time k + m of the coupon
payments k + 1, k + 2, . . . , k + m, i.e. the coupons payments
from when the outstanding was B
k
until when the
outstanding is B
k+m
.
c 2009. Miguel A. Arcones. All rights reserved. Manual for SOA Exam FM/CAS Exam 2.
16/??
Chapter 5. Bonds. Section 5.3. Book value and amortization schedules.
Example 4
An nyear 4000 par value bond with 9% semiannual coupons has
an annual nominal yield of i , i > 0, convertible semiannually. The
book value of the bond at the end of year 4 is 3812.13 and the
book value at the end of year 7 is 3884.27. Calculate i .
Solution: We have that F = C = 4000, r = 4.5% and
Fr = 4000(0.045) = 180. The end of year 4 is the end of the 8th
period. The end of year 7 is the end of the 14th period. Hence,
B
8
= 3812.13 and B
14
= 3884.27 and
3884.27 = 3812.13(1 + i )
6
(180)s
6

|i
.
using the calculator with
6 N 3812.13 PV 180 PMT 3884.27 FV CPT I/Y
we get that i = 5% (this is the eective interest rate per period).
The annual nominal rate of interest convertible semiannually is
i = 10%.
c 2009. Miguel A. Arcones. All rights reserved. Manual for SOA Exam FM/CAS Exam 2.
17/??
Chapter 5. Bonds. Section 5.3. Book value and amortization schedules.
Example 4
An nyear 4000 par value bond with 9% semiannual coupons has
an annual nominal yield of i , i > 0, convertible semiannually. The
book value of the bond at the end of year 4 is 3812.13 and the
book value at the end of year 7 is 3884.27. Calculate i .
Solution: We have that F = C = 4000, r = 4.5% and
Fr = 4000(0.045) = 180. The end of year 4 is the end of the 8th
period. The end of year 7 is the end of the 14th period. Hence,
B
8
= 3812.13 and B
14
= 3884.27 and
3884.27 = 3812.13(1 + i )
6
(180)s
6

|i
.
using the calculator with
6 N 3812.13 PV 180 PMT 3884.27 FV CPT I/Y
we get that i = 5% (this is the eective interest rate per period).
The annual nominal rate of interest convertible semiannually is
i = 10%.
c 2009. Miguel A. Arcones. All rights reserved. Manual for SOA Exam FM/CAS Exam 2.
18/??
Chapter 5. Bonds. Section 5.3. Book value and amortization schedules.
Amount of interest contained in the kth coupon.
The amount of interest contained in the kth coupon is
I
k
= iB
k1
, which can obtained using any of the following
formulas:
I
k
= iB
k1
= Fr (Fr Ci )
nk+1
= Cg C(g i )
nk+1
.
Notice that
iB
k1
= iFra
n+1k

|i
+ iC
nk+1
= Fr (1
nk+1
) + iC
nk+1
=Fr (Fr Ci )
nk+1
= Cg (Cg Ci )
nk+1
=Cg C(g i )
nk+1
.
c 2009. Miguel A. Arcones. All rights reserved. Manual for SOA Exam FM/CAS Exam 2.
19/??
Chapter 5. Bonds. Section 5.3. Book value and amortization schedules.
Principal portion in the the kth coupon.
The principal portion in the kth coupon is
P
k
= B
k1
B
k
= Fr B
k1
i = Fr I
k
and it can be obtained
using any of the following formulas:
P
k
= (Fr Ci )
nk+1
= C(g i )
nk+1
.
P
k
is the change in the book value of the bond (principal
adjustment) between times k 1 and k. P
k
could be either
negative, or zero or positive. P
k
is the amortization in the kth
payment.
c 2009. Miguel A. Arcones. All rights reserved. Manual for SOA Exam FM/CAS Exam 2.
20/??
Chapter 5. Bonds. Section 5.3. Book value and amortization schedules.
Example 5
Kendal buys a 5000 parvalue 10 year bond with 8% semiannual
coupons to yield 4% converted semiannually. Find the amount of
interest and principal in the 5th coupon.
Solution: We have that F = C = 5000, r = g = 0.04, Fr = 200,
n = 20 and i = 2%. Using that I
k
= Fr (Fr Ci )
n+1k
and
P
k
= (Fr Ci )
n+1k
, we get that
I
5
= 200 (200 (5000)(0.02))(1.02)
(20+15)
= 127.1554
and
P
5
= (200 (5000)(0.02))(1.02)
(20+15)
= 72.8446.
c 2009. Miguel A. Arcones. All rights reserved. Manual for SOA Exam FM/CAS Exam 2.
21/??
Chapter 5. Bonds. Section 5.3. Book value and amortization schedules.
Example 5
Kendal buys a 5000 parvalue 10 year bond with 8% semiannual
coupons to yield 4% converted semiannually. Find the amount of
interest and principal in the 5th coupon.
Solution: We have that F = C = 5000, r = g = 0.04, Fr = 200,
n = 20 and i = 2%. Using that I
k
= Fr (Fr Ci )
n+1k
and
P
k
= (Fr Ci )
n+1k
, we get that
I
5
= 200 (200 (5000)(0.02))(1.02)
(20+15)
= 127.1554
and
P
5
= (200 (5000)(0.02))(1.02)
(20+15)
= 72.8446.
c 2009. Miguel A. Arcones. All rights reserved. Manual for SOA Exam FM/CAS Exam 2.
22/??
Chapter 5. Bonds. Section 5.3. Book value and amortization schedules.
Example 6
For a bond pays annual coupons. The principal amortized in the
rst coupon is half that amortized in the 10th coupon. Calculate
the annual yield rate.
Solution: We know that (Fr Ci )
n
=
1
2
(Fr Ci )
n9
. Hence,
(1 + i )
9
= 2 and i = 8.005973889%.
c 2009. Miguel A. Arcones. All rights reserved. Manual for SOA Exam FM/CAS Exam 2.
23/??
Chapter 5. Bonds. Section 5.3. Book value and amortization schedules.
Example 6
For a bond pays annual coupons. The principal amortized in the
rst coupon is half that amortized in the 10th coupon. Calculate
the annual yield rate.
Solution: We know that (Fr Ci )
n
=
1
2
(Fr Ci )
n9
. Hence,
(1 + i )
9
= 2 and i = 8.005973889%.
c 2009. Miguel A. Arcones. All rights reserved. Manual for SOA Exam FM/CAS Exam 2.
24/??
Chapter 5. Bonds. Section 5.3. Book value and amortization schedules.
The amortization schedule of a bond is
Time Payment Interest paid (iB
k1
) Principal repaid Book value (B
k
)
0 C + (Fr Ci )a
n

|i
1 Fr Fr (Fr Ci )
n
(Fr Ci )
n
C + (Fr Ci )a
n1

|i
2 Fr Fr (Fr Ci )
n1
(Fr Ci )
n1
C + (Fr Ci )a
n2

|i

k Fr Fr (Fr Ci )
nk+1
(Fr Ci )
n+1k
C + (Fr Ci )a
nk

|i

n 1 Fr Fr (Fr Ci )
2
(Fr Ci )
2
C + (Fr Ci )a
1

|i
n Fr + C Fr (Fr Ci ) (Fr Ci ) + C 0
The total payments in a bond are nFr + C.
The total coupon interest (sum of the column of interest
payments) is nFr + C P.
The total coupon principal (sum of the column of payments to
principal) is P.
c 2009. Miguel A. Arcones. All rights reserved. Manual for SOA Exam FM/CAS Exam 2.
25/??
Chapter 5. Bonds. Section 5.3. Book value and amortization schedules.
Notice that for each time k the interest paid plus the principal paid
equal to the total payment. We also have that
I
k
= iB
k1
= i

C + (Fr Ci )a
nk+1

|i

= iC + i (Fr Ci )a
nk+1

|i
=Fr + (Fr Ci )(1
nk+1
) = Fr (Fr Ci )
nk+1
.
and
B
k
= B
k1
(Fr Ci )
n+1k
=C + (Fr Ci )a
nk+1

|i
(Fr Ci )
n+1k
=C + (Fr Ci )(a
nk+1

|i

n+1k
) = C + (Fr Ci )a
nk

|i
.
c 2009. Miguel A. Arcones. All rights reserved. Manual for SOA Exam FM/CAS Exam 2.
26/??
Chapter 5. Bonds. Section 5.3. Book value and amortization schedules.
Example 7
A 1000 parvalue 3year bond pays 6%, convertible semiannually,
and has a yield rate of 8%, convertible semiannually.
(i) What is the interest paid in the 3
rd
coupon?
(ii) What is the change in book value contained in the 3
rd
coupon?
(iii) Construct a bond amortization schedule.
Solution:
c 2009. Miguel A. Arcones. All rights reserved. Manual for SOA Exam FM/CAS Exam 2.
27/??
Chapter 5. Bonds. Section 5.3. Book value and amortization schedules.
Example 7
A 1000 parvalue 3year bond pays 6%, convertible semiannually,
and has a yield rate of 8%, convertible semiannually.
(i) What is the interest paid in the 3
rd
coupon?
(ii) What is the change in book value contained in the 3
rd
coupon?
(iii) Construct a bond amortization schedule.
Solution:
(i) We know that F = C = 1000, r = 0.03, Fr = 30, n = 6,
i = 0.04 and Ci = 40. The interest paid in the 3
rd
coupon is
I
3
= Fr (Fr Ci )
nk+1
= 30(3040)(1.04)
4
= 38.54804191.
We also can do
B
2
= 30a
4

|0.04
+ 1000(1.04)
4
= 963.7010478
and I
3
= B
2
(0.04) = 38.54804191.
c 2009. Miguel A. Arcones. All rights reserved. Manual for SOA Exam FM/CAS Exam 2.
28/??
Chapter 5. Bonds. Section 5.3. Book value and amortization schedules.
Example 7
A 1000 parvalue 3year bond pays 6%, convertible semiannually,
and has a yield rate of 8%, convertible semiannually.
(i) What is the interest paid in the 3
rd
coupon?
(ii) What is the change in book value contained in the 3
rd
coupon?
(iii) Construct a bond amortization schedule.
Solution:
(ii) Since I
3
> Fr , the book value increases in the 3
rd
coupon.
The increase in the book value in the 3
rd
coupon is I
3
Fr =
38.54804191 30 = 8.54804191. We also can do
B
3
= 30a
3

|0.04
+ 1000(1.04)
3
= 972.2490897
and B
3
B
2
= 972.2490897 963.7010478 = 8.5480419.
c 2009. Miguel A. Arcones. All rights reserved. Manual for SOA Exam FM/CAS Exam 2.
29/??
Chapter 5. Bonds. Section 5.3. Book value and amortization schedules.
(iii) Here is the bond amortization schedule:
Principal Book
Time Coupon Interest Adjustment Value
0 947.58
1 30 37.90 7.90 955.48
2 30 38.22 8.22 963.70
3 30 38.59 8.59 972.25
4 30 38.89 8.89 981.14
5 30 39.25 9.25 990.37
6 30 39.62 9.62 1000.00
c 2009. Miguel A. Arcones. All rights reserved. Manual for SOA Exam FM/CAS Exam 2.
30/??
Chapter 5. Bonds. Section 5.3. Book value and amortization schedules.
The price of a typical bond changes in the opposite direction from
a change in interest rates.
As interest rates rise, the price of a bond falls. A bond assures a
determined number of payments in the future, if interest rates rise,
the present value of these payments decreases. We make an
unrealized capital loss, if the market value is less than the book
value.
Reciprocally, if interest rates decline, the price of a bond rises. The
market value of a bond is the price at which a bond is
bought/sold. When rates of interest change, the market value of a
bond changes. We make an unrealized capital gain, if the market
value is bigger than the book value.
c 2009. Miguel A. Arcones. All rights reserved. Manual for SOA Exam FM/CAS Exam 2.
31/??
Chapter 5. Bonds. Section 5.3. Book value and amortization schedules.
Example 8
Oliver buys a tenyear 5000 face value bond with semiannual
coupons at annual rate of 6%. He buys his bond to yield 8%
compounded semiannually and immediately sell them to an
investor to yield 4% compounded semiannually. What is Olivers
prot in this investment?
Solution: We have that F = C = 5000, r = 0.03, Fr = 150 and
n = 20. Oliver buys his bond for
(150)a
20

|0.04
+ 5000(1.04)
20
= 4320.484
Oliver sells his bond for
(150)a
20

|0.02
+ 5000(1.02)
20
= 5817.572
Olivers prot is 5817.572 4320.484 = 1497.088.
c 2009. Miguel A. Arcones. All rights reserved. Manual for SOA Exam FM/CAS Exam 2.
32/??
Chapter 5. Bonds. Section 5.3. Book value and amortization schedules.
Example 8
Oliver buys a tenyear 5000 face value bond with semiannual
coupons at annual rate of 6%. He buys his bond to yield 8%
compounded semiannually and immediately sell them to an
investor to yield 4% compounded semiannually. What is Olivers
prot in this investment?
Solution: We have that F = C = 5000, r = 0.03, Fr = 150 and
n = 20. Oliver buys his bond for
(150)a
20

|0.04
+ 5000(1.04)
20
= 4320.484
Oliver sells his bond for
(150)a
20

|0.02
+ 5000(1.02)
20
= 5817.572
Olivers prot is 5817.572 4320.484 = 1497.088.
c 2009. Miguel A. Arcones. All rights reserved. Manual for SOA Exam FM/CAS Exam 2.
33/??
Chapter 5. Bonds. Section 5.3. Book value and amortization schedules.
Example 9
On January 1, 2000, Maxwell bought a 10year $5000 noncallable
bond with coupons at 7% convertible semiannually. Maxwell
bought the bond to yield 7%, compounded semiannually. On July
1, 2005, the market value of bonds is based on a 5% interest rate,
compounded semiannually. Calculate the unrealized capital gain on
July 1, 2005.
Solution: We have that F = 5000, r = 0.035 and Fr = 175. On
July 1, 2005, there are nine remaining coupons. On July 1, 2005,
the book value of the bond is
175a
9

|3.5%
+ 5000(1.035)
9
= 5000.
On July 1, 2005, the market value of the bond is
175a
9

|2.5%
+ 5000(1.025)
9
= 5398.543276.
The unrealized capital gain is 5398.543276 5000 = 398.543276.
c 2009. Miguel A. Arcones. All rights reserved. Manual for SOA Exam FM/CAS Exam 2.
34/??
Chapter 5. Bonds. Section 5.3. Book value and amortization schedules.
Example 9
On January 1, 2000, Maxwell bought a 10year $5000 noncallable
bond with coupons at 7% convertible semiannually. Maxwell
bought the bond to yield 7%, compounded semiannually. On July
1, 2005, the market value of bonds is based on a 5% interest rate,
compounded semiannually. Calculate the unrealized capital gain on
July 1, 2005.
Solution: We have that F = 5000, r = 0.035 and Fr = 175. On
July 1, 2005, there are nine remaining coupons. On July 1, 2005,
the book value of the bond is
175a
9

|3.5%
+ 5000(1.035)
9
= 5000.
On July 1, 2005, the market value of the bond is
175a
9

|2.5%
+ 5000(1.025)
9
= 5398.543276.
The unrealized capital gain is 5398.543276 5000 = 398.543276.
c 2009. Miguel A. Arcones. All rights reserved. Manual for SOA Exam FM/CAS Exam 2.
35/??
Chapter 5. Bonds. Section 5.3. Book value and amortization schedules.
Premium.
If the investor is paying more than the redemption value, i.e. if
P > C, we say that the bond has being bought at premium. The
premium is P C. We have that
P C = (Fr Ci )a
n

|i
= C(g i )a
n

|i
.
A bond has been bought at premium if and only if g > i . For a
bond bought at premium Fr > Ci and
P = B
0
> B
1
> B
2
> > B
n
= C. Notice that
B
k
C = (Fr Ci )a
nk

|i
. P
k
= B
k1
B
k
= (Fr Ci )
n+1k
is
the writeup in premium in the kth coupon. The premium is
P C =

n
j =1
(B
j 1
B
j
).
c 2009. Miguel A. Arcones. All rights reserved. Manual for SOA Exam FM/CAS Exam 2.
36/??
Chapter 5. Bonds. Section 5.3. Book value and amortization schedules.
Discount.
If the investor is paying less than the redemption value, i.e. if
P < C, we say that the bond has being bought at discount. The
discount is C P. We have that
C P = (Ci Fr )a
n

|i
= C(i g)a
n

|i
.
A bond has been bought at discount if and only if g < i . For a
bond bought at discount Fr < Ci and
P = B
0
< B
1
< B
2
< < B
n
= C.
|P
k
| = B
k
B
k1
= (Ci Fr )
n+1k
is the writeup in discount
in the kth coupon. The discount is C P =

n
j =1
(B
j
B
j 1
).
c 2009. Miguel A. Arcones. All rights reserved. Manual for SOA Exam FM/CAS Exam 2.
37/??
Chapter 5. Bonds. Section 5.3. Book value and amortization schedules.
Example 10
A 10 year 50000 face value bond pays semiannual coupons of
3000. The bond is bought to yield a nominal annual interest rate
of 6% convertible semiannually. Calculate the premium paid for
the bond.
Solution: We know that F = C = 50000, Fr = 3000, n = 20, and
i = 3%. The price of the bond is
P = Fra
n

|i
+ C(1 + i )
n
= (3000)a
20

|3%
+ (50000)(1.03)
20
=72316.21229.
The premium of the bond is
P C = 72316.21229 50000 = 22316.21229.
c 2009. Miguel A. Arcones. All rights reserved. Manual for SOA Exam FM/CAS Exam 2.
38/??
Chapter 5. Bonds. Section 5.3. Book value and amortization schedules.
Example 10
A 10 year 50000 face value bond pays semiannual coupons of
3000. The bond is bought to yield a nominal annual interest rate
of 6% convertible semiannually. Calculate the premium paid for
the bond.
Solution: We know that F = C = 50000, Fr = 3000, n = 20, and
i = 3%. The price of the bond is
P = Fra
n

|i
+ C(1 + i )
n
= (3000)a
20

|3%
+ (50000)(1.03)
20
=72316.21229.
The premium of the bond is
P C = 72316.21229 50000 = 22316.21229.
c 2009. Miguel A. Arcones. All rights reserved. Manual for SOA Exam FM/CAS Exam 2.
39/??
Chapter 5. Bonds. Section 5.3. Book value and amortization schedules.
Example 11
Oprah buys a 10000 parvalue 15 year bond with 9% semiannual
coupons to yield 5% converted semiannually.
(i) Find the premium in the bond.
(ii) Find the write up in premium in the 8th coupon.
Solution: (i) We have that F = C = 10000, r = 0.045, Fr = 450
and n = 30. Oprah buys her bond for
(450)a
30|2.5%
+ 10000(1.025)
30
= 14186.06
The premium of the bond is P C = 14186.06 10000 = 4186.06.
(ii) The writeup in premium in the 8th coupon is
(Fr Ci )
n+1k
= (10000)(0.045 0.025)(1.025)
23
= 113.34.
We also can do the problem in the following way:
B
7
= (450)a
23

|2.5%
+ 10000(1.025)
23
= 13466.42
B
8
= (450)a
22

|2.5%
+ 10000(1.025)
22
= 13353.08
and B
7
B
8
= 13466.42 13353.08 = 113.34.
c 2009. Miguel A. Arcones. All rights reserved. Manual for SOA Exam FM/CAS Exam 2.
40/??
Chapter 5. Bonds. Section 5.3. Book value and amortization schedules.
Example 11
Oprah buys a 10000 parvalue 15 year bond with 9% semiannual
coupons to yield 5% converted semiannually.
(i) Find the premium in the bond.
(ii) Find the write up in premium in the 8th coupon.
Solution: (i) We have that F = C = 10000, r = 0.045, Fr = 450
and n = 30. Oprah buys her bond for
(450)a
30|2.5%
+ 10000(1.025)
30
= 14186.06
The premium of the bond is P C = 14186.06 10000 = 4186.06.
(ii) The writeup in premium in the 8th coupon is
(Fr Ci )
n+1k
= (10000)(0.045 0.025)(1.025)
23
= 113.34.
We also can do the problem in the following way:
B
7
= (450)a
23

|2.5%
+ 10000(1.025)
23
= 13466.42
B
8
= (450)a
22

|2.5%
+ 10000(1.025)
22
= 13353.08
and B
7
B
8
= 13466.42 13353.08 = 113.34.
c 2009. Miguel A. Arcones. All rights reserved. Manual for SOA Exam FM/CAS Exam 2.
41/??
Chapter 5. Bonds. Section 5.3. Book value and amortization schedules.
Example 11
Oprah buys a 10000 parvalue 15 year bond with 9% semiannual
coupons to yield 5% converted semiannually.
(i) Find the premium in the bond.
(ii) Find the write up in premium in the 8th coupon.
Solution: (i) We have that F = C = 10000, r = 0.045, Fr = 450
and n = 30. Oprah buys her bond for
(450)a
30|2.5%
+ 10000(1.025)
30
= 14186.06
The premium of the bond is P C = 14186.06 10000 = 4186.06.
(ii) The writeup in premium in the 8th coupon is
(Fr Ci )
n+1k
= (10000)(0.045 0.025)(1.025)
23
= 113.34.
We also can do the problem in the following way:
B
7
= (450)a
23

|2.5%
+ 10000(1.025)
23
= 13466.42
B
8
= (450)a
22

|2.5%
+ 10000(1.025)
22
= 13353.08
and B
7
B
8
= 13466.42 13353.08 = 113.34.
c 2009. Miguel A. Arcones. All rights reserved. Manual for SOA Exam FM/CAS Exam 2.
1/10
Chapter 5. Bonds.
Manual for SOA Exam FM/CAS Exam 2.
Chapter 5. Bonds.
Section 5.4. Book value between coupon payments dates.
c 2009. Miguel A. Arcones. All rights reserved.
Extract from:
Arcones Manual for the SOA Exam FM/CAS Exam 2,
Financial Mathematics. Fall 2009 Edition,
available at http://www.actexmadriver.com/
c 2009. Miguel A. Arcones. All rights reserved. Manual for SOA Exam FM/CAS Exam 2.
2/10
Chapter 5. Bonds. Section 5.4. Book value between coupon payments dates.
Book value between coupon payments dates
We know that the book value of a bond immediately after a
coupon payment is given by
B
k
= Fra
nk|i
+C
nk
= C +(Fr Ci )a
nk|i
= C +C(g i )a
nk|i
.
We want to determine the book value of a bond between
successive coupon dates. We use as a unit of time coupon periods.
We want to determine the value of a bond at time k + t periods,
where k is an integer and 0 t < 1. The present value at time
k + t of the remaining payments is B
k
(1 + i )
t
. Immediately before
the payment of the (k + 1)th coupon, the price of the bond is
B
k
(1 + i ). Immediately after the payment of the (k + 1)th
coupon, the price of the bond is B
k
(1 + i ) Fr . Hence, the price
of a bond is a discontinuous function (see Figure 1).
c 2009. Miguel A. Arcones. All rights reserved. Manual for SOA Exam FM/CAS Exam 2.
3/10
Chapter 5. Bonds. Section 5.4. Book value between coupon payments dates.
Figure 1: Theoretical at and market values
c 2009. Miguel A. Arcones. All rights reserved. Manual for SOA Exam FM/CAS Exam 2.
4/10
Chapter 5. Bonds. Section 5.4. Book value between coupon payments dates.
During the time period (k, k + 1) a bond accrues a coupon
payment. The accrued value at time t [0, 1) of the next coupon
is denoted by Fr
t
. We have that Fr
0
= 0, lim
t1
Fr
t
= Fr . The
at price of a bond is the money that actually changes hands at
the date of sale. The market price B
m
k+t
is the price of a bond
excluding the accrued value of the next coupon. Hence, we have
that B
f
k+t
= B
m
k+t
+ Fr
t
.
The at price (also known as the dirty price) is the book value of a
bond. It is the price that an investor pays for a bond. The market
price (also known as the clean price) is the price of bond quoted in
a newspaper.
c 2009. Miguel A. Arcones. All rights reserved. Manual for SOA Exam FM/CAS Exam 2.
5/10
Chapter 5. Bonds. Section 5.4. Book value between coupon payments dates.
There are three methods to determine the at value, accrued
coupon and market value of a bond:

Theoretical method:
B
f
k+t
= B
k
(1 + i )
t
, Fr
t
= Fr

(1 + i )
t
1
i

B
m
k+t
= B
k
(1 + i )
t
Fr

(1 + i )
t
1
i

Practical method:
B
f
k+t
= B
k
(1 + ti ), Fr
t
= tFr , B
m
k+t
= B
k
(1 + ti ) tFr

Semitheoretical method:
B
f
k+t
= B
k
(1 + i )
t
, Fr
t
= tFr , B
m
k+t
= B
k
(1 + i )
t
tFr
c 2009. Miguel A. Arcones. All rights reserved. Manual for SOA Exam FM/CAS Exam 2.
6/10
Chapter 5. Bonds. Section 5.4. Book value between coupon payments dates.
The practical method assumes that the at price accrues under
simple interest. It assumes that the accrued coupon is proportional
to the time since the last coupon payment.
As to the theoretical method, B
k
(1 +i )
t
is the present value of the
payments to be made. This is actual outstanding balance in the
loan at time k + t. We have that
B
m
k+t
= B
k
(1 + i )
t
Fr

(1 + i )
t
1
i

Fr

1
nk
i

+ C
nk

t
Fr

t
1
i

=Fr

1
nkt
i

+ C
nkt
= Fra
nkt|i
+ C
nkt
,
where a
s|i
=
1
s
i
, s > 0 and s is not necessarily a positive integer.
The market price according with the theoretical method has a
continuous function.
c 2009. Miguel A. Arcones. All rights reserved. Manual for SOA Exam FM/CAS Exam 2.
7/10
Chapter 5. Bonds. Section 5.4. Book value between coupon payments dates.
Example 1
Find the at price, the accrued interest and the market price of a
1000 10year bond with 4% annual coupons, bought to yield 3%,
four months after the second coupon has been issued. Use all
three methods.
Solution: We have that F = 1000, n = 10, r = 0.04, Fr = 40,
i = 3%, k = 2 and t =
1
3
. So,
B
2
= Fra
nk

|i
+ C
nk
= 40a
8

|3%
+ (1000)(1.03)
8
= 1070.20.
c 2009. Miguel A. Arcones. All rights reserved. Manual for SOA Exam FM/CAS Exam 2.
8/10
Chapter 5. Bonds. Section 5.4. Book value between coupon payments dates.
Example 1
Find the at price, the accrued interest and the market price of a
1000 10year bond with 4% annual coupons, bought to yield 3%,
four months after the second coupon has been issued. Use all
three methods.
Solution: We have that F = 1000, n = 10, r = 0.04, Fr = 40,
i = 3%, k = 2 and t =
1
3
. So,
B
2
= Fra
nk

|i
+ C
nk
= 40a
8

|3%
+ (1000)(1.03)
8
= 1070.20.
Using the theoretical method,
B
f
2+(1/3)
= B
2
(1 + i )
1/3
= (1070.20)(1.03)
1/3
= 1080.80,
Fr
(1/3)
= Fr

(1 + i )
1/3
1
i

= (40)

(1 + 0.03)
1/3
1
0.03

= 13.20
B
m
2+(1/3)
= B
f
2+(1/3)
Fr
(1/3)
= 1080.80 13.20 = 1067.60.
c 2009. Miguel A. Arcones. All rights reserved. Manual for SOA Exam FM/CAS Exam 2.
9/10
Chapter 5. Bonds. Section 5.4. Book value between coupon payments dates.
Example 1
Find the at price, the accrued interest and the market price of a
1000 10year bond with 4% annual coupons, bought to yield 3%,
four months after the second coupon has been issued. Use all
three methods.
Solution: We have that F = 1000, n = 10, r = 0.04, Fr = 40,
i = 3%, k = 2 and t =
1
3
. So,
B
2
= Fra
nk

|i
+ C
nk
= 40a
8

|3%
+ (1000)(1.03)
8
= 1070.20.
Using the practical method,
B
f
2+(1/3)
= B
2
(1 + (1/3)i ) = (1070.20)(1 + (0.03)/3)) = 1080.90,
Fr
(1/3)
= (1/3)Fr = (1/3)(40) = 13.33
B
m
2+(1/3)
= B
f
2+(1/3)
Fr
(1/3)
= 1080.90 13.33 = 1067.57
c 2009. Miguel A. Arcones. All rights reserved. Manual for SOA Exam FM/CAS Exam 2.
10/10
Chapter 5. Bonds. Section 5.4. Book value between coupon payments dates.
Example 1
Find the at price, the accrued interest and the market price of a
1000 10year bond with 4% annual coupons, bought to yield 3%,
four months after the second coupon has been issued. Use all
three methods.
Solution: We have that F = 1000, n = 10, r = 0.04, Fr = 40,
i = 3%, k = 2 and t =
1
3
. So,
B
2
= Fra
nk

|i
+ C
nk
= 40a
8

|3%
+ (1000)(1.03)
8
= 1070.20.
Using the semitheoretical method:
B
f
2+(1/3)
= B
2
(1 + i )
1/3
= (1070.20)(1.03)
1/3
= 1080.80,
Fr
(1/3)
= (1/3)Fr = (1/3)(40) = 13.33,
B
m
2+(1/3)
= B
f
2+(1/3)
Fr
(1/3)
= 1080.79 13.33 = 1067.46.
c 2009. Miguel A. Arcones. All rights reserved. Manual for SOA Exam FM/CAS Exam 2.
1/10
Chapter 5. Bonds.
Manual for SOA Exam FM/CAS Exam 2.
Chapter 5. Bonds.
Section 5.5. Callable bonds.
c 2009. Miguel A. Arcones. All rights reserved.
Extract from:
Arcones Manual for the SOA Exam FM/CAS Exam 2,
Financial Mathematics. Fall 2009 Edition,
available at http://www.actexmadriver.com/
c 2009. Miguel A. Arcones. All rights reserved. Manual for SOA Exam FM/CAS Exam 2.
2/10
Chapter 5. Bonds. Section 5.5. Callable bonds.
Callable bonds
A callable bond is a bond which gives the issuer (not the investor)
the right to redeem prior to its maturity date, under certain
conditions. When issued, the call provisions explain when the
bond can be redeemed and what the price will be. In most cases,
there is some period of time during which the bond cannot be
called. This period of time is named the call protection period.
The earliest time to call the bond is named the call date. The call
price is the amount of money the insurer must pay to buy the
bond back.
Usually, bonds can be only called immediately after the payment of
a coupon. We will study the computation of the yield rate of
return for the investor in this situation. Since the investor does not
know the cashow obtained from his investment, he will assume
that the issuer calls the bond under the worst possible situation (in
the sense of lowest possible interest rates).
c 2009. Miguel A. Arcones. All rights reserved. Manual for SOA Exam FM/CAS Exam 2.
3/10
Chapter 5. Bonds. Section 5.5. Callable bonds.
If the bond is called immediately after the payment of kth
coupon, the present value of the obtained payments is
P
k
= Fra
k

|i
+ C
k
= C + (Fr Ci )a
k

|i
= C + C(g i )a
k

|i
.
P
k
is the price which the investor would pay for the bond assuming
that the bond is called immediately after the k coupon. As smaller
as P
k
is, as worst for the lender is. Between all possible choices to
recall a bond, the borrower will choose the option with the smallest
price. Assuming that the redemption value is a constant and that a
bond can be called after any coupon payment:
(i) if Fr > Ci (bond sells at a premium), P
k
increases with k, and
we assume the redemption date is the earliest possible.
(ii) if Fr < Ci (bond sells at a discount), P
k
decreases with k, and
we assume that the redemption date is the latest possible.
If one investor wants to get an eective rate of interest of i per
period, then the maximum price which the investor should pay is
the lowest possible P
k
under that particular rate i .
c 2009. Miguel A. Arcones. All rights reserved. Manual for SOA Exam FM/CAS Exam 2.
4/10
Chapter 5. Bonds. Section 5.5. Callable bonds.
Example 1
Consider a 100 parvalue 8% bond with semiannual coupons
callable at 120 on any coupon date starting 5 years after issue for
the next 5 years, at 110 starting 10 years after issue for the next 5
years and maturing at 105 at the end of 15 years. What is the
highest price which an investor can pay and still be certain of a
yield of 9% converted semiannually.
Solution: We have that F = 100, r = 4%, Fr = 4, n = 30 and
i = 4.5%. Let k be the number of the half year when the bond is
called.
c 2009. Miguel A. Arcones. All rights reserved. Manual for SOA Exam FM/CAS Exam 2.
5/10
Chapter 5. Bonds. Section 5.5. Callable bonds.
Example 1
Consider a 100 parvalue 8% bond with semiannual coupons
callable at 120 on any coupon date starting 5 years after issue for
the next 5 years, at 110 starting 10 years after issue for the next 5
years and maturing at 105 at the end of 15 years. What is the
highest price which an investor can pay and still be certain of a
yield of 9% converted semiannually.
Solution: We have that F = 100, r = 4%, Fr = 4, n = 30 and
i = 4.5%. Let k be the number of the half year when the bond is
called.
If 10 k 19, then C = 120 and Ci = (120)(0.045) = 5.4. So,
the bond sells at a discount. We have that lowest price which the
investor can get is
P
19
= 4a
19

|4.5%
+ 120(1.045)
19
= 102.37.
c 2009. Miguel A. Arcones. All rights reserved. Manual for SOA Exam FM/CAS Exam 2.
6/10
Chapter 5. Bonds. Section 5.5. Callable bonds.
Example 1
Consider a 100 parvalue 8% bond with semiannual coupons
callable at 120 on any coupon date starting 5 years after issue for
the next 5 years, at 110 starting 10 years after issue for the next 5
years and maturing at 105 at the end of 15 years. What is the
highest price which an investor can pay and still be certain of a
yield of 9% converted semiannually.
Solution: We have that F = 100, r = 4%, Fr = 4, n = 30 and
i = 4.5%. Let k be the number of the half year when the bond is
called.
If 20 k 29, then C = 110 and Ci = (110)(0.045) = 4.95. So,
the bond sells at a discount. We have that lowest price which the
investor can get is
P
29
= 4a
29

|4.5%
+ (110)(1.045)
29
= 94.78.
c 2009. Miguel A. Arcones. All rights reserved. Manual for SOA Exam FM/CAS Exam 2.
7/10
Chapter 5. Bonds. Section 5.5. Callable bonds.
Example 1
Consider a 100 parvalue 8% bond with semiannual coupons
callable at 120 on any coupon date starting 5 years after issue for
the next 5 years, at 110 starting 10 years after issue for the next 5
years and maturing at 105 at the end of 15 years. What is the
highest price which an investor can pay and still be certain of a
yield of 9% converted semiannually.
Solution: We have that F = 100, r = 4%, Fr = 4, n = 30 and
i = 4.5%. Let k be the number of the half year when the bond is
called.
If k = 30, then the price is
P
30
= 4a
30

|4.5%
+ 105(1.045)
30
= 93.19.
c 2009. Miguel A. Arcones. All rights reserved. Manual for SOA Exam FM/CAS Exam 2.
8/10
Chapter 5. Bonds. Section 5.5. Callable bonds.
Example 1
Consider a 100 parvalue 8% bond with semiannual coupons
callable at 120 on any coupon date starting 5 years after issue for
the next 5 years, at 110 starting 10 years after issue for the next 5
years and maturing at 105 at the end of 15 years. What is the
highest price which an investor can pay and still be certain of a
yield of 9% converted semiannually.
Solution: We have that F = 100, r = 4%, Fr = 4, n = 30 and
i = 4.5%. Let k be the number of the half year when the bond is
called.
We conclude that the highest price which an investor can pay and
still be certain of a yield of 9% converted semiannually is 93.19.
c 2009. Miguel A. Arcones. All rights reserved. Manual for SOA Exam FM/CAS Exam 2.
9/10
Chapter 5. Bonds. Section 5.5. Callable bonds.
Example 2
Joshua paid 800 for a 15-year 1000 par value bond with
semiannual coupons at a nominal annual rate of 4% convertible
semiannually. The bond can be called at 1300 on any coupon date
starting at the end of year 7. What is the minimum annual
nominal rate convertible semiannually yield that Joshua could
receive? Answer: 7.3521%.
Solution: We have that F = 1000, P = 800, C = 1300, r = 0.02
and n = 30. Since P < C, the bond was bought at discount. We
assume that the redemption value is as late as possible. From the
equation
800 = 20a
30

|i
+ 1300(1 + i )
30
we get that i = 3.6760 and i
(2)
= 7.3521%.
c 2009. Miguel A. Arcones. All rights reserved. Manual for SOA Exam FM/CAS Exam 2.
10/10
Chapter 5. Bonds. Section 5.5. Callable bonds.
Example 2
Joshua paid 800 for a 15-year 1000 par value bond with
semiannual coupons at a nominal annual rate of 4% convertible
semiannually. The bond can be called at 1300 on any coupon date
starting at the end of year 7. What is the minimum annual
nominal rate convertible semiannually yield that Joshua could
receive? Answer: 7.3521%.
Solution: We have that F = 1000, P = 800, C = 1300, r = 0.02
and n = 30. Since P < C, the bond was bought at discount. We
assume that the redemption value is as late as possible. From the
equation
800 = 20a
30

|i
+ 1300(1 + i )
30
we get that i = 3.6760 and i
(2)
= 7.3521%.
c 2009. Miguel A. Arcones. All rights reserved. Manual for SOA Exam FM/CAS Exam 2.
1/11
More securities
Manual for SOA Exam FM/CAS Exam 2.
Chapter 5. Bonds.
Section 5.6. More securities.
c 2009. Miguel A. Arcones. All rights reserved.
Extract from:
Arcones Manual for the SOA Exam FM/CAS Exam 2,
Financial Mathematics. Fall 2009 Edition,
available at http://www.actexmadriver.com/
c 2009. Miguel A. Arcones. All rights reserved. Manual for SOA Exam FM/CAS Exam 2.
2/11
More securities
Serial bonds
A serial bond is a collection of bonds issued at the same time but
with dierent redemption dates. The price of a serial bond is the
sum of the prices of the individual bonds.
Let P
k
, C
k
, K
k
be the price value, the redemption value and the
present value of redemption value of the kth bond. Let P

, C

, K

be the price value, redemption value and present value of


redemption value of the serial bond. For each k we have that
P
k
= K
k
+
g
i
(C
k
K
k
).
Let P

k
j =1
P
j
, C

k
j =1
C
j
and K

k
j =1
K
j
. Hence,
P

= K

+
g
i
(C

).
c 2009. Miguel A. Arcones. All rights reserved. Manual for SOA Exam FM/CAS Exam 2.
3/11
More securities
Example 1
A 12% serial bond with semiannual coupons and par value of 1000
will be redeemed by the following schedule:
(i) 100 at the end of years 10 through 14; and
(ii) 500 at the end of year 15.
Calculate the price of the bond on the issue date to yield 10% per
annum convertible semiannually.
Solution: We have that g =
Fr
C
= 6% and i = 5%. Since the
annual nominal rate compounded semiannually is 10%, the annual
eective rate of interest is 10.25%. The redemption values and
times of redemption are given by the following table:
Redemption value 100 100 100 100 100 500
Time in years 10 11 12 13 14 15
c 2009. Miguel A. Arcones. All rights reserved. Manual for SOA Exam FM/CAS Exam 2.
4/11
More securities
Example 1
A 12% serial bond with semiannual coupons and par value of 1000
will be redeemed by the following schedule:
(i) 100 at the end of years 10 through 14; and
(ii) 500 at the end of year 15.
Calculate the price of the bond on the issue date to yield 10% per
annum convertible semiannually.
Solution: We have that g =
Fr
C
= 6% and i = 5%. Since the
annual nominal rate compounded semiannually is 10%, the annual
eective rate of interest is 10.25%. The redemption values and
times of redemption are given by the following table:
Redemption value 100 100 100 100 100 500
Time in years 10 11 12 13 14 15
c 2009. Miguel A. Arcones. All rights reserved. Manual for SOA Exam FM/CAS Exam 2.
5/11
More securities
Solution: We have that g =
Fr
C
= 6% and i = 5%. Since the
annual nominal rate compounded semiannually is 10%, the annual
eective rate of interest is 10.25%. The redemption values and
times of redemption are given by the following table:
Redemption value 100 100 100 100 100 500
Time in years 10 11 12 13 14 15
Hence,
K

=
14

k=10
(100)(1 + 0.1025)
k
+ (500)(1 + 0.1025)
15
=(100)(1.1025)
9
a
5

|10.25%
+ (500)(1.1025)
15
=156.51 + 115.69 = 272.20.
Hence,
P

= K

+
g
i
(C

) = 272.20 +
0.06
0.05
(1000 272.20) = 1145.56.
c 2009. Miguel A. Arcones. All rights reserved. Manual for SOA Exam FM/CAS Exam 2.
6/11
More securities
Preferred stock
Preferred stock is like a perpetual bond. This stock pays dividends
forever. The price of the stock is the present value of future
dividends. If a preferred stock pays an annual dividend D, then the
price of this stock is
P = Da

|i
=
D
i
,
where i the annual eective rate of interest.
c 2009. Miguel A. Arcones. All rights reserved. Manual for SOA Exam FM/CAS Exam 2.
7/11
More securities
Example 2
You have acquired some Microsot preferred stock that pays $3000
per year forever. What is the present value of this investment?
Assume that i = 6%.
Solution:
3000
0.06
= 50000.
c 2009. Miguel A. Arcones. All rights reserved. Manual for SOA Exam FM/CAS Exam 2.
8/11
More securities
Example 2
You have acquired some Microsot preferred stock that pays $3000
per year forever. What is the present value of this investment?
Assume that i = 6%.
Solution:
3000
0.06
= 50000.
c 2009. Miguel A. Arcones. All rights reserved. Manual for SOA Exam FM/CAS Exam 2.
9/11
More securities
Common stock
For common stock the dividends are not known in advance. So
one has to project what these dividends will be in the future. For
example, let D be the dividend at the end of the current period and
assume that the next dividends change geometrically with common
ratio 1 + k with 1 < k < i , then the cashow of dividends is:
Contributions D D(1 + k) D(1 + k)
2

Time, in years 1 2 3
The price of the common stock is
P =
D
1 + k
a

|
i k
1+k
= D
1
i k
.
Recall that
P = D
1
1 + i
+D
1 + k
(1 + i )
2
+D
(1 + k)
2
(1 + i )
3
+. . . =
D
1 + i

1
1
1+k
1+i
= D
1
i k
.
c 2009. Miguel A. Arcones. All rights reserved. Manual for SOA Exam FM/CAS Exam 2.
10/11
More securities
Example 3
Each quarter the corporation plans to pay 45% of its earnings as a
stock dividend. The earnings of a corporation increase at 1% per
quarter indenitely. At the start of a quarter, an investor purchases
the stock to yield a nominal rate of 5% compounded quarterly.
The rst stock dividend is 2.4 payable at the end of the quarter.
Calculate the theoretical price of the stock.
Solution: Since the earnings of a corporation increase at 1% per
quarter, the dividends also increase at 1% per quarter. The
cashow of dividends is
Dividends 2.4 (2.4)(1.01) (2.4)(1.01)
2

Time 1 2 3
The present value of the cashow of dividends is
P
(i
(4)
/4)k
=
2.4
(0.05/4)0.01
= 960.
c 2009. Miguel A. Arcones. All rights reserved. Manual for SOA Exam FM/CAS Exam 2.
11/11
More securities
Example 3
Each quarter the corporation plans to pay 45% of its earnings as a
stock dividend. The earnings of a corporation increase at 1% per
quarter indenitely. At the start of a quarter, an investor purchases
the stock to yield a nominal rate of 5% compounded quarterly.
The rst stock dividend is 2.4 payable at the end of the quarter.
Calculate the theoretical price of the stock.
Solution: Since the earnings of a corporation increase at 1% per
quarter, the dividends also increase at 1% per quarter. The
cashow of dividends is
Dividends 2.4 (2.4)(1.01) (2.4)(1.01)
2

Time 1 2 3
The present value of the cashow of dividends is
P
(i
(4)
/4)k
=
2.4
(0.05/4)0.01
= 960.
c 2009. Miguel A. Arcones. All rights reserved. Manual for SOA Exam FM/CAS Exam 2.
1/6
Chapter 6. Variable interest rates and portfolio insurance.
Manual for SOA Exam FM/CAS Exam 2.
Chapter 6. Variable interest rates and portfolio insurance.
Section 6.1. Ination.
c 2009. Miguel A. Arcones. All rights reserved.
Extract from:
Arcones Manual for the SOA Exam FM/CAS Exam 2,
Financial Mathematics. Fall 2009 Edition,
available at http://www.actexmadriver.com/
c 2009. Miguel A. Arcones. All rights reserved. Manual for SOA Exam FM/CAS Exam 2.
2/6
Chapter 6. Variable interest rates and portfolio insurance. Section 6.1. Ination.
Ination
Ination is the fall in the purchasing power of money over time. It
is usually measured with reference to an index representing the
cost of certain goods and services. One the most frequently used
index is the consumer index price. The consumer index price is
released by the Bureau of Labor Statistics monthly. The Bureau of
Labor Statistics nds the consumer index price by averaging the
changes of prices of a market basket of goods and services.
Sometimes, it is convenient for all the calculations related with an
investment to nd the units of real purchasing power rather than
units of ordinary currency.
c 2009. Miguel A. Arcones. All rights reserved. Manual for SOA Exam FM/CAS Exam 2.
3/6
Chapter 6. Variable interest rates and portfolio insurance. Section 6.1. Ination.
Example 1
Peter uses his 450000 in his retirement fund to buy a
perpetuityimmediate. The perpetuity is expected to pay dividends
at the end of each year forever. The next payment (payable one
year from now) is x, and is expected to increase at a rate of 3%
per year. This increase is made to take in account for the ination.
The current annual eective rate of interest is 4.5%. Calculate x.
Solution: The cashow of payments is
Payments x x(1.03)
1
x(1.03)
2

Time 1 2 3
Using the formula for the present value of a geometric perpetuity,
450000 =
x
0.0450.03
and x = (450000)(0.045 0.03) = 6750.
c 2009. Miguel A. Arcones. All rights reserved. Manual for SOA Exam FM/CAS Exam 2.
4/6
Chapter 6. Variable interest rates and portfolio insurance. Section 6.1. Ination.
Example 1
Peter uses his 450000 in his retirement fund to buy a
perpetuityimmediate. The perpetuity is expected to pay dividends
at the end of each year forever. The next payment (payable one
year from now) is x, and is expected to increase at a rate of 3%
per year. This increase is made to take in account for the ination.
The current annual eective rate of interest is 4.5%. Calculate x.
Solution: The cashow of payments is
Payments x x(1.03)
1
x(1.03)
2

Time 1 2 3
Using the formula for the present value of a geometric perpetuity,
450000 =
x
0.0450.03
and x = (450000)(0.045 0.03) = 6750.
c 2009. Miguel A. Arcones. All rights reserved. Manual for SOA Exam FM/CAS Exam 2.
5/6
Chapter 6. Variable interest rates and portfolio insurance. Section 6.1. Ination.
Example 2
Richard invests $10,000 at the end of each year for 10 years into
an account earning an eective rate of interest is 6.5%. The
annual rate of ination is 3.5% over the 10 year period. Calculate
the value at the end of 10 years of Richards investment in todays
dollars.
Solution: The balance at the end of ten years is
(10000)s
10|6.5%
= 134944.2254. The value at the end of 10 years
of Richards investment in todays dollars is
134944.2254(1.035)
10
= 95664.50019.
c 2009. Miguel A. Arcones. All rights reserved. Manual for SOA Exam FM/CAS Exam 2.
6/6
Chapter 6. Variable interest rates and portfolio insurance. Section 6.1. Ination.
Example 2
Richard invests $10,000 at the end of each year for 10 years into
an account earning an eective rate of interest is 6.5%. The
annual rate of ination is 3.5% over the 10 year period. Calculate
the value at the end of 10 years of Richards investment in todays
dollars.
Solution: The balance at the end of ten years is
(10000)s
10|6.5%
= 134944.2254. The value at the end of 10 years
of Richards investment in todays dollars is
134944.2254(1.035)
10
= 95664.50019.
c 2009. Miguel A. Arcones. All rights reserved. Manual for SOA Exam FM/CAS Exam 2.
1/2
Chapter 6. Variable interest rates and portfolio insurance.
Manual for SOA Exam FM/CAS Exam 2.
Chapter 6. Variable interest rates and portfolio insurance.
Section 6.2. Arbitrage.
c 2009. Miguel A. Arcones. All rights reserved.
Extract from:
Arcones Manual for the SOA Exam FM/CAS Exam 2,
Financial Mathematics. Fall 2009 Edition,
available at http://www.actexmadriver.com/
c 2009. Miguel A. Arcones. All rights reserved. Manual for SOA Exam FM/CAS Exam 2.
2/2
Chapter 6. Variable interest rates and portfolio insurance. Section 6.2. Arbitrage.
Arbitrage
In economics, arbitrage is the practice of taking advantage of a
state of imbalance between two (or possibly more) markets by a
combination of matching deals to make a prot. A simple case of
arbitrage consists in buying something in one place and selling it in
another place at the same time. Suppose that the exchange rates
(after taking out the fees for making the exchange) in London are
5 = 10 and the exchange rates in Tokyo are 6 = 10.
Converting 10 to 6 in Tokyo and converting that 6 into 12
in London, for a prot of 2, would be arbitrage.
c 2009. Miguel A. Arcones. All rights reserved. Manual for SOA Exam FM/CAS Exam 2.
1/25
Chapter 6. Variable interest rates and portfolio insurance.
Manual for SOA Exam FM/CAS Exam 2.
Chapter 6. Variable interest rates and portfolio insurance.
Section 6.3. Term structure of interest rates.
c 2009. Miguel A. Arcones. All rights reserved.
Extract from:
Arcones Manual for the SOA Exam FM/CAS Exam 2,
Financial Mathematics. Fall 2009 Edition,
available at http://www.actexmadriver.com/
c 2009. Miguel A. Arcones. All rights reserved. Manual for SOA Exam FM/CAS Exam 2.
2/25
Chapter 6. Variable interest rates and portfolio insurance. Section 6.3. Term structure of interest rates.
Term structure of interest rates
The relationship between yield and time to mature is called the
term structure of interest rates.
As larger as money is tied up in an investment as more likely a
default is. Usually, interest rates increase with maturity date.
For US Treasury zerocoupons bonds, dierent interest rates are
given according with the maturity date.
Denition 1
A yield curve is a graph that shows interest rates (vertical axis)
versus (maturity date) duration of a investment/loan (horizontal
axis).
Yield curves are studied to predict of changes in economic activity
(economic growth, ination, etc.).
c 2009. Miguel A. Arcones. All rights reserved. Manual for SOA Exam FM/CAS Exam 2.
3/25
Chapter 6. Variable interest rates and portfolio insurance. Section 6.3. Term structure of interest rates.
Example 1
A bank oers CDs with the following interest rates
length of the investment (in years) 1 year 2 years 3 years 4 years 5 years
Interest rate 7% 8% 8.5% 9% 9.25%
Graph the yield curve for these interest rates.
Solution: Assuming that the interest rate is a linear function
between the given points, the yield curve is given by the graph in
Figure 1.
c 2009. Miguel A. Arcones. All rights reserved. Manual for SOA Exam FM/CAS Exam 2.
4/25
Chapter 6. Variable interest rates and portfolio insurance. Section 6.3. Term structure of interest rates.
Example 1
A bank oers CDs with the following interest rates
length of the investment (in years) 1 year 2 years 3 years 4 years 5 years
Interest rate 7% 8% 8.5% 9% 9.25%
Graph the yield curve for these interest rates.
Solution: Assuming that the interest rate is a linear function
between the given points, the yield curve is given by the graph in
Figure 1.
c 2009. Miguel A. Arcones. All rights reserved. Manual for SOA Exam FM/CAS Exam 2.
5/25
Chapter 6. Variable interest rates and portfolio insurance. Section 6.3. Term structure of interest rates.
Figure 1: Yield curve for Example 1
c 2009. Miguel A. Arcones. All rights reserved. Manual for SOA Exam FM/CAS Exam 2.
6/25
Chapter 6. Variable interest rates and portfolio insurance. Section 6.3. Term structure of interest rates.
The interest rates appearing in the yield curve are called the spot
rates. Thus, for Example 1, the spot rates are 7%, 8%, 8.5%, 9%
and 9.25%.
Denition 2
The j year spot rate s
j
is the rate of interest charged in a loan
paid with a unique payment at the end of j years.
Note that is the j year spot rate s
j
is as an eective annual rate
of interest, the current j year interest factor is (1 + s
j
)
j
. Money
invested now multiply by (1 + s
j
)
j
in j years. The price of a zero
coupon j year bond with face value F is P = F(1 + s
j
)
j
.
c 2009. Miguel A. Arcones. All rights reserved. Manual for SOA Exam FM/CAS Exam 2.
7/25
Chapter 6. Variable interest rates and portfolio insurance. Section 6.3. Term structure of interest rates.
Spot rates refer to a xed maturity date. Usually, bonds have
coupon payments over time. But, often strip bonds are traded.
Strip or zero coupon bonds are bonds that have being
separated into their component parts (each coupon payment and
the face value). Often strip bonds are obtained from US Treasury
bonds. A nancial trader (strips) separates the coupons from a
US Treasury bond, by accumulating a large number of US Treasury
bonds and selling the rights of obtaining a particular payment to
an investor. In this way, the investor can buy a strip bond as an
individual security. The strip bond market consists of coupons and
residuals, with coupons representing the interest portion of the
original bond and the residual representing the principal portion.
An investor will get a unique payment from a strip bond. In this
situation, interest rates of a strip bond depend on the maturity
date. The yield rate of a zerocoupon bond is called its spot rate.
c 2009. Miguel A. Arcones. All rights reserved. Manual for SOA Exam FM/CAS Exam 2.
8/25
Chapter 6. Variable interest rates and portfolio insurance. Section 6.3. Term structure of interest rates.
The following table consists of the Daily Treasury Yield Curve
Rates, which can be found at
http://www.treas.gov/oces/domestic-nance/debt-
management/interest-rate/yield.html
Date 1 mo 3 mo 6 mo 1 yr 2 yr 3 yr 5 yr 7 yr 10 yr 20 yr
07/01/04 1.01 1.22 1.64 2.07 2.64 3.08 3.74 4.18 4.57 5.31
07/02/04 1.07 1.30 1.61 2.02 2.54 2.96 3.62 4.08 4.48 5.22
07/06/04 1.11 1.34 1.68 2.15 2.56 2.99 3.65 4.10 4.49 5.24
07/07/04 1.16 1.30 1.64 2.00 2.56 2.99 3.67 4.10 4.50 5.24
07/08/04 1.14 1.27 1.63 1.99 2.55 2.97 3.65 4.09 4.49 5.24
07/09/04 1.14 1.28 1.63 2.00 2.55 2.96 3.64 4.08 4.49 5.23
c 2009. Miguel A. Arcones. All rights reserved. Manual for SOA Exam FM/CAS Exam 2.
9/25
Chapter 6. Variable interest rates and portfolio insurance. Section 6.3. Term structure of interest rates.
Suppose that a zerocoupon bond with a face value of F and
maturity j years has a price of P
j
. Then, P
j
(1 + s
j
)
j
= F, where s
j
is the j year spot rate. Note that a payment of P
j
now is
exchanged by a payment of F in j years. (1 + s
j
)
j
is the interest
factor from year zero to year j . If the current interest rates follow
the accumulation function is a(t), t 0, then a(j ) = (1 + s
j
)
j
, i.e.
s
j
= (a(j ))
1/j
1.
c 2009. Miguel A. Arcones. All rights reserved. Manual for SOA Exam FM/CAS Exam 2.
10/25
Chapter 6. Variable interest rates and portfolio insurance. Section 6.3. Term structure of interest rates.
Example 2
The following table lists prices of zero-coupon $1000 bonds with
their respective maturities:
Number of years to maturity Price
1 $980.39
2 $957.41
5 $888.18
10 $781.20
Calculate the 1year, 2year, 5year, and 10year spot rates of
interest.
Solution: Since (1000)(1 + s
1
)
1
= 980.39,
(1000)(1 + s
2
)
2
= 957.41, (1000)(1 + s
5
)
5
= 888.17, and
(1000)(1 + s
10
)
10
= 781.1984, we get s
1
= 2.00%, s
2
= 2.20% ,
s
5
= 2.40%, s
10
= 2.50%.
c 2009. Miguel A. Arcones. All rights reserved. Manual for SOA Exam FM/CAS Exam 2.
11/25
Chapter 6. Variable interest rates and portfolio insurance. Section 6.3. Term structure of interest rates.
Example 2
The following table lists prices of zero-coupon $1000 bonds with
their respective maturities:
Number of years to maturity Price
1 $980.39
2 $957.41
5 $888.18
10 $781.20
Calculate the 1year, 2year, 5year, and 10year spot rates of
interest.
Solution: Since (1000)(1 + s
1
)
1
= 980.39,
(1000)(1 + s
2
)
2
= 957.41, (1000)(1 + s
5
)
5
= 888.17, and
(1000)(1 + s
10
)
10
= 781.1984, we get s
1
= 2.00%, s
2
= 2.20% ,
s
5
= 2.40%, s
10
= 2.50%.
c 2009. Miguel A. Arcones. All rights reserved. Manual for SOA Exam FM/CAS Exam 2.
12/25
Chapter 6. Variable interest rates and portfolio insurance. Section 6.3. Term structure of interest rates.
The present value at time zero of a cashow
Contributions 0 C
1
C
2
C
n
Time 0 1 2 n
following the spot rates
spot rate s
1
s
2
s
n
maturity time 1 2 n
is given by the formula
PV =
n

j =1
(1 + s
j
)
j
C
j
.
c 2009. Miguel A. Arcones. All rights reserved. Manual for SOA Exam FM/CAS Exam 2.
13/25
Chapter 6. Variable interest rates and portfolio insurance. Section 6.3. Term structure of interest rates.
Example 3
(i) Find the price of a 2year 1000 par value 6% bond with
semiannual coupons using the spot rates:
nominal annual interest
rate convertible semiannually
4% 5% 6% 7%
maturity time (in half years) 1 2 3 4
c 2009. Miguel A. Arcones. All rights reserved. Manual for SOA Exam FM/CAS Exam 2.
14/25
Chapter 6. Variable interest rates and portfolio insurance. Section 6.3. Term structure of interest rates.
Example 3
(i) Find the price of a 2year 1000 par value 6% bond with
semiannual coupons using the spot rates:
nominal annual interest
rate convertible semiannually
4% 5% 6% 7%
maturity time (in half years) 1 2 3 4
Solution: (i) The cashow of payments is
Payments 30 30 30 1030
Time (in half years) 1 2 3 4
The present value of these payments is
PV = (30)

1 +
0.04
2

1
+ (30)

1 +
0.05
2

2
+ (30)

1 +
0.06
2

3
+ (1030)

1 +
0.07
2

4
=29.41176 + 28.55443 + 27.45425 + 897.5855 = 983.0059
c 2009. Miguel A. Arcones. All rights reserved. Manual for SOA Exam FM/CAS Exam 2.
15/25
Chapter 6. Variable interest rates and portfolio insurance. Section 6.3. Term structure of interest rates.
Example 3
(i) Find the price of a 2year 1000 par value 6% bond with
semiannual coupons using the spot rates:
nominal annual interest
rate convertible semiannually
4% 5% 6% 7%
maturity time (in half years) 1 2 3 4
(ii) Find the annual eective yield rate of the previous bond, if
bought at the price in (i).
c 2009. Miguel A. Arcones. All rights reserved. Manual for SOA Exam FM/CAS Exam 2.
16/25
Chapter 6. Variable interest rates and portfolio insurance. Section 6.3. Term structure of interest rates.
Example 3
(i) Find the price of a 2year 1000 par value 6% bond with
semiannual coupons using the spot rates:
nominal annual interest
rate convertible semiannually
4% 5% 6% 7%
maturity time (in half years) 1 2 3 4
(ii) Find the annual eective yield rate of the previous bond, if
bought at the price in (i).
Solution: (ii) To nd the yield rate, we solve for i
(2)
in 983.0059 =
30a
4

|i
(2)
/2
+1000(1+i
(2)
/2)
4
, to get i
(2)
= 6.92450%. The annual
eective yield rate is i = 7.0443%.
Note that i
(2)
= 6.92450% is a sort of average of the spot rates
used to nd the price of the bond. Since the biggest payment is at
time t = 4 half years, i
(2)
= 6.92450% is close to 7%.
c 2009. Miguel A. Arcones. All rights reserved. Manual for SOA Exam FM/CAS Exam 2.
17/25
Chapter 6. Variable interest rates and portfolio insurance. Section 6.3. Term structure of interest rates.
The one year forward rate for the j th year f
j
is dened as
f
j
=
(1 + s
j
)
j
(1 + s
j 1
)
j 1
1.
f
j
is also called the 1 year forward rate from time j 1 to time j .
f
j
is also called the 1 year forward rate from the j th year.
f
j
is also called the (j 1)year forward rate.
f
j
is also called the (j 1)year deferred 1year forward rate.
f
j
is also called the (j 1)year forward rate, 1year interest rate.
1 + f
j
is the interest factor from year j 1 to year j .
c 2009. Miguel A. Arcones. All rights reserved. Manual for SOA Exam FM/CAS Exam 2.
18/25
Chapter 6. Variable interest rates and portfolio insurance. Section 6.3. Term structure of interest rates.
(1 + s
j 1
)
j 1
is the interest factor from year zero to year j 1.
(1 + s
j
)
j
is the interest factor from year zero to year j .
Hence, we have that (1 + s
j 1
)
j 1
(1 + f
j
) = (1 + s
j
)
j
.
Notice that by the denition of the one year forward rate
(1 + s
2
)
2
= (1 + s
1
)(1 + f
2
),
(1 + s
3
)
3
= (1 + s
2
)
2
(1 + f
3
) = (1 + s
1
)(1 + f
2
)(1 + f
3
),
(1 + s
4
)
4
= (1 + s
3
)
3
(1 + f
4
) = (1 + s
1
)(1 + f
2
)(1 + f
3
)(1 + f
4
).
In general,
(1 + s
n
)
n
= (1 + s
1
)(1 + f
2
)(1 + f
3
) (1 + f
n
).
c 2009. Miguel A. Arcones. All rights reserved. Manual for SOA Exam FM/CAS Exam 2.
19/25
Chapter 6. Variable interest rates and portfolio insurance. Section 6.3. Term structure of interest rates.
Example 4
Suppose that the following spot rates are given:
maturity time
(in years)
1 2 3 4 5
Interest rate 12.00% 11.75% 11.25% 10.00% 9.25%
Calculate the oneyear forward rates for years 2 through 5.
Solution:
f
2
=
(1.1175)
2
1.12
1 = 0.115006
f
3
=
(1.1125)
3
(1.1175)
2
1 = 0.102567
f
4
=
(1.1)
4
(1.1125)
3
1 = 0.063336
f
5
=
(1.0925)
5
(1.1)
4
1 = 0.063008
The oneyear forward rate for year 2 is 11.5006%.
The oneyear forward rate for year 3 is 10.2567%.
The oneyear forward rate for year 4 is 6.3336%.
The oneyear forward rate for year 5 is 6.3008%.
c 2009. Miguel A. Arcones. All rights reserved. Manual for SOA Exam FM/CAS Exam 2.
20/25
Chapter 6. Variable interest rates and portfolio insurance. Section 6.3. Term structure of interest rates.
Example 4
Suppose that the following spot rates are given:
maturity time
(in years)
1 2 3 4 5
Interest rate 12.00% 11.75% 11.25% 10.00% 9.25%
Calculate the oneyear forward rates for years 2 through 5.
Solution:
f
2
=
(1.1175)
2
1.12
1 = 0.115006
f
3
=
(1.1125)
3
(1.1175)
2
1 = 0.102567
f
4
=
(1.1)
4
(1.1125)
3
1 = 0.063336
f
5
=
(1.0925)
5
(1.1)
4
1 = 0.063008
The oneyear forward rate for year 2 is 11.5006%.
The oneyear forward rate for year 3 is 10.2567%.
The oneyear forward rate for year 4 is 6.3336%.
The oneyear forward rate for year 5 is 6.3008%.
c 2009. Miguel A. Arcones. All rights reserved. Manual for SOA Exam FM/CAS Exam 2.
21/25
Chapter 6. Variable interest rates and portfolio insurance. Section 6.3. Term structure of interest rates.
Example 5
The following table lists prices of zero-coupon $100 bonds with
their respective maturities:
Number of years
to maturity
Price
1 $96.15
2 $92.10
3 $87.63
4 $82.27
c 2009. Miguel A. Arcones. All rights reserved. Manual for SOA Exam FM/CAS Exam 2.
22/25
Chapter 6. Variable interest rates and portfolio insurance. Section 6.3. Term structure of interest rates.
Example 5
The following table lists prices of zero-coupon $100 bonds with
their respective maturities:
Number of years
to maturity
Price
1 $96.15
2 $92.10
3 $87.63
4 $82.27
(i) Calculate the 1year, 2year, 3year, and 4year spot rates of
interest.
c 2009. Miguel A. Arcones. All rights reserved. Manual for SOA Exam FM/CAS Exam 2.
23/25
Chapter 6. Variable interest rates and portfolio insurance. Section 6.3. Term structure of interest rates.
Example 5
The following table lists prices of zero-coupon $100 bonds with
their respective maturities:
Number of years
to maturity
Price
1 $96.15
2 $92.10
3 $87.63
4 $82.27
(i) Calculate the 1year, 2year, 3year, and 4year spot rates of
interest.
Solution: (i) Note that the price of j th bond is P
j
= (100)(1 +
s
j
)
j
. To get the j year spot rate s
j
, we solve (100)(1 + s
1
)
1
=
96.15, (100)(1 + s
2
)
2
= 92.10, (100)(1 + s
3
)
3
= 87.63, and
(100)(1 + s
4
)
4
= 82.27, to get s
1
= 4.00%, s
2
= 4.20% , s
3
=
4.50%, s
4
= 5.00%.
c 2009. Miguel A. Arcones. All rights reserved. Manual for SOA Exam FM/CAS Exam 2.
24/25
Chapter 6. Variable interest rates and portfolio insurance. Section 6.3. Term structure of interest rates.
Example 5
The following table lists prices of zero-coupon $100 bonds with
their respective maturities:
Number of years
to maturity
Price
1 $96.15
2 $92.10
3 $87.63
4 $82.27
(ii) Calculate the 1year, 2year, and 3year forward rates of interest.
c 2009. Miguel A. Arcones. All rights reserved. Manual for SOA Exam FM/CAS Exam 2.
25/25
Chapter 6. Variable interest rates and portfolio insurance. Section 6.3. Term structure of interest rates.
Example 5
The following table lists prices of zero-coupon $100 bonds with
their respective maturities:
Number of years
to maturity
Price
1 $96.15
2 $92.10
3 $87.63
4 $82.27
(ii) Calculate the 1year, 2year, and 3year forward rates of interest.
Solution: (ii) To get the j 1 year forward rate f
j
, we do f
j
=
(1+s
j
)
j
(1+s
j 1
)
j 1
1 =
P
j 1
P
j
1, where P
j
is the price of the j th bond.
We get that:
f
2
=
96.15
92.10
1 = 4.397394%,
f
3
=
92.10
87.63
1 = 5.100993%,
f
4
=
87.63
82.27
1 = 6.515133%.
c 2009. Miguel A. Arcones. All rights reserved. Manual for SOA Exam FM/CAS Exam 2.
1/90
Chapter 6. Variable interest rates and portfolio insurance.
Manual for SOA Exam FM/CAS Exam 2.
Chapter 6. Variable interest rates and portfolio insurance.
Section 6.4. Duration, convexity.
c 2009. Miguel A. Arcones. All rights reserved.
Extract from:
Arcones Manual for the SOA Exam FM/CAS Exam 2,
Financial Mathematics. Fall 2009 Edition,
available at http://www.actexmadriver.com/
c 2009. Miguel A. Arcones. All rights reserved. Manual for SOA Exam FM/CAS Exam 2.
2/90
Chapter 6. Variable interest rates and portfolio insurance. Section 6.4. Duration, convexity.
Duration
Next we will assume that the rate of interest is constant over
maturity.
Denition 1
The duration (or Macaulays duration) of a cashow
Contributions C
1
C
2
C
n
Time in years 1 2 n
with C
j
0 for each 1 j m, is dened as

d =

n
j =1
jC
j

n
j =1
C
j

j
=

n
j =1
jC
j
(1 + i )
j

n
j =1
C
j
(1 + i )
j
=
n

j =1
j
C
j

n
k=1
C
k

k
.
c 2009. Miguel A. Arcones. All rights reserved. Manual for SOA Exam FM/CAS Exam 2.
3/90
Chapter 6. Variable interest rates and portfolio insurance. Section 6.4. Duration, convexity.
Main Properties of volatility

The duration is an average of the times when the payments of


the cashow are made:

d =
n

j =1
j
C
j

n
k=1
C
k

k
=
n

j =1
jw
j
,
where w
j
=
C
j

j
P
n
k=1
C
k

k
satisfy w
j
0 and

n
j =1
w
j
= 1.

w
j
is the fraction of the present value of contribution at time
t over the present value of the whole cashow.

If C
j
0
> 0 and C
j
= 0, for each j = j
0
, then

d = j
0
, for each
rate of interest i .

The units of the duration are years.

The Macaulay duration is a measure of the price sensitivity of


a cashow to interest rate changes.
c 2009. Miguel A. Arcones. All rights reserved. Manual for SOA Exam FM/CAS Exam 2.
4/90
Chapter 6. Variable interest rates and portfolio insurance. Section 6.4. Duration, convexity.
Example 1
An investment pays 1000 at the end of year two and 1000 at the
end of year 12. The annual eective rate of interest is 8%.
Calculate the Macaulay duration for this investment.
Solution:

d =

n
j =1
jC
j

n
j =1
C
j

j
=
(2)(1000)(1.08)
2
+ (12)(1000)(1.08)
12
(1000)(1.08)
2
+ (1000)(1.08)
12
=5.165633881 years.
c 2009. Miguel A. Arcones. All rights reserved. Manual for SOA Exam FM/CAS Exam 2.
5/90
Chapter 6. Variable interest rates and portfolio insurance. Section 6.4. Duration, convexity.
Example 1
An investment pays 1000 at the end of year two and 1000 at the
end of year 12. The annual eective rate of interest is 8%.
Calculate the Macaulay duration for this investment.
Solution:

d =

n
j =1
jC
j

n
j =1
C
j

j
=
(2)(1000)(1.08)
2
+ (12)(1000)(1.08)
12
(1000)(1.08)
2
+ (1000)(1.08)
12
=5.165633881 years.
c 2009. Miguel A. Arcones. All rights reserved. Manual for SOA Exam FM/CAS Exam 2.
6/90
Chapter 6. Variable interest rates and portfolio insurance. Section 6.4. Duration, convexity.
Theorem 1
Let r > 0. If the Macaulay duration of the cashow
Contributions C
1
C
2
C
n
Time in years 1 2 n
is

d, then the Macaulay duration of the cashow
Contributions rC
1
rC
2
rC
n
Time in years 1 2 n
is

d.
Proof: The duration of the modied cashow is

n
j =1
jrC
j

n
j =1
rC
j

j
=

n
j =1
jC
j

n
j =1
C
j

j
=

d.
c 2009. Miguel A. Arcones. All rights reserved. Manual for SOA Exam FM/CAS Exam 2.
7/90
Chapter 6. Variable interest rates and portfolio insurance. Section 6.4. Duration, convexity.
Theorem 1
Let r > 0. If the Macaulay duration of the cashow
Contributions C
1
C
2
C
n
Time in years 1 2 n
is

d, then the Macaulay duration of the cashow
Contributions rC
1
rC
2
rC
n
Time in years 1 2 n
is

d.
Proof: The duration of the modied cashow is

n
j =1
jrC
j

n
j =1
rC
j

j
=

n
j =1
jC
j

n
j =1
C
j

j
=

d.
c 2009. Miguel A. Arcones. All rights reserved. Manual for SOA Exam FM/CAS Exam 2.
8/90
Chapter 6. Variable interest rates and portfolio insurance. Section 6.4. Duration, convexity.
Example 2
The Macaulay duration of a 10year annuityimmediate with
annual payments of $1000 is 5.6 years. Calculate the Macaulay
duration of a 10year annuityimmediate with annual payments of
$50000.
Solution: Both cashows have duration 5.6 years.
c 2009. Miguel A. Arcones. All rights reserved. Manual for SOA Exam FM/CAS Exam 2.
9/90
Chapter 6. Variable interest rates and portfolio insurance. Section 6.4. Duration, convexity.
Example 2
The Macaulay duration of a 10year annuityimmediate with
annual payments of $1000 is 5.6 years. Calculate the Macaulay
duration of a 10year annuityimmediate with annual payments of
$50000.
Solution: Both cashows have duration 5.6 years.
c 2009. Miguel A. Arcones. All rights reserved. Manual for SOA Exam FM/CAS Exam 2.
10/90
Chapter 6. Variable interest rates and portfolio insurance. Section 6.4. Duration, convexity.
Theorem 2
If the Macaulay duration of the cashow
Contributions C
1
C
2
C
n
Time in years 1 2 n
is

d, then the Macaulay duration of the cashow
Contributions C
1
C
2
C
n
Time in years t + 1 t + 2 t + n
is

d + t.
Proof:

n
j =1
(t + j )C
j

n
j =1
C
j

j
=
t

n
j =1
C
j

j
+

n
j =1
jC
j

n
j =1
C
j

j
= t +

n
j =1
jC
j

n
j =1
C
j

j
.
c 2009. Miguel A. Arcones. All rights reserved. Manual for SOA Exam FM/CAS Exam 2.
11/90
Chapter 6. Variable interest rates and portfolio insurance. Section 6.4. Duration, convexity.
Theorem 2
If the Macaulay duration of the cashow
Contributions C
1
C
2
C
n
Time in years 1 2 n
is

d, then the Macaulay duration of the cashow
Contributions C
1
C
2
C
n
Time in years t + 1 t + 2 t + n
is

d + t.
Proof:

n
j =1
(t + j )C
j

n
j =1
C
j

j
=
t

n
j =1
C
j

j
+

n
j =1
jC
j

n
j =1
C
j

j
= t +

n
j =1
jC
j

n
j =1
C
j

j
.
c 2009. Miguel A. Arcones. All rights reserved. Manual for SOA Exam FM/CAS Exam 2.
12/90
Chapter 6. Variable interest rates and portfolio insurance. Section 6.4. Duration, convexity.
Example 3
The Macaulay duration of a 10year annuityimmediate with
annual payments of $1000 is 5.6 years. Calculate the Macaulay
duration of a 10year annuitydue with annual payments of $5000.
Solution: The Macaulay duration of the two annuities does not
dependent on the amount of the payment. So, we may assume
that the two annual payments agree. Since the cashow of an
annuitydue is obtained from the cashow of an annuityimmediate
by translating payments 1 year, the answer is 5.6 1 = 4.6 years.
c 2009. Miguel A. Arcones. All rights reserved. Manual for SOA Exam FM/CAS Exam 2.
13/90
Chapter 6. Variable interest rates and portfolio insurance. Section 6.4. Duration, convexity.
Example 3
The Macaulay duration of a 10year annuityimmediate with
annual payments of $1000 is 5.6 years. Calculate the Macaulay
duration of a 10year annuitydue with annual payments of $5000.
Solution: The Macaulay duration of the two annuities does not
dependent on the amount of the payment. So, we may assume
that the two annual payments agree. Since the cashow of an
annuitydue is obtained from the cashow of an annuityimmediate
by translating payments 1 year, the answer is 5.6 1 = 4.6 years.
c 2009. Miguel A. Arcones. All rights reserved. Manual for SOA Exam FM/CAS Exam 2.
14/90
Chapter 6. Variable interest rates and portfolio insurance. Section 6.4. Duration, convexity.
Theorem 3
Suppose that two cashows have durations

d
1
and

d
2
, respectively,
present values P
1
and P
2
, respectively. Then, the duration of the
combined cashow is

d =
P
1

d
1
+ P
2

d
2
P
1
+ P
2
.
By induction the previous formula holds for a combination of
nitely many cashows. Suppose that we have n cashows. The
j the cashow has present value P
j
and duration

d
j
. Then, the
duration of the combined cashow is

d =

n
j =1
P
j
(i )

d
j

n
j =1
P
j
(i )
.
c 2009. Miguel A. Arcones. All rights reserved. Manual for SOA Exam FM/CAS Exam 2.
15/90
Chapter 6. Variable interest rates and portfolio insurance. Section 6.4. Duration, convexity.
Proof: Suppose that the considered cashows are
Contributions 0 C
1
C
2
C
n
Time 0 1 2 n
and
Contributions 0 D
1
D
2
D
n
Time 0 1 2 n
Then, the combined cashow is
Contributions 0 C
1
+ D
1
C
2
+ D
2
C
n
+ D
n
Time 0 1 2 n
c 2009. Miguel A. Arcones. All rights reserved. Manual for SOA Exam FM/CAS Exam 2.
16/90
Chapter 6. Variable interest rates and portfolio insurance. Section 6.4. Duration, convexity.
We have that P
1
=

n
j =1
C
j

j
and P
2
=

n
j =1
D
j

j
. By denition
of duration,

d
1
=

n
j =1
jC
j

n
j =1
C
j

j
=

n
j =1
jC
j

j
P
1
and

d
2
=

n
j =1
jD
j

n
j =1
D
j

j
=

n
j =1
jD
j

j
P
2
.
Hence,

d =

n
j =1
j (C
j
+ D
j
)
j

n
j =1
(C
j
+ D
j
)
j
=

n
j =1
jC
j

j
+

n
j =1
jD
j

n
j =1
C
j

j
+

n
j =1
D
j

j
=

d
1
P
1
+

d
2
P
2
P
1
+ P
2
.
c 2009. Miguel A. Arcones. All rights reserved. Manual for SOA Exam FM/CAS Exam 2.
17/90
Chapter 6. Variable interest rates and portfolio insurance. Section 6.4. Duration, convexity.
Example 4
An insurance has the following portfolio of investments:
(i) Bonds with a value of $1,520,000 and duration 4.5 years.
(ii) Stock dividends payments with a value of $1,600,000 and
duration 14.5 years.
(iii) Certicate of deposits payments with a value of $2,350,000
and duration 2 years.
Calculate the duration of the portfolio of investments.
Solution: The duration of the portfolio is

d =

n
j =1
P
j
(i )

d
j

n
j =1
P
j
(i )
=
(4.5)(1520000) + (14.5)(1600000) + (2)(2350000)
1520000 + 1600000 + 2350000
=6.351005484 years.
c 2009. Miguel A. Arcones. All rights reserved. Manual for SOA Exam FM/CAS Exam 2.
18/90
Chapter 6. Variable interest rates and portfolio insurance. Section 6.4. Duration, convexity.
Example 4
An insurance has the following portfolio of investments:
(i) Bonds with a value of $1,520,000 and duration 4.5 years.
(ii) Stock dividends payments with a value of $1,600,000 and
duration 14.5 years.
(iii) Certicate of deposits payments with a value of $2,350,000
and duration 2 years.
Calculate the duration of the portfolio of investments.
Solution: The duration of the portfolio is

d =

n
j =1
P
j
(i )

d
j

n
j =1
P
j
(i )
=
(4.5)(1520000) + (14.5)(1600000) + (2)(2350000)
1520000 + 1600000 + 2350000
=6.351005484 years.
c 2009. Miguel A. Arcones. All rights reserved. Manual for SOA Exam FM/CAS Exam 2.
19/90
Chapter 6. Variable interest rates and portfolio insurance. Section 6.4. Duration, convexity.
Theorem 4
The Macaulay duration of a level payments annuityimmediate is

d =
(Ia)
n

|i
a
n

|i
.
Proof.
We have that

d =

n
j =1
jP
j

n
j =1
P
j
=
(Ia)
n

|i
a
n

|i
.
c 2009. Miguel A. Arcones. All rights reserved. Manual for SOA Exam FM/CAS Exam 2.
20/90
Chapter 6. Variable interest rates and portfolio insurance. Section 6.4. Duration, convexity.
Theorem 4
The Macaulay duration of a level payments annuityimmediate is

d =
(Ia)
n

|i
a
n

|i
.
Proof.
We have that

d =

n
j =1
jP
j

n
j =1
P
j
=
(Ia)
n

|i
a
n

|i
.
c 2009. Miguel A. Arcones. All rights reserved. Manual for SOA Exam FM/CAS Exam 2.
21/90
Chapter 6. Variable interest rates and portfolio insurance. Section 6.4. Duration, convexity.
Example 5
Calculate Macaulay the duration of a 15year annuity immediate
with level payments if the current eective interest rate per annum
is 5%.
Solution: The Macaulay the duration is

d =
(Ia)
n

|i
a
n

|i
=
(Ia)
15

|5%
a
15

|5%
=
73.66768937
10.37965804
= 7.097313716.
c 2009. Miguel A. Arcones. All rights reserved. Manual for SOA Exam FM/CAS Exam 2.
22/90
Chapter 6. Variable interest rates and portfolio insurance. Section 6.4. Duration, convexity.
Example 5
Calculate Macaulay the duration of a 15year annuity immediate
with level payments if the current eective interest rate per annum
is 5%.
Solution: The Macaulay the duration is

d =
(Ia)
n

|i
a
n

|i
=
(Ia)
15

|5%
a
15

|5%
=
73.66768937
10.37965804
= 7.097313716.
c 2009. Miguel A. Arcones. All rights reserved. Manual for SOA Exam FM/CAS Exam 2.
23/90
Chapter 6. Variable interest rates and portfolio insurance. Section 6.4. Duration, convexity.
Theorem 5
The duration of a level payments perpetuityimmediate is

d =
1 + i
i
.
Proof.
We have that

d =

j =1
jP
j

j =1
P
j
=
(Ia)

|i
a

|i
=
1+i
i
2
1
i
=
1 + i
i
.
c 2009. Miguel A. Arcones. All rights reserved. Manual for SOA Exam FM/CAS Exam 2.
24/90
Chapter 6. Variable interest rates and portfolio insurance. Section 6.4. Duration, convexity.
Example 6
Suppose that the Macaulay duration of a perpetuity immediate
with level payments of 1000 at the end of each year is 21. Find the
current eective rate of interest.
Solution: We have that 21 =

d =
1+i
i
. So, i =
1
20
= 5%.
c 2009. Miguel A. Arcones. All rights reserved. Manual for SOA Exam FM/CAS Exam 2.
25/90
Chapter 6. Variable interest rates and portfolio insurance. Section 6.4. Duration, convexity.
Example 6
Suppose that the Macaulay duration of a perpetuity immediate
with level payments of 1000 at the end of each year is 21. Find the
current eective rate of interest.
Solution: We have that 21 =

d =
1+i
i
. So, i =
1
20
= 5%.
c 2009. Miguel A. Arcones. All rights reserved. Manual for SOA Exam FM/CAS Exam 2.
26/90
Chapter 6. Variable interest rates and portfolio insurance. Section 6.4. Duration, convexity.
Theorem 6
The duration of n year bond with r % annual coupons, face value F
and redemption value C is

d =
Fr (Ia)
n

|i
+ Cn
n
Fra
n

|i
+ C
n
.
Proof.
Since the cashow
Contributions Fr Fr Fr Fr + C
Time 1 2 n 1 n
the duration is

d =
Fr

n
j =1
j
j
+ Cn
n
Fr

n
j =1

j
+ C
n
=
Fr (Ia)
n

|i
+ Cn
n
Fra
n

|i
+ C
n
.
c 2009. Miguel A. Arcones. All rights reserved. Manual for SOA Exam FM/CAS Exam 2.
27/90
Chapter 6. Variable interest rates and portfolio insurance. Section 6.4. Duration, convexity.
Example 7
Megan buys a 10year 1000facevalue bond with a redemption
value of 1200 which pay annual coupons at rate 7.5%. Calculate
the Macaulay duration if the eective rate of interest per annum is
8%.
Solution: We have that

d =
Fr (Ia)
n

|i
+ Cn
n
Fra
n

|i
+ C
n
=
(1000)(0.075)(Ia)
10

|8%
+ (1200)(10)(1.08)
10
(1000)(0.075)a
10

|8%
+ (1200)(1.08)
10
=
(75)(32.68691288) + 5558.321857
(75)(6.710081399) + 555.8321857
= 7.562958059.
c 2009. Miguel A. Arcones. All rights reserved. Manual for SOA Exam FM/CAS Exam 2.
28/90
Chapter 6. Variable interest rates and portfolio insurance. Section 6.4. Duration, convexity.
Example 7
Megan buys a 10year 1000facevalue bond with a redemption
value of 1200 which pay annual coupons at rate 7.5%. Calculate
the Macaulay duration if the eective rate of interest per annum is
8%.
Solution: We have that

d =
Fr (Ia)
n

|i
+ Cn
n
Fra
n

|i
+ C
n
=
(1000)(0.075)(Ia)
10

|8%
+ (1200)(10)(1.08)
10
(1000)(0.075)a
10

|8%
+ (1200)(1.08)
10
=
(75)(32.68691288) + 5558.321857
(75)(6.710081399) + 555.8321857
= 7.562958059.
c 2009. Miguel A. Arcones. All rights reserved. Manual for SOA Exam FM/CAS Exam 2.
29/90
Chapter 6. Variable interest rates and portfolio insurance. Section 6.4. Duration, convexity.
We have the following table:
Cashow Duration

d
zerocoupon bond

d = n
level payments annuityimmediate

d =
(Ia)
n

|i
a
n

|i
level payments perpetuityimmediate

d =
1+i
i
regular bond

d =
Fr (Ia)
n

|i
+Cn
n
Fra
n

|i
+C
n
c 2009. Miguel A. Arcones. All rights reserved. Manual for SOA Exam FM/CAS Exam 2.
30/90
Chapter 6. Variable interest rates and portfolio insurance. Section 6.4. Duration, convexity.
Volatility
Consider a cashow
Contributions C
1
C
2
C
n
Time in years 1 2 n
Denition 2
The quantity =
d ln P(i )
di
=
P

(i )
P(i )
is called the volatility or
modied duration.
Notice that P(i ) =

n
j =1
C
j

j
=

n
j =1
C
j
(1 + i )
j
and
P

(i ) =

n
j =1
C
j
(j )(1 + i )
j 1
. So,
=

n
j =1
jC
j

j +1

n
j =1
C
j

j
.
c 2009. Miguel A. Arcones. All rights reserved. Manual for SOA Exam FM/CAS Exam 2.
31/90
Chapter 6. Variable interest rates and portfolio insurance. Section 6.4. Duration, convexity.
Since

d =

n
j =1
jC
j

n
j =1
C
j

j
and
=

n
j =1
jC
j

j +1

n
j =1
C
j

j
,
=

d.
c 2009. Miguel A. Arcones. All rights reserved. Manual for SOA Exam FM/CAS Exam 2.
32/90
Chapter 6. Variable interest rates and portfolio insurance. Section 6.4. Duration, convexity.
Main Properties of volatility

=
P

(i )
P(i )
.

The volatility measures the loss of present value of the


cashow as i increases relative to the PV of the cashow.

n
j =1
jC
j

j +1

n
j =1
C
j

j
.

If C
j
0, for each 1 j n, > 0.

d.

Volatility is measured in years.


c 2009. Miguel A. Arcones. All rights reserved. Manual for SOA Exam FM/CAS Exam 2.
33/90
Chapter 6. Variable interest rates and portfolio insurance. Section 6.4. Duration, convexity.
The present value P(i ) of the above cashow as a function on i ,
i.e.
P(i ) =
n

j =1
C
j

j
=
n

j =1
C
j
(1 + i )
j
.
It is easy to see that
P

(i ) =
n

j =1
C
j
(j )(1+i )
j 1
, and P

(i ) =
n

j =1
C
j
j (j +1)(1+i )
j 2
.
If C
j
> 0, for each 1 j n, then P

(i ) < 0 and P

(i ) > 0, for
each i 0. This implies that P(i ) is a decreasing convex function
on i .
c 2009. Miguel A. Arcones. All rights reserved. Manual for SOA Exam FM/CAS Exam 2.
34/90
Chapter 6. Variable interest rates and portfolio insurance. Section 6.4. Duration, convexity.
Since P(i ), i 0, is a decreasing function of i , so is ln P(i ). Hence,
0 <
d ln P(i )
di
=
P

(i )
P(i )
=

n
j =1
C
j
(j )(1 + i )
j 1

n
j =1
C
j
(1 + i )
j
=

n
j =1
jC
j

j +1

n
j =1
C
j

j
.
Hence, > 0.
c 2009. Miguel A. Arcones. All rights reserved. Manual for SOA Exam FM/CAS Exam 2.
35/90
Chapter 6. Variable interest rates and portfolio insurance. Section 6.4. Duration, convexity.
Since =

d, we have that the volatility satises some of the


properties of the duration. Suppose that we have n cashows. The
j the cashow has present value P
j
and duration
j
. Then, the
duration of the combined cashow is
=

n
j =1
P
j
(i )
j

n
j =1
P
j
(i )
.
c 2009. Miguel A. Arcones. All rights reserved. Manual for SOA Exam FM/CAS Exam 2.
36/90
Chapter 6. Variable interest rates and portfolio insurance. Section 6.4. Duration, convexity.
Example 8
A perpetuity pays 100 immediately. Each subsequent payment in
increased by ination. The current annual eective rate of interest
is 6.5%. Calculate the modied duration of the perpetuity
assuming that ination will be 5% annually.
Solution: The present value of the perpetuity is P(i ) =
100
i 0.05
, if
i > 0.05. Hence, P

(i ) =
100
(i 0.05)
2
,
=
P

(0.065)
P(0.065)
=
1
0.0650.05
= 66.66666667.
c 2009. Miguel A. Arcones. All rights reserved. Manual for SOA Exam FM/CAS Exam 2.
37/90
Chapter 6. Variable interest rates and portfolio insurance. Section 6.4. Duration, convexity.
Example 8
A perpetuity pays 100 immediately. Each subsequent payment in
increased by ination. The current annual eective rate of interest
is 6.5%. Calculate the modied duration of the perpetuity
assuming that ination will be 5% annually.
Solution: The present value of the perpetuity is P(i ) =
100
i 0.05
, if
i > 0.05. Hence, P

(i ) =
100
(i 0.05)
2
,
=
P

(0.065)
P(0.065)
=
1
0.0650.05
= 66.66666667.
c 2009. Miguel A. Arcones. All rights reserved. Manual for SOA Exam FM/CAS Exam 2.
38/90
Chapter 6. Variable interest rates and portfolio insurance. Section 6.4. Duration, convexity.
Example 9
A portfolio consists of four bonds. The prices and modied
durations of the four bonds are given by the table:
Bond Present value Modied duration in years
Bond A $15050 4.3
Bond B $10350 10.4
Bond C $67080 7.6
Bond D $16750 6.5
Find the volatility of the whole portfolio.
Solution: We have that
=

n
j =1
P
j
(i )
j

n
j =1
P
j
(i )
=
(15050)(4.3) + (10350)(10.4) + (67080)(7.6) + (16750)(6.5)
15050 + 10350 + 67080 + 16750
=7.241948183 years.
c 2009. Miguel A. Arcones. All rights reserved. Manual for SOA Exam FM/CAS Exam 2.
39/90
Chapter 6. Variable interest rates and portfolio insurance. Section 6.4. Duration, convexity.
Example 9
A portfolio consists of four bonds. The prices and modied
durations of the four bonds are given by the table:
Bond Present value Modied duration in years
Bond A $15050 4.3
Bond B $10350 10.4
Bond C $67080 7.6
Bond D $16750 6.5
Find the volatility of the whole portfolio.
Solution: We have that
=

n
j =1
P
j
(i )
j

n
j =1
P
j
(i )
=
(15050)(4.3) + (10350)(10.4) + (67080)(7.6) + (16750)(6.5)
15050 + 10350 + 67080 + 16750
=7.241948183 years.
c 2009. Miguel A. Arcones. All rights reserved. Manual for SOA Exam FM/CAS Exam 2.
40/90
Chapter 6. Variable interest rates and portfolio insurance. Section 6.4. Duration, convexity.
Let P(i ) be the present value of a portfolio, when i is the eective
rate of interest. By a Taylor expansion, for h close to zero,
P(i + h) P(i ) + P

(i )h = P(i )
_
1

dh
_
= P(i ) (1 h) .
Example 10
A portfolio of bonds is worth 535000 at the current rate of interest
of 4.75%. Its Macaulay duration is 6.375. Estimate the value of
the portfolio if interest rates decrease by 0.10%.
Solution: We have that P(i + h) P(i )
_
1

dh
_
. In our case,
P(0.0475 0.0010) 535000(1 (1.0475)
1
(6.375)(0.001))
=538255.9666.
c 2009. Miguel A. Arcones. All rights reserved. Manual for SOA Exam FM/CAS Exam 2.
41/90
Chapter 6. Variable interest rates and portfolio insurance. Section 6.4. Duration, convexity.
Let P(i ) be the present value of a portfolio, when i is the eective
rate of interest. By a Taylor expansion, for h close to zero,
P(i + h) P(i ) + P

(i )h = P(i )
_
1

dh
_
= P(i ) (1 h) .
Example 10
A portfolio of bonds is worth 535000 at the current rate of interest
of 4.75%. Its Macaulay duration is 6.375. Estimate the value of
the portfolio if interest rates decrease by 0.10%.
Solution: We have that P(i + h) P(i )
_
1

dh
_
. In our case,
P(0.0475 0.0010) 535000(1 (1.0475)
1
(6.375)(0.001))
=538255.9666.
c 2009. Miguel A. Arcones. All rights reserved. Manual for SOA Exam FM/CAS Exam 2.
42/90
Chapter 6. Variable interest rates and portfolio insurance. Section 6.4. Duration, convexity.
Let P(i ) be the present value of a portfolio, when i is the eective
rate of interest. By a Taylor expansion, for h close to zero,
P(i + h) P(i ) + P

(i )h = P(i )
_
1

dh
_
= P(i ) (1 h) .
Example 10
A portfolio of bonds is worth 535000 at the current rate of interest
of 4.75%. Its Macaulay duration is 6.375. Estimate the value of
the portfolio if interest rates decrease by 0.10%.
Solution: We have that P(i + h) P(i )
_
1

dh
_
. In our case,
P(0.0475 0.0010) 535000(1 (1.0475)
1
(6.375)(0.001))
=538255.9666.
c 2009. Miguel A. Arcones. All rights reserved. Manual for SOA Exam FM/CAS Exam 2.
43/90
Chapter 6. Variable interest rates and portfolio insurance. Section 6.4. Duration, convexity.
If interest rates change from i into i + h, the percentage of change
in the present value of the portfolio is
P(i + h) P(i )
P(i )

P(i ) + P

(i )h P(i )
P(i )
=

dh = h.
Example 11
A bond has a volatility of 4.5 years, at the current annual interest
rate of 5%. Calculate the percentage of loss of value of the bond if
the annual eective interest rate increase 250 basis points.
Solution: The percentage of change is
h = (4.5)(0.025) = 0.1125 = 11.25%. The bond loses
11.25% of its value.
c 2009. Miguel A. Arcones. All rights reserved. Manual for SOA Exam FM/CAS Exam 2.
44/90
Chapter 6. Variable interest rates and portfolio insurance. Section 6.4. Duration, convexity.
If interest rates change from i into i + h, the percentage of change
in the present value of the portfolio is
P(i + h) P(i )
P(i )

P(i ) + P

(i )h P(i )
P(i )
=

dh = h.
Example 11
A bond has a volatility of 4.5 years, at the current annual interest
rate of 5%. Calculate the percentage of loss of value of the bond if
the annual eective interest rate increase 250 basis points.
Solution: The percentage of change is
h = (4.5)(0.025) = 0.1125 = 11.25%. The bond loses
11.25% of its value.
c 2009. Miguel A. Arcones. All rights reserved. Manual for SOA Exam FM/CAS Exam 2.
45/90
Chapter 6. Variable interest rates and portfolio insurance. Section 6.4. Duration, convexity.
If interest rates change from i into i + h, the percentage of change
in the present value of the portfolio is
P(i + h) P(i )
P(i )

P(i ) + P

(i )h P(i )
P(i )
=

dh = h.
Example 11
A bond has a volatility of 4.5 years, at the current annual interest
rate of 5%. Calculate the percentage of loss of value of the bond if
the annual eective interest rate increase 250 basis points.
Solution: The percentage of change is
h = (4.5)(0.025) = 0.1125 = 11.25%. The bond loses
11.25% of its value.
c 2009. Miguel A. Arcones. All rights reserved. Manual for SOA Exam FM/CAS Exam 2.
46/90
Chapter 6. Variable interest rates and portfolio insurance. Section 6.4. Duration, convexity.
Duration is a measurement of how long in years it takes for the
payments to be made. Mainly, we will consider applications to the
bond market. Duration is an important measure for investors to
consider, as bonds with higher durations are riskier and have a
higher price volatility than bonds with lower durations. We have
the following rules of thumb:

Higher coupon rates lead to lower duration.

Longer terms to maturity usually lead to longer duration.

Higher yields lead to lower duration.


c 2009. Miguel A. Arcones. All rights reserved. Manual for SOA Exam FM/CAS Exam 2.
47/90
Chapter 6. Variable interest rates and portfolio insurance. Section 6.4. Duration, convexity.
The price of a bond decreases as the rate of interest increases.
Suppose that you believe that interest rates will drop soon. You
want to make a benet by buying a bond today and selling it later
for a higher price. The prot you make is P(i + h) P(i ), where i
is the interest you buy the bond and i + h is the interest rate when
you sell the bond. Notice that you make a benet if h < 0. The
rate of return in your investment is
P(i + h) P(i )
P(i )
h.
So, between all possible bonds, you will make a biggest prot
investing in the bond with the highest possible volatility.
c 2009. Miguel A. Arcones. All rights reserved. Manual for SOA Exam FM/CAS Exam 2.
48/90
Chapter 6. Variable interest rates and portfolio insurance. Section 6.4. Duration, convexity.
Example 12
Suppose that you are comparing two veyear bonds with a face
value of 1000, and are expecting a drop in yields of 1% almost
immediately. The current yield is 8%. Bond 1 has 6% annual
coupons and bond 2 has annual 12% coupons. You would like to
invest 100,000 in the bond giving you the biggest return.
(i) Which would provide you with the highest potential gain if your
outlook for rates actually occurs?
(ii) Find the duration of each bond.
c 2009. Miguel A. Arcones. All rights reserved. Manual for SOA Exam FM/CAS Exam 2.
49/90
Chapter 6. Variable interest rates and portfolio insurance. Section 6.4. Duration, convexity.
Solution: (i) The price of the bond 1 is
(60)a
5

|8%
+ 1000(1.08)
5
= 920.15.
Its price after the change of interest rates is
(60)a
5

|7%
+ 1000(1.07)
5
= 959.00.
The gain is 38.85. The percentage of change is
959.00920.15
920.15
= 4.22%
The price of the bond 2 is
(120)a
5

|8%
+ 1000(1.08)
5
= 1159.71.
Its price after the change of interest rates is
(120)a
5

|7%
+ 1000(1.07)
5
= 1205.01.
The gain is 45.30. The percentage of change is
1205.011159.71
1159.71
= 3.91%.
Bond 1 is a better investment than bond 2 (if we believe that the
interest rates are going to fall to 1%).
c 2009. Miguel A. Arcones. All rights reserved. Manual for SOA Exam FM/CAS Exam 2.
50/90
Chapter 6. Variable interest rates and portfolio insurance. Section 6.4. Duration, convexity.
Solution: (i) The price of the bond 1 is
(60)a
5

|8%
+ 1000(1.08)
5
= 920.15.
Its price after the change of interest rates is
(60)a
5

|7%
+ 1000(1.07)
5
= 959.00.
The gain is 38.85. The percentage of change is
959.00920.15
920.15
= 4.22%
The price of the bond 2 is
(120)a
5

|8%
+ 1000(1.08)
5
= 1159.71.
Its price after the change of interest rates is
(120)a
5

|7%
+ 1000(1.07)
5
= 1205.01.
The gain is 45.30. The percentage of change is
1205.011159.71
1159.71
= 3.91%.
Bond 1 is a better investment than bond 2 (if we believe that the
interest rates are going to fall to 1%).
c 2009. Miguel A. Arcones. All rights reserved. Manual for SOA Exam FM/CAS Exam 2.
51/90
Chapter 6. Variable interest rates and portfolio insurance. Section 6.4. Duration, convexity.
(ii) For the rst bond, F = C = 1000, Fr = 60, i = 8% and n = 5.
Its price is
Fra
n

|i
+ F
n
= 60a
5

|8%
+ 1000(1.08)
5
= 920.15.
and its Macaulay duration is

d =
Fr (Ia)
n

|i
+ Fn
n
Fra
n

|i
+ F
n
=
60(Ia)
5

|0.08
+ (1000)(5)(1.08)
5
920.15
=
60(11.3651) + 3402.92
920.15
= 4.4393.
For the second bond, F = 1000, Fr = 120, i = 8% and n = 5. Its
price is
Fra
n

|i
+ F
n
= 120a
5

|8%
+ 1000(1.08)
5
= 1159.71.
and its Macaulay duration is

d =
120(Ia)
5

|0.08
+ (1000)(5)(1.08)
5
1159.71
=
120(11.3651) + 3402.92
1159.71
=4.1103.
c 2009. Miguel A. Arcones. All rights reserved. Manual for SOA Exam FM/CAS Exam 2.
52/90
Chapter 6. Variable interest rates and portfolio insurance. Section 6.4. Duration, convexity.
(ii) For the rst bond, F = C = 1000, Fr = 60, i = 8% and n = 5.
Its price is
Fra
n

|i
+ F
n
= 60a
5

|8%
+ 1000(1.08)
5
= 920.15.
and its Macaulay duration is

d =
Fr (Ia)
n

|i
+ Fn
n
Fra
n

|i
+ F
n
=
60(Ia)
5

|0.08
+ (1000)(5)(1.08)
5
920.15
=
60(11.3651) + 3402.92
920.15
= 4.4393.
For the second bond, F = 1000, Fr = 120, i = 8% and n = 5. Its
price is
Fra
n

|i
+ F
n
= 120a
5

|8%
+ 1000(1.08)
5
= 1159.71.
and its Macaulay duration is

d =
120(Ia)
5

|0.08
+ (1000)(5)(1.08)
5
1159.71
=
120(11.3651) + 3402.92
1159.71
=4.1103.
c 2009. Miguel A. Arcones. All rights reserved. Manual for SOA Exam FM/CAS Exam 2.
53/90
Chapter 6. Variable interest rates and portfolio insurance. Section 6.4. Duration, convexity.
Convexity
Denition 3
The convexity of the cashow
Contributions C
1
C
2
C
n
Time in years 1 2 n
is dened as
c =
P

(i )
P(i )
=

n
j =1
C
j
j (j + 1)(1 + i )
j 2

n
j =1
C
j
(1 + i )
j
=

n
j =1
C
j
j (j + 1)
j 2

n
j =1
C
j

j
.
Convexity is measured in years
2
.
c 2009. Miguel A. Arcones. All rights reserved. Manual for SOA Exam FM/CAS Exam 2.
54/90
Chapter 6. Variable interest rates and portfolio insurance. Section 6.4. Duration, convexity.
Main Properties of volatility

Convexity measures the rate of change of the volatility:


d
di
=
d
di
P

(i )
P(i )
=
P

(i )P(i ) P

(i )P

(i )
(P(i ))
2
= c ( )
2
.

The second order Taylor expansion of the present value with


respect to the yield is:
P(i +h) P(i ) +P

(i )h +
h
2
2
P

(i ) = P(i )
_
1 h +
h
2
2
c
_
.

Convexity is a measure of the curvature of the priceyield


curve for a bond. Convexity is related with the second term in
the Taylor expansion of the PV.

Using duration and convexity, we measure of how sensitive the


present value of a cashow is to interest rate changes.
c 2009. Miguel A. Arcones. All rights reserved. Manual for SOA Exam FM/CAS Exam 2.
55/90
Chapter 6. Variable interest rates and portfolio insurance. Section 6.4. Duration, convexity.
Main Properties of volatility

Using duration and convexity, we have the following Taylor


expansion:
P(i + h) P(i )
_
1 h +
h
2
2
c
_
.

The percentage change in the PV of a cashow is


P(i + h) P(i )
P(i )
h +
h
2
2
c.

Convexity can be used to compare bonds. If two bonds oer


the same duration and yield but one exhibits greater
convexity, the bond with greater convexity is more aected by
interest rates.
c 2009. Miguel A. Arcones. All rights reserved. Manual for SOA Exam FM/CAS Exam 2.
56/90
Chapter 6. Variable interest rates and portfolio insurance. Section 6.4. Duration, convexity.
Example 13
A portfolio of bonds is worth 350000 at the current rate of interest
of 5.2%. Its modied duration is 7.22. Its convexity is 370.
Estimate the value of the portfolio if interest rates increase by
0.2%.
Solution: We have that P(i + h) P(i )
_
1 h +
h
2
2
c
_
. In our
case,
P(0.052 + 0.002) = 350000
_
1 (7.22)(0.002) + (370)
(0.002)
2
2
_
=345205.
c 2009. Miguel A. Arcones. All rights reserved. Manual for SOA Exam FM/CAS Exam 2.
57/90
Chapter 6. Variable interest rates and portfolio insurance. Section 6.4. Duration, convexity.
Example 13
A portfolio of bonds is worth 350000 at the current rate of interest
of 5.2%. Its modied duration is 7.22. Its convexity is 370.
Estimate the value of the portfolio if interest rates increase by
0.2%.
Solution: We have that P(i + h) P(i )
_
1 h +
h
2
2
c
_
. In our
case,
P(0.052 + 0.002) = 350000
_
1 (7.22)(0.002) + (370)
(0.002)
2
2
_
=345205.
c 2009. Miguel A. Arcones. All rights reserved. Manual for SOA Exam FM/CAS Exam 2.
58/90
Chapter 6. Variable interest rates and portfolio insurance. Section 6.4. Duration, convexity.
Example 14
Calculate the duration, the modied duration and the convexity of
a $5000 face value 15year zerocoupon bond if the current
eective annual rate of interest is 7.5%.
Solution: Since P(i ) = (5000)(1 + i )
15
,
P

(i ) = (5000)(15)(1 + i )
16
,
P

(i ) = (5000)(15)(16)(1 + i )
17
, we have that
=
P

(0.75)
P(0.75)
= (15)(1 + 0.075)
1
= 13.95348837 years,

d = (1 + i ) = (1.075)(13.95348837) = 15 years and


c = (15)(16)(1 + 0.75)
2
= 78.36734694 years
2
.
c 2009. Miguel A. Arcones. All rights reserved. Manual for SOA Exam FM/CAS Exam 2.
59/90
Chapter 6. Variable interest rates and portfolio insurance. Section 6.4. Duration, convexity.
Example 14
Calculate the duration, the modied duration and the convexity of
a $5000 face value 15year zerocoupon bond if the current
eective annual rate of interest is 7.5%.
Solution: Since P(i ) = (5000)(1 + i )
15
,
P

(i ) = (5000)(15)(1 + i )
16
,
P

(i ) = (5000)(15)(16)(1 + i )
17
, we have that
=
P

(0.75)
P(0.75)
= (15)(1 + 0.075)
1
= 13.95348837 years,

d = (1 + i ) = (1.075)(13.95348837) = 15 years and


c = (15)(16)(1 + 0.75)
2
= 78.36734694 years
2
.
c 2009. Miguel A. Arcones. All rights reserved. Manual for SOA Exam FM/CAS Exam 2.
60/90
Chapter 6. Variable interest rates and portfolio insurance. Section 6.4. Duration, convexity.
Example 15
Calculate the duration, the modied duration and the convexity of
a level payments perpetuityimmediate with payments at the end
of the year if the current eective annual rate of interest is 5%.
Solution: Since P(i ) =
C
i
, P

(i ) =
C
i
2
, P

(i ) =
2C
i
3
, we have that
=
P

(0.05)
P(0.05)
=
1
0.05
= 20 years,

d = (1 + i ) = (1.05)(20) = 21
ears and c =
2
i
2
=
2
(0.05)
2
= 800 years
2
.
c 2009. Miguel A. Arcones. All rights reserved. Manual for SOA Exam FM/CAS Exam 2.
61/90
Chapter 6. Variable interest rates and portfolio insurance. Section 6.4. Duration, convexity.
Example 15
Calculate the duration, the modied duration and the convexity of
a level payments perpetuityimmediate with payments at the end
of the year if the current eective annual rate of interest is 5%.
Solution: Since P(i ) =
C
i
, P

(i ) =
C
i
2
, P

(i ) =
2C
i
3
, we have that
=
P

(0.05)
P(0.05)
=
1
0.05
= 20 years,

d = (1 + i ) = (1.05)(20) = 21
ears and c =
2
i
2
=
2
(0.05)
2
= 800 years
2
.
c 2009. Miguel A. Arcones. All rights reserved. Manual for SOA Exam FM/CAS Exam 2.
62/90
Chapter 6. Variable interest rates and portfolio insurance. Section 6.4. Duration, convexity.
Example 16
A 100 par value 3 year bond pays annual coupons at a rate 7%
coupon rate (with annual coupon payments). The current annual
eective interest rate is 7%.
c 2009. Miguel A. Arcones. All rights reserved. Manual for SOA Exam FM/CAS Exam 2.
63/90
Chapter 6. Variable interest rates and portfolio insurance. Section 6.4. Duration, convexity.
Example 16
A 100 par value 3 year bond pays annual coupons at a rate 7%
coupon rate (with annual coupon payments). The current annual
eective interest rate is 7%.
(i) Calculate the duration, the modied duration and the convexity
of the bond.
c 2009. Miguel A. Arcones. All rights reserved. Manual for SOA Exam FM/CAS Exam 2.
64/90
Chapter 6. Variable interest rates and portfolio insurance. Section 6.4. Duration, convexity.
Example 16
A 100 par value 3 year bond pays annual coupons at a rate 7%
coupon rate (with annual coupon payments). The current annual
eective interest rate is 7%.
(i) Calculate the duration, the modied duration and the convexity
of the bond.
Solution: (i) The cashow is
Contributions 7 7 107
Time 1 2 3
The duration is

d =
(7)(1.07)
1
+ 2(7)(1.07)
2
+ 3(107)(1.07)
3
100
= 2.808018.
The modied duration is =
2.808018
1.07
= 2.6243. The convexity is
c =
(7)(1)(2)(1.07)
3
+ (7)(2)(3)(1.07)
4
+ (107)(3)(4)(1.07)
5
100
=9.58944.
c 2009. Miguel A. Arcones. All rights reserved. Manual for SOA Exam FM/CAS Exam 2.
65/90
Chapter 6. Variable interest rates and portfolio insurance. Section 6.4. Duration, convexity.
Example 16
A 100 par value 3 year bond pays annual coupons at a rate 7%
coupon rate (with annual coupon payments). The current annual
eective interest rate is 7%.
(ii) If the interest rate change from 7% to 8%, what is the percentage
change in the price of the bond?
c 2009. Miguel A. Arcones. All rights reserved. Manual for SOA Exam FM/CAS Exam 2.
66/90
Chapter 6. Variable interest rates and portfolio insurance. Section 6.4. Duration, convexity.
Example 16
A 100 par value 3 year bond pays annual coupons at a rate 7%
coupon rate (with annual coupon payments). The current annual
eective interest rate is 7%.
(ii) If the interest rate change from 7% to 8%, what is the percentage
change in the price of the bond?
Solution: (ii) If i = 7%, the price of the bond is
7a
3

|7%
+ 100(1.07)
3
= 100.
If i = 8%, the price of the bond is
7a
3

|8%
+ 100(1.08)
3
= 97.4229.
The change in percentage is
97.4229
100
1 = 2.5771%.
c 2009. Miguel A. Arcones. All rights reserved. Manual for SOA Exam FM/CAS Exam 2.
67/90
Chapter 6. Variable interest rates and portfolio insurance. Section 6.4. Duration, convexity.
Example 16
A 100 par value 3 year bond pays annual coupons at a rate 7%
coupon rate (with annual coupon payments). The current annual
eective interest rate is 7%.
(iii) Using the duration rule, including convexity, what is the per-
centage change in the bond price?
c 2009. Miguel A. Arcones. All rights reserved. Manual for SOA Exam FM/CAS Exam 2.
68/90
Chapter 6. Variable interest rates and portfolio insurance. Section 6.4. Duration, convexity.
Example 16
A 100 par value 3 year bond pays annual coupons at a rate 7%
coupon rate (with annual coupon payments). The current annual
eective interest rate is 7%.
(iii) Using the duration rule, including convexity, what is the per-
centage change in the bond price?
Solution: (iii) The estimation in the change in percentage is
h +
h
2
2
c = (2.6243)(0.01) +
(0.01)
2
2
(9.58944)
=0.025760528 = 2.576353%.
c 2009. Miguel A. Arcones. All rights reserved. Manual for SOA Exam FM/CAS Exam 2.
69/90
Chapter 6. Variable interest rates and portfolio insurance. Section 6.4. Duration, convexity.
Theorem 7
Suppose that we have n dierent investments. The j th
investment has present value P
j
and convexity c
j
. Then, the
convexity of the combined investments is
c =

n
j =1
P
j
(i ) c
j

n
j =1
P
j
(i )
.
Proof.
Notice that
c =

n
j =1
P

j
(i )
P
j
(i )
=

n
j =1
P
j
(i ) c
j
P
j
(i )
.
c 2009. Miguel A. Arcones. All rights reserved. Manual for SOA Exam FM/CAS Exam 2.
70/90
Chapter 6. Variable interest rates and portfolio insurance. Section 6.4. Duration, convexity.
Theorem 7
Suppose that we have n dierent investments. The j th
investment has present value P
j
and convexity c
j
. Then, the
convexity of the combined investments is
c =

n
j =1
P
j
(i ) c
j

n
j =1
P
j
(i )
.
Proof.
Notice that
c =

n
j =1
P

j
(i )
P
j
(i )
=

n
j =1
P
j
(i ) c
j
P
j
(i )
.
c 2009. Miguel A. Arcones. All rights reserved. Manual for SOA Exam FM/CAS Exam 2.
71/90
Chapter 6. Variable interest rates and portfolio insurance. Section 6.4. Duration, convexity.
Example 17
A company has issued debt using the following bonds:
Bond Present value Macaulays duration convexity
Bond A 100000 5.3 1.2
Bond B 50000 3.4 3.2
Bond C 120000 12.2 6.2
Bond D 80000 2.3 3.6
Find the Macaulays duration and the convexity for the entire
portfolio.
c 2009. Miguel A. Arcones. All rights reserved. Manual for SOA Exam FM/CAS Exam 2.
72/90
Chapter 6. Variable interest rates and portfolio insurance. Section 6.4. Duration, convexity.
Solution: Let P
j
,

d
j
and c
j
be the present value, Macaulays
duration and convexity, respectively, of the j th bond, 1 j 4.
Then, the Macaulays duration of the whole portfolio is

d =

n
j =1
P
j

d
j

n
j =1
P
j
=
100000(5.3) + 50000(3.4) + 120000(12.2) + 80000(2.3)
100000 + 50000 + 120000 + 80000
=6.708571429.
The convexity of the whole portfolio is
c =

n
j =1
P
j
c
j

n
j =1
P
j
=
100000(1.2) + 50000(3.2) + 120000(6.2) + 80000(3.6)
100000 + 50000 + 120000 + 80000
=3.748571429.
c 2009. Miguel A. Arcones. All rights reserved. Manual for SOA Exam FM/CAS Exam 2.
73/90
Chapter 6. Variable interest rates and portfolio insurance. Section 6.4. Duration, convexity.
In the case of payments made every
1
m
years, it is usual to use the
nominal rate of interest i
(m)
as the variable. The present value of
the cashow
Contributions C
1
C
2
C
n
Time (in years)
1
m
2
m

n
m
is
P(i
(m)
) =
n

j =1
C
j
_
1 +
i
(m)
m
_
j
.
c 2009. Miguel A. Arcones. All rights reserved. Manual for SOA Exam FM/CAS Exam 2.
74/90
Chapter 6. Variable interest rates and portfolio insurance. Section 6.4. Duration, convexity.
The duration (or Macaulays duration) of the cashow is

d =

n
j =1
j
m
C
j
_
1 +
i
(m)
m
_
j

n
j =1
C
j
_
1 +
i
(m)
m
_
j
=
1
m

n
j =1
jC
j
_
1 +
i
(m)
m
_
j

n
j =1
C
j
_
1 +
i
(m)
m
_
j
years.
The volatility is
=
P

(i
(m)
)
P(i
(m)
)
=

n
j =1
C
j
(j )(1 +
i
(m)
m
)
j 1 1
m

n
j =1
C
j
(1 +
i
(m)
m
)
j
=
_
1 +
i
(m)
m
_
1

d.
The convexity is
c =
P

(i
(m)
)
P(i
(m)
)
=

n
j =1
C
j
(j )(j 1)(1 +
i
(m)
m
)
j 2 1
m
2

n
j =1
C
j
(1 +
i
(m)
m
)
j
=
1
m
2

n
j =1
C
j
j (j + 1)(1 +
i
(m)
m
)
j 2

n
j =1
C
j
(1 +
i
(m)
m
)
j
.
c 2009. Miguel A. Arcones. All rights reserved. Manual for SOA Exam FM/CAS Exam 2.
75/90
Chapter 6. Variable interest rates and portfolio insurance. Section 6.4. Duration, convexity.
Example 18
You are given the following information about a bond:

The termtomaturity is 2 years.

The bond has a 9% annual coupon rate, paid semiannually.

The annual bondequivalent yieldtomaturity is 8%.

The par value is $100.


c 2009. Miguel A. Arcones. All rights reserved. Manual for SOA Exam FM/CAS Exam 2.
76/90
Chapter 6. Variable interest rates and portfolio insurance. Section 6.4. Duration, convexity.
Example 18
You are given the following information about a bond:

The termtomaturity is 2 years.

The bond has a 9% annual coupon rate, paid semiannually.

The annual bondequivalent yieldtomaturity is 8%.

The par value is $100.


(i) Calculate the current price of the bond.
c 2009. Miguel A. Arcones. All rights reserved. Manual for SOA Exam FM/CAS Exam 2.
77/90
Chapter 6. Variable interest rates and portfolio insurance. Section 6.4. Duration, convexity.
Example 18
You are given the following information about a bond:

The termtomaturity is 2 years.

The bond has a 9% annual coupon rate, paid semiannually.

The annual bondequivalent yieldtomaturity is 8%.

The par value is $100.


(i) Calculate the current price of the bond.
Solution: (i) Since F = 100, Fr = 4.5, i = 4% and n = 4, the
price is is
Fra
n

|i
+ P
n
= (4.5)a
4

|4%
+ 100(1.04)
4
= 101.8149.
c 2009. Miguel A. Arcones. All rights reserved. Manual for SOA Exam FM/CAS Exam 2.
78/90
Chapter 6. Variable interest rates and portfolio insurance. Section 6.4. Duration, convexity.
Example 18
You are given the following information about a bond:

The termtomaturity is 2 years.

The bond has a 9% annual coupon rate, paid semiannually.

The annual bondequivalent yieldtomaturity is 8%.

The par value is $100.


(ii) Calculate the Macaulay duration of the bond.
c 2009. Miguel A. Arcones. All rights reserved. Manual for SOA Exam FM/CAS Exam 2.
79/90
Chapter 6. Variable interest rates and portfolio insurance. Section 6.4. Duration, convexity.
Example 18
You are given the following information about a bond:

The termtomaturity is 2 years.

The bond has a 9% annual coupon rate, paid semiannually.

The annual bondequivalent yieldtomaturity is 8%.

The par value is $100.


(ii) Calculate the Macaulay duration of the bond.
Solution: (ii) The Macaulay duration is (in years)

d =
1
2
Fr (Ia)
n

|i
+nF
n
Fra
n

|i
+F
n
=
1
2
(4.5)(8.896856)+(4)(100)(0.854804)
101.8149
= 1.875744.
c 2009. Miguel A. Arcones. All rights reserved. Manual for SOA Exam FM/CAS Exam 2.
80/90
Chapter 6. Variable interest rates and portfolio insurance. Section 6.4. Duration, convexity.
Example 18
You are given the following information about a bond:

The termtomaturity is 2 years.

The bond has a 9% annual coupon rate, paid semiannually.

The annual bondequivalent yieldtomaturity is 8%.

The par value is $100.


(iii) Calculate the convexity of the bond.
c 2009. Miguel A. Arcones. All rights reserved. Manual for SOA Exam FM/CAS Exam 2.
81/90
Chapter 6. Variable interest rates and portfolio insurance. Section 6.4. Duration, convexity.
Example 18
You are given the following information about a bond:

The termtomaturity is 2 years.

The bond has a 9% annual coupon rate, paid semiannually.

The annual bondequivalent yieldtomaturity is 8%.

The par value is $100.


(iii) Calculate the convexity of the bond.
Solution: (iii) The convexity is (in years
2
)
c =
1
4

(4.5)(1)(2)(1.04)
3
+(4.5)(2)(3)(1.04)
4
+(4.5)(3)(4)(1.04)
5
+(104.5)(4)(5)(1.04)
6
101.8149
=
1
4
8.0009 + 23.0797 + 44.3841 + 1651.7574
101.8149
= 4.241083
where we have used
1
4
because the time is in half years.
c 2009. Miguel A. Arcones. All rights reserved. Manual for SOA Exam FM/CAS Exam 2.
82/90
Chapter 6. Variable interest rates and portfolio insurance. Section 6.4. Duration, convexity.
Example 18
You are given the following information about a bond:

The termtomaturity is 2 years.

The bond has a 9% annual coupon rate, paid semiannually.

The annual bondequivalent yieldtomaturity is 8%.

The par value is $100.


(iv) For a 200 basis point increase in yield, determine the amount of
error in using duration and convexity to estimate the price change.
c 2009. Miguel A. Arcones. All rights reserved. Manual for SOA Exam FM/CAS Exam 2.
83/90
Chapter 6. Variable interest rates and portfolio insurance. Section 6.4. Duration, convexity.
(iv) For a 200 basis point increase in yield, determine the amount of
error in using duration and convexity to estimate the price change.
Solution: (iv) We need to nd
P(i +h)P(i )P

(i )hP

(i )
h
2
2
= P(i +h)P(i )
_
1 h + c
h
2
2
_
.
The price of the bond after the change in interest rates is
P(i +h) = Fra
n

|i +h
+P(1+i +h)
n
= (4.5)a
4

|6%
+100(1.06)
4
= 94.8023.
So, the error is
P(i + h) P(i )
_
1 h + c
h
2
2
_
=94.8023 101.8149
_
1 (1.875744)(1.04)
1
(0.02) + 4.241083
(0.02)
2
2
_
=94.8023 101.8149(0.9647762) = 3.426292.
c 2009. Miguel A. Arcones. All rights reserved. Manual for SOA Exam FM/CAS Exam 2.
84/90
Chapter 6. Variable interest rates and portfolio insurance. Section 6.4. Duration, convexity.
Example 18
Find the price and Macaulay duration of the following
xedincome securities, given the annual eective rate of interest
4.75% and par value of each bond is $1,000.
c 2009. Miguel A. Arcones. All rights reserved. Manual for SOA Exam FM/CAS Exam 2.
85/90
Chapter 6. Variable interest rates and portfolio insurance. Section 6.4. Duration, convexity.
Example 18
Find the price and Macaulay duration of the following
xedincome securities, given the annual eective rate of interest
4.75% and par value of each bond is $1,000.
(i) 3year bond with 5.00% annual coupons
c 2009. Miguel A. Arcones. All rights reserved. Manual for SOA Exam FM/CAS Exam 2.
86/90
Chapter 6. Variable interest rates and portfolio insurance. Section 6.4. Duration, convexity.
Example 18
Find the price and Macaulay duration of the following
xedincome securities, given the annual eective rate of interest
4.75% and par value of each bond is $1,000.
(i) 3year bond with 5.00% annual coupons
Solution: (i) We have F = 1000, r = 0.05, i = 4.75%, Fr = 50
and n = 3. The price of the bond is
Fra
n

|i
+ F
n
= 50a
3

|4.75%
+ 1000(1.0475)
3
= 1006.84.
The Macaulay duration is (in years)

d =
Fr (Ia)
n

|i
+ Fn
n
Fra
n

|i
+ F
n
=
50(Ia)
3

|0.0475
+ (1000)(3)(1.0475)
3
1006.84
=
50(5.3875) + 2610.11
1006.84
= 2.8599.
c 2009. Miguel A. Arcones. All rights reserved. Manual for SOA Exam FM/CAS Exam 2.
87/90
Chapter 6. Variable interest rates and portfolio insurance. Section 6.4. Duration, convexity.
Example 18
Find the price and Macaulay duration of the following
xedincome securities, given the annual eective rate of interest
4.75% and par value of each bond is $1,000.
(ii) 3year bond with 5.00% semiannual coupons
c 2009. Miguel A. Arcones. All rights reserved. Manual for SOA Exam FM/CAS Exam 2.
88/90
Chapter 6. Variable interest rates and portfolio insurance. Section 6.4. Duration, convexity.
Example 18
Find the price and Macaulay duration of the following
xedincome securities, given the annual eective rate of interest
4.75% and par value of each bond is $1,000.
(ii) 3year bond with 5.00% semiannual coupons
Solution: (ii) We have i = 0.0475, i
(2)
= 0.046949,
i
(2)
2
=
0.0234745, F = 1000, r = 0.025, Fr = 25 and n = 6. The price of
the bond is
25a
6

|0.0234745
+ 1000(1.0234745)
6
= 1008.45.
The Macaulay duration is (in years)

d =
1
2
Fr (Ia)
n

|i
+ Fn
n
Fra
n

|i
+ F
n
=
1
2
25(Ia)
6

|0.0234745
+ (1000)(6)(1.0234745)
6
1008.45
=
(12.5)(19.0026) + 2610.11
1008.45
= 2.823782.
c 2009. Miguel A. Arcones. All rights reserved. Manual for SOA Exam FM/CAS Exam 2.
89/90
Chapter 6. Variable interest rates and portfolio insurance. Section 6.4. Duration, convexity.
Example 18
Find the price and Macaulay duration of the following
xedincome securities, given the annual eective rate of interest
4.75% and par value of each bond is $1,000.
(iii) 3year bond with 5.00% quarter coupons.
c 2009. Miguel A. Arcones. All rights reserved. Manual for SOA Exam FM/CAS Exam 2.
90/90
Chapter 6. Variable interest rates and portfolio insurance. Section 6.4. Duration, convexity.
Example 18
Find the price and Macaulay duration of the following
xedincome securities, given the annual eective rate of interest
4.75% and par value of each bond is $1,000.
(iii) 3year bond with 5.00% quarter coupons.
Solution: (iii) We have i = 0.0475, i
(4)
= 0.046677,
i
(4)
4
=
0.0116692, F = 1000, Fr = 12.5 and n = 6. The price of the
bond is
12.5a
12

|0.0116692
+ 1000(1.0116692)
12
= 1009.25.
The Macaulay duration is (in years)

d =
1
4
Fr (Ia)
n

|i
+ Fn
n
Fra
n

|i
+ F
n
=
1
4
12.5(Ia)
12

|0.0116692
+ (1000)(12)(1.0116692)
12
1009.25
=
(3.125)(70.8531) + 2610.11
1009.25
= 2.8056.
c 2009. Miguel A. Arcones. All rights reserved. Manual for SOA Exam FM/CAS Exam 2.
1/20
Chapter 6. Variable interest rates and portfolio insurance.
Manual for SOA Exam FM/CAS Exam 2.
Chapter 6. Variable interest rates and portfolio insurance.
Section 6.5. Assetliability management.
c 2009. Miguel A. Arcones. All rights reserved.
Extract from:
Arcones Manual for the SOA Exam FM/CAS Exam 2,
Financial Mathematics. Fall 2009 Edition,
available at http://www.actexmadriver.com/
c 2009. Miguel A. Arcones. All rights reserved. Manual for SOA Exam FM/CAS Exam 2.
2/20
Chapter 6. Variable interest rates and portfolio insurance. Section 6.5. Assetliability management.
Hedging
The money collected by insurance companies is invested and
subject to risk (possible losses). For example, bonds can drop in
value if the rate of interest changes. Several methods have being
developed to minimize the risk of investing. A method, often
sophisticated, employed to minimize investment risk is called
hedging. In this section, we study several hedging methods.
c 2009. Miguel A. Arcones. All rights reserved. Manual for SOA Exam FM/CAS Exam 2.
3/20
Chapter 6. Variable interest rates and portfolio insurance. Section 6.5. Assetliability management.
Match assets and liabilities
If possible we would like to match assets and liabilities. i.e. the
total amount of contributions in assets equals the total amount of
contributions in liabilities.

If an insurance company has more liabilities than assets, it


may fail to meet its commitments to its policyholders.

If an insurance company has more assets than liabilities, it will


not the make the appropriate prot for the capital available.
c 2009. Miguel A. Arcones. All rights reserved. Manual for SOA Exam FM/CAS Exam 2.
4/20
Chapter 6. Variable interest rates and portfolio insurance. Section 6.5. Assetliability management.
Example 1
A company has liabilities of 2000, 5000 and 10000 payable at the
end of years 1, 2 and 5 respectively. The investments available to
the company are the following zerocoupon 1000 par value bonds:
Bond Maturity (years) Eective Annual Yield
Bond A 1 year 4.5%
Bond B 2 years 5.0
Bond C 3 years 5.5
Bond D 4 years 6.0
Bond E 5 years 6.5
Determine the cost for matching these liabilities exactly.
Solution: We need to buy 2 Bonds A, 5 Bonds B and 10 Bonds E.
The cost is
2000(1.045)
1
+ 5000(1.05)
2
+ 10000(1.065)
5
= 13747.83136.
c 2009. Miguel A. Arcones. All rights reserved. Manual for SOA Exam FM/CAS Exam 2.
5/20
Chapter 6. Variable interest rates and portfolio insurance. Section 6.5. Assetliability management.
Example 1
A company has liabilities of 2000, 5000 and 10000 payable at the
end of years 1, 2 and 5 respectively. The investments available to
the company are the following zerocoupon 1000 par value bonds:
Bond Maturity (years) Eective Annual Yield
Bond A 1 year 4.5%
Bond B 2 years 5.0
Bond C 3 years 5.5
Bond D 4 years 6.0
Bond E 5 years 6.5
Determine the cost for matching these liabilities exactly.
Solution: We need to buy 2 Bonds A, 5 Bonds B and 10 Bonds E.
The cost is
2000(1.045)
1
+ 5000(1.05)
2
+ 10000(1.065)
5
= 13747.83136.
c 2009. Miguel A. Arcones. All rights reserved. Manual for SOA Exam FM/CAS Exam 2.
6/20
Chapter 6. Variable interest rates and portfolio insurance. Section 6.5. Assetliability management.
Example 2
A bond portfolio manager in a pension fund is designing a bond
portfolio. His company has an obligation to pay 50000 at the end
of each year for 3 years. He can purchase a combination of the
following three bonds in order to exactly match its obligation:
(i) 1year 5% annual coupon bond with a yield rate of 6%.
(ii) 2year 7% annual coupon bond with a yield rate of 7%.
(ii) 3year 9% annual coupon bond with a yield rate of 8%.
(i) How much of each bond should you purchase in order to
exactly match the liabilities?
(ii) Find the cost of such a combination of bonds.
c 2009. Miguel A. Arcones. All rights reserved. Manual for SOA Exam FM/CAS Exam 2.
7/20
Chapter 6. Variable interest rates and portfolio insurance. Section 6.5. Assetliability management.
Solution: (i) Suppose that the face values of the bonds we buy are
x, y, z, in each of the bonds, respectively. Then, the cashows are
Liabilities 50000 50000 50000
Bond 1 (1.05)x 0 0
Bond 2 (0.07)y (1.07)y 0
Bond 3 (0.09)z (0.09)z (1.09)z
Time 1 2 3
In order to match assets and liabilities,
50000 = (1.05)x + (0.07)y + (0.09)z,
50000 = (1.07)y + (0.09)z,
50000 = (1.09)z.
Hence,
z =
50000
1.09
= 45871.56,
y =
50000(0.09)(45871.56)
1.07
= 42870.61645,
x =
50000(0.07)(42870.61645)(0.09)(45871.56)
1.05
= 40829.15852.
c 2009. Miguel A. Arcones. All rights reserved. Manual for SOA Exam FM/CAS Exam 2.
8/20
Chapter 6. Variable interest rates and portfolio insurance. Section 6.5. Assetliability management.
(ii) Find the cost of such a combination of bonds.
Solution: (ii) The total price of the bonds is
(40829.15852)(1.05)(1.06)
1
+ (42870.61645)((0.07)a
2

|7%
+ (1.07)
2
)
+ (45871.56)((0.09)a
3

|8%
+ (1.08)
3
)
=40035.97426 + 42870.61645 + 47053.71459 = 129960.3053.
c 2009. Miguel A. Arcones. All rights reserved. Manual for SOA Exam FM/CAS Exam 2.
9/20
Chapter 6. Variable interest rates and portfolio insurance. Section 6.5. Assetliability management.
Theory of immunization
Fluctuation in interest rates can cause losses to a nancial
institution. Suppose that a nancial institution has a cashow of
assets and a cashow of liabilities. The present values of these
cashows is sensitive to changes of interest rates. If interest rates
fall, the present value of the cashow of liabilities increases. If
interest rates increase, the present value of the cashow of assets
decreases. To mitigate the risk associated with a change in the
interest rates, Redington (1954) introduced the theory of
immunization. Immunization is a hedging method against the risk
associated with changes in interest rates.
c 2009. Miguel A. Arcones. All rights reserved. Manual for SOA Exam FM/CAS Exam 2.
10/20
Chapter 6. Variable interest rates and portfolio insurance. Section 6.5. Assetliability management.
According with the traditional immunization theory, a portfolio is
immunized against uctuations in interest rates if the 3 criteria are
satised:
1. The present value of the assets must equal the present value
of the liabilities.
2. The duration of the assets must equal the duration of the
liabilities.
3. The convexity of the assets must be greater than that of the
liabilities.
The rst of the previous 3 conditions is an eciency condition.
The last conditions are imposed so that interest rate risk for the
assets osets the interest rate risk for the liabilities.
c 2009. Miguel A. Arcones. All rights reserved. Manual for SOA Exam FM/CAS Exam 2.
11/20
Chapter 6. Variable interest rates and portfolio insurance. Section 6.5. Assetliability management.
If the the immunization conditions are satised, then
P
A
(i ) = P
L
(i ),
A
=
L
, c
A
> c
L
.
The approximations to the present value of assets and liabilities are:
P
A
(i + h) P
A
(i )

1
A
h +
h
2
2
c
A

and
P
L
(i + h) P
L
(i )

1
L
h +
h
2
2
c
L

.
Hence
P
A
(i + h) P
L
(i + h) = P
A
(i )
h
2
2
( c c
L
) > 0.
For small variation in interest rates the previous approximations are
accurate. Since high variations in interest rates in short periods of
time are unlikely, it is possible to hedge against interest rate
variations by immunizing periodically.
c 2009. Miguel A. Arcones. All rights reserved. Manual for SOA Exam FM/CAS Exam 2.
12/20
Chapter 6. Variable interest rates and portfolio insurance. Section 6.5. Assetliability management.
Example 3
An actuarial department needs to setup an investment program to
pay for a loan of $20000 due in 2 years. The only available
investments are:
(i) a money market fund paying the current rate of interest.
(ii) 5year zerocoupon bonds earning 4%.
Assume that the current rate of interest is 4%. Develop an
investment program satisfying the theory of immunization. Graph
the present value of asset minus liabilities versus interest rates.
c 2009. Miguel A. Arcones. All rights reserved. Manual for SOA Exam FM/CAS Exam 2.
13/20
Chapter 6. Variable interest rates and portfolio insurance. Section 6.5. Assetliability management.
Solution: The investment program invest x in the money maker
fund, and y in the zero coupon bond. The PV of the cashow of
assets and liabilities is
P(i ) = x + y(1.04)
5
(1 + i )
5
20000(1 + i )
2
. We solve for x
and y such that P(0.04) = 0 and P

(0.04) = 0. Since
P

(i ) = (5)y(1.04)
5
(1 + i )
6
+ (2)(20000)(1 + i )
3
, we need to
solve
x+y = 20000(1.04)
2
, and (5)y(1.04)
1
+(2)(20000)(1.04)
3
= 0.
We get y =
(2)(20000)(1.04)
2
5
= 7396.45 and
x = 20000(1.04)
2
y = 11094.67. Since
P

(i ) = (5)(6)y(1.04)
5
(1 + i )
7
(2)(3)(20000)(1 + i )
4
,
P

(0.04) = (5)(6)7396.45(1.04)
2
(2)(3)(20000)(1.04)
4
= 102576.5.
The convexity of the cashow is positive. The investment strategy
consisting in allocate 11094.67 in the money market account and
7396.45 in bonds satises the immunization requirements.
c 2009. Miguel A. Arcones. All rights reserved. Manual for SOA Exam FM/CAS Exam 2.
14/20
Chapter 6. Variable interest rates and portfolio insurance. Section 6.5. Assetliability management.
The graph of the present value of asset minus liabilities versus
interest rates is Figure 1.
Figure 1: Present value of assets minus liabilities
c 2009. Miguel A. Arcones. All rights reserved. Manual for SOA Exam FM/CAS Exam 2.
15/20
Chapter 6. Variable interest rates and portfolio insurance. Section 6.5. Assetliability management.
Example 4
An actuarial department has determined that the company has a
liability of $10,000 that will be payable in seven years. The
company has two choices of assets to invest in: a 5year
zero-coupon bond and a 10year zero coupon bond. The interest
rate is 5%. How can the actuarial department immunize its
portfolio?
c 2009. Miguel A. Arcones. All rights reserved. Manual for SOA Exam FM/CAS Exam 2.
16/20
Chapter 6. Variable interest rates and portfolio insurance. Section 6.5. Assetliability management.
Solution: The investment program invest x in the 5year bond,
and y in the 10year bond. The PV of the cashow of assets and
liabilities is
P(i ) = x(1.05)
5
(1 + i )
5
+ y(1.05)
10
(1 + i )
10
10000(1 + i )
7
.
So,
P

(i ) = (5)x(1.05)
5
(1 + i )
6
(10)y(1.05)
10
(1 + i )
11
+ (7)10000(1 + i )
8
and
P

(i ) = (5)(6)x(1.05)
5
(1 + i )
7
+ (10)(11)y(1.05)
10
(1 + i )
12
(7)(8)10000(1 + i )
9
.
We solve for x and y in the equations P(0.05) = 0 and
P

(0.05) = 0, i.e.
x + y = (10000)(1.05)
7
, and 5x + 10y = (70000)(1.05)
7
.
We get x = (6000)(1.05)
7
= 4264.08 and
y = (4000)(1.05)
7
= 2842.72.
c 2009. Miguel A. Arcones. All rights reserved. Manual for SOA Exam FM/CAS Exam 2.
17/20
Chapter 6. Variable interest rates and portfolio insurance. Section 6.5. Assetliability management.
We have that
P

(0.05) = (5)(6)(6000)(1.05)
7
(1.05)
5
(1.05)
7
+ (10)(11)(4000)(1.05)
7
(1.05)
10
(1.05)
12
(7)(8)(10000)(1.05)
9
=(60000)(1.05)
9
> 0.
So, the investment program satises the immunization
requirements.
c 2009. Miguel A. Arcones. All rights reserved. Manual for SOA Exam FM/CAS Exam 2.
18/20
Chapter 6. Variable interest rates and portfolio insurance. Section 6.5. Assetliability management.
Example 5
An actuarial department has determined that the company has a
liability of $10,000 that will be payable in two years. The company
has two choices of assets to invest in: a oneyear zerocoupon
bond and a threeyear zerocoupon bond. The interest rate is 6%.
(i) Find an investment portfolio which immunizes this portfolio.
(ii) Find the interval of interest rates at which the present value of
assets is bigger than the present value of liabilities.
c 2009. Miguel A. Arcones. All rights reserved. Manual for SOA Exam FM/CAS Exam 2.
19/20
Chapter 6. Variable interest rates and portfolio insurance. Section 6.5. Assetliability management.
Solution: (i) The investment program invest x in the oneyear
zerocoupon bond, and y in the threeyear zerocoupon bond.
The PV of the cashow of assets and liabilities is
P(i ) = x(1.06)(1 + i )
1
+ y(1.06)
3
(1 + i )
3
10000(1 + i )
2
.
So,
P

(i ) = x(1.06)(1)(1 + i )
2
+ y(1.06)
3
(3)(1 + i )
4
10000(2)(1 + i )
3
and
P

(i ) = x(1.06)(1)(2)(1 + i )
3
+ y(1.06)
3
(3)(4)(1 + i )
5
10000(2)(3)(1 + i )
4
.
We solve for x and y in the equations P(0.06) = 0 and
P

(0.06) = 0, i.e.
x + y = (10000)(1.06)
2
, and x + 3y = (20000)(1.06)
2
.
We get x = y = (5000)(1.06)
2
= 4449.98222.
c 2009. Miguel A. Arcones. All rights reserved. Manual for SOA Exam FM/CAS Exam 2.
20/20
Chapter 6. Variable interest rates and portfolio insurance. Section 6.5. Assetliability management.
We have that
P

(0.06) = (10000)(1.06)
2
> 0.
So, the investment program satises the immunization
requirements.
(ii) The present value of assets is bigger than the present value of
liabilities if
0 (5000)(1.06)
1
(1 + i )
1
+ (5000)(1.06)(1 + i )
3
10000(1 + i )
2
which is equivalent to
(5000)(1.06)
1
(1 + i )
2
+ (5000)(1.06) 10000(1 + i )
=(5000)(1.06)
1

(1 + i )
2
(2)(1.06) + (1.06)
2

=(5000)(1.06)
1
(i 0.06)
2
0.
The present value of assets is bigger than the present value of
liabilities for any interest rate.
c 2009. Miguel A. Arcones. All rights reserved. Manual for SOA Exam FM/CAS Exam 2.
1/52
Chapter 7. Derivatives markets.
Manual for SOA Exam FM/CAS Exam 2.
Chapter 7. Derivatives markets.
Section 7.1. Derivatives.
c 2009. Miguel A. Arcones. All rights reserved.
Extract from:
Arcones Manual for the SOA Exam FM/CAS Exam 2,
Financial Mathematics. Fall 2009 Edition,
available at http://www.actexmadriver.com/
c 2009. Miguel A. Arcones. All rights reserved. Manual for SOA Exam FM/CAS Exam 2.
2/52
Chapter 7. Derivatives markets. Section 7.1. Derivatives.
Risk sharing

A risk is a contingent nancial loss. Changes in commodity


prices, currency exchange rates and interest rates are potential
risks for a business. A farmer faces the possible fall of the
price of his/her crop. Surging oil prices can wipe out airlines
prots. Manufacturing companies face high rising prices of
commodities. These changes in prices could hurt the viability
of a business.

Many of the risks faced by business are diversiable. A risk is


diversiable if it is unrelated to another risk. Markets permit
diversiable risks to be widely shared.

Risk is nondiversiable when it does vanish when spread


across many investors.

A way to do risk sharing for companies is to do contracts to


avoid risks.
c 2009. Miguel A. Arcones. All rights reserved. Manual for SOA Exam FM/CAS Exam 2.
3/52
Chapter 7. Derivatives markets. Section 7.1. Derivatives.
Risk sharing

A risk is a contingent nancial loss. Changes in commodity


prices, currency exchange rates and interest rates are potential
risks for a business. A farmer faces the possible fall of the
price of his/her crop. Surging oil prices can wipe out airlines
prots. Manufacturing companies face high rising prices of
commodities. These changes in prices could hurt the viability
of a business.

Many of the risks faced by business are diversiable. A risk is


diversiable if it is unrelated to another risk. Markets permit
diversiable risks to be widely shared.

Risk is nondiversiable when it does vanish when spread


across many investors.

A way to do risk sharing for companies is to do contracts to


avoid risks.
c 2009. Miguel A. Arcones. All rights reserved. Manual for SOA Exam FM/CAS Exam 2.
4/52
Chapter 7. Derivatives markets. Section 7.1. Derivatives.
Risk sharing

A risk is a contingent nancial loss. Changes in commodity


prices, currency exchange rates and interest rates are potential
risks for a business. A farmer faces the possible fall of the
price of his/her crop. Surging oil prices can wipe out airlines
prots. Manufacturing companies face high rising prices of
commodities. These changes in prices could hurt the viability
of a business.

Many of the risks faced by business are diversiable. A risk is


diversiable if it is unrelated to another risk. Markets permit
diversiable risks to be widely shared.

Risk is nondiversiable when it does vanish when spread


across many investors.

A way to do risk sharing for companies is to do contracts to


avoid risks.
c 2009. Miguel A. Arcones. All rights reserved. Manual for SOA Exam FM/CAS Exam 2.
5/52
Chapter 7. Derivatives markets. Section 7.1. Derivatives.
Risk sharing

A risk is a contingent nancial loss. Changes in commodity


prices, currency exchange rates and interest rates are potential
risks for a business. A farmer faces the possible fall of the
price of his/her crop. Surging oil prices can wipe out airlines
prots. Manufacturing companies face high rising prices of
commodities. These changes in prices could hurt the viability
of a business.

Many of the risks faced by business are diversiable. A risk is


diversiable if it is unrelated to another risk. Markets permit
diversiable risks to be widely shared.

Risk is nondiversiable when it does vanish when spread


across many investors.

A way to do risk sharing for companies is to do contracts to


avoid risks.
c 2009. Miguel A. Arcones. All rights reserved. Manual for SOA Exam FM/CAS Exam 2.
6/52
Chapter 7. Derivatives markets. Section 7.1. Derivatives.
Derivatives
Denition 1
A derivative is a contract which species the right or obligation to
receive or deliver certain asset for a certain price. The value of a
derivative contract depends on the value of another asset.
They are several possible reasons to enter into a derivative market:

Risk management. Parties enter derivatives to avoid risks.

Speculation. Parties enter derivatives to make money. A


marketmaker enters into derivatives to make money.

Reduce transaction costs. Derivatives can be used to reduce


commodity costs, borrowing costs, etc.

Arbitrage. When derivatives are miss priced, investors can


make a prot.

Regulatory arbitrage. Sometimes business enter into


derivatives to get around regulatory limitations, accounting
regulations and taxes.
c 2009. Miguel A. Arcones. All rights reserved. Manual for SOA Exam FM/CAS Exam 2.
7/52
Chapter 7. Derivatives markets. Section 7.1. Derivatives.
Derivatives
Denition 1
A derivative is a contract which species the right or obligation to
receive or deliver certain asset for a certain price. The value of a
derivative contract depends on the value of another asset.
They are several possible reasons to enter into a derivative market:

Risk management. Parties enter derivatives to avoid risks.

Speculation. Parties enter derivatives to make money. A


marketmaker enters into derivatives to make money.

Reduce transaction costs. Derivatives can be used to reduce


commodity costs, borrowing costs, etc.

Arbitrage. When derivatives are miss priced, investors can


make a prot.

Regulatory arbitrage. Sometimes business enter into


derivatives to get around regulatory limitations, accounting
regulations and taxes.
c 2009. Miguel A. Arcones. All rights reserved. Manual for SOA Exam FM/CAS Exam 2.
8/52
Chapter 7. Derivatives markets. Section 7.1. Derivatives.
Derivatives
Denition 1
A derivative is a contract which species the right or obligation to
receive or deliver certain asset for a certain price. The value of a
derivative contract depends on the value of another asset.
They are several possible reasons to enter into a derivative market:

Risk management. Parties enter derivatives to avoid risks.

Speculation. Parties enter derivatives to make money. A


marketmaker enters into derivatives to make money.

Reduce transaction costs. Derivatives can be used to reduce


commodity costs, borrowing costs, etc.

Arbitrage. When derivatives are miss priced, investors can


make a prot.

Regulatory arbitrage. Sometimes business enter into


derivatives to get around regulatory limitations, accounting
regulations and taxes.
c 2009. Miguel A. Arcones. All rights reserved. Manual for SOA Exam FM/CAS Exam 2.
9/52
Chapter 7. Derivatives markets. Section 7.1. Derivatives.
Derivatives
Denition 1
A derivative is a contract which species the right or obligation to
receive or deliver certain asset for a certain price. The value of a
derivative contract depends on the value of another asset.
They are several possible reasons to enter into a derivative market:

Risk management. Parties enter derivatives to avoid risks.

Speculation. Parties enter derivatives to make money. A


marketmaker enters into derivatives to make money.

Reduce transaction costs. Derivatives can be used to reduce


commodity costs, borrowing costs, etc.

Arbitrage. When derivatives are miss priced, investors can


make a prot.

Regulatory arbitrage. Sometimes business enter into


derivatives to get around regulatory limitations, accounting
regulations and taxes.
c 2009. Miguel A. Arcones. All rights reserved. Manual for SOA Exam FM/CAS Exam 2.
10/52
Chapter 7. Derivatives markets. Section 7.1. Derivatives.
Derivatives
Denition 1
A derivative is a contract which species the right or obligation to
receive or deliver certain asset for a certain price. The value of a
derivative contract depends on the value of another asset.
They are several possible reasons to enter into a derivative market:

Risk management. Parties enter derivatives to avoid risks.

Speculation. Parties enter derivatives to make money. A


marketmaker enters into derivatives to make money.

Reduce transaction costs. Derivatives can be used to reduce


commodity costs, borrowing costs, etc.

Arbitrage. When derivatives are miss priced, investors can


make a prot.

Regulatory arbitrage. Sometimes business enter into


derivatives to get around regulatory limitations, accounting
regulations and taxes.
c 2009. Miguel A. Arcones. All rights reserved. Manual for SOA Exam FM/CAS Exam 2.
11/52
Chapter 7. Derivatives markets. Section 7.1. Derivatives.
Derivatives
Denition 1
A derivative is a contract which species the right or obligation to
receive or deliver certain asset for a certain price. The value of a
derivative contract depends on the value of another asset.
They are several possible reasons to enter into a derivative market:

Risk management. Parties enter derivatives to avoid risks.

Speculation. Parties enter derivatives to make money. A


marketmaker enters into derivatives to make money.

Reduce transaction costs. Derivatives can be used to reduce


commodity costs, borrowing costs, etc.

Arbitrage. When derivatives are miss priced, investors can


make a prot.

Regulatory arbitrage. Sometimes business enter into


derivatives to get around regulatory limitations, accounting
regulations and taxes.
c 2009. Miguel A. Arcones. All rights reserved. Manual for SOA Exam FM/CAS Exam 2.
12/52
Chapter 7. Derivatives markets. Section 7.1. Derivatives.
Derivatives
Denition 1
A derivative is a contract which species the right or obligation to
receive or deliver certain asset for a certain price. The value of a
derivative contract depends on the value of another asset.
They are several possible reasons to enter into a derivative market:

Risk management. Parties enter derivatives to avoid risks.

Speculation. Parties enter derivatives to make money. A


marketmaker enters into derivatives to make money.

Reduce transaction costs. Derivatives can be used to reduce


commodity costs, borrowing costs, etc.

Arbitrage. When derivatives are miss priced, investors can


make a prot.

Regulatory arbitrage. Sometimes business enter into


derivatives to get around regulatory limitations, accounting
regulations and taxes.
c 2009. Miguel A. Arcones. All rights reserved. Manual for SOA Exam FM/CAS Exam 2.
13/52
Chapter 7. Derivatives markets. Section 7.1. Derivatives.
Example 1
Suppose that a farmer grows wheat and a baker makes bread using
wheat and other ingredients. If the price of the wheat decreases,
the farmer loses money. If the price of the wheat increases, the
baker loses money. In order to avoid possible nancial losses which
may jeopardy their businesss protability, the farmer and the baker
can agree to sell/buy wheat one year from now at a certain price.
The contract they enter is a derivative. It is a winwin contract for
both of them. The two risks are diversiable.
Usually, the contract is not made directly between them. A
marketmaker or scalper makes a contract with the farmer and
another with the baker. The farmer and the baker enter into this
contract to do hedging.
c 2009. Miguel A. Arcones. All rights reserved. Manual for SOA Exam FM/CAS Exam 2.
14/52
Chapter 7. Derivatives markets. Section 7.1. Derivatives.
Example 1
Suppose that a farmer grows wheat and a baker makes bread using
wheat and other ingredients. If the price of the wheat decreases,
the farmer loses money. If the price of the wheat increases, the
baker loses money. In order to avoid possible nancial losses which
may jeopardy their businesss protability, the farmer and the baker
can agree to sell/buy wheat one year from now at a certain price.
The contract they enter is a derivative. It is a winwin contract for
both of them. The two risks are diversiable.
Usually, the contract is not made directly between them. A
marketmaker or scalper makes a contract with the farmer and
another with the baker. The farmer and the baker enter into this
contract to do hedging.
c 2009. Miguel A. Arcones. All rights reserved. Manual for SOA Exam FM/CAS Exam 2.
15/52
Chapter 7. Derivatives markets. Section 7.1. Derivatives.
Derivatives in Practice.

Derivatives are often traded on commodities, stock, stock


indexes, currency exchange rates and interest rates. Common
commodities in derivatives are agricultural (corn, soybeans,
wheat, live cattle, cattle feeder, hogs lean, sugar, coee,
orange juice), metals (gold, silver, copper, lead, aluminum,
platinum) and energy (crude oil, ethanol, natural gas,
gasoline).

Derivative contracts for agricultural commodities have been


traded in the U.S. for more than 100 years. The biggest
markets in derivatives are the Chicago Board for Trade, the
Chicago Mercantile Exchange, the New York Mercantile
Exchange, and the Eurex (Frankfurt, Germany).

The market in derivatives is regulated by the (SEC) Securities


and Exchange Commission and the (CFTC) Commodity
Futures Trading Commission.
c 2009. Miguel A. Arcones. All rights reserved. Manual for SOA Exam FM/CAS Exam 2.
16/52
Chapter 7. Derivatives markets. Section 7.1. Derivatives.
Derivatives in Practice.

Derivatives are often traded on commodities, stock, stock


indexes, currency exchange rates and interest rates. Common
commodities in derivatives are agricultural (corn, soybeans,
wheat, live cattle, cattle feeder, hogs lean, sugar, coee,
orange juice), metals (gold, silver, copper, lead, aluminum,
platinum) and energy (crude oil, ethanol, natural gas,
gasoline).

Derivative contracts for agricultural commodities have been


traded in the U.S. for more than 100 years. The biggest
markets in derivatives are the Chicago Board for Trade, the
Chicago Mercantile Exchange, the New York Mercantile
Exchange, and the Eurex (Frankfurt, Germany).

The market in derivatives is regulated by the (SEC) Securities


and Exchange Commission and the (CFTC) Commodity
Futures Trading Commission.
c 2009. Miguel A. Arcones. All rights reserved. Manual for SOA Exam FM/CAS Exam 2.
17/52
Chapter 7. Derivatives markets. Section 7.1. Derivatives.
Derivatives in Practice.

Derivatives are often traded on commodities, stock, stock


indexes, currency exchange rates and interest rates. Common
commodities in derivatives are agricultural (corn, soybeans,
wheat, live cattle, cattle feeder, hogs lean, sugar, coee,
orange juice), metals (gold, silver, copper, lead, aluminum,
platinum) and energy (crude oil, ethanol, natural gas,
gasoline).

Derivative contracts for agricultural commodities have been


traded in the U.S. for more than 100 years. The biggest
markets in derivatives are the Chicago Board for Trade, the
Chicago Mercantile Exchange, the New York Mercantile
Exchange, and the Eurex (Frankfurt, Germany).

The market in derivatives is regulated by the (SEC) Securities


and Exchange Commission and the (CFTC) Commodity
Futures Trading Commission.
c 2009. Miguel A. Arcones. All rights reserved. Manual for SOA Exam FM/CAS Exam 2.
18/52
Chapter 7. Derivatives markets. Section 7.1. Derivatives.
Buying an asset

(Scalpers) Marketmakers buy/sell stock and derivatives


making transactions possible. Usually, scalpers are nancial
institutions.

In order to make a living, scalpers buy low and sell high.

The price at which the scalper buys is called the bid price.
The bid price of an asset is the price at which a scalper takes
bids for an asset (a bid is an oer).

The price at which the scalper sells is called the oer price or
ask price.

The bid price is lower than the oer price. The dierence
between the bid price and the oer price is called the bidask
spread.

bidask percentage spread is the bidask spread divided


over the ask price. Scalpers also ask for commissions.
c 2009. Miguel A. Arcones. All rights reserved. Manual for SOA Exam FM/CAS Exam 2.
19/52
Chapter 7. Derivatives markets. Section 7.1. Derivatives.
Buying an asset

(Scalpers) Marketmakers buy/sell stock and derivatives


making transactions possible. Usually, scalpers are nancial
institutions.

In order to make a living, scalpers buy low and sell high.

The price at which the scalper buys is called the bid price.
The bid price of an asset is the price at which a scalper takes
bids for an asset (a bid is an oer).

The price at which the scalper sells is called the oer price or
ask price.

The bid price is lower than the oer price. The dierence
between the bid price and the oer price is called the bidask
spread.

bidask percentage spread is the bidask spread divided


over the ask price. Scalpers also ask for commissions.
c 2009. Miguel A. Arcones. All rights reserved. Manual for SOA Exam FM/CAS Exam 2.
20/52
Chapter 7. Derivatives markets. Section 7.1. Derivatives.
Buying an asset

(Scalpers) Marketmakers buy/sell stock and derivatives


making transactions possible. Usually, scalpers are nancial
institutions.

In order to make a living, scalpers buy low and sell high.

The price at which the scalper buys is called the bid price.
The bid price of an asset is the price at which a scalper takes
bids for an asset (a bid is an oer).

The price at which the scalper sells is called the oer price or
ask price.

The bid price is lower than the oer price. The dierence
between the bid price and the oer price is called the bidask
spread.

bidask percentage spread is the bidask spread divided


over the ask price. Scalpers also ask for commissions.
c 2009. Miguel A. Arcones. All rights reserved. Manual for SOA Exam FM/CAS Exam 2.
21/52
Chapter 7. Derivatives markets. Section 7.1. Derivatives.
Buying an asset

(Scalpers) Marketmakers buy/sell stock and derivatives


making transactions possible. Usually, scalpers are nancial
institutions.

In order to make a living, scalpers buy low and sell high.

The price at which the scalper buys is called the bid price.
The bid price of an asset is the price at which a scalper takes
bids for an asset (a bid is an oer).

The price at which the scalper sells is called the oer price or
ask price.

The bid price is lower than the oer price. The dierence
between the bid price and the oer price is called the bidask
spread.

bidask percentage spread is the bidask spread divided


over the ask price. Scalpers also ask for commissions.
c 2009. Miguel A. Arcones. All rights reserved. Manual for SOA Exam FM/CAS Exam 2.
22/52
Chapter 7. Derivatives markets. Section 7.1. Derivatives.
Buying an asset

(Scalpers) Marketmakers buy/sell stock and derivatives


making transactions possible. Usually, scalpers are nancial
institutions.

In order to make a living, scalpers buy low and sell high.

The price at which the scalper buys is called the bid price.
The bid price of an asset is the price at which a scalper takes
bids for an asset (a bid is an oer).

The price at which the scalper sells is called the oer price or
ask price.

The bid price is lower than the oer price. The dierence
between the bid price and the oer price is called the bidask
spread.

bidask percentage spread is the bidask spread divided


over the ask price. Scalpers also ask for commissions.
c 2009. Miguel A. Arcones. All rights reserved. Manual for SOA Exam FM/CAS Exam 2.
23/52
Chapter 7. Derivatives markets. Section 7.1. Derivatives.
Buying an asset

(Scalpers) Marketmakers buy/sell stock and derivatives


making transactions possible. Usually, scalpers are nancial
institutions.

In order to make a living, scalpers buy low and sell high.

The price at which the scalper buys is called the bid price.
The bid price of an asset is the price at which a scalper takes
bids for an asset (a bid is an oer).

The price at which the scalper sells is called the oer price or
ask price.

The bid price is lower than the oer price. The dierence
between the bid price and the oer price is called the bidask
spread.

bidask percentage spread is the bidask spread divided


over the ask price. Scalpers also ask for commissions.
c 2009. Miguel A. Arcones. All rights reserved. Manual for SOA Exam FM/CAS Exam 2.
24/52
Chapter 7. Derivatives markets. Section 7.1. Derivatives.
bid price at which the scalper buys
ask or oer price price at which the scalper sells
The bid price is lower than the oer price.
c 2009. Miguel A. Arcones. All rights reserved. Manual for SOA Exam FM/CAS Exam 2.
25/52
Chapter 7. Derivatives markets. Section 7.1. Derivatives.
Example 2
An online currency exchange services bid rate for Japanese yens is
$0.00838 and its ask rate is $0.00847. Find the bidask percentage
spread.
Solution: The ask percentage spread is
0.008470.00838
0.00847
= 1.062574%.
c 2009. Miguel A. Arcones. All rights reserved. Manual for SOA Exam FM/CAS Exam 2.
26/52
Chapter 7. Derivatives markets. Section 7.1. Derivatives.
Example 2
An online currency exchange services bid rate for Japanese yens is
$0.00838 and its ask rate is $0.00847. Find the bidask percentage
spread.
Solution: The ask percentage spread is
0.008470.00838
0.00847
= 1.062574%.
c 2009. Miguel A. Arcones. All rights reserved. Manual for SOA Exam FM/CAS Exam 2.
27/52
Chapter 7. Derivatives markets. Section 7.1. Derivatives.
Example 3
$1 million face value six month T bill is traded by a government
security dealer who give the following annual nominal discount
yield convertible semiannually:
Bid Ask
4.96% 4.94%
(i) Find the price the dealer is asking for the T bill
(ii) Find the price the dealer is buying the T bill.
(iii) Find the bidask percentage spread.
Solution: (i) (1000000)(1 (0.0494/2)) = 975300
(ii) (1000000)(1 (0.0496/2)) = 975200.
Notice that the dealer sells the T bill for money than he buys it.
(iii) The bidask percentage spread is
975300975200
975300
= 0.01025325541%.
c 2009. Miguel A. Arcones. All rights reserved. Manual for SOA Exam FM/CAS Exam 2.
28/52
Chapter 7. Derivatives markets. Section 7.1. Derivatives.
Example 3
$1 million face value six month T bill is traded by a government
security dealer who give the following annual nominal discount
yield convertible semiannually:
Bid Ask
4.96% 4.94%
(i) Find the price the dealer is asking for the T bill
(ii) Find the price the dealer is buying the T bill.
(iii) Find the bidask percentage spread.
Solution: (i) (1000000)(1 (0.0494/2)) = 975300
(ii) (1000000)(1 (0.0496/2)) = 975200.
Notice that the dealer sells the T bill for money than he buys it.
(iii) The bidask percentage spread is
975300975200
975300
= 0.01025325541%.
c 2009. Miguel A. Arcones. All rights reserved. Manual for SOA Exam FM/CAS Exam 2.
29/52
Chapter 7. Derivatives markets. Section 7.1. Derivatives.
Example 3
$1 million face value six month T bill is traded by a government
security dealer who give the following annual nominal discount
yield convertible semiannually:
Bid Ask
4.96% 4.94%
(i) Find the price the dealer is asking for the T bill
(ii) Find the price the dealer is buying the T bill.
(iii) Find the bidask percentage spread.
Solution: (i) (1000000)(1 (0.0494/2)) = 975300
(ii) (1000000)(1 (0.0496/2)) = 975200.
Notice that the dealer sells the T bill for money than he buys it.
(iii) The bidask percentage spread is
975300975200
975300
= 0.01025325541%.
c 2009. Miguel A. Arcones. All rights reserved. Manual for SOA Exam FM/CAS Exam 2.
30/52
Chapter 7. Derivatives markets. Section 7.1. Derivatives.
Example 3
$1 million face value six month T bill is traded by a government
security dealer who give the following annual nominal discount
yield convertible semiannually:
Bid Ask
4.96% 4.94%
(i) Find the price the dealer is asking for the T bill
(ii) Find the price the dealer is buying the T bill.
(iii) Find the bidask percentage spread.
Solution: (i) (1000000)(1 (0.0494/2)) = 975300
(ii) (1000000)(1 (0.0496/2)) = 975200.
Notice that the dealer sells the T bill for money than he buys it.
(iii) The bidask percentage spread is
975300975200
975300
= 0.01025325541%.
c 2009. Miguel A. Arcones. All rights reserved. Manual for SOA Exam FM/CAS Exam 2.
31/52
Chapter 7. Derivatives markets. Section 7.1. Derivatives.
Example 3
$1 million face value six month T bill is traded by a government
security dealer who give the following annual nominal discount
yield convertible semiannually:
Bid Ask
4.96% 4.94%
(i) Find the price the dealer is asking for the T bill
(ii) Find the price the dealer is buying the T bill.
(iii) Find the bidask percentage spread.
Solution: (i) (1000000)(1 (0.0494/2)) = 975300
(ii) (1000000)(1 (0.0496/2)) = 975200.
Notice that the dealer sells the T bill for money than he buys it.
(iii) The bidask percentage spread is
975300975200
975300
= 0.01025325541%.
c 2009. Miguel A. Arcones. All rights reserved. Manual for SOA Exam FM/CAS Exam 2.
32/52
Chapter 7. Derivatives markets. Section 7.1. Derivatives.
Long and short positions

When some one owns an asset, we say to this person has a


long position in this asset. Later, he may sell the asset and
receive cash. Buying an asset means to make an investment.
Buying an asset is like lending money.

When some one needs to buy an asset in the future, it is said


that this person has a short position in the asset.
c 2009. Miguel A. Arcones. All rights reserved. Manual for SOA Exam FM/CAS Exam 2.
33/52
Chapter 7. Derivatives markets. Section 7.1. Derivatives.
Long and short positions

When some one owns an asset, we say to this person has a


long position in this asset. Later, he may sell the asset and
receive cash. Buying an asset means to make an investment.
Buying an asset is like lending money.

When some one needs to buy an asset in the future, it is said


that this person has a short position in the asset.
c 2009. Miguel A. Arcones. All rights reserved. Manual for SOA Exam FM/CAS Exam 2.
34/52
Chapter 7. Derivatives markets. Section 7.1. Derivatives.
Short sale
A short sale of an asset entails borrowing an asset and then
immediately selling the asset receiving cash. Later, the short seller
must buy back the asset paying cash for it and return it to the
lender. The act of buying the replacement asset and return it to
the lender is called to close or cover the short position.
believe derivative desired outcome
price will increase purchase buy low now and sell high later
price will decrease short sale sell high now and buy low later
Table: An investor makes money buying low and selling high.
c 2009. Miguel A. Arcones. All rights reserved. Manual for SOA Exam FM/CAS Exam 2.
35/52
Chapter 7. Derivatives markets. Section 7.1. Derivatives.
Short sale
A short sale of an asset entails borrowing an asset and then
immediately selling the asset receiving cash. Later, the short seller
must buy back the asset paying cash for it and return it to the
lender. The act of buying the replacement asset and return it to
the lender is called to close or cover the short position.
believe derivative desired outcome
price will increase purchase buy low now and sell high later
price will decrease short sale sell high now and buy low later
Table: An investor makes money buying low and selling high.
c 2009. Miguel A. Arcones. All rights reserved. Manual for SOA Exam FM/CAS Exam 2.
36/52
Chapter 7. Derivatives markets. Section 7.1. Derivatives.
There are three main reasons to short sell:

Speculation. An investor enters a short sale because he


believes that the price of the stock will drop and desires to
prot from this price movement.

Financing. A short sale is a way to borrow money.

Hedging. A short sale can be undertaken to oset the risk of


owning an asset.
c 2009. Miguel A. Arcones. All rights reserved. Manual for SOA Exam FM/CAS Exam 2.
37/52
Chapter 7. Derivatives markets. Section 7.1. Derivatives.
There are three main reasons to short sell:

Speculation. An investor enters a short sale because he


believes that the price of the stock will drop and desires to
prot from this price movement.

Financing. A short sale is a way to borrow money.

Hedging. A short sale can be undertaken to oset the risk of


owning an asset.
c 2009. Miguel A. Arcones. All rights reserved. Manual for SOA Exam FM/CAS Exam 2.
38/52
Chapter 7. Derivatives markets. Section 7.1. Derivatives.
There are three main reasons to short sell:

Speculation. An investor enters a short sale because he


believes that the price of the stock will drop and desires to
prot from this price movement.

Financing. A short sale is a way to borrow money.

Hedging. A short sale can be undertaken to oset the risk of


owning an asset.
c 2009. Miguel A. Arcones. All rights reserved. Manual for SOA Exam FM/CAS Exam 2.
39/52
Chapter 7. Derivatives markets. Section 7.1. Derivatives.
There are three main reasons to short sell:

Speculation. An investor enters a short sale because he


believes that the price of the stock will drop and desires to
prot from this price movement.

Financing. A short sale is a way to borrow money.

Hedging. A short sale can be undertaken to oset the risk of


owning an asset.
c 2009. Miguel A. Arcones. All rights reserved. Manual for SOA Exam FM/CAS Exam 2.
40/52
Chapter 7. Derivatives markets. Section 7.1. Derivatives.
In order to a short sale to take place several conditions must be
taken into account:

Availability of a lender of stock. A lender must interested in


making some money and losing temporarily his voting rights
on the company issuing the stock.

Credit risk of the short seller. The short seller make be


required to setup a bank account with a deposit as collateral.

Scarcity of shares.
If the stock pays dividends, the short seller must return the paid
dividend payments to the stock lender.
c 2009. Miguel A. Arcones. All rights reserved. Manual for SOA Exam FM/CAS Exam 2.
41/52
Chapter 7. Derivatives markets. Section 7.1. Derivatives.
In order to a short sale to take place several conditions must be
taken into account:

Availability of a lender of stock. A lender must interested in


making some money and losing temporarily his voting rights
on the company issuing the stock.

Credit risk of the short seller. The short seller make be


required to setup a bank account with a deposit as collateral.

Scarcity of shares.
If the stock pays dividends, the short seller must return the paid
dividend payments to the stock lender.
c 2009. Miguel A. Arcones. All rights reserved. Manual for SOA Exam FM/CAS Exam 2.
42/52
Chapter 7. Derivatives markets. Section 7.1. Derivatives.
In order to a short sale to take place several conditions must be
taken into account:

Availability of a lender of stock. A lender must interested in


making some money and losing temporarily his voting rights
on the company issuing the stock.

Credit risk of the short seller. The short seller make be


required to setup a bank account with a deposit as collateral.

Scarcity of shares.
If the stock pays dividends, the short seller must return the paid
dividend payments to the stock lender.
c 2009. Miguel A. Arcones. All rights reserved. Manual for SOA Exam FM/CAS Exam 2.
43/52
Chapter 7. Derivatives markets. Section 7.1. Derivatives.
In order to a short sale to take place several conditions must be
taken into account:

Availability of a lender of stock. A lender must interested in


making some money and losing temporarily his voting rights
on the company issuing the stock.

Credit risk of the short seller. The short seller make be


required to setup a bank account with a deposit as collateral.

Scarcity of shares.
If the stock pays dividends, the short seller must return the paid
dividend payments to the stock lender.
c 2009. Miguel A. Arcones. All rights reserved. Manual for SOA Exam FM/CAS Exam 2.
44/52
Chapter 7. Derivatives markets. Section 7.1. Derivatives.
Example 4
Mary short sells 200 shares of XYZ stock which has a bid price of
$18.12 and ask price of $18.56. Her broker charges her a 0.5%
commission to take on the short sale. Mary covers her position 6
months later when the bid price is $15.74 and the ask price is
$15.93. The commission to close the short sale is $15. XYZ stock
did not pay any dividends in those six months. How much does
Mary earn in this short sale?
Solution: Mary short sells the stock for
(200)(18.12)(1 0.005) = $3605.88.
Mary covers her position for
(200)(15.93) + 15 = $3201.
Mary earns 3605.88 3201 = $404.88.
c 2009. Miguel A. Arcones. All rights reserved. Manual for SOA Exam FM/CAS Exam 2.
45/52
Chapter 7. Derivatives markets. Section 7.1. Derivatives.
Example 4
Mary short sells 200 shares of XYZ stock which has a bid price of
$18.12 and ask price of $18.56. Her broker charges her a 0.5%
commission to take on the short sale. Mary covers her position 6
months later when the bid price is $15.74 and the ask price is
$15.93. The commission to close the short sale is $15. XYZ stock
did not pay any dividends in those six months. How much does
Mary earn in this short sale?
Solution: Mary short sells the stock for
(200)(18.12)(1 0.005) = $3605.88.
Mary covers her position for
(200)(15.93) + 15 = $3201.
Mary earns 3605.88 3201 = $404.88.
c 2009. Miguel A. Arcones. All rights reserved. Manual for SOA Exam FM/CAS Exam 2.
46/52
Chapter 7. Derivatives markets. Section 7.1. Derivatives.

Often, short sellers are required to make a deposit as


collateral into an account with the lender. This account is
called the margin account. This deposit is called the margin
requirement. Usually, the margin requirement is a percentage
of the current price of the stock. The margin requirement
could be bigger than the current asset price. If this is so, the
excess of the margin over the current asset price is called
haircut.

This margin account generates an interest for the investor.


Demand on short sales is a factor to determine the margin
account interest rate. The margin interest rate is called
the repo rate for bonds and the short rebate for stock.

When borrowing, the lender can require payment of certain


benets lost by lending the asset. This payment requirement
is called the lease rate of the asset. Usually, the lease rate of
a stock is the payment of the dividends obtained while the
stock was shorted. Usually, the lease rate of a bond is the
payment of the coupons obtained while the bond was shorted.
c 2009. Miguel A. Arcones. All rights reserved. Manual for SOA Exam FM/CAS Exam 2.
47/52
Chapter 7. Derivatives markets. Section 7.1. Derivatives.

Often, short sellers are required to make a deposit as


collateral into an account with the lender. This account is
called the margin account. This deposit is called the margin
requirement. Usually, the margin requirement is a percentage
of the current price of the stock. The margin requirement
could be bigger than the current asset price. If this is so, the
excess of the margin over the current asset price is called
haircut.

This margin account generates an interest for the investor.


Demand on short sales is a factor to determine the margin
account interest rate. The margin interest rate is called
the repo rate for bonds and the short rebate for stock.

When borrowing, the lender can require payment of certain


benets lost by lending the asset. This payment requirement
is called the lease rate of the asset. Usually, the lease rate of
a stock is the payment of the dividends obtained while the
stock was shorted. Usually, the lease rate of a bond is the
payment of the coupons obtained while the bond was shorted.
c 2009. Miguel A. Arcones. All rights reserved. Manual for SOA Exam FM/CAS Exam 2.
48/52
Chapter 7. Derivatives markets. Section 7.1. Derivatives.

Often, short sellers are required to make a deposit as


collateral into an account with the lender. This account is
called the margin account. This deposit is called the margin
requirement. Usually, the margin requirement is a percentage
of the current price of the stock. The margin requirement
could be bigger than the current asset price. If this is so, the
excess of the margin over the current asset price is called
haircut.

This margin account generates an interest for the investor.


Demand on short sales is a factor to determine the margin
account interest rate. The margin interest rate is called
the repo rate for bonds and the short rebate for stock.

When borrowing, the lender can require payment of certain


benets lost by lending the asset. This payment requirement
is called the lease rate of the asset. Usually, the lease rate of
a stock is the payment of the dividends obtained while the
stock was shorted. Usually, the lease rate of a bond is the
payment of the coupons obtained while the bond was shorted.
c 2009. Miguel A. Arcones. All rights reserved. Manual for SOA Exam FM/CAS Exam 2.
49/52
Chapter 7. Derivatives markets. Section 7.1. Derivatives.
Sometimes short sales are subject to a margin requirement (or
deposit). The investor has to setup an account with a percentage
of the current price of the stock. This margin account generates
an interest for the investor. If the stock which the investor borrows
pays a dividend, the investor must pay the dividend to the
brokerage rm making the loan.
We have the following variables in a short sale of a stock:

P = prot on sale=price soldprice bought.

M = margin requirement= deposit on the short sale.

D = dividend paid by the short seller to the securitys owner.

j = rate of interest earned in the margin account.

I = Mj interest earned by the short seller on the margin


deposit.

i = yield rate on the short sale.


c 2009. Miguel A. Arcones. All rights reserved. Manual for SOA Exam FM/CAS Exam 2.
50/52
Chapter 7. Derivatives markets. Section 7.1. Derivatives.
We have that the net prot is
Net prot = gain on short sale+interest on margindividend on stock = P+I D.
Hence, the yield rate earner in a short sale is
i =
net prot
margin
=
P + I D
M
.
c 2009. Miguel A. Arcones. All rights reserved. Manual for SOA Exam FM/CAS Exam 2.
51/52
Chapter 7. Derivatives markets. Section 7.1. Derivatives.
Example 5
Jason sold short 1,000 shares of FinanTech at $75 a share on
January 2, 2006. Jason is required to hold a margin account with
his broker equal to 50% of the short securitys initial value. Jasons
margin account earns an annual eective interest rate of i . There
is a $0.25 per share dividend paid on December 31, 2006. On
January 2, 2007, Jason buys back stock to cover his position at a
price of $70 per share. Jasons annual eective yield in this
investment is 17.6667%. Find i .
Solution: We have that P = (75)(1000) (70)(10000) = 5000,
M = (75)(1000)(0.50) = 37500, I = 37500i and
D = (1000)(0.25) = 250. Jasons annual eective yield is
0.176667 =
P + I D
M
=
5000 + 37500i 250
37500
.
Hence, i =
(0.176667)(37500)5000+250
37500
= 5%.
c 2009. Miguel A. Arcones. All rights reserved. Manual for SOA Exam FM/CAS Exam 2.
52/52
Chapter 7. Derivatives markets. Section 7.1. Derivatives.
Example 5
Jason sold short 1,000 shares of FinanTech at $75 a share on
January 2, 2006. Jason is required to hold a margin account with
his broker equal to 50% of the short securitys initial value. Jasons
margin account earns an annual eective interest rate of i . There
is a $0.25 per share dividend paid on December 31, 2006. On
January 2, 2007, Jason buys back stock to cover his position at a
price of $70 per share. Jasons annual eective yield in this
investment is 17.6667%. Find i .
Solution: We have that P = (75)(1000) (70)(10000) = 5000,
M = (75)(1000)(0.50) = 37500, I = 37500i and
D = (1000)(0.25) = 250. Jasons annual eective yield is
0.176667 =
P + I D
M
=
5000 + 37500i 250
37500
.
Hence, i =
(0.176667)(37500)5000+250
37500
= 5%.
c 2009. Miguel A. Arcones. All rights reserved. Manual for SOA Exam FM/CAS Exam 2.
1/89
Chapter 7. Derivatives markets.
Manual for SOA Exam FM/CAS Exam 2.
Chapter 7. Derivatives markets.
Section 7.2. Forwards.
c 2009. Miguel A. Arcones. All rights reserved.
Extract from:
Arcones Manual for the SOA Exam FM/CAS Exam 2,
Financial Mathematics. Fall 2009 Edition,
available at http://www.actexmadriver.com/
c 2009. Miguel A. Arcones. All rights reserved. Manual for SOA Exam FM/CAS Exam 2.
2/89
Chapter 7. Derivatives markets. Section 7.2. Forwards.
Forwards
Denition 1
A forward is a contract between a buyer and seller in which they
agree upon the sale of an asset of a specied quality for a specied
price at a specied future date.
Forward contracts are privately negotiated and are not
standardized.
Common forwards are in commodities, currency exchange, stock
shares and stock indices.
c 2009. Miguel A. Arcones. All rights reserved. Manual for SOA Exam FM/CAS Exam 2.
3/89
Chapter 7. Derivatives markets. Section 7.2. Forwards.
A forward contract states the following:

forces the seller to sell and the buyer to buy.

spells out the quantity, quality and exact type of asset to be


sold.

states the delivery price and the time, date and place for the
transfer of ownership of the asset.

specify the time, date, place for payment.


Usually a forward contract has more terms.
Sometimes instead of the asset to be delivered, there exists a cash
settlement between the parties engaging in a forward contract.
Either physical settlement or cash settlement can be used to
settle a forward contract.
When entering in a forward contract, parties must check
counterparts for credit risk (sometimes using a collateral, bank
letters, real state guarantee, etc).
c 2009. Miguel A. Arcones. All rights reserved. Manual for SOA Exam FM/CAS Exam 2.
4/89
Chapter 7. Derivatives markets. Section 7.2. Forwards.
A forward contract states the following:

forces the seller to sell and the buyer to buy.

spells out the quantity, quality and exact type of asset to be


sold.

states the delivery price and the time, date and place for the
transfer of ownership of the asset.

specify the time, date, place for payment.


Usually a forward contract has more terms.
Sometimes instead of the asset to be delivered, there exists a cash
settlement between the parties engaging in a forward contract.
Either physical settlement or cash settlement can be used to
settle a forward contract.
When entering in a forward contract, parties must check
counterparts for credit risk (sometimes using a collateral, bank
letters, real state guarantee, etc).
c 2009. Miguel A. Arcones. All rights reserved. Manual for SOA Exam FM/CAS Exam 2.
5/89
Chapter 7. Derivatives markets. Section 7.2. Forwards.
A forward contract states the following:

forces the seller to sell and the buyer to buy.

spells out the quantity, quality and exact type of asset to be


sold.

states the delivery price and the time, date and place for the
transfer of ownership of the asset.

specify the time, date, place for payment.


Usually a forward contract has more terms.
Sometimes instead of the asset to be delivered, there exists a cash
settlement between the parties engaging in a forward contract.
Either physical settlement or cash settlement can be used to
settle a forward contract.
When entering in a forward contract, parties must check
counterparts for credit risk (sometimes using a collateral, bank
letters, real state guarantee, etc).
c 2009. Miguel A. Arcones. All rights reserved. Manual for SOA Exam FM/CAS Exam 2.
6/89
Chapter 7. Derivatives markets. Section 7.2. Forwards.
A forward contract states the following:

forces the seller to sell and the buyer to buy.

spells out the quantity, quality and exact type of asset to be


sold.

states the delivery price and the time, date and place for the
transfer of ownership of the asset.

specify the time, date, place for payment.


Usually a forward contract has more terms.
Sometimes instead of the asset to be delivered, there exists a cash
settlement between the parties engaging in a forward contract.
Either physical settlement or cash settlement can be used to
settle a forward contract.
When entering in a forward contract, parties must check
counterparts for credit risk (sometimes using a collateral, bank
letters, real state guarantee, etc).
c 2009. Miguel A. Arcones. All rights reserved. Manual for SOA Exam FM/CAS Exam 2.
7/89
Chapter 7. Derivatives markets. Section 7.2. Forwards.
A forward contract states the following:

forces the seller to sell and the buyer to buy.

spells out the quantity, quality and exact type of asset to be


sold.

states the delivery price and the time, date and place for the
transfer of ownership of the asset.

specify the time, date, place for payment.


Usually a forward contract has more terms.
Sometimes instead of the asset to be delivered, there exists a cash
settlement between the parties engaging in a forward contract.
Either physical settlement or cash settlement can be used to
settle a forward contract.
When entering in a forward contract, parties must check
counterparts for credit risk (sometimes using a collateral, bank
letters, real state guarantee, etc).
c 2009. Miguel A. Arcones. All rights reserved. Manual for SOA Exam FM/CAS Exam 2.
8/89
Chapter 7. Derivatives markets. Section 7.2. Forwards.
A forward contract states the following:

forces the seller to sell and the buyer to buy.

spells out the quantity, quality and exact type of asset to be


sold.

states the delivery price and the time, date and place for the
transfer of ownership of the asset.

specify the time, date, place for payment.


Usually a forward contract has more terms.
Sometimes instead of the asset to be delivered, there exists a cash
settlement between the parties engaging in a forward contract.
Either physical settlement or cash settlement can be used to
settle a forward contract.
When entering in a forward contract, parties must check
counterparts for credit risk (sometimes using a collateral, bank
letters, real state guarantee, etc).
c 2009. Miguel A. Arcones. All rights reserved. Manual for SOA Exam FM/CAS Exam 2.
9/89
Chapter 7. Derivatives markets. Section 7.2. Forwards.
A forward contract states the following:

forces the seller to sell and the buyer to buy.

spells out the quantity, quality and exact type of asset to be


sold.

states the delivery price and the time, date and place for the
transfer of ownership of the asset.

specify the time, date, place for payment.


Usually a forward contract has more terms.
Sometimes instead of the asset to be delivered, there exists a cash
settlement between the parties engaging in a forward contract.
Either physical settlement or cash settlement can be used to
settle a forward contract.
When entering in a forward contract, parties must check
counterparts for credit risk (sometimes using a collateral, bank
letters, real state guarantee, etc).
c 2009. Miguel A. Arcones. All rights reserved. Manual for SOA Exam FM/CAS Exam 2.
10/89
Chapter 7. Derivatives markets. Section 7.2. Forwards.
A forward contract states the following:

forces the seller to sell and the buyer to buy.

spells out the quantity, quality and exact type of asset to be


sold.

states the delivery price and the time, date and place for the
transfer of ownership of the asset.

specify the time, date, place for payment.


Usually a forward contract has more terms.
Sometimes instead of the asset to be delivered, there exists a cash
settlement between the parties engaging in a forward contract.
Either physical settlement or cash settlement can be used to
settle a forward contract.
When entering in a forward contract, parties must check
counterparts for credit risk (sometimes using a collateral, bank
letters, real state guarantee, etc).
c 2009. Miguel A. Arcones. All rights reserved. Manual for SOA Exam FM/CAS Exam 2.
11/89
Chapter 7. Derivatives markets. Section 7.2. Forwards.

The asset in which the forward contract is based is called the


underlier or underlying asset.

The nominal amount (also called notional amount) of a


forward contract is the quantity of the asset traded in the
forward contract.

The price of the asset in the forward contract is called the


forward price.

The time at which the contract settles is called the expiration


date.
For example, if a forward contract involves 10,000 barrels of oil to
be delivered in one year, oil is the underlying asset, 10000 barrels is
the notional amount and one year is the expiration date.
c 2009. Miguel A. Arcones. All rights reserved. Manual for SOA Exam FM/CAS Exam 2.
12/89
Chapter 7. Derivatives markets. Section 7.2. Forwards.

The asset in which the forward contract is based is called the


underlier or underlying asset.

The nominal amount (also called notional amount) of a


forward contract is the quantity of the asset traded in the
forward contract.

The price of the asset in the forward contract is called the


forward price.

The time at which the contract settles is called the expiration


date.
For example, if a forward contract involves 10,000 barrels of oil to
be delivered in one year, oil is the underlying asset, 10000 barrels is
the notional amount and one year is the expiration date.
c 2009. Miguel A. Arcones. All rights reserved. Manual for SOA Exam FM/CAS Exam 2.
13/89
Chapter 7. Derivatives markets. Section 7.2. Forwards.

The asset in which the forward contract is based is called the


underlier or underlying asset.

The nominal amount (also called notional amount) of a


forward contract is the quantity of the asset traded in the
forward contract.

The price of the asset in the forward contract is called the


forward price.

The time at which the contract settles is called the expiration


date.
For example, if a forward contract involves 10,000 barrels of oil to
be delivered in one year, oil is the underlying asset, 10000 barrels is
the notional amount and one year is the expiration date.
c 2009. Miguel A. Arcones. All rights reserved. Manual for SOA Exam FM/CAS Exam 2.
14/89
Chapter 7. Derivatives markets. Section 7.2. Forwards.

The asset in which the forward contract is based is called the


underlier or underlying asset.

The nominal amount (also called notional amount) of a


forward contract is the quantity of the asset traded in the
forward contract.

The price of the asset in the forward contract is called the


forward price.

The time at which the contract settles is called the expiration


date.
For example, if a forward contract involves 10,000 barrels of oil to
be delivered in one year, oil is the underlying asset, 10000 barrels is
the notional amount and one year is the expiration date.
c 2009. Miguel A. Arcones. All rights reserved. Manual for SOA Exam FM/CAS Exam 2.
15/89
Chapter 7. Derivatives markets. Section 7.2. Forwards.
The two main reasons why an investor might be interested in
forward contracts are: speculation and (hedging) reduce
investment risk.
c 2009. Miguel A. Arcones. All rights reserved. Manual for SOA Exam FM/CAS Exam 2.
16/89
Chapter 7. Derivatives markets. Section 7.2. Forwards.
Apart from commission, a forward contract requires no initial
payment. The current price of an asset is called its spot price.
Besides the spot price, to price a forward contract, several factors,
such as delivery cost and time of delivery must be taken into
account.
The dierence between the spot and the forward price is called the
forward premium or forward discount.
If the forward price is higher than the spot price, the asset is
forwarded at a premium. The premium is the forward price
minus the current spot price.
If the forward price is lower than the spot price, the asset is
forwarded at a discount. The discount is the current spot price
minus the forward price.
c 2009. Miguel A. Arcones. All rights reserved. Manual for SOA Exam FM/CAS Exam 2.
17/89
Chapter 7. Derivatives markets. Section 7.2. Forwards.
The buyer of the asset in a forward contract is called the long
forward. The long forward benets when prices rise.
The seller of the asset in a forward contract is called the short
forward. The short forward benets when prices decline.
c 2009. Miguel A. Arcones. All rights reserved. Manual for SOA Exam FM/CAS Exam 2.
18/89
Chapter 7. Derivatives markets. Section 7.2. Forwards.
We will denote by S
T
the spot price of an asset at time T. S
0
is
the current price of the asset. S
0
is a xed quantity. For T > 0,
S
T
is a random variable. We will denote by F
0,T
to the price at
time zero of a forward with expiration time T paid at time T.
The payo of a derivative is the value of this position at expiration.
The payo of a long forward contract is S
T
F
0,T
. Notice that
the bearer of a long forward contract buys an asset at time T with
value S
T
for F
0,T
. Assuming that there are no expenses setting the
forward contract, the prot for a long forward is S
T
F
0,T
.
The payo of a short forward is F
0,T
S
T
. The holder of a short
forward contract sells at time T an asset with value S
T
for F
0,T
.
Assuming that there are no expenses setting the forward contract,
the prot for a short forward contract is F
0,T
S
T
.
c 2009. Miguel A. Arcones. All rights reserved. Manual for SOA Exam FM/CAS Exam 2.
19/89
Chapter 7. Derivatives markets. Section 7.2. Forwards.
The prots of the long and the short in a forward contract are the
opposite of each other. The sum of their prots is zero. A forward
contract is a zerosum game.

The minimum long forwards prot F


0,T
, which is attained
when S
T
= 0.

The maximum long forwards prot is innity, which is


attained when S
T
= .

The minimum short forwards prot is , which is attained


when S
T
= .

The maximum short forwards prot is F


0,T
, which is attained
when S
T
= 0.
long forwards prot = S
T
F
0,T
short forwards prot = F
0,T
S
T
minimum prot maximum prot
long forward F
0,T

short forward F
0,T
c 2009. Miguel A. Arcones. All rights reserved. Manual for SOA Exam FM/CAS Exam 2.
20/89
Chapter 7. Derivatives markets. Section 7.2. Forwards.
Example 1
A gold miner enters a forward contract with a jeweler to sell him
200 ounces of gold in six months for $600 per ounce.
(i) Find the jewelers payo in the forward contract if the spot
price at expiration of a gold ounce is $590, $595, $600, $605,
$610. Graph the jewelers payo.
(ii) Find the gold miners payo in the forward contract if the spot
price at expiration of a gold ounce is $590, $595, $600, $605,
$610. Graph the gold miners payo.
c 2009. Miguel A. Arcones. All rights reserved. Manual for SOA Exam FM/CAS Exam 2.
21/89
Chapter 7. Derivatives markets. Section 7.2. Forwards.
Solution: (i) The jewelers payo is (200)(S
T
600). A table of
the jewelers payo is
S
T
590 595 600 605 610
Payo 2000 1000 0 1000 2000
The graph of the jewelers payo is given in Figure 1.
(ii) The gold miners payo is (200)(600 S
T
). A table of the
gold miners payo is
S
T
590 595 600 605 610
Payo 2000 1000 0 1000 2000
c 2009. Miguel A. Arcones. All rights reserved. Manual for SOA Exam FM/CAS Exam 2.
22/89
Chapter 7. Derivatives markets. Section 7.2. Forwards.
Figure 1: Example 1. (Long forward) Jewelers payo
c 2009. Miguel A. Arcones. All rights reserved. Manual for SOA Exam FM/CAS Exam 2.
23/89
Chapter 7. Derivatives markets. Section 7.2. Forwards.
Figure 2: Example 1. (Short forward) Gold miners payo
Notice how gures 1 and 2 are the opposite of each other. The
gold miners payo equals minus the jewelers payo.
c 2009. Miguel A. Arcones. All rights reserved. Manual for SOA Exam FM/CAS Exam 2.
24/89
Chapter 7. Derivatives markets. Section 7.2. Forwards.
Usually, a commissions has to be paid to enter a forward contract.
Suppose that the long forward has to paid C
L
to the marketmaker
at negotiation time to enter into the forward contract. Then, the
prot for a long forward is
Prot of a long forward = S
T
F
0,T
C
L
(1 + i )
T
,
where i is the annual eective rate of interest.
If the short forward has to paid C
S
at negotiation time to the
marketmaker to enter the forward contract, then prot for a short
forward is
Prot of a short forward = F
0,T
S
T
C
S
(1 + i )
T
.
c 2009. Miguel A. Arcones. All rights reserved. Manual for SOA Exam FM/CAS Exam 2.
25/89
Chapter 7. Derivatives markets. Section 7.2. Forwards.
Sometimes, instead of using the annual eective rate of interest,
we will use the annual interest rate compounded continuously.
This is another name for the force of interest. This rate is also
called the annual continuous interest rate. If r is the annual
continuously compounded interest rate, then the future value at
time T of a payment of P made at time zero is Pe
rT
.
c 2009. Miguel A. Arcones. All rights reserved. Manual for SOA Exam FM/CAS Exam 2.
26/89
Chapter 7. Derivatives markets. Section 7.2. Forwards.
Alternative ways to buy an asset.
Suppose that you want to buy an asset. Suppose that the buyers
payment can be made at either time zero or time T. Suppose that
the transfer of ownership of an asset can be made either at time
zero or at time T. There are four possible ways to buy an asset
(see Table 1):
Pay at time
Receive
the asset
at time
0 T
0 Outright purchase Fully leveraged purchase
T Prepaid forward contract Forward contract
Table 1: Alternative ways to buy an asset.
c 2009. Miguel A. Arcones. All rights reserved. Manual for SOA Exam FM/CAS Exam 2.
27/89
Chapter 7. Derivatives markets. Section 7.2. Forwards.
1. Outright purchase. Both the payment and the transfer of
ownership are made at time zero. The price paid per share is
the current spot price S
0
.
2. Fully leveraged purchase. The transfer of ownership is made
at time zero. The payment is made at time T. The payment
is S
0
e
rT
, where S
0
is the current spot price and r is the
riskfree continuously compounded annual interest rate.
3. Prepaid forward contract. A payment of F
P
0,T
is made at
time zero. The transfer of ownership is made at time T. The
payment F
P
0,T
is not necessarily the current spot price S
0
.
4. Forward contract. Both the payment and the transfer of
ownership happen at time T. The price of a forward contract
is denoted by F
0,T
. We have that F
0,T
= e
rT
F
P
0,T
, where r is
the riskfree annual interest rate continuously compounded.
c 2009. Miguel A. Arcones. All rights reserved. Manual for SOA Exam FM/CAS Exam 2.
28/89
Chapter 7. Derivatives markets. Section 7.2. Forwards.
Pricing a forward contract.
Whenever an asset is delivered and paid at time zero, the fair price
of the asset is its (spot) market price. The market of an outright
purchase is S
0
. Commissions and bidask spreads must be taken
into account.
The case of a fully leverage purchase, the price of a forward
contract is just the price of a loan of S
0
. The price of a fully
leveraged purchase is S
0
e
rT
, which is the price of a loan of S
0
taken at time zero and paid at time T.
c 2009. Miguel A. Arcones. All rights reserved. Manual for SOA Exam FM/CAS Exam 2.
29/89
Chapter 7. Derivatives markets. Section 7.2. Forwards.
We are interested in determining F
0,T
, the price of a forward
contract. Many dierent factors such as the cost of storing,
delivering, the convenience yield and the scarcity of the asset.
Some commodities like oil have high storage costs. The
convenience yield measures the cost of not having the asset, but a
forward contract on it. For example, if instead of having a forward
on gasoline, we have the physical asset, we may use it in case of
scarcity. In the case of stock paying dividends, an stock owner
receives dividend payments, and a long forward holder does not.
c 2009. Miguel A. Arcones. All rights reserved. Manual for SOA Exam FM/CAS Exam 2.
30/89
Chapter 7. Derivatives markets. Section 7.2. Forwards.
Pricing a prepaid forward contract.
To nd F
P
0,T
, we make three cases.
1. Price of prepaid forward contract if there are no
dividends. We consider an asset with no cost/benet in holding
the asset. This applies to the the price of a stock which does pay
any dividends. It is irrelevant whether the transfer of ownership
happens now or later
The no arbitrage price of a prepaid forward contract is F
P
0,T
= S
0
,
where S
0
is the price of an asset today.
c 2009. Miguel A. Arcones. All rights reserved. Manual for SOA Exam FM/CAS Exam 2.
31/89
Chapter 7. Derivatives markets. Section 7.2. Forwards.
Example 2
XYZ stock costs $55 per share. XYZ stock does not pay any
dividends. The riskfree interest rate continuously compounded
8%. Calculate the price of a prepaid forward contract that expires
30 months from today.
Solution: The prepaid forward price is F
P
0,T
= S
0
= 55.
c 2009. Miguel A. Arcones. All rights reserved. Manual for SOA Exam FM/CAS Exam 2.
32/89
Chapter 7. Derivatives markets. Section 7.2. Forwards.
Example 2
XYZ stock costs $55 per share. XYZ stock does not pay any
dividends. The riskfree interest rate continuously compounded
8%. Calculate the price of a prepaid forward contract that expires
30 months from today.
Solution: The prepaid forward price is F
P
0,T
= S
0
= 55.
c 2009. Miguel A. Arcones. All rights reserved. Manual for SOA Exam FM/CAS Exam 2.
33/89
Chapter 7. Derivatives markets. Section 7.2. Forwards.
2. Price of prepaid forward contract when there are discrete
dividends. Suppose that the stock is expected to make a dividend
payment of D
T
i
at the time t
i
, i = 1 . . . , n. A prepaid forward
contract will entitle you receive the stock at time T without
receiving the interim dividends. The prepaid forward price is
F
P
0,T
= S
0

i =1
D
t
i
e
rt
i
.
c 2009. Miguel A. Arcones. All rights reserved. Manual for SOA Exam FM/CAS Exam 2.
34/89
Chapter 7. Derivatives markets. Section 7.2. Forwards.
Example 3
XYZ stock cost $55 per share. It pays $2 in dividends every 3
months. The rst dividend is paid in 3 months. The riskfree
interest rate continuously compounded 8%. Calculate the price of
a prepaid forward contract that expires 18 months from today,
immediately after the dividend is paid.
Solution: The prepaid forward price is
F
P
0,T
= S
0

i =1
D
t
i
e
rt
i
= 55
6

j =1
2e
(0.08)j (1/4)
=55
6

j =1
2e
(0.02)j
= 55 2a
6

|e
0.02
1
= 43.80474631.
Recall that a
n

|i
=

n
j =1
(1 + i )
j
.
c 2009. Miguel A. Arcones. All rights reserved. Manual for SOA Exam FM/CAS Exam 2.
35/89
Chapter 7. Derivatives markets. Section 7.2. Forwards.
Example 3
XYZ stock cost $55 per share. It pays $2 in dividends every 3
months. The rst dividend is paid in 3 months. The riskfree
interest rate continuously compounded 8%. Calculate the price of
a prepaid forward contract that expires 18 months from today,
immediately after the dividend is paid.
Solution: The prepaid forward price is
F
P
0,T
= S
0

i =1
D
t
i
e
rt
i
= 55
6

j =1
2e
(0.08)j (1/4)
=55
6

j =1
2e
(0.02)j
= 55 2a
6

|e
0.02
1
= 43.80474631.
Recall that a
n

|i
=

n
j =1
(1 + i )
j
.
c 2009. Miguel A. Arcones. All rights reserved. Manual for SOA Exam FM/CAS Exam 2.
36/89
Chapter 7. Derivatives markets. Section 7.2. Forwards.
3. Price of prepaid forward contract when there are
continuous dividends. In the case of an index stock, dividends
are given almost daily. We may model the dividend payments as a
continuous ow. Let be the rate of dividends given per unit of
time. Suppose that dividends payments are reinvested into stock.
Let t
j
=
jT
n
, 1 j n. If A
j
is the amount of shares at time t
j
,
then A
j +1
= A
j
(1 +
T
n
). Hence, the total amount of shares
multiplies by 1 +
T
n
in each period. Hence, one share at time zero
grows to
_
1 +
T
n
_
n
at time T. Letting n , we get that one
share at time zero grows to e
T
shares at time T. With $K at
time 0, we can buy
K
S
0
shares in the market at time 0. These
shares grow to
K
S
0
e
T
at time T. With $K at time 0, we can buy
K
F
P
0,T
shares to be delivered at time T using a prepaid forward.
Hence, is there exists no arbitrage,
K
S
0
e
T
=
K
F
P
0,T
and
F
P
0,T
= S
0
e
T
.
c 2009. Miguel A. Arcones. All rights reserved. Manual for SOA Exam FM/CAS Exam 2.
37/89
Chapter 7. Derivatives markets. Section 7.2. Forwards.
Example 4
An investor is interested in buying XYZ stock. The current price of
stock is $45 per share. This stock pays dividends at an annual
continuous rate of 5%. Calculate the price of a prepaid forward
contract which expires in 18 months.
Solution: The price of the prepaid forward contract is
F
P
0,T
= S
0
e
T
= 45e
(0.05)(18/12)
= 41.74845688.
c 2009. Miguel A. Arcones. All rights reserved. Manual for SOA Exam FM/CAS Exam 2.
38/89
Chapter 7. Derivatives markets. Section 7.2. Forwards.
Example 4
An investor is interested in buying XYZ stock. The current price of
stock is $45 per share. This stock pays dividends at an annual
continuous rate of 5%. Calculate the price of a prepaid forward
contract which expires in 18 months.
Solution: The price of the prepaid forward contract is
F
P
0,T
= S
0
e
T
= 45e
(0.05)(18/12)
= 41.74845688.
c 2009. Miguel A. Arcones. All rights reserved. Manual for SOA Exam FM/CAS Exam 2.
39/89
Chapter 7. Derivatives markets. Section 7.2. Forwards.
Example 5
XYZ stock costs $55 per share. The annual continuous interest
rate is 0.055. This stock pays dividends at an annual continuous
rate of 3.5%. A one year prepaid forward has a price of $52.60. Is
there any arbitrage opportunity? If so, describe the position an
arbitrageur would take and his prot per share.
Solution: The no arbitrage prepaid forward price is
F
P
0,T
= S
0
e
T
= 55e
0.035
= 53.10829789.
An arbitrage portfolio consists of entering a prepaid long forward
contract for one share of stock and shorting e
0.035
shares of
stock. The return of this transaction is
55e
0.035
52.60 = 0.5082978942. At redemption time, the
arbitrageur covers his short position after executing the prepaid
forward contract.
c 2009. Miguel A. Arcones. All rights reserved. Manual for SOA Exam FM/CAS Exam 2.
40/89
Chapter 7. Derivatives markets. Section 7.2. Forwards.
Example 5
XYZ stock costs $55 per share. The annual continuous interest
rate is 0.055. This stock pays dividends at an annual continuous
rate of 3.5%. A one year prepaid forward has a price of $52.60. Is
there any arbitrage opportunity? If so, describe the position an
arbitrageur would take and his prot per share.
Solution: The no arbitrage prepaid forward price is
F
P
0,T
= S
0
e
T
= 55e
0.035
= 53.10829789.
An arbitrage portfolio consists of entering a prepaid long forward
contract for one share of stock and shorting e
0.035
shares of
stock. The return of this transaction is
55e
0.035
52.60 = 0.5082978942. At redemption time, the
arbitrageur covers his short position after executing the prepaid
forward contract.
c 2009. Miguel A. Arcones. All rights reserved. Manual for SOA Exam FM/CAS Exam 2.
41/89
Chapter 7. Derivatives markets. Section 7.2. Forwards.
Example 6
XYZ stock costs $55 per share. The annual continuous interest
rate is 0.035. This stock pays dividends at an annual continuous
rate of 5.5%. A one year prepaid forward has a price of $52.60. Is
there any arbitrage opportunity? If so, describe the position an
arbitrageur would take and his prot per share.
Solution: The no arbitrage prepaid forward price is
F
P
0,T
= S
0
e
T
= 55e
0.055
= 52.05668314.
An arbitrage portfolio consists of entering a prepaid short forward
contract for one share of stock and buying e
0.055
shares of stock.
The return of this transaction is
52.60 55e
0.055
= 0.5433168626. At redemption time, we use
the bought stock to meet the short forward.
Notice that in the previous questions, we can make arbitrage
without making any investment of capital. The total price of
setting the portfolios at time zero is zero.
c 2009. Miguel A. Arcones. All rights reserved. Manual for SOA Exam FM/CAS Exam 2.
42/89
Chapter 7. Derivatives markets. Section 7.2. Forwards.
Example 6
XYZ stock costs $55 per share. The annual continuous interest
rate is 0.035. This stock pays dividends at an annual continuous
rate of 5.5%. A one year prepaid forward has a price of $52.60. Is
there any arbitrage opportunity? If so, describe the position an
arbitrageur would take and his prot per share.
Solution: The no arbitrage prepaid forward price is
F
P
0,T
= S
0
e
T
= 55e
0.055
= 52.05668314.
An arbitrage portfolio consists of entering a prepaid short forward
contract for one share of stock and buying e
0.055
shares of stock.
The return of this transaction is
52.60 55e
0.055
= 0.5433168626. At redemption time, we use
the bought stock to meet the short forward.
Notice that in the previous questions, we can make arbitrage
without making any investment of capital. The total price of
setting the portfolios at time zero is zero.
c 2009. Miguel A. Arcones. All rights reserved. Manual for SOA Exam FM/CAS Exam 2.
43/89
Chapter 7. Derivatives markets. Section 7.2. Forwards.
Pricing a forward contract.
Both the payment and the transfer of ownership happen at time
T. The price of a forward contract is the future value of the
prepaid forward contract, i.e. F
0,T
= e
rT
F
P
0,T
. So,

The price of a forward contract for a stock with no dividends


is F
0,T
= e
rT
S
0
.

The price of a forward contract for a stock with discrete


dividends is F
0,T
= e
rT
S
0

n
i =1
D
t
i
e
r (Tt
i
)
.

The price of a forward contract for a stock with continuous


dividends is F
0,T
= e
(r )T
S
0
.
c 2009. Miguel A. Arcones. All rights reserved. Manual for SOA Exam FM/CAS Exam 2.
44/89
Chapter 7. Derivatives markets. Section 7.2. Forwards.
Example 7
The current price of one share of XYZ stock is 55.34. The price of
a ninemonth forward contract on one share of XYZ stock is 57.6.
XYZ stock is not going to pay any dividends on the next 2 years.
(i) Calculate the annual compounded continuously interest rate
implied by this forward contract.
(ii) Calculate the price of a twoyear forward contract on one share
of XYZ stock.
Solution: (i) Since F
0,T
= e
rT
S
0
, 57.6 = e
(3/4)r
55.34 and
r = (4/3) ln(57.6/55.34) = 0.05336879112.
(ii) We have that
F
0,2
= e
r 2
S
0
= e
(0.05336879112)(2)
55.34 = 61.57362151.
c 2009. Miguel A. Arcones. All rights reserved. Manual for SOA Exam FM/CAS Exam 2.
45/89
Chapter 7. Derivatives markets. Section 7.2. Forwards.
Example 7
The current price of one share of XYZ stock is 55.34. The price of
a ninemonth forward contract on one share of XYZ stock is 57.6.
XYZ stock is not going to pay any dividends on the next 2 years.
(i) Calculate the annual compounded continuously interest rate
implied by this forward contract.
(ii) Calculate the price of a twoyear forward contract on one share
of XYZ stock.
Solution: (i) Since F
0,T
= e
rT
S
0
, 57.6 = e
(3/4)r
55.34 and
r = (4/3) ln(57.6/55.34) = 0.05336879112.
(ii) We have that
F
0,2
= e
r 2
S
0
= e
(0.05336879112)(2)
55.34 = 61.57362151.
c 2009. Miguel A. Arcones. All rights reserved. Manual for SOA Exam FM/CAS Exam 2.
46/89
Chapter 7. Derivatives markets. Section 7.2. Forwards.
Example 7
The current price of one share of XYZ stock is 55.34. The price of
a ninemonth forward contract on one share of XYZ stock is 57.6.
XYZ stock is not going to pay any dividends on the next 2 years.
(i) Calculate the annual compounded continuously interest rate
implied by this forward contract.
(ii) Calculate the price of a twoyear forward contract on one share
of XYZ stock.
Solution: (i) Since F
0,T
= e
rT
S
0
, 57.6 = e
(3/4)r
55.34 and
r = (4/3) ln(57.6/55.34) = 0.05336879112.
(ii) We have that
F
0,2
= e
r 2
S
0
= e
(0.05336879112)(2)
55.34 = 61.57362151.
c 2009. Miguel A. Arcones. All rights reserved. Manual for SOA Exam FM/CAS Exam 2.
47/89
Chapter 7. Derivatives markets. Section 7.2. Forwards.
Example 8
A stock is expected to pay a dividend of $1 per share in 2 months
and again in 5 months. The current stock price is $59 per share.
The risk free eective annual rate of interest is 6%.
(i) What is the fair price of a 6month forward contract?
(ii) Assume that 3 months from now the stock price is $57 per
share, what is the fair price of the same forward contract at that
time?
Solution: (i) The forward price is the future value of the payments
associated with owning the stock in six months:
F
0,0.5
= (59)(1.06)
0.5
(1)(1.06)
4/12
(1)(1.06)
1/12
= 58.71974.
(ii) (57)(1.06)
3/12
(1)(1.06)
1/12
= 56.83154.
c 2009. Miguel A. Arcones. All rights reserved. Manual for SOA Exam FM/CAS Exam 2.
48/89
Chapter 7. Derivatives markets. Section 7.2. Forwards.
Example 8
A stock is expected to pay a dividend of $1 per share in 2 months
and again in 5 months. The current stock price is $59 per share.
The risk free eective annual rate of interest is 6%.
(i) What is the fair price of a 6month forward contract?
(ii) Assume that 3 months from now the stock price is $57 per
share, what is the fair price of the same forward contract at that
time?
Solution: (i) The forward price is the future value of the payments
associated with owning the stock in six months:
F
0,0.5
= (59)(1.06)
0.5
(1)(1.06)
4/12
(1)(1.06)
1/12
= 58.71974.
(ii) (57)(1.06)
3/12
(1)(1.06)
1/12
= 56.83154.
c 2009. Miguel A. Arcones. All rights reserved. Manual for SOA Exam FM/CAS Exam 2.
49/89
Chapter 7. Derivatives markets. Section 7.2. Forwards.
Example 8
A stock is expected to pay a dividend of $1 per share in 2 months
and again in 5 months. The current stock price is $59 per share.
The risk free eective annual rate of interest is 6%.
(i) What is the fair price of a 6month forward contract?
(ii) Assume that 3 months from now the stock price is $57 per
share, what is the fair price of the same forward contract at that
time?
Solution: (i) The forward price is the future value of the payments
associated with owning the stock in six months:
F
0,0.5
= (59)(1.06)
0.5
(1)(1.06)
4/12
(1)(1.06)
1/12
= 58.71974.
(ii) (57)(1.06)
3/12
(1)(1.06)
1/12
= 56.83154.
c 2009. Miguel A. Arcones. All rights reserved. Manual for SOA Exam FM/CAS Exam 2.
50/89
Chapter 7. Derivatives markets. Section 7.2. Forwards.
Example 9
An investor is interested in buying XYZ stock. The current price of
stock is $30 per share. The riskfree annual interest rate
continuously compounded is 0.03. The price of a fourteenmonth
forward contract is 30.352. Calculate the continuous dividend yield
.
Solution: We have that
30.352 = F
0,T
= S
0
e
(r )T
= 30e
(0.03)(14/12)
.
and
= 0.03 (12/14) ln(30.352/30) = 0.02000140155.
c 2009. Miguel A. Arcones. All rights reserved. Manual for SOA Exam FM/CAS Exam 2.
51/89
Chapter 7. Derivatives markets. Section 7.2. Forwards.
Example 9
An investor is interested in buying XYZ stock. The current price of
stock is $30 per share. The riskfree annual interest rate
continuously compounded is 0.03. The price of a fourteenmonth
forward contract is 30.352. Calculate the continuous dividend yield
.
Solution: We have that
30.352 = F
0,T
= S
0
e
(r )T
= 30e
(0.03)(14/12)
.
and
= 0.03 (12/14) ln(30.352/30) = 0.02000140155.
c 2009. Miguel A. Arcones. All rights reserved. Manual for SOA Exam FM/CAS Exam 2.
52/89
Chapter 7. Derivatives markets. Section 7.2. Forwards.
Example 10
An investor is interested in buying XYZ stock. The current price of
stock is $30 per share. This stock pays dividends at an annual
continuous rate of 0.02. The riskfree annual eective rate of
interest is 0.045.
(i) What is the price of prepaid forward contract which expires in
18 months?
(ii) What is the price of forward contract which expires in 18
months?
Solution: (i) The prepaid forward price is
F
P
0,T
= S
0
e
T
= 30e
(0.02)(18/12)
= 29.11336601.
(ii) The 18month forward price is
29.11336601(1.045)
18/12
= 31.1004631.
c 2009. Miguel A. Arcones. All rights reserved. Manual for SOA Exam FM/CAS Exam 2.
53/89
Chapter 7. Derivatives markets. Section 7.2. Forwards.
Example 10
An investor is interested in buying XYZ stock. The current price of
stock is $30 per share. This stock pays dividends at an annual
continuous rate of 0.02. The riskfree annual eective rate of
interest is 0.045.
(i) What is the price of prepaid forward contract which expires in
18 months?
(ii) What is the price of forward contract which expires in 18
months?
Solution: (i) The prepaid forward price is
F
P
0,T
= S
0
e
T
= 30e
(0.02)(18/12)
= 29.11336601.
(ii) The 18month forward price is
29.11336601(1.045)
18/12
= 31.1004631.
c 2009. Miguel A. Arcones. All rights reserved. Manual for SOA Exam FM/CAS Exam 2.
54/89
Chapter 7. Derivatives markets. Section 7.2. Forwards.
Example 10
An investor is interested in buying XYZ stock. The current price of
stock is $30 per share. This stock pays dividends at an annual
continuous rate of 0.02. The riskfree annual eective rate of
interest is 0.045.
(i) What is the price of prepaid forward contract which expires in
18 months?
(ii) What is the price of forward contract which expires in 18
months?
Solution: (i) The prepaid forward price is
F
P
0,T
= S
0
e
T
= 30e
(0.02)(18/12)
= 29.11336601.
(ii) The 18month forward price is
29.11336601(1.045)
18/12
= 31.1004631.
c 2009. Miguel A. Arcones. All rights reserved. Manual for SOA Exam FM/CAS Exam 2.
55/89
Chapter 7. Derivatives markets. Section 7.2. Forwards.
omarket forward contract
A forward contract where either you pay a premium or you collect
a premium for entering into the deal is called an omarket
forward contract.
c 2009. Miguel A. Arcones. All rights reserved. Manual for SOA Exam FM/CAS Exam 2.
56/89
Chapter 7. Derivatives markets. Section 7.2. Forwards.
Example 11
Suppose that the current value of a certain amount of a
commodity is $45000. The annual eective rate of interest is 4.5%.
(i) You are oered a 2year long forward contract at a forward
price of $50000. How much would you need to be paid to enter
into this contract?
(ii) You are oered a 2year long forward contract at a forward
price of $48000. How much would you need be willing to pay to
enter into this contract?
c 2009. Miguel A. Arcones. All rights reserved. Manual for SOA Exam FM/CAS Exam 2.
57/89
Chapter 7. Derivatives markets. Section 7.2. Forwards.
Example 11
Suppose that the current value of a certain amount of a
commodity is $45000. The annual eective rate of interest is 4.5%.
(i) You are oered a 2year long forward contract at a forward
price of $50000. How much would you need to be paid to enter
into this contract?
(ii) You are oered a 2year long forward contract at a forward
price of $48000. How much would you need be willing to pay to
enter into this contract?
Solution: (i) Let x be how much you need to be paid to enter into
this contract. The current value of the commodity should be equal
to the present value of the expenses needed to get the commodity
using the long forward contract. Hence, 50000(1.045)
2
x =
45000. So, x = 50000(1.045)
2
45000 = 786.4976.
c 2009. Miguel A. Arcones. All rights reserved. Manual for SOA Exam FM/CAS Exam 2.
58/89
Chapter 7. Derivatives markets. Section 7.2. Forwards.
Example 11
Suppose that the current value of a certain amount of a
commodity is $45000. The annual eective rate of interest is 4.5%.
(i) You are oered a 2year long forward contract at a forward
price of $50000. How much would you need to be paid to enter
into this contract?
(ii) You are oered a 2year long forward contract at a forward
price of $48000. How much would you need be willing to pay to
enter into this contract?
Solution: (ii) Let y be how much would you need be willing to
pay to enter into this contract. The current value of the commod-
ity should be equal to the present value of the expenses needed
to get the commodity using the long forward contract. Hence,
48000(1.045)
2
+ y = 45000. So, y = 45000 48000(1.045)
2
=
1044.962341.
c 2009. Miguel A. Arcones. All rights reserved. Manual for SOA Exam FM/CAS Exam 2.
59/89
Chapter 7. Derivatives markets. Section 7.2. Forwards.
Arbitrage
If the price of a forward contract does not follow the previous
formulas, an arbitrageur can do arbitrage.
c 2009. Miguel A. Arcones. All rights reserved. Manual for SOA Exam FM/CAS Exam 2.
60/89
Chapter 7. Derivatives markets. Section 7.2. Forwards.
Example 12
XYZ stock pays no dividends and has a current price of $42.5 per
share. A long position in a forward contract is available to buy
1000 shares of stock six months from now for $43 per share. A
bank pays interest at the rate of 5% per annum (continuously
compounded) on a 6month certicate of deposit. Describe a
strategy for creating an arbitrage prot and determine the amount
of the prot.
Solution: The no arbitrage price of a forward contract is
S
0
e
rT
= (42.5)e
0.05(0.5)
= 43.57589262. Hence, it is possible to do
arbitrage by entering into the long forward position. An arbitrageur
can: sell 1000 shares of stock for (1000)(42.5) = 42500, deposit
42500 in the bank for six months, and sign up a forward contract
for a long position for 1000 shares of stock. In six months, the CD
returns (1000)(42.5)e
0.05(0.5)
= 43575.89262. The cost of the
forward is (1000)(43) = 430000. Hence, the prot is
43575.89262 430000 = 575.89262.
c 2009. Miguel A. Arcones. All rights reserved. Manual for SOA Exam FM/CAS Exam 2.
61/89
Chapter 7. Derivatives markets. Section 7.2. Forwards.
Example 12
XYZ stock pays no dividends and has a current price of $42.5 per
share. A long position in a forward contract is available to buy
1000 shares of stock six months from now for $43 per share. A
bank pays interest at the rate of 5% per annum (continuously
compounded) on a 6month certicate of deposit. Describe a
strategy for creating an arbitrage prot and determine the amount
of the prot.
Solution: The no arbitrage price of a forward contract is
S
0
e
rT
= (42.5)e
0.05(0.5)
= 43.57589262. Hence, it is possible to do
arbitrage by entering into the long forward position. An arbitrageur
can: sell 1000 shares of stock for (1000)(42.5) = 42500, deposit
42500 in the bank for six months, and sign up a forward contract
for a long position for 1000 shares of stock. In six months, the CD
returns (1000)(42.5)e
0.05(0.5)
= 43575.89262. The cost of the
forward is (1000)(43) = 430000. Hence, the prot is
43575.89262 430000 = 575.89262.
c 2009. Miguel A. Arcones. All rights reserved. Manual for SOA Exam FM/CAS Exam 2.
62/89
Chapter 7. Derivatives markets. Section 7.2. Forwards.
Example 13
Suppose that the riskfree eective rate of interest is 5% per
annum. XYZ stock is currently trading for $45.34 per share. XYZ
stock is expected to pay a dividend of $1.20 per share six months
from now. The price of a ninemonth forward contract on one
share of XYZ stock is $47.56. Is there an arbitrage opportunity on
the forward contract? If so, describe the strategy to realize prot
and nd the arbitrage prot.
Solution: The no arbitrage forward price is
F
0,T
= e
rT
S
0

i =1
D
t
i
e
r (Tt
i
)
= 45.34(1.05)
9/12
1.2(1.05)
3/12
=45.81511211.
We can make arbitrage by buying stock and entering a short
forward contract. The prot per share at expiration is
47.56 45.81511211 = 1.74488789.
c 2009. Miguel A. Arcones. All rights reserved. Manual for SOA Exam FM/CAS Exam 2.
63/89
Chapter 7. Derivatives markets. Section 7.2. Forwards.
Example 13
Suppose that the riskfree eective rate of interest is 5% per
annum. XYZ stock is currently trading for $45.34 per share. XYZ
stock is expected to pay a dividend of $1.20 per share six months
from now. The price of a ninemonth forward contract on one
share of XYZ stock is $47.56. Is there an arbitrage opportunity on
the forward contract? If so, describe the strategy to realize prot
and nd the arbitrage prot.
Solution: The no arbitrage forward price is
F
0,T
= e
rT
S
0

i =1
D
t
i
e
r (Tt
i
)
= 45.34(1.05)
9/12
1.2(1.05)
3/12
=45.81511211.
We can make arbitrage by buying stock and entering a short
forward contract. The prot per share at expiration is
47.56 45.81511211 = 1.74488789.
c 2009. Miguel A. Arcones. All rights reserved. Manual for SOA Exam FM/CAS Exam 2.
64/89
Chapter 7. Derivatives markets. Section 7.2. Forwards.
Suppose that a stock pays dividends at the continuous rate . In
the absence of arbitrage, entering a forward for one share for F
0,T
is equivalent to buying e
T
shares of stock for S
0
e
T
and
(borrowing S
0
e
T
) selling a zerocoupon bond for S
0
e
T
with
expiration in T years. In both cases, at time T we have one share
of stock after we make a payment of F
0,T
. There exists no
arbitrage if S
0
e
T
= F
0,T
e
rT
. If S
0
e
T
= F
0,T
e
rT
, we can
make arbitrage.
c 2009. Miguel A. Arcones. All rights reserved. Manual for SOA Exam FM/CAS Exam 2.
65/89
Chapter 7. Derivatives markets. Section 7.2. Forwards.
If F
0,T
< e
(r )T
S
0
, we can enter into a long forward for one share
of stock, and short e
T
shares of stock. At redemption time, we
cover the short position by paying F
0,T
for the stock. It is like we
have borrowed S
0
e
T
and pay the loan for F
0,T
. In some sense
we have created a zerocoupon bond. The position is called a
synthetic zerocoupon bond. Let r

be the continuous annual


rate of interest of the synthetic bond. This rate is called the
implied repo rate. We have that
S
0
e
T
e
r

T
= F
0,T
.
Hence, if F
0,T
< e
(r )T
S
0
,
r

=
1
T
log
_
F
0,T
S
0
e
T
_
<
1
T
log
_
S
0
e
(r )T
S
0
e
T
_
< r .
By doing an arbitrage, we are able to reduce the interest rate at
which we borrow. Technically, this is not call arbitrage. It is called
quasiarbitrage. We benet from this portfolio, only if we are
already borrowing.
c 2009. Miguel A. Arcones. All rights reserved. Manual for SOA Exam FM/CAS Exam 2.
66/89
Chapter 7. Derivatives markets. Section 7.2. Forwards.
Notice that the synthetic bond is created observing that
Long forward = Buy stock + Issue a bond
implies that
Issue a bond = Short stock + Long forward
c 2009. Miguel A. Arcones. All rights reserved. Manual for SOA Exam FM/CAS Exam 2.
67/89
Chapter 7. Derivatives markets. Section 7.2. Forwards.
Reciprocally, if F
0,T
> e
(r )T
S
0
, we can create a portfolio earning
a rate of interest bigger than the riskfree interest rate. We can
enter a short forward contract for one share of stock and buy e
T
shares of stock. At redemption time, we get an inow of F
0,T
.
Since we invested S
0
e
T
, the continuous annual interest rate r

,
which we earned in the investment satises
S
0
e
T
e
r

T
= F
0,T
.
Hence, if F
0,T
> e
(r )T
S
0
,
r

=
1
T
ln
_
F
0,T
S
0
e
T
_
>
1
T
ln
_
S
0
e
(r )T
S
0
e
T
_
= r .
Again this rate is called the implied repo rate.
c 2009. Miguel A. Arcones. All rights reserved. Manual for SOA Exam FM/CAS Exam 2.
68/89
Chapter 7. Derivatives markets. Section 7.2. Forwards.
Example 14
XYZ stock costs $123.118 per share. This stock pays dividends at
an annual continuous rate of 2.5%. A 18 month forward has a
price of $130.242. You own 10000 shares of XYZ stock. Calculate
the annual continuous rate of interest at which you can borrow by
shorting your stock.
Solution: We have that
r

= +
1
T
ln
_
F
0,T
S
0
_
= 0.025+
1
1.5
ln
_
130.242
123.118
_
= 6.250067554%.
c 2009. Miguel A. Arcones. All rights reserved. Manual for SOA Exam FM/CAS Exam 2.
69/89
Chapter 7. Derivatives markets. Section 7.2. Forwards.
Example 14
XYZ stock costs $123.118 per share. This stock pays dividends at
an annual continuous rate of 2.5%. A 18 month forward has a
price of $130.242. You own 10000 shares of XYZ stock. Calculate
the annual continuous rate of interest at which you can borrow by
shorting your stock.
Solution: We have that
r

= +
1
T
ln
_
F
0,T
S
0
_
= 0.025+
1
1.5
ln
_
130.242
123.118
_
= 6.250067554%.
c 2009. Miguel A. Arcones. All rights reserved. Manual for SOA Exam FM/CAS Exam 2.
70/89
Chapter 7. Derivatives markets. Section 7.2. Forwards.
Example 15
XYZ stock costs $124 per share. This stock pays dividends at an
annual continuous rate of 1.5%. A 2year forward has a price of
$135.7 per share. Calculate the annual continuous rate of interest
which you earn by buying stock and entering into a short forward
contract, both positions for the same nominal amount.
Solution: We have that
r

= +
1
T
ln
_
F
0,T
S
0
_
= 0.015 +
1
2
ln
_
135.7
124
_
= 6%.
c 2009. Miguel A. Arcones. All rights reserved. Manual for SOA Exam FM/CAS Exam 2.
71/89
Chapter 7. Derivatives markets. Section 7.2. Forwards.
Example 15
XYZ stock costs $124 per share. This stock pays dividends at an
annual continuous rate of 1.5%. A 2year forward has a price of
$135.7 per share. Calculate the annual continuous rate of interest
which you earn by buying stock and entering into a short forward
contract, both positions for the same nominal amount.
Solution: We have that
r

= +
1
T
ln
_
F
0,T
S
0
_
= 0.015 +
1
2
ln
_
135.7
124
_
= 6%.
c 2009. Miguel A. Arcones. All rights reserved. Manual for SOA Exam FM/CAS Exam 2.
72/89
Chapter 7. Derivatives markets. Section 7.2. Forwards.
The forward premium is
F
0,T
S
0
. Notice that this is not a price.
Prices of options are called premiums. But, here the nomenclature
is dierent. If a stock index pays dividends according with a
continuous rate , then
F
0,T
S
0
= e
T(r )
.
c 2009. Miguel A. Arcones. All rights reserved. Manual for SOA Exam FM/CAS Exam 2.
73/89
Chapter 7. Derivatives markets. Section 7.2. Forwards.
Example 16
XYZ stock cost $55 per share. A fourmonth forward on XYZ
stock costs $57.5. XYZ stock pays dividends according a
continuous rate.
(i) Calculate the fourmonth forward premium.
(ii) Calculate the eightmonth forward premium.
(iii) Calculate the eightmonth forward price.
Solution: (i) The fourmonth forward premium is
F
0,4/12
S
0
=
57.5
55
= 1.045454545.
(ii) The eightmonth forward premium is
F
0,8/12
S
0
= e
(8/12)(r )
=
_
e
(4/12)(r )
_
2
=
_
57.5
55
_
2
= 1.092975206.
(iii) The eightmonth forward price is
F
0,8/12
= (55)(1.092975206) = 60.11363633.
c 2009. Miguel A. Arcones. All rights reserved. Manual for SOA Exam FM/CAS Exam 2.
74/89
Chapter 7. Derivatives markets. Section 7.2. Forwards.
Example 16
XYZ stock cost $55 per share. A fourmonth forward on XYZ
stock costs $57.5. XYZ stock pays dividends according a
continuous rate.
(i) Calculate the fourmonth forward premium.
(ii) Calculate the eightmonth forward premium.
(iii) Calculate the eightmonth forward price.
Solution: (i) The fourmonth forward premium is
F
0,4/12
S
0
=
57.5
55
= 1.045454545.
(ii) The eightmonth forward premium is
F
0,8/12
S
0
= e
(8/12)(r )
=
_
e
(4/12)(r )
_
2
=
_
57.5
55
_
2
= 1.092975206.
(iii) The eightmonth forward price is
F
0,8/12
= (55)(1.092975206) = 60.11363633.
c 2009. Miguel A. Arcones. All rights reserved. Manual for SOA Exam FM/CAS Exam 2.
75/89
Chapter 7. Derivatives markets. Section 7.2. Forwards.
Example 16
XYZ stock cost $55 per share. A fourmonth forward on XYZ
stock costs $57.5. XYZ stock pays dividends according a
continuous rate.
(i) Calculate the fourmonth forward premium.
(ii) Calculate the eightmonth forward premium.
(iii) Calculate the eightmonth forward price.
Solution: (i) The fourmonth forward premium is
F
0,4/12
S
0
=
57.5
55
= 1.045454545.
(ii) The eightmonth forward premium is
F
0,8/12
S
0
= e
(8/12)(r )
=
_
e
(4/12)(r )
_
2
=
_
57.5
55
_
2
= 1.092975206.
(iii) The eightmonth forward price is
F
0,8/12
= (55)(1.092975206) = 60.11363633.
c 2009. Miguel A. Arcones. All rights reserved. Manual for SOA Exam FM/CAS Exam 2.
76/89
Chapter 7. Derivatives markets. Section 7.2. Forwards.
Example 16
XYZ stock cost $55 per share. A fourmonth forward on XYZ
stock costs $57.5. XYZ stock pays dividends according a
continuous rate.
(i) Calculate the fourmonth forward premium.
(ii) Calculate the eightmonth forward premium.
(iii) Calculate the eightmonth forward price.
Solution: (i) The fourmonth forward premium is
F
0,4/12
S
0
=
57.5
55
= 1.045454545.
(ii) The eightmonth forward premium is
F
0,8/12
S
0
= e
(8/12)(r )
=
_
e
(4/12)(r )
_
2
=
_
57.5
55
_
2
= 1.092975206.
(iii) The eightmonth forward price is
F
0,8/12
= (55)(1.092975206) = 60.11363633.
c 2009. Miguel A. Arcones. All rights reserved. Manual for SOA Exam FM/CAS Exam 2.
77/89
Chapter 7. Derivatives markets. Section 7.2. Forwards.
The annualized forward premium is
1
T
ln
_
F
0,T
S
0
_
. Note that in
the case of continuous dividends, the annualized forward premium
is r .
c 2009. Miguel A. Arcones. All rights reserved. Manual for SOA Exam FM/CAS Exam 2.
78/89
Chapter 7. Derivatives markets. Section 7.2. Forwards.
Example 17
XYZ stock cost $55 per share. A fourmonth forward on XYZ
stock costs $57.5.
(i) Calculate the annualized forward premium
(ii) Calculate the twelvemonth forward price.
Solution: (i) The annualized forward premium is
r =
1
T
ln
_
F
0,T
S
0
_
=
1
4/12
ln
_
57.5
55
_
= 0.1333552877.
(ii) The twelvemonth forward price is
F
0,T
= S
0
e
r
= 55e
0.1333552877
= 62.84607438.
c 2009. Miguel A. Arcones. All rights reserved. Manual for SOA Exam FM/CAS Exam 2.
79/89
Chapter 7. Derivatives markets. Section 7.2. Forwards.
Example 17
XYZ stock cost $55 per share. A fourmonth forward on XYZ
stock costs $57.5.
(i) Calculate the annualized forward premium
(ii) Calculate the twelvemonth forward price.
Solution: (i) The annualized forward premium is
r =
1
T
ln
_
F
0,T
S
0
_
=
1
4/12
ln
_
57.5
55
_
= 0.1333552877.
(ii) The twelvemonth forward price is
F
0,T
= S
0
e
r
= 55e
0.1333552877
= 62.84607438.
c 2009. Miguel A. Arcones. All rights reserved. Manual for SOA Exam FM/CAS Exam 2.
80/89
Chapter 7. Derivatives markets. Section 7.2. Forwards.
Example 17
XYZ stock cost $55 per share. A fourmonth forward on XYZ
stock costs $57.5.
(i) Calculate the annualized forward premium
(ii) Calculate the twelvemonth forward price.
Solution: (i) The annualized forward premium is
r =
1
T
ln
_
F
0,T
S
0
_
=
1
4/12
ln
_
57.5
55
_
= 0.1333552877.
(ii) The twelvemonth forward price is
F
0,T
= S
0
e
r
= 55e
0.1333552877
= 62.84607438.
c 2009. Miguel A. Arcones. All rights reserved. Manual for SOA Exam FM/CAS Exam 2.
81/89
Chapter 7. Derivatives markets. Section 7.2. Forwards.
Hedging a forward contract.
A (scalper) market maker must be able to oset the risk of trading
forward contracts. Assume continuous dividends. Suppose that a
scalper enters into a short forward contract. The prot at
expiration for a long forward position is S
T
F
0,T
. In order to
obtain this same payo a scalper can borrow S
0
e
T
and use this
money to get e
T
shares of stock. At time T, he sells the stock
which he owns for S
0
e
(r )T
= F
0,T
. Notice that by investing the
dividends, e
T
shares of stock have grown to one share at time
T. Borrowing S
0
e
T
and buying e
T
shares of stock is called a
synthetic long forward. So, if a scalper enters into a short
forward contract with a client, the scalper either matches this
position with another clients long forward contract or creates a
synthetic long forward
c 2009. Miguel A. Arcones. All rights reserved. Manual for SOA Exam FM/CAS Exam 2.
82/89
Chapter 7. Derivatives markets. Section 7.2. Forwards.
The prot of a short forward position is F
0,T
S
T
. A scalper can
get this prot, by (lending) buying a zerocoupon bond for S
0
e
T
and shorting e
T
shares receiving S
0
e
T
. Buying a zerocoupon
bond for S
0
e
T
and shorting a tailed position for e
T
shares is
called a synthetic short forward. Again, a scalper may need to
create this position to match a clients long forward position.
c 2009. Miguel A. Arcones. All rights reserved. Manual for SOA Exam FM/CAS Exam 2.
83/89
Chapter 7. Derivatives markets. Section 7.2. Forwards.
Using these strategies, a marketmaker can hedge his clients
positions.
A transaction in which you buy the asset and short the forward
contract is called cashandcarry (or cashandcarry hedge). It
is called cashandcarry, because the cash is used to buy the asset
and the asset is kept. A cashandcarry has no risk. You have
obligation to deliver the asset, but you also own the asset. An
arbitrage that involves buying the asset and selling it forward is
called cashandcarry arbitrage. A (reverse cashandcarry
hedge) reverse cashandcarry involves shortselling and asset
and entering into a long forward position.
c 2009. Miguel A. Arcones. All rights reserved. Manual for SOA Exam FM/CAS Exam 2.
84/89
Chapter 7. Derivatives markets. Section 7.2. Forwards.
An arbitrageur can make money if F
0,T
= S
0
e
(r )T
. But, in the
real world, transaction costs have to be taken into account.
Suppose that:
(i) The stock bid and ask prices are S
b
0
and S
a
0
, where S
b
0
< S
a
0
.
(ii) The forward bid and ask prices are F
b
0,T
< F
a
0,T
.
(iii) The cost of a transaction in the stock is K
S
.
(iv) The cost of a transaction in the forward is K
F
.
(v) The interest rates for borrowing and lending are r
b
> r
l
,
respectively.
c 2009. Miguel A. Arcones. All rights reserved. Manual for SOA Exam FM/CAS Exam 2.
85/89
Chapter 7. Derivatives markets. Section 7.2. Forwards.
Suppose that the arbitrageur believes that the observed forward
price F
0,T
is too high. Then, he could:
(a) contract a short forward for F
b
0,T
(b) buy a tailed position in stock for S
a
0
e
T
.
(c) borrow S
ae
T
0
+ K
F
+ K
S
.
The payo of this combined transaction is:
F
b
0,T
S
T
+ S
T
(S
a
0
e
T
+ K
f
+ K
S
)e
r
b
T
=F
b
0,T
(S
a
0
e
T
+ K
f
+ K
S
)e
r
b
T
.
The scalper makes money if F
b
0,T
> (S
a
0
e
T
+ K
f
+ K
S
)e
r
b
T
. The
previous strategy is cashandcarry arbitrage.
c 2009. Miguel A. Arcones. All rights reserved. Manual for SOA Exam FM/CAS Exam 2.
86/89
Chapter 7. Derivatives markets. Section 7.2. Forwards.
Suppose that the scalper believes that the observed forward price
F
0,T
is too low. Then, he could:
(a) enter a long forward for F
a
0,T
(b) short a tailed position in stock for S
b
0
e
T
.
(c) lend S
b
0
e
T
K
F
K
S
.
The payo of this combined transaction is:
S
T
F
a
0,T
S
T
+ (S
a
0
e
T
K
F
K
S
)e
r
l
T
=F
a
0,T
+ (S
a
0
e
T
K
F
K
S
)e
r
l
T
.
The arbitrageur makes money if (S
a
0
e
T
K
F
K
S
)e
r
l
T
> F
a
0,T
.
The previous strategy is reverse cashandcarry arbitrage.
c 2009. Miguel A. Arcones. All rights reserved. Manual for SOA Exam FM/CAS Exam 2.
87/89
Chapter 7. Derivatives markets. Section 7.2. Forwards.
Example 18
Suppose that an arbitrageur would like to enter a cashandcarry
for 10000 barrels of oil for delivery in six months. Suppose that he
can borrow at an annual eective rate of interest of 4.5%. The
current price of a barrel of oil is $55.
(i) What is the minimum forward price at which he would make a
prot?
(ii) What is his prot if the forward price is $57?
Solution: (i) He would make a prot if
F
0,T
> 55(1.045)
1/2
= 56.22388283.
(ii) The prot is (10000)(57 (55)(1.045)
1/2
) = 7761.171743.
c 2009. Miguel A. Arcones. All rights reserved. Manual for SOA Exam FM/CAS Exam 2.
88/89
Chapter 7. Derivatives markets. Section 7.2. Forwards.
Example 18
Suppose that an arbitrageur would like to enter a cashandcarry
for 10000 barrels of oil for delivery in six months. Suppose that he
can borrow at an annual eective rate of interest of 4.5%. The
current price of a barrel of oil is $55.
(i) What is the minimum forward price at which he would make a
prot?
(ii) What is his prot if the forward price is $57?
Solution: (i) He would make a prot if
F
0,T
> 55(1.045)
1/2
= 56.22388283.
(ii) The prot is (10000)(57 (55)(1.045)
1/2
) = 7761.171743.
c 2009. Miguel A. Arcones. All rights reserved. Manual for SOA Exam FM/CAS Exam 2.
89/89
Chapter 7. Derivatives markets. Section 7.2. Forwards.
Example 18
Suppose that an arbitrageur would like to enter a cashandcarry
for 10000 barrels of oil for delivery in six months. Suppose that he
can borrow at an annual eective rate of interest of 4.5%. The
current price of a barrel of oil is $55.
(i) What is the minimum forward price at which he would make a
prot?
(ii) What is his prot if the forward price is $57?
Solution: (i) He would make a prot if
F
0,T
> 55(1.045)
1/2
= 56.22388283.
(ii) The prot is (10000)(57 (55)(1.045)
1/2
) = 7761.171743.
c 2009. Miguel A. Arcones. All rights reserved. Manual for SOA Exam FM/CAS Exam 2.
1/15
Chapter 7. Derivatives markets.
Manual for SOA Exam FM/CAS Exam 2.
Chapter 7. Derivative markets.
Section 7.3. Futures.
c 2009. Miguel A. Arcones. All rights reserved.
Extract from:
Arcones Manual for the SOA Exam FM/CAS Exam 2,
Financial Mathematics. Fall 2009 Edition,
available at http://www.actexmadriver.com/
c 2009. Miguel A. Arcones. All rights reserved. Manual for SOA Exam FM/CAS Exam 2.
2/15
Chapter 7. Derivatives markets. Section 7.3. Futures.
Futures
A future is a standardized contract in which two counterparts
agree to buy/sell an asset for a specied price (the future price) at
a specied date (the delivery date).
The buyer in the future contract is called the long future. The
seller in the future contract is called the short future.
The main reasons to enter into a future contract are hedging and
speculation.
At dierence of futures, forward contracts are privately negotiated
and are not standardized. Forward contracts are entirely exible.
Forward contracts are tailormade contracts.
c 2009. Miguel A. Arcones. All rights reserved. Manual for SOA Exam FM/CAS Exam 2.
3/15
Chapter 7. Derivatives markets. Section 7.3. Futures.
Futures are bought and sold in organized futures exchanges. The
biggest future exchanges are the Chicago Mercantile Exchange, the
Chicago Board of Trade, the International Petroleum Exchange of
London, the New York Mercantile Exchange, the London Metal
Exchange and the Tokyo Commodity Exchange.
Futures transactions in the USA are regulated by the (CFTC)
Commodity Futures Trading Commission, an agency of the USA
government. The CFTC also regulates option markets.
A future contract is negotiated through a brokerage rm that holds
a seat on the exchange. A future contract is settled by a
clearinghouse owned by or associated with the exchange. The
clearinghouse matches the purchases and the sales which take
place during the day. By matching trades, the clearinghouse never
takes market risk because it always has osetting positions with
dierent counterparts. By having the clearinghouse as counterpart,
an individual entering a future contract does not face the possible
credit risk of its counterpart.
c 2009. Miguel A. Arcones. All rights reserved. Manual for SOA Exam FM/CAS Exam 2.
4/15
Chapter 7. Derivatives markets. Section 7.3. Futures.
Let us consider some common futures.
Crude oil futures trade in units of 1,000 U.S. barrels (42,000
gallons). The underlying is a US barrel. The notional amount is
1000 barrels. The current price is $70/barrel. Hence, the current
value of a future contract on crude oil is $70000.
S & P 500 future contracts trade on 250 units of the index. They
are cash settled. At expiration time, instead of a sale, one of the
future counterpart receive a payment according with S & P 500
spot price at expiration. The current price of S & P 500 is 1500.
The current value of a future contract on S & P 500 is
(250)(1500) = $375000.
c 2009. Miguel A. Arcones. All rights reserved. Manual for SOA Exam FM/CAS Exam 2.
5/15
Chapter 7. Derivatives markets. Section 7.3. Futures.
Suppose that two parties agree in a future contact for crude oil for
delivery in 18 months. The contract is worth $70000. Each
(investor) party makes a trade with the clearinghouse. This
contract has two risks: market risk and credit risk. The market
risk is related with the volatility of the price of the asset. The
credit risk is related with the solvency of each party. To avoid
credit risk, an individual or corporation entering a future contract
must make a deposit into an account called the margin account.
This deposit is called the initial margin. The margin account
earns interest. The amount of the initial margin is determined by
the exchange. It is usually a fraction of the market value of the
futures underlying asset. Usually future positions are settled into
the margin account either every day or every week. By every day
we mean every day which the market is open. Let us suppose that
a clearinghouse settles accounts daily. Suppose that the annual
continuously compounded interest rate is r .
c 2009. Miguel A. Arcones. All rights reserved. Manual for SOA Exam FM/CAS Exam 2.
6/15
Chapter 7. Derivatives markets. Section 7.3. Futures.
Every day, the prot or loss is calculated on the investors futures
position. If there exists a loss, the investors broker transfers that
amount from the investors margin account to the clearinghouse. If
a prot, the clearinghouse transfers that amount to investors
broker who then deposits it into the investors margin account.
The prot for a long position in a future contract is
M
t(1/365)
(e
r /365
1) + N(S
t
S
t(1/365)
),
where M
t(1/365)
is the yesterdays balance in the margin account,
N is the nominal amount, S
t
is the current price, S
t(1/365)
is the
yesterday price. Hence, after the settlement, the balance in the
investors margin account is
M
t
= M
t(1/365)
e
r /365
+ N(S
t
S
t(1/365)
).
The prot for a short position in a future contract is
M
t(1/365)
(1 e
r /365
) + N(S
t(1/365)
S
t
).
Markingtomarket is to calculate the value of a future contract
according with the current value of the asset.
c 2009. Miguel A. Arcones. All rights reserved. Manual for SOA Exam FM/CAS Exam 2.
7/15
Chapter 7. Derivatives markets. Section 7.3. Futures.
Example 1
On July 5, 2007, John enters a long future contract for 1,000 U.S.
barrels of oil at $71.6 a barrel. The margin account is 50% of the
market value of the futures underlier. The annual continuously
compounded rate of return is 0.06.
(i) On July 6, 2007, the price of oil is $70.3. What is the balance
in Johns margin account after settlement?
(ii) On July 7, 2007, the price of oil is $72.1. What is the balance
in Johns margin account after settlement?
c 2009. Miguel A. Arcones. All rights reserved. Manual for SOA Exam FM/CAS Exam 2.
8/15
Chapter 7. Derivatives markets. Section 7.3. Futures.
Solution: (i) The initial balance in Johns margin account is
(0.50)(1000)(71.6) = 35800.
The balance in Johns margin account on July 6, 2007, after
settlement, is
M
t(1/365)
e
r /365
+ N(S
t
S
t(1/365)
)
=(35800)e
0.06/365
+ (1000)(70.3 71.6) = 35105.89.
Since the price of the oil decreases, the value of having 1000
barrels in 18 months decreases.
c 2009. Miguel A. Arcones. All rights reserved. Manual for SOA Exam FM/CAS Exam 2.
9/15
Chapter 7. Derivatives markets. Section 7.3. Futures.
Solution: (ii) The balance in Johns margin account on July 6,
2007, after settlement, is
M
t(1/365)
e
r /365
+ n(S
t
S
t(1/365)
)
=(35105.89)e
0.06/365
+ (1000)(72.1 70.3) = 35711.56.
Notice that this balance is dierent from
(35800)e
(0.06)(2/365)
+ (1000)(72.1 71.6) = 36311.77.
In the rst day, Johns account balance was smaller. So, John lost
interest because the drop on price on July 6, 2007.
c 2009. Miguel A. Arcones. All rights reserved. Manual for SOA Exam FM/CAS Exam 2.
10/15
Chapter 7. Derivatives markets. Section 7.3. Futures.
If the balance in the margin account falls the clearinghouse has less
protection against default. Investors are required to keep the
margin account to a minimum level. This level is a fraction of the
initial margin. The maintenance margin is the fraction of the
initial margin which participants are asked to hold in their
accounts. If the balance in the margin account falls below this
level, an investors broker will require the investor to deposit funds
sucient to restore the balance to the initial margin level. Such a
demand is called a margin call. If an investor fail to the deposit,
the investors broker will immediately liquidate some or all of the
investors positions.
c 2009. Miguel A. Arcones. All rights reserved. Manual for SOA Exam FM/CAS Exam 2.
11/15
Chapter 7. Derivatives markets. Section 7.3. Futures.
Example 2
A company enters into a short futures contract to sell 100,000
pounds of frozen orange juice for $1.4 cents per pound. The initial
margin is 30% and the maintenance margin is 20%. The annual
eective rate of interest is 4.5%. The account is settled every
week. What is the minimum next week price which would lead to a
margin call?
c 2009. Miguel A. Arcones. All rights reserved. Manual for SOA Exam FM/CAS Exam 2.
12/15
Chapter 7. Derivatives markets. Section 7.3. Futures.
Solution: The initial balance in the margin account is
(0.30)(100000)(1.4) = 42000. The minimum balance in the
margin account is (0.20)(100000)(1.4) = 28000. After settlement
next week balance is
42000(1.045)
1/52
+ 100000(1.4 S
1/52
).
A margin call happens if
28000 > 42000(1.045)
1/52
+ 100000(1.4 S
1/52
),
or
S
1/52
> 1.4
28000 42000(1.045)
1/52
100000
= 1.540355672.
c 2009. Miguel A. Arcones. All rights reserved. Manual for SOA Exam FM/CAS Exam 2.
13/15
Chapter 7. Derivatives markets. Section 7.3. Futures.
Besides holding the contract until expiration, there are two ways to
close a future contract: oset the contract and exchange for
physicals. To oset the contract means to enter a reverse position
with the same broker. Since future contracts are standardized, it is
possible to nd a reserve position on a contract. Exchange for
physicals consists selling/buying the commodity.
c 2009. Miguel A. Arcones. All rights reserved. Manual for SOA Exam FM/CAS Exam 2.
14/15
Chapter 7. Derivatives markets. Section 7.3. Futures.
The two main advantages of futures versus forwards are liquidity
and counterparty risk. It is much easier to cancel before
expiration a future contract than a forward contract. Since the
trade is made against a clearinghouse, a participant does face
credit risk. At the same time, the margin and the marking to
market reduces the default risk.
c 2009. Miguel A. Arcones. All rights reserved. Manual for SOA Exam FM/CAS Exam 2.
15/15
Chapter 7. Derivatives markets. Section 7.3. Futures.
Having a margin account makes the prot/losses of the investment
higher for a future than for a forward. The oscillations of the price
of the asset make the earnings in the margin account more
variable. Usually, if there is a prot from the change of the price of
the asset, there is also a prot in the balance account.
Reciprocally, if there is a loss from the price change, there exists a
loss in the margin account.
c 2009. Miguel A. Arcones. All rights reserved. Manual for SOA Exam FM/CAS Exam 2.
1/112
Chapter 7. Derivatives markets.
Manual for SOA Exam FM/CAS Exam 2.
Chapter 7. Derivatives markets.
Section 7.4. Call options.
c 2009. Miguel A. Arcones. All rights reserved.
Extract from:
Arcones Manual for the SOA Exam FM/CAS Exam 2,
Financial Mathematics. Fall 2009 Edition,
available at http://www.actexmadriver.com/
c 2009. Miguel A. Arcones. All rights reserved. Manual for SOA Exam FM/CAS Exam 2.
2/112
Chapter 7. Derivatives markets. Section 7.4. Call options.
Minimums and maximums
Denition 1
Given two real numbers a and b,
(i) min(a, b) denotes the (minimum) smallest of the two numbers.
(ii) max(a, b) denotes the (maximum) biggest of the two numbers.
Example 1
min(10, 5) = 5, max(10, 5) = 10, min(1, 5) = 1,
max(1, 5) = 5, min(2, 100) = 100, max(2, 100) = 2.
c 2009. Miguel A. Arcones. All rights reserved. Manual for SOA Exam FM/CAS Exam 2.
3/112
Chapter 7. Derivatives markets. Section 7.4. Call options.
Denition 2
Given real numbers a
1
, . . . , a
n
,
(i) min(a
1
, . . . , a
n
) denotes the (minimum) smallest of these
numbers.
(ii) max(a
1
, . . . , a
n
) denotes the (maximum) biggest of these
numbers.
Example 2
min(1, 5, 3, 6) = 6, max(1, 5, 3, 6) = 5,
min(2, 100, 50) = 100 and max(2, 100, 50) = 2.
c 2009. Miguel A. Arcones. All rights reserved. Manual for SOA Exam FM/CAS Exam 2.
4/112
Chapter 7. Derivatives markets. Section 7.4. Call options.
Theorem 1
For each a, b, c R and each 0,

min(a, b) = min(b, a).

max(a, b) = max(b, a).

min(min(a, b), c) = min(a, min(b, c)) = min(a, b, c).

max(max(a, b), c) = max(a, max(b, c)) = max(a, b, c).

min(a + c, b + c) = min(a, b) + c.

max(a + c, b + c) = max(a, b) + c.

min(a, b) = min(a, b).

max(a, b) = max(a, b).

min(a, b) = max(a, b).

max(a, b) = min(a, b).


c 2009. Miguel A. Arcones. All rights reserved. Manual for SOA Exam FM/CAS Exam 2.
5/112
Chapter 7. Derivatives markets. Section 7.4. Call options.
Denition 3
Given a real number a, |a| = a, if a 0; and |a| = a, if a 0
Example 3
|23| = 23, | 4| = 4.
Theorem 2
For each a, b R, min(a, b) + max(a, b) = a + b.
Proof.
min(a, b) and max(a, b) are a and b in some order. Hence,
min(a, b) + max(a, b) = a + b.
Theorem 3
For each a R, |a| = max(a, 0) min(a, 0).
Proof.
If a 0, then max(a, 0) = a, min(a, 0) = 0, and
max(a, 0) min(a, 0) = a = |a|. If a 0, then max(a, 0) = 0,
min(a, 0) = a, and max(a, 0) min(a, 0) = a = |a|.
c 2009. Miguel A. Arcones. All rights reserved. Manual for SOA Exam FM/CAS Exam 2.
6/112
Chapter 7. Derivatives markets. Section 7.4. Call options.
Call options
Denition 4
A call option is a nancial contract which gives the owner the
right, but not the obligation, to buy a specied amount of a given
asset at a specied price during a specied period of time.
The call option owner exercises the option by buying the asset at
the specied call price from the call writer. A call option is
executed only if the call owner decides to do so. A call option
owner executes a call option only when it benets him, i.e. when
the specied call price is smaller than the current (market value)
spot price. Since the owner of a call option can make money if the
option is exercised, call options are sold. The owner of the call
option must pay to its counterpart for holding a call option. The
price of a call option is called its premium.
c 2009. Miguel A. Arcones. All rights reserved. Manual for SOA Exam FM/CAS Exam 2.
7/112
Chapter 7. Derivatives markets. Section 7.4. Call options.
Call options
Denition 4
A call option is a nancial contract which gives the owner the
right, but not the obligation, to buy a specied amount of a given
asset at a specied price during a specied period of time.
The call option owner exercises the option by buying the asset at
the specied call price from the call writer. A call option is
executed only if the call owner decides to do so. A call option
owner executes a call option only when it benets him, i.e. when
the specied call price is smaller than the current (market value)
spot price. Since the owner of a call option can make money if the
option is exercised, call options are sold. The owner of the call
option must pay to its counterpart for holding a call option. The
price of a call option is called its premium.
c 2009. Miguel A. Arcones. All rights reserved. Manual for SOA Exam FM/CAS Exam 2.
8/112
Chapter 7. Derivatives markets. Section 7.4. Call options.

The (owner) buyer of a call option is called the option call


holder. The holder of a call option is said to have a long call
position.

The seller of a call option is called the option call writer.


The writer of a call is said to have a short call position.

Assets used in call options are in commodities, currency


exchange, stock shares and stock indices.

A call option needs to specify the type and quality of the


underlying.

The asset used in the call option is called the underlier or


underlying asset.

The amount of the underlying asset to which the call option


applies is called the notional amount.

The specied price of an asset in a call option is called the


strike price, or exercise price.

A forward contract forces the buyer and seller to execute the


sale. A call option is executed only if the call holder decides
to do so.
c 2009. Miguel A. Arcones. All rights reserved. Manual for SOA Exam FM/CAS Exam 2.
9/112
Chapter 7. Derivatives markets. Section 7.4. Call options.

For an European option, the exercise of the option must


occur at a certain time (the expiration date).

For an American option, the exercise of the option must


occur any time by the expiration date.

For a Bermudan option, the buyer can exercise the call


option during specied periods.
Unless say otherwise, we will assume that an option is an European
option. European options are simpler and easier to study.
c 2009. Miguel A. Arcones. All rights reserved. Manual for SOA Exam FM/CAS Exam 2.
10/112
Chapter 7. Derivatives markets. Section 7.4. Call options.
Example 4
Suppose that an investor buys a call option of 100 shares of XYZ
stock with a strike price of $76 per share. The exercise date is one
year from now.
(i) If the spot price at expiration is $70 per share, the call option
holder does not exercise the option. The option is worthless. The
call option holder can buy stock in the market for a price smaller
than the call option price.
(ii) If the (the market price) spot price at expiration is $80 per
share, the call option holder exercises the call option, i.e. he buys
100 shares of XYZ stock for $76 from the option seller. Since the
call option holder can sell these shares for $80 per share, the call
option holder gets a payo of 100(80 76) = $400.
c 2009. Miguel A. Arcones. All rights reserved. Manual for SOA Exam FM/CAS Exam 2.
11/112
Chapter 7. Derivatives markets. Section 7.4. Call options.
Let K be the strike price of a call option. Let S
T
be the price of
the asset at expiration.

The call option holders payo is

0 if S
T
< K,
S
T
K if S
T
K.
We also can write this as max(0, S
T
K).

The payo for the call option writer is the opposite of the
holders payo. The payo for the call option writer is
max(0, S
T
K).

A calloption is a zerosum game. The sum of the two


payos is zero.
Figure 1 shows a graph of the call option payo as a function of
S
T
.
c 2009. Miguel A. Arcones. All rights reserved. Manual for SOA Exam FM/CAS Exam 2.
12/112
Chapter 7. Derivatives markets. Section 7.4. Call options.
T
E

max(S
T
K, 0)
S
T
K
Payo for the call option holder
T
E
d
d
d
d
d
max(S
T
K, 0)
S
T
Payo for the call option writer
K
Figure 1: Payos of a call option
c 2009. Miguel A. Arcones. All rights reserved. Manual for SOA Exam FM/CAS Exam 2.
13/112
Chapter 7. Derivatives markets. Section 7.4. Call options.
Recall:

The call option holders payo is


max(0, S
T
K).

The call option writers payo is


max(0, S
T
K).
We get from Figure 1 that:

The minimum payo for the call option holder is 0. The


maximum payo for the call option holder is .

The minimum payo for the call option writer is . The


maximum payo for the call option writer is 0.
minimum payo maximum payo
call option holder 0
call option writer 0
c 2009. Miguel A. Arcones. All rights reserved. Manual for SOA Exam FM/CAS Exam 2.
14/112
Chapter 7. Derivatives markets. Section 7.4. Call options.
Example 5
Andrew buys a 45strike call option for XYZ stock with a nominal
amount of 2000 shares. The expiration date is 6 months from now.
(i) Calculate Andrews payo for the following spot prices per share
at expiration: 35, 40, 45, 55, 60.
(ii) Calculate Andrews minimum and maximum payos.
Solution: (i) Andrews payo is (2000) max(S
T
45, 0). The
corresponding payos are:
if S
T
= 35, payo = (2000) max(35 45, 0) = 0,
if S
T
= 40, payo = (2000) max(40 45, 0) = 0,
if S
T
= 45, payo = (2000) max(45 45, 0) = 0,
if S
T
= 50, payo = (2000) max(50 45, 0) = 10000,
if S
T
= 55, payo = (2000) max(55 45, 0) = 20000.
(ii) Andrews minimum payo is 0. Andrews maximum payo is
.
c 2009. Miguel A. Arcones. All rights reserved. Manual for SOA Exam FM/CAS Exam 2.
15/112
Chapter 7. Derivatives markets. Section 7.4. Call options.
Example 5
Andrew buys a 45strike call option for XYZ stock with a nominal
amount of 2000 shares. The expiration date is 6 months from now.
(i) Calculate Andrews payo for the following spot prices per share
at expiration: 35, 40, 45, 55, 60.
(ii) Calculate Andrews minimum and maximum payos.
Solution: (i) Andrews payo is (2000) max(S
T
45, 0). The
corresponding payos are:
if S
T
= 35, payo = (2000) max(35 45, 0) = 0,
if S
T
= 40, payo = (2000) max(40 45, 0) = 0,
if S
T
= 45, payo = (2000) max(45 45, 0) = 0,
if S
T
= 50, payo = (2000) max(50 45, 0) = 10000,
if S
T
= 55, payo = (2000) max(55 45, 0) = 20000.
(ii) Andrews minimum payo is 0. Andrews maximum payo is
.
c 2009. Miguel A. Arcones. All rights reserved. Manual for SOA Exam FM/CAS Exam 2.
16/112
Chapter 7. Derivatives markets. Section 7.4. Call options.
Example 6
Madison sells a 45strike call option for XYZ stock with a nominal
amount of 2000 shares. The expiration date is 6 months from now.
(i) Calculate Madisons payo for the following spot prices at
expiration: 35, 40, 45, 55, 60.
(ii) Calculate Madisons minimum and maximum payos.
Solution: (i) Madisons payo is (2000) max(S
T
45, 0). The
corresponding payos are:
if S
T
= 35, payo = (2000) max(35 45, 0) = 0,
if S
T
= 40, payo = (2000) max(40 45, 0) = 0,
if S
T
= 45, payo = (2000) max(45 45, 0) = 0,
if S
T
= 50, payo = (2000) max(50 45, 0) = 10000,
if S
T
= 55, payo = (2000) max(55 45, 0) = 20000.
(ii) Madisons payo is (2000) max(S
T
45, 0). Madisons
minimum payo is . Madisons maximum payo is 0.
c 2009. Miguel A. Arcones. All rights reserved. Manual for SOA Exam FM/CAS Exam 2.
17/112
Chapter 7. Derivatives markets. Section 7.4. Call options.
Example 6
Madison sells a 45strike call option for XYZ stock with a nominal
amount of 2000 shares. The expiration date is 6 months from now.
(i) Calculate Madisons payo for the following spot prices at
expiration: 35, 40, 45, 55, 60.
(ii) Calculate Madisons minimum and maximum payos.
Solution: (i) Madisons payo is (2000) max(S
T
45, 0). The
corresponding payos are:
if S
T
= 35, payo = (2000) max(35 45, 0) = 0,
if S
T
= 40, payo = (2000) max(40 45, 0) = 0,
if S
T
= 45, payo = (2000) max(45 45, 0) = 0,
if S
T
= 50, payo = (2000) max(50 45, 0) = 10000,
if S
T
= 55, payo = (2000) max(55 45, 0) = 20000.
(ii) Madisons payo is (2000) max(S
T
45, 0). Madisons
minimum payo is . Madisons maximum payo is 0.
c 2009. Miguel A. Arcones. All rights reserved. Manual for SOA Exam FM/CAS Exam 2.
18/112
Chapter 7. Derivatives markets. Section 7.4. Call options.
Let Call(K, T) be the premium per unit paid by the buyer of a call
option with strike price K and expiration time T years. Notice that
Call(K, T) > 0. The premium of a call option for N units is
NCall(K, T). Let i be the riskfree annual eective rate of
interest.

The call option holders prot per unit is


max(S
T
K, 0) Call(K, T)(1 + i )
T
=

Call(K, T)(1 + i )
T
if S
T
< K,
S
T
K Call(K, T)(1 + i )
T
if S
T
K.

The call option sellers prot per unit is


Call(K, T)(1 + i )
T
max(0, S
T
K)
=

Call(K, T)(1 + i )
T
if S
T
< K,
Call(K, T)(1 + i )
T
(S
T
K) if S
T
K.
.
c 2009. Miguel A. Arcones. All rights reserved. Manual for SOA Exam FM/CAS Exam 2.
19/112
Chapter 7. Derivatives markets. Section 7.4. Call options.
The call option holder prot max(S
T
K, 0) Call(K, T)(1 + i )
T
as a function of S
T
is nondecreasing. The call option holder
benets from the increase of the spot price.

The minimum call option holder prot is


Call(K, T)(1 + i )
T
.

The maximum call option holder prot is .

The prot for the call option holder is positive if


S
T
> K + Call(K, T)(1 + i )
T
.

If S
T
< K + Call(K, T)(1 + i )
T
, the call option holder prot
is negative.
c 2009. Miguel A. Arcones. All rights reserved. Manual for SOA Exam FM/CAS Exam 2.
20/112
Chapter 7. Derivatives markets. Section 7.4. Call options.
The call option writers prot Call(K, T)(1+i )
T
max(0, S
T
K)
as a function of S
T
is nonincreasing. The call option writer
benets from the decrease of the spot price.

The minimum call option writer prot is . The call option


writer position is riskier than his counterpart. A call option
writer can assumed unbounded loses.

The maximum call option writer prot is Call(K, T)(1 + i )


T
.

The prot for the call option writer is positive if


S
T
< K + Call(K, T)(1 + i )
T
.

The prot for the call option writer is negative if


S
T
> K + Call(K, T)(1 + i )
T
.
c 2009. Miguel A. Arcones. All rights reserved. Manual for SOA Exam FM/CAS Exam 2.
21/112
Chapter 7. Derivatives markets. Section 7.4. Call options.
prot
call option holder max(S
T
K, 0) Call(K, T)(1 + i )
T
call option writer max(S
T
K, 0) + Call(K, T)(1 + i )
T
minimum prot maximum prot
call option holder Call(K, T)(1 + i )
T

call option writer Call(K, T)(1 + i )


T
Figure 2 shows the graph of the prot of a call option as a function
of S
T
.
c 2009. Miguel A. Arcones. All rights reserved. Manual for SOA Exam FM/CAS Exam 2.
22/112
Chapter 7. Derivatives markets. Section 7.4. Call options.
T
E

max(S
T
K, 0) C(1 + i )
T
S
T
K
C(1 + i )
T
Prot for the call option holder
T
E
d
d
d
d
d
C(1 + i )
T
max(S
T
K, 0)
S
T
C(1 + i )
T
K
Prot for the call option writer
Figure 2: Prot of a call option
c 2009. Miguel A. Arcones. All rights reserved. Manual for SOA Exam FM/CAS Exam 2.
23/112
Chapter 7. Derivatives markets. Section 7.4. Call options.
If r is the annual interest rate compounded continuously, then the
prot for the call option holder is
max(0, S
T
K) Call(K, T)e
rT
and the prot of the call option writer is
Call(K, T)e
rT
max(0, S
T
K).
c 2009. Miguel A. Arcones. All rights reserved. Manual for SOA Exam FM/CAS Exam 2.
24/112
Chapter 7. Derivatives markets. Section 7.4. Call options.
Example 7
Ethan buys a 35strike call option for XYZ stock for 4.337 per
share. The nominal amount of this call option is 2000 shares. The
expiration date of this option is 18 months. The annual eective
interest rate is 5.5%.
c 2009. Miguel A. Arcones. All rights reserved. Manual for SOA Exam FM/CAS Exam 2.
25/112
Chapter 7. Derivatives markets. Section 7.4. Call options.
Example 7
Ethan buys a 35strike call option for XYZ stock for 4.337 per
share. The nominal amount of this call option is 2000 shares. The
expiration date of this option is 18 months. The annual eective
interest rate is 5.5%.
(i) Calculate Ethans prot function.
c 2009. Miguel A. Arcones. All rights reserved. Manual for SOA Exam FM/CAS Exam 2.
26/112
Chapter 7. Derivatives markets. Section 7.4. Call options.
Example 7
Ethan buys a 35strike call option for XYZ stock for 4.337 per
share. The nominal amount of this call option is 2000 shares. The
expiration date of this option is 18 months. The annual eective
interest rate is 5.5%.
(i) Calculate Ethans prot function.
Solution: (i) Ethans prot function is
(2000)(max(S
T
35, 0) 4.337(1.055)
1.5
)
=(2000) max(S
T
35, 0) 9400.
c 2009. Miguel A. Arcones. All rights reserved. Manual for SOA Exam FM/CAS Exam 2.
27/112
Chapter 7. Derivatives markets. Section 7.4. Call options.
Example 7
Ethan buys a 35strike call option for XYZ stock for 4.337 per
share. The nominal amount of this call option is 2000 shares. The
expiration date of this option is 18 months. The annual eective
interest rate is 5.5%.
(ii) Calculate Ethans prot for the following spot prices at expira-
tion: 25, 30, 35, 40, 45.
c 2009. Miguel A. Arcones. All rights reserved. Manual for SOA Exam FM/CAS Exam 2.
28/112
Chapter 7. Derivatives markets. Section 7.4. Call options.
Example 7
Ethan buys a 35strike call option for XYZ stock for 4.337 per
share. The nominal amount of this call option is 2000 shares. The
expiration date of this option is 18 months. The annual eective
interest rate is 5.5%.
(ii) Calculate Ethans prot for the following spot prices at expira-
tion: 25, 30, 35, 40, 45.
Solution: (ii) Since Ethans prot is (2000) max(S
T
35, 0)9400,
Ethans prot for the considered spot prices is:
if S
T
= 25, prot = (2000) max(25 35, 0) 9400 = 9400,
if S
T
= 30, prot = (2000) max(30 35, 0) 9400 = 9400,
if S
T
= 35, prot = (2000) max(35 35, 0) 9400 = 9400,
if S
T
= 40, prot = (2000) max(40 35, 0) 9400 = 600,
if S
T
= 45, prot = (2000) max(45 35, 0) 9400 = 10600.
c 2009. Miguel A. Arcones. All rights reserved. Manual for SOA Exam FM/CAS Exam 2.
29/112
Chapter 7. Derivatives markets. Section 7.4. Call options.
Example 7
Ethan buys a 35strike call option for XYZ stock for 4.337 per
share. The nominal amount of this call option is 2000 shares. The
expiration date of this option is 18 months. The annual eective
interest rate is 5.5%.
(iii) Calculate Ethans minimum and maximum prots.
c 2009. Miguel A. Arcones. All rights reserved. Manual for SOA Exam FM/CAS Exam 2.
30/112
Chapter 7. Derivatives markets. Section 7.4. Call options.
Example 7
Ethan buys a 35strike call option for XYZ stock for 4.337 per
share. The nominal amount of this call option is 2000 shares. The
expiration date of this option is 18 months. The annual eective
interest rate is 5.5%.
(iii) Calculate Ethans minimum and maximum prots.
Solution: (iii) Since Ethans prot is (2000) max(S
T
35, 0)9400,
Ethans minimum prot is 9400 and Ethans maximum prot is .
c 2009. Miguel A. Arcones. All rights reserved. Manual for SOA Exam FM/CAS Exam 2.
31/112
Chapter 7. Derivatives markets. Section 7.4. Call options.
Example 7
Ethan buys a 35strike call option for XYZ stock for 4.337 per
share. The nominal amount of this call option is 2000 shares. The
expiration date of this option is 18 months. The annual eective
interest rate is 5.5%.
(iv) Find the spot prices at which Ethans prot is positive.
c 2009. Miguel A. Arcones. All rights reserved. Manual for SOA Exam FM/CAS Exam 2.
32/112
Chapter 7. Derivatives markets. Section 7.4. Call options.
Example 7
Ethan buys a 35strike call option for XYZ stock for 4.337 per
share. The nominal amount of this call option is 2000 shares. The
expiration date of this option is 18 months. The annual eective
interest rate is 5.5%.
(iv) Find the spot prices at which Ethans prot is positive.
Solution: (iv) Since Ethans prot is (2000) max(S
T
35, 0)9400,
Ethans prot is positive if (2000) max(S
T
35, 0) 9400 > 0, i.e.
if S
T
> 35 +
9400
2000
= 39.7.
c 2009. Miguel A. Arcones. All rights reserved. Manual for SOA Exam FM/CAS Exam 2.
33/112
Chapter 7. Derivatives markets. Section 7.4. Call options.
Example 7
Ethan buys a 35strike call option for XYZ stock for 4.337 per
share. The nominal amount of this call option is 2000 shares. The
expiration date of this option is 18 months. The annual eective
interest rate is 5.5%.
(v) Calculate the spot price at expiration at which Ethan does not
make or lose money on this contract.
c 2009. Miguel A. Arcones. All rights reserved. Manual for SOA Exam FM/CAS Exam 2.
34/112
Chapter 7. Derivatives markets. Section 7.4. Call options.
Example 7
Ethan buys a 35strike call option for XYZ stock for 4.337 per
share. The nominal amount of this call option is 2000 shares. The
expiration date of this option is 18 months. The annual eective
interest rate is 5.5%.
(v) Calculate the spot price at expiration at which Ethan does not
make or lose money on this contract.
Solution: (v) Since Ethans prot is (2000) max(S
T
35, 0)9400,
Ethan breaks even if (2000)(S
T
35) 9400 = 0, i.e. if S
T
=
35 +
9400
2000
= 39.7.
c 2009. Miguel A. Arcones. All rights reserved. Manual for SOA Exam FM/CAS Exam 2.
35/112
Chapter 7. Derivatives markets. Section 7.4. Call options.
Example 7
Ethan buys a 35strike call option for XYZ stock for 4.337 per
share. The nominal amount of this call option is 2000 shares. The
expiration date of this option is 18 months. The annual eective
interest rate is 5.5%.
(vi) Find the spot price at expiration at which Ethan makes an annual
eective yield of 4.75%.
c 2009. Miguel A. Arcones. All rights reserved. Manual for SOA Exam FM/CAS Exam 2.
36/112
Chapter 7. Derivatives markets. Section 7.4. Call options.
Example 7
Ethan buys a 35strike call option for XYZ stock for 4.337 per
share. The nominal amount of this call option is 2000 shares. The
expiration date of this option is 18 months. The annual eective
interest rate is 5.5%.
(vi) Find the spot price at expiration at which Ethan makes an annual
eective yield of 4.75%.
Solution: (vi) Ethan invests (2000)(4.337) = 8674. If his yield is
4.75%, his payo is
(8674)(1.0475)
18/12
= 9300 = (2000) max(S
T
35, 0)
and
S
T
= 35 +
9300
2000
= 39.65.
c 2009. Miguel A. Arcones. All rights reserved. Manual for SOA Exam FM/CAS Exam 2.
37/112
Chapter 7. Derivatives markets. Section 7.4. Call options.
Example 7
Ethan buys a 35strike call option for XYZ stock for 4.337 per
share. The nominal amount of this call option is 2000 shares. The
expiration date of this option is 18 months. The annual eective
interest rate is 5.5%.
(vii) Find the annual eective rate of return earned by Ethan if the
spot price at expiration is 38.
c 2009. Miguel A. Arcones. All rights reserved. Manual for SOA Exam FM/CAS Exam 2.
38/112
Chapter 7. Derivatives markets. Section 7.4. Call options.
Example 7
Ethan buys a 35strike call option for XYZ stock for 4.337 per
share. The nominal amount of this call option is 2000 shares. The
expiration date of this option is 18 months. The annual eective
interest rate is 5.5%.
(vii) Find the annual eective rate of return earned by Ethan if the
spot price at expiration is 38.
Solution: (vii) Let i be Ethans annual eective rate of re-
turn. Ethan invests (2000)(4.337) = 8674. His payo is
(2000) max(38 35, 0) = 6000. Hence, 8674(1 + i )
1.5
= 6000
and i = 21.78538923%.
c 2009. Miguel A. Arcones. All rights reserved. Manual for SOA Exam FM/CAS Exam 2.
39/112
Chapter 7. Derivatives markets. Section 7.4. Call options.
Example 8
Hannah sells a 35strike call option for XYZ stock for 4.337 per
share. The nominal amount of this call option is 2000 shares. The
expiration date of this option is 18 months. The annual eective
interest rate is 5.5%. Hannah invests the proceeds of the sale in a
zerocoupon bond.
c 2009. Miguel A. Arcones. All rights reserved. Manual for SOA Exam FM/CAS Exam 2.
40/112
Chapter 7. Derivatives markets. Section 7.4. Call options.
Example 8
Hannah sells a 35strike call option for XYZ stock for 4.337 per
share. The nominal amount of this call option is 2000 shares. The
expiration date of this option is 18 months. The annual eective
interest rate is 5.5%. Hannah invests the proceeds of the sale in a
zerocoupon bond.
(i) Calculate Hannahs prot function.
c 2009. Miguel A. Arcones. All rights reserved. Manual for SOA Exam FM/CAS Exam 2.
41/112
Chapter 7. Derivatives markets. Section 7.4. Call options.
Example 8
Hannah sells a 35strike call option for XYZ stock for 4.337 per
share. The nominal amount of this call option is 2000 shares. The
expiration date of this option is 18 months. The annual eective
interest rate is 5.5%. Hannah invests the proceeds of the sale in a
zerocoupon bond.
(i) Calculate Hannahs prot function.
Solution: (i) Hannahs prot is
(2000)(max(S
T
35, 0) 4.337(1.055)
1.5
)
=9400 (2000) max(S
T
35, 0).
c 2009. Miguel A. Arcones. All rights reserved. Manual for SOA Exam FM/CAS Exam 2.
42/112
Chapter 7. Derivatives markets. Section 7.4. Call options.
Example 8
Hannah sells a 35strike call option for XYZ stock for 4.337 per
share. The nominal amount of this call option is 2000 shares. The
expiration date of this option is 18 months. The annual eective
interest rate is 5.5%. Hannah invests the proceeds of the sale in a
zerocoupon bond.
(ii) Calculate Hannahs prot for the following spot prices at expi-
ration: 25, 30, 35, 40, 45.
c 2009. Miguel A. Arcones. All rights reserved. Manual for SOA Exam FM/CAS Exam 2.
43/112
Chapter 7. Derivatives markets. Section 7.4. Call options.
Example 8
Hannah sells a 35strike call option for XYZ stock for 4.337 per
share. The nominal amount of this call option is 2000 shares. The
expiration date of this option is 18 months. The annual eective
interest rate is 5.5%. Hannah invests the proceeds of the sale in a
zerocoupon bond.
(ii) Calculate Hannahs prot for the following spot prices at expi-
ration: 25, 30, 35, 40, 45.
Solution: (ii) Since Hannahs prot is 9400 (2000) max(S
T

35, 0), Hannahs prot for the considered spot prices is:
if S
T
= 25, prot = 9400 (2000) max(25 35, 0) = 9400,
if S
T
= 30, prot = 9400 (2000) max(30 35, 0) = 9400,
if S
T
= 35, prot = 9400 (2000) max(35 35, 0) = 9400,
if S
T
= 40, prot = 9400 (2000) max(40 35, 0) = 600,
if S
T
= 45, prot = 9400 (2000) max(45 35, 0) = 10600.
c 2009. Miguel A. Arcones. All rights reserved. Manual for SOA Exam FM/CAS Exam 2.
44/112
Chapter 7. Derivatives markets. Section 7.4. Call options.
Example 8
Hannah sells a 35strike call option for XYZ stock for 4.337 per
share. The nominal amount of this call option is 2000 shares. The
expiration date of this option is 18 months. The annual eective
interest rate is 5.5%. Hannah invests the proceeds of the sale in a
zerocoupon bond.
(iii) Calculate Hannahs minimum and maximum prots.
c 2009. Miguel A. Arcones. All rights reserved. Manual for SOA Exam FM/CAS Exam 2.
45/112
Chapter 7. Derivatives markets. Section 7.4. Call options.
Example 8
Hannah sells a 35strike call option for XYZ stock for 4.337 per
share. The nominal amount of this call option is 2000 shares. The
expiration date of this option is 18 months. The annual eective
interest rate is 5.5%. Hannah invests the proceeds of the sale in a
zerocoupon bond.
(iii) Calculate Hannahs minimum and maximum prots.
Solution: (iii) Since Hannahs prot is 9400 (2000) max(S
T

35, 0), Hannahs minimum prot is and Hannahs maximum
prot is 9400.
c 2009. Miguel A. Arcones. All rights reserved. Manual for SOA Exam FM/CAS Exam 2.
46/112
Chapter 7. Derivatives markets. Section 7.4. Call options.
Next we consider the pricing of a call option. The prot of a call
option depends on S
T
, which is random. In the case of uncertain
scenarios, an arbitrage portfolio consists of a zero investment
portfolio, which shows nonnegative payos in all scenarios. This
implies that if there exists no arbitrage, the prot function of a
portfolio is either constantly zero, or its minimum is negative and
its maximum positive.
c 2009. Miguel A. Arcones. All rights reserved. Manual for SOA Exam FM/CAS Exam 2.
47/112
Chapter 7. Derivatives markets. Section 7.4. Call options.
Theorem 4
If there exist no arbitrage, then
max(S
0
(1 + i )
T
K, 0) < Call(K, T) < S
0
.
c 2009. Miguel A. Arcones. All rights reserved. Manual for SOA Exam FM/CAS Exam 2.
48/112
Chapter 7. Derivatives markets. Section 7.4. Call options.
Proof: Consider the portfolio consisting of selling a call option and
buying the asset. The prot per unit at expiration is
S
T
max(S
T
K, 0) (S
0
Call(K, T))(1 + i )
T
=S
T
+ K max(S
T
, K) (S
0
Call(K, T))(1 + i )
T
=min(S
T
, K) (S
0
Call(K, T))(1 + i )
T
.
The prot is nondecreasing on S
T
. The minimum of this portfolio
is (S
0
Call(K, T))(1 + i )
T
. The maximum of this portfolio is
K (S
0
Call(K, T))(1 +i )
T
. If there exists no arbitrage and the
prot function is not constant, the minimum prot is negative and
the maximum prot is positive. Hence,
(S
0
Call(K, T))(1 + i )
T
< 0 < K (S
0
Call(K, T))(1 + i )
T
which is equivalent to
S
0
(1 + i )
T
K < Call(K, T) < S
0
.
c 2009. Miguel A. Arcones. All rights reserved. Manual for SOA Exam FM/CAS Exam 2.
49/112
Chapter 7. Derivatives markets. Section 7.4. Call options.
If the bounds in Theorem 4 do not hold, we can make arbitrage.
For example, if the price of the call is bigger than the spot price, we
can make money by buying the asset, selling the call and investing
the proceeds in a zerocoupon bond. At redemption time, we have
the asset which can use to satisfy the requirements of the call.
c 2009. Miguel A. Arcones. All rights reserved. Manual for SOA Exam FM/CAS Exam 2.
50/112
Chapter 7. Derivatives markets. Section 7.4. Call options.
Example 9
Consider an European call option on a stock worth S
0
=32, with
expiration date exactly one year from now, and with strike price
$30. The riskfree annual rate of interest compounded
continuously is r = 5%.
c 2009. Miguel A. Arcones. All rights reserved. Manual for SOA Exam FM/CAS Exam 2.
51/112
Chapter 7. Derivatives markets. Section 7.4. Call options.
Example 9
Consider an European call option on a stock worth S
0
=32, with
expiration date exactly one year from now, and with strike price
$30. The riskfree annual rate of interest compounded
continuously is r = 5%.
(i) If the call is worth $3, nd an arbitrage portfolio.
c 2009. Miguel A. Arcones. All rights reserved. Manual for SOA Exam FM/CAS Exam 2.
52/112
Chapter 7. Derivatives markets. Section 7.4. Call options.
Example 9
Consider an European call option on a stock worth S
0
=32, with
expiration date exactly one year from now, and with strike price
$30. The riskfree annual rate of interest compounded
continuously is r = 5%.
(i) If the call is worth $3, nd an arbitrage portfolio.
Solution: (i) We have that
Call(K, T)S
0
+(1+i )
T
K = 332+e
0.05
30 = 0.463117265 < 0.
We can do arbitrage by buying the call and shorting stock. If the
spot price at expiration is more than 30, we buy the stock using the
call option. If the spot price at expiration is less than 30, we buy
the stock at market price. Any case, we buy stock for min(S
T
, 30).
Hence, the prot is
(32 3)e
0.05
min(S
T
, 30) (32 3)e
0.05
30 = 0.4868617949.
c 2009. Miguel A. Arcones. All rights reserved. Manual for SOA Exam FM/CAS Exam 2.
53/112
Chapter 7. Derivatives markets. Section 7.4. Call options.
Example 9
Consider an European call option on a stock worth S
0
=32, with
expiration date exactly one year from now, and with strike price
$30. The riskfree annual rate of interest compounded
continuously is r = 5%.
(i) If the call is worth $35, nd an arbitrage portfolio.
c 2009. Miguel A. Arcones. All rights reserved. Manual for SOA Exam FM/CAS Exam 2.
54/112
Chapter 7. Derivatives markets. Section 7.4. Call options.
Example 9
Consider an European call option on a stock worth S
0
=32, with
expiration date exactly one year from now, and with strike price
$30. The riskfree annual rate of interest compounded
continuously is r = 5%.
(i) If the call is worth $35, nd an arbitrage portfolio.
Solution: (ii) In this case Call(K, T) > S
0
. We can do arbitrage
by selling the call and buying stock. If the spot price at expiration is
more than 30, we sell the stock to the call option holder. If the spot
price at expiration is less than 30, we sell the stock at the market
price. In any case, we sell stock for min(S
T
, 30). The prot is
(35 32)e
0.05
+ min(S
T
, 30) (35 32)e
0.05
= 3.153813289.
c 2009. Miguel A. Arcones. All rights reserved. Manual for SOA Exam FM/CAS Exam 2.
55/112
Chapter 7. Derivatives markets. Section 7.4. Call options.
A call option is a way to buy stock in the future. A long forward is
another way to buy stock in the future. Buying a call option, you
are guaranteed that the price you pay is not bigger than the strike
price. If you buy a call option, you can buy the asset at expiration
for min(S
T
, K). The baker in the example in Section 7.1, instead
of buying a long forward for F
0,T
, he can buy a call option to hedge
against high wheat prices. Doing this we will be able to buy wheat
at time T for min(S
T
, K). The cost of this investment strategy is
Call(K, T)e
rT
+ min(S
T
, K).
Recall that F
0,T
is the price of a forward contract with delivery in
T years. The prot of a long forward is S
T
F
0,T
. The minimum
prot of a long forward is F
0,T
. The maximum prot of a long
forward is .
c 2009. Miguel A. Arcones. All rights reserved. Manual for SOA Exam FM/CAS Exam 2.
56/112
Chapter 7. Derivatives markets. Section 7.4. Call options.
Example 10
Joseph buys a oneyear long forward for 100 shares of a stock at
$74 per share. Samantha buys a call option of 100 shares of XYZ
stock for $76 per share. The exercise date is one year from now.
The risk free eective annual interest rate is 6%. The premium of
this call is $6.4133 per share.
c 2009. Miguel A. Arcones. All rights reserved. Manual for SOA Exam FM/CAS Exam 2.
57/112
Chapter 7. Derivatives markets. Section 7.4. Call options.
Example 10
Joseph buys a oneyear long forward for 100 shares of a stock at
$74 per share. Samantha buys a call option of 100 shares of XYZ
stock for $76 per share. The exercise date is one year from now.
The risk free eective annual interest rate is 6%. The premium of
this call is $6.4133 per share.
(i) Make a table with Josephs prot and Samanthas prot when
the spot price at expiration is $50, $70, $90 and $110. Compare
these prots.
c 2009. Miguel A. Arcones. All rights reserved. Manual for SOA Exam FM/CAS Exam 2.
58/112
Chapter 7. Derivatives markets. Section 7.4. Call options.
Example 10
Joseph buys a oneyear long forward for 100 shares of a stock at
$74 per share. Samantha buys a call option of 100 shares of XYZ
stock for $76 per share. The exercise date is one year from now.
The risk free eective annual interest rate is 6%. The premium of
this call is $6.4133 per share.
(i) Make a table with Josephs prot and Samanthas prot when
the spot price at expiration is $50, $70, $90 and $110. Compare
these prots.
Solution: (i) Josephs prot is given by the formula
(100)(S
T
74).
Samanthas prot is
100 max(0, S
T
K) 100Call(K, T)(1 + i )
T
= 100 max(0, S
T
76) (100)(6.4133)(1.06)
= 100 max(0, S
T
76) 679.81.
c 2009. Miguel A. Arcones. All rights reserved. Manual for SOA Exam FM/CAS Exam 2.
59/112
Chapter 7. Derivatives markets. Section 7.4. Call options.
Example 10
Joseph buys a oneyear long forward for 100 shares of a stock at
$74 per share. Samantha buys a call option of 100 shares of XYZ
stock for $76 per share. The exercise date is one year from now.
The risk free eective annual interest rate is 6%. The premium of
this call is $6.4133 per share.
(i) Make a table with Josephs prot and Samanthas prot when
the spot price at expiration is $50, $70, $90 and $110. Compare
these prots.
Solution: (i) (continuation)
Josephs prot 2400 400 1600 3600
Samanthas prot 679.81 679.81 720.19 2720.19
Spot Price 50 70 90 110
For high spot prices at expiration, Samanthas prots are smaller
than Johns prots. For low prices, Samanthas losses are smaller
than Josephs losses.
c 2009. Miguel A. Arcones. All rights reserved. Manual for SOA Exam FM/CAS Exam 2.
60/112
Chapter 7. Derivatives markets. Section 7.4. Call options.
Example 10
Joseph buys a oneyear long forward for 100 shares of a stock at
$74 per share. Samantha buys a call option of 100 shares of XYZ
stock for $76 per share. The exercise date is one year from now.
The risk free eective annual interest rate is 6%. The premium of
this call is $6.4133 per share.
(ii) Calculate Josephs prot and Samanthas minimum and maxi-
mum payos.
c 2009. Miguel A. Arcones. All rights reserved. Manual for SOA Exam FM/CAS Exam 2.
61/112
Chapter 7. Derivatives markets. Section 7.4. Call options.
Example 10
Joseph buys a oneyear long forward for 100 shares of a stock at
$74 per share. Samantha buys a call option of 100 shares of XYZ
stock for $76 per share. The exercise date is one year from now.
The risk free eective annual interest rate is 6%. The premium of
this call is $6.4133 per share.
(ii) Calculate Josephs prot and Samanthas minimum and maxi-
mum payos.
Solution: (ii) Josephs minimum prot is 7400. Josephs maxi-
mum prot is . Samanthas minimum prot is 679.81. Saman-
thas maximum prot is .
c 2009. Miguel A. Arcones. All rights reserved. Manual for SOA Exam FM/CAS Exam 2.
62/112
Chapter 7. Derivatives markets. Section 7.4. Call options.
Example 10
Joseph buys a oneyear long forward for 100 shares of a stock at
$74 per share. Samantha buys a call option of 100 shares of XYZ
stock for $76 per share. The exercise date is one year from now.
The risk free eective annual interest rate is 6%. The premium of
this call is $6.4133 per share.
(iii) Which is the minimum spot price at expiration at which Joseph
makes a prot? Which is the minimum spot price at expiration at
which Samantha makes a prot?
c 2009. Miguel A. Arcones. All rights reserved. Manual for SOA Exam FM/CAS Exam 2.
63/112
Chapter 7. Derivatives markets. Section 7.4. Call options.
Example 10
Joseph buys a oneyear long forward for 100 shares of a stock at
$74 per share. Samantha buys a call option of 100 shares of XYZ
stock for $76 per share. The exercise date is one year from now.
The risk free eective annual interest rate is 6%. The premium of
this call is $6.4133 per share.
(iii) Which is the minimum spot price at expiration at which Joseph
makes a prot? Which is the minimum spot price at expiration at
which Samantha makes a prot?
Solution: (iii) Joseph is even if S
T
= 74. Samantha is even if
100(S
T
76)679.81 = 0, i.e. S
T
= 76+(679.81/100) = 82.7981.
c 2009. Miguel A. Arcones. All rights reserved. Manual for SOA Exam FM/CAS Exam 2.
64/112
Chapter 7. Derivatives markets. Section 7.4. Call options.
Example 10
Joseph buys a oneyear long forward for 100 shares of a stock at
$74 per share. Samantha buys a call option of 100 shares of XYZ
stock for $76 per share. The exercise date is one year from now.
The risk free eective annual interest rate is 6%. The premium of
this call is $6.4133 per share.
(iv) Draw the graph of the prot versus the spot price at expiration
for Joseph and Samantha.
c 2009. Miguel A. Arcones. All rights reserved. Manual for SOA Exam FM/CAS Exam 2.
65/112
Chapter 7. Derivatives markets. Section 7.4. Call options.
Example 10
Joseph buys a oneyear long forward for 100 shares of a stock at
$74 per share. Samantha buys a call option of 100 shares of XYZ
stock for $76 per share. The exercise date is one year from now.
The risk free eective annual interest rate is 6%. The premium of
this call is $6.4133 per share.
(iv) Draw the graph of the prot versus the spot price at expiration
for Joseph and Samantha.
Solution: (iv) The graphs of (long forward) Josephs prot and
(purchased call) Samanthas prot are in Figure 3.
c 2009. Miguel A. Arcones. All rights reserved. Manual for SOA Exam FM/CAS Exam 2.
66/112
Chapter 7. Derivatives markets. Section 7.4. Call options.
Example 10
Joseph buys a oneyear long forward for 100 shares of a stock at
$74 per share. Samantha buys a call option of 100 shares of XYZ
stock for $76 per share. The exercise date is one year from now.
The risk free eective annual interest rate is 6%. The premium of
this call is $6.4133 per share.
(v) Find the spot price at redemption at which both prots are equal.
c 2009. Miguel A. Arcones. All rights reserved. Manual for SOA Exam FM/CAS Exam 2.
67/112
Chapter 7. Derivatives markets. Section 7.4. Call options.
Example 10
Joseph buys a oneyear long forward for 100 shares of a stock at
$74 per share. Samantha buys a call option of 100 shares of XYZ
stock for $76 per share. The exercise date is one year from now.
The risk free eective annual interest rate is 6%. The premium of
this call is $6.4133 per share.
(v) Find the spot price at redemption at which both prots are equal.
Solution: (v) We solve (100)(S
T
74) = 100 max(0, S
T
76)
679.81 for S
T
. There is not solution with S
T
76. If S
T
< 76
we have the equation (100)(S
T
74) = 679.81, or S
T
= 74
6.7981 = 67.2019.
c 2009. Miguel A. Arcones. All rights reserved. Manual for SOA Exam FM/CAS Exam 2.
68/112
Chapter 7. Derivatives markets. Section 7.4. Call options.
Figure 3: Example 10. Prot for long forward and purchased call.
c 2009. Miguel A. Arcones. All rights reserved. Manual for SOA Exam FM/CAS Exam 2.
69/112
Chapter 7. Derivatives markets. Section 7.4. Call options.
A purchased call option reduces losses over a long forward. Notice
that in Figure 3 the losses for a long forward holder can be big if
the spot price at redemption is small. A call option is an insured
long position in an asset. In return for not having large losses, the
possible prots in a call option are smaller. The spot price needed
to make money is bigger for a purchased call than for a long
forward. The prot for the call option holder is positive if
S
T
> K + Call(K, T)(1 + i )
T
.
The prot for the long forward is positive if S
T
> F
0,T
. By
Theorem 5,
K + Call(K, T)(1 + i )
T
> F
0,T
.
To make a positive prot, a call option holder needs a bigger
increase on the spot price than a long forward holder.
c 2009. Miguel A. Arcones. All rights reserved. Manual for SOA Exam FM/CAS Exam 2.
70/112
Chapter 7. Derivatives markets. Section 7.4. Call options.
Theorem 5
If there exists no arbitrage, then
(1 + i )
T
max(F
0,T
K, 0) < Call(K, T) < (1 + i )
T
F
0,T
.
c 2009. Miguel A. Arcones. All rights reserved. Manual for SOA Exam FM/CAS Exam 2.
71/112
Chapter 7. Derivatives markets. Section 7.4. Call options.
Proof: Suppose that you enter into a short forward contract and
you buy a call option. Both contracts have the same expiration
time and nominal amount. At expiration, the prot of this strategy
is
F
0,T
S
T
+ max(S
T
K, 0) (1 + i )
T
Call(K, T)
=F
0,T
+ max(K, S
T
) (1 + i )
T
Call(K, T)
=F
0,T
min(K, S
T
) (1 + i )
T
Call(K, T).
This prot function is increasing on S
T
and it not constant. The
minimum prot of this portfolio is
F
0,T
K (1 + i )
T
Call(K, T).
The maximum prot of this portfolio is
F
0,T
(1 + i )
T
Call(K, T).
If there is no arbitrage,
F
0,T
K (1 + i )
T
Call(K, T) < 0 < F
0,T
(1 + i )
T
Call(K, T).
c 2009. Miguel A. Arcones. All rights reserved. Manual for SOA Exam FM/CAS Exam 2.
72/112
Chapter 7. Derivatives markets. Section 7.4. Call options.
Example 11
The current price of a forward contract for 1000 units of an asset
with expiration date two years from now is $120000. The riskfree
annual rate of interest compounded continuously is 5%. The price
of a twoyear 100strike European call option for 1000 units of the
asset is $15000. Find an arbitrage portfolio and its minimum prot.
Solution: Since
e
rT
(F
0,T
K) = e
(2)(0.05)
(120000 (100)(1000))
=18096.74836 > 15000,
the call option is under priced. Consider the portfolio consisting of
buying the call and entering into a short forward. The prot is
1000 max(S
T
100, 0) 15000e
(2)(0.05)
+ 120000 1000S
T
=1000 max(100, S
T
) + 103422.4362
=103422.4362 1000 min(100, S
T
).
The minimum prot is 103422.4362 1000(100) = 3422.4362.
c 2009. Miguel A. Arcones. All rights reserved. Manual for SOA Exam FM/CAS Exam 2.
73/112
Chapter 7. Derivatives markets. Section 7.4. Call options.
Example 11
The current price of a forward contract for 1000 units of an asset
with expiration date two years from now is $120000. The riskfree
annual rate of interest compounded continuously is 5%. The price
of a twoyear 100strike European call option for 1000 units of the
asset is $15000. Find an arbitrage portfolio and its minimum prot.
Solution: Since
e
rT
(F
0,T
K) = e
(2)(0.05)
(120000 (100)(1000))
=18096.74836 > 15000,
the call option is under priced. Consider the portfolio consisting of
buying the call and entering into a short forward. The prot is
1000 max(S
T
100, 0) 15000e
(2)(0.05)
+ 120000 1000S
T
=1000 max(100, S
T
) + 103422.4362
=103422.4362 1000 min(100, S
T
).
The minimum prot is 103422.4362 1000(100) = 3422.4362.
c 2009. Miguel A. Arcones. All rights reserved. Manual for SOA Exam FM/CAS Exam 2.
74/112
Chapter 7. Derivatives markets. Section 7.4. Call options.
Another motive to buy call options is to speculate. Call options
allow betting in the increase of the price of a particular asset for a
small cash outlay. Buying a call option, a speculator achieves
leverage. Call options provide price exposure without having to
pay, hold and warehouse the underlying asset. If a speculator
believes that an asset price is going to increase and it is right, he
can get a much higher yield of return buying a call option than
buying the asset.
c 2009. Miguel A. Arcones. All rights reserved. Manual for SOA Exam FM/CAS Exam 2.
75/112
Chapter 7. Derivatives markets. Section 7.4. Call options.
Example 12
Rachel is a speculator. She anticipates XYZ stock to appreciate
from its current level of $130 per share in four months. Rachel
buys a fourmonth 1000share call option with a strike price of
$150 per share and a premium of $1.8074 per share. Luke is also a
speculator. He also expects XYZ stock to appreciate and buys
XYZ stock at the current market price.
c 2009. Miguel A. Arcones. All rights reserved. Manual for SOA Exam FM/CAS Exam 2.
76/112
Chapter 7. Derivatives markets. Section 7.4. Call options.
Example 12
Rachel is a speculator. She anticipates XYZ stock to appreciate
from its current level of $130 per share in four months. Rachel
buys a fourmonth 1000share call option with a strike price of
$150 per share and a premium of $1.8074 per share. Luke is also a
speculator. He also expects XYZ stock to appreciate and buys
XYZ stock at the current market price.
(i) Find Rachels annual eective rate of return in her investment
for the following spot prices at expiration 130, 150, 160 and 170.
c 2009. Miguel A. Arcones. All rights reserved. Manual for SOA Exam FM/CAS Exam 2.
77/112
Chapter 7. Derivatives markets. Section 7.4. Call options.
Example 12
Rachel is a speculator. She anticipates XYZ stock to appreciate
from its current level of $130 per share in four months. Rachel
buys a fourmonth 1000share call option with a strike price of
$150 per share and a premium of $1.8074 per share. Luke is also a
speculator. He also expects XYZ stock to appreciate and buys
XYZ stock at the current market price.
(i) Find Rachels annual eective rate of return in her investment
for the following spot prices at expiration 130, 150, 160 and 170.
Solution: (i) Rachel invests (1000)(1.8074) = 1807.4. Four months
later, she receives (1000) max(S
T
150, 0).
If S
T
150, Rachel loses all her money and her yield of return
is 100%. If S
T
= 160, Rachel receives (1000)(160 150) =
10000 at expiration. Rachels annual rate of return is

10000
1807.4

1 = 168.3702647 = 16837.02647%. If S
T
= 170, Rachel receives
(1000)(170 150) = 20000 at expiration. Rachels annual rate of
return is

20000
1807.4

3
1 = 1353.962117 = 135396.2117%.
c 2009. Miguel A. Arcones. All rights reserved. Manual for SOA Exam FM/CAS Exam 2.
78/112
Chapter 7. Derivatives markets. Section 7.4. Call options.
Example 12
Rachel is a speculator. She anticipates XYZ stock to appreciate
from its current level of $130 per share in four months. Rachel
buys a fourmonth 1000share call option with a strike price of
$150 per share and a premium of $1.8074 per share. Luke is also a
speculator. He also expects XYZ stock to appreciate and buys
XYZ stock at the current market price.
(ii) Luke sells his stock at the end of four months. Find Lukes
annual eective rate of return in his investment for the spot prices
in (i).
c 2009. Miguel A. Arcones. All rights reserved. Manual for SOA Exam FM/CAS Exam 2.
79/112
Chapter 7. Derivatives markets. Section 7.4. Call options.
Example 12
Rachel is a speculator. She anticipates XYZ stock to appreciate
from its current level of $130 per share in four months. Rachel
buys a fourmonth 1000share call option with a strike price of
$150 per share and a premium of $1.8074 per share. Luke is also a
speculator. He also expects XYZ stock to appreciate and buys
XYZ stock at the current market price.
(ii) Luke sells his stock at the end of four months. Find Lukes
annual eective rate of return in his investment for the spot prices
in (i).
Solution: (ii) Luke invests 130 per share. His annual rate of return
j satises S
T
= 130(1 + j )
1/3
. So, j =

S
T
130

3
1.
If S
T
= 130, j = 0%.
If S
T
= 150, j = 53.61857078%.
If S
T
= 160, j = 86.43604916%.
If S
T
= 170, j = 123.6231224%.
c 2009. Miguel A. Arcones. All rights reserved. Manual for SOA Exam FM/CAS Exam 2.
80/112
Chapter 7. Derivatives markets. Section 7.4. Call options.
Example 12
Rachel is a speculator. She anticipates XYZ stock to appreciate
from its current level of $130 per share in four months. Rachel
buys a fourmonth 1000share call option with a strike price of
$150 per share and a premium of $1.8074 per share. Luke is also a
speculator. He also expects XYZ stock to appreciate and buys
XYZ stock at the current market price.
(iii) Compare the rates in (i) and (ii).
c 2009. Miguel A. Arcones. All rights reserved. Manual for SOA Exam FM/CAS Exam 2.
81/112
Chapter 7. Derivatives markets. Section 7.4. Call options.
Example 12
Rachel is a speculator. She anticipates XYZ stock to appreciate
from its current level of $130 per share in four months. Rachel
buys a fourmonth 1000share call option with a strike price of
$150 per share and a premium of $1.8074 per share. Luke is also a
speculator. He also expects XYZ stock to appreciate and buys
XYZ stock at the current market price.
(iii) Compare the rates in (i) and (ii).
Solution: (iii) In the case that XYZ stock does not appreciate,
Rachel loses all her money. But in the cases where XYZ stock
appreciates, Rachel makes a much higher yield than Luke.
c 2009. Miguel A. Arcones. All rights reserved. Manual for SOA Exam FM/CAS Exam 2.
82/112
Chapter 7. Derivatives markets. Section 7.4. Call options.
Let j
C
be the rate of return which an investor makes buying a call
option. Since the payo per share is max(S
T
K, 0), we have that
max(S
T
K, 0)
Call(K, T)
= (1 + j
C
)
T
.
Let j
B
be the rate of return which an investor makes buying an
asset and holding it for T years. Since the payo per share is S
T
,
we have that
S
T
= (1 + j
B
)
T
S
0
.
We have that j
C
> j
B
if
max(S
T
K, 0)
Call(K, T)
>
S
T
S
0
,
which is equivalent to S
0
> Call(K, T) and
S
T
>
K
Call(K,T)
1
Call(K,T)

1
S
0
=
KS
0
S
0
Call(K, T)
.
We conclude that if S
T
is large enough, investing in an option call
gives a larger yield than buying an asset.
c 2009. Miguel A. Arcones. All rights reserved. Manual for SOA Exam FM/CAS Exam 2.
83/112
Chapter 7. Derivatives markets. Section 7.4. Call options.
Next we consider call options with dierent strike prices. If
0 < K
1
< K
2
, then
max(S
T
K
2
, 0) max(S
T
K
1
, 0),
i.e. the payo of a K
1
strike call option is higher than the payo
of a K
2
strike call option (see Figure 4). Hence, the price of the
call is bigger for the call with smaller strike price (see Theorem 6).
c 2009. Miguel A. Arcones. All rights reserved. Manual for SOA Exam FM/CAS Exam 2.
84/112
Chapter 7. Derivatives markets. Section 7.4. Call options.
Example 13
The current price of XYZ stock is $75 per share. The annual
eective rate of interest is 5%. The redemption time is one year
from now. Draw the payo and prot diagrams for the buyer of:
c 2009. Miguel A. Arcones. All rights reserved. Manual for SOA Exam FM/CAS Exam 2.
85/112
Chapter 7. Derivatives markets. Section 7.4. Call options.
Example 13
The current price of XYZ stock is $75 per share. The annual
eective rate of interest is 5%. The redemption time is one year
from now. Draw the payo and prot diagrams for the buyer of:
(i) a $70 strike call option with a premium of $10.755.
c 2009. Miguel A. Arcones. All rights reserved. Manual for SOA Exam FM/CAS Exam 2.
86/112
Chapter 7. Derivatives markets. Section 7.4. Call options.
Example 13
The current price of XYZ stock is $75 per share. The annual
eective rate of interest is 5%. The redemption time is one year
from now. Draw the payo and prot diagrams for the buyer of:
(i) a $70 strike call option with a premium of $10.755.
Solution: (i) The payo is max(S
T
70, 0). The diagram of this
payo is in Figure 4. The prot is
max(S
T
70, 0) (10.755)(1.05) = max(S
T
70, 0) 11.29275.
The diagram of this prot is in Figure 5.
c 2009. Miguel A. Arcones. All rights reserved. Manual for SOA Exam FM/CAS Exam 2.
87/112
Chapter 7. Derivatives markets. Section 7.4. Call options.
Example 13
The current price of XYZ stock is $75 per share. The annual
eective rate of interest is 5%. The redemption time is one year
from now. Draw the payo and prot diagrams for the buyer of:
(ii) a $80 strike call option with a premium of $5.445.
c 2009. Miguel A. Arcones. All rights reserved. Manual for SOA Exam FM/CAS Exam 2.
88/112
Chapter 7. Derivatives markets. Section 7.4. Call options.
Example 13
The current price of XYZ stock is $75 per share. The annual
eective rate of interest is 5%. The redemption time is one year
from now. Draw the payo and prot diagrams for the buyer of:
(ii) a $80 strike call option with a premium of $5.445.
Solution: (ii) The payo is max(S
T
80, 0). The diagram of this
payo is in Figure 4. The prot is
max(S
T
80, 0) (5.445)(1.05) = max(S
T
80, 0) 5.71725.
The diagram of this prot is in Figure 5.
c 2009. Miguel A. Arcones. All rights reserved. Manual for SOA Exam FM/CAS Exam 2.
89/112
Chapter 7. Derivatives markets. Section 7.4. Call options.
Example 13
The current price of XYZ stock is $75 per share. The annual
eective rate of interest is 5%. The redemption time is one year
from now. Draw the payo and prot diagrams for the buyer of:
(iii) Find the spot price at redemption at which both prots are
equal.
c 2009. Miguel A. Arcones. All rights reserved. Manual for SOA Exam FM/CAS Exam 2.
90/112
Chapter 7. Derivatives markets. Section 7.4. Call options.
Example 13
The current price of XYZ stock is $75 per share. The annual
eective rate of interest is 5%. The redemption time is one year
from now. Draw the payo and prot diagrams for the buyer of:
(iii) Find the spot price at redemption at which both prots are
equal.
Solution: (iii) The prot amounts are equal for some S
T
(70, 80).
So,
S
T
70 11.29275 = max(S
T
70, 0) 11.29275
=max(S
T
80, 0) 5.71725 = 5.71725
and S
T
= 70 + 11.29275 5.71725 = 75.5755.
c 2009. Miguel A. Arcones. All rights reserved. Manual for SOA Exam FM/CAS Exam 2.
91/112
Chapter 7. Derivatives markets. Section 7.4. Call options.
Figure 4: Example 13. Payo for two calls with dierent strikes.
c 2009. Miguel A. Arcones. All rights reserved. Manual for SOA Exam FM/CAS Exam 2.
92/112
Chapter 7. Derivatives markets. Section 7.4. Call options.
Figure 5: Example 13. Prot for two calls with dierent strikes.
c 2009. Miguel A. Arcones. All rights reserved. Manual for SOA Exam FM/CAS Exam 2.
93/112
Chapter 7. Derivatives markets. Section 7.4. Call options.
Theorem 6
If 0 < K
1
< K
2
, then
Call(K
2
, T) Call(K
1
, T) Call(K
2
, T) + (K
2
K
1
)e
rT
.
c 2009. Miguel A. Arcones. All rights reserved. Manual for SOA Exam FM/CAS Exam 2.
94/112
Chapter 7. Derivatives markets. Section 7.4. Call options.
Proof.
We have that
max(S
T
K
2
, 0)
max(S
T
K
1
, 0) = K
2
K
1
+ max(S
T
K
2
, K
1
K
2
)
K
2
K
1
+ max(S
T
K
2
, 0).
In other words,
(i) The payo for a K
2
strike call is smaller than the payo for a
K
1
strike call.
(ii) The payo for a K
1
strike call is smaller than K
2
K
1
plus the
payo for a K
2
strike call.
Hence, if there exist no arbitrage, then
Call(K
2
, T) Call(K
1
, T) Call(K
2
, T) + (K
2
K
1
)e
rT
.
c 2009. Miguel A. Arcones. All rights reserved. Manual for SOA Exam FM/CAS Exam 2.
95/112
Chapter 7. Derivatives markets. Section 7.4. Call options.
Example 14
Consider two European call options on a stock worth S
0
=32, both
with expiration date exactly two years from now and the same
nominal amount. The riskfree annual rate of interest
compounded continuously is 5%. One call option has strike price
$30 and the other one $35. The price of the 30strike call is 7.
c 2009. Miguel A. Arcones. All rights reserved. Manual for SOA Exam FM/CAS Exam 2.
96/112
Chapter 7. Derivatives markets. Section 7.4. Call options.
Example 14
Consider two European call options on a stock worth S
0
=32, both
with expiration date exactly two years from now and the same
nominal amount. The riskfree annual rate of interest
compounded continuously is 5%. One call option has strike price
$30 and the other one $35. The price of the 30strike call is 7.
(i) Suppose that the price of the 35strike call option is 8, nd an
arbitrage portfolio.
c 2009. Miguel A. Arcones. All rights reserved. Manual for SOA Exam FM/CAS Exam 2.
97/112
Chapter 7. Derivatives markets. Section 7.4. Call options.
Example 14
Consider two European call options on a stock worth S
0
=32, both
with expiration date exactly two years from now and the same
nominal amount. The riskfree annual rate of interest
compounded continuously is 5%. One call option has strike price
$30 and the other one $35. The price of the 30strike call is 7.
(i) Suppose that the price of the 35strike call option is 8, nd an
arbitrage portfolio.
Solution: (i) Here, Call(35, T) Call(30, T) does not hold. We
can do arbitrage by a buying a 30strike call option and selling a
35strike call option, both for the same nominal amount. The prot
per share is
max(S
T
30, 0) max(S
T
35, 0) + (8 7)e
0.05
(8 7)e
0.05
= 1.051271096.
c 2009. Miguel A. Arcones. All rights reserved. Manual for SOA Exam FM/CAS Exam 2.
98/112
Chapter 7. Derivatives markets. Section 7.4. Call options.
Example 14
Consider two European call options on a stock worth S
0
=32, both
with expiration date exactly two years from now and the same
nominal amount. The riskfree annual rate of interest
compounded continuously is 5%. One call option has strike price
$30 and the other one $35. The price of the 30strike call is 7.
(ii) Suppose that the price of the 35strike call option is 1, nd an
arbitrage portfolio.
c 2009. Miguel A. Arcones. All rights reserved. Manual for SOA Exam FM/CAS Exam 2.
99/112
Chapter 7. Derivatives markets. Section 7.4. Call options.
Example 14
Consider two European call options on a stock worth S
0
=32, both
with expiration date exactly two years from now and the same
nominal amount. The riskfree annual rate of interest
compounded continuously is 5%. One call option has strike price
$30 and the other one $35. The price of the 30strike call is 7.
(ii) Suppose that the price of the 35strike call option is 1, nd an
arbitrage portfolio.
Solution: (ii) We have that
Call(K
2
, T) Call(K
1
, T) + (K
2
K
1
)e
rT
=1 7 + (35 30)e
0.05
= 1.243852877 < 0.
We can do arbitrage by buying a 35strike call option and selling a
30strike call option.
c 2009. Miguel A. Arcones. All rights reserved. Manual for SOA Exam FM/CAS Exam 2.
100/112
Chapter 7. Derivatives markets. Section 7.4. Call options.
Example 14
Consider two European call options on a stock worth S
0
=32, both
with expiration date exactly two years from now and the same
nominal amount. The riskfree annual rate of interest
compounded continuously is 5%. One call option has strike price
$30 and the other one $35. The price of the 30strike call is 7.
(ii) Suppose that the price of the 35strike call option is 1, nd an
arbitrage portfolio.
Solution: (ii) (continuation) We can do arbitrage by buying a 35
strike call option and selling a 30strike call option. The prot per
share is
max(S
T
35, 0) max(S
T
30, 0) + (7 1)e
0.05
=5 + max(S
T
30, 5) max(S
T
30, 0) + (7 1)e
0.05
5 + (7 1)e
0.05
= 5 + 6.307626578 = 1.307626578.
c 2009. Miguel A. Arcones. All rights reserved. Manual for SOA Exam FM/CAS Exam 2.
101/112
Chapter 7. Derivatives markets. Section 7.4. Call options.
The premium of a call option of an asset depends on several
factors, like asset price, interest rate, expiration time, strike price,
and asset price variability. We have the following rules of thump
for the price of a call:

Higher asset prices lead to higher call option prices.

Higher strike prices lead to lower call option prices.

Higher interest rates lead to higher call option prices.

Higher expiration time leads to higher call option prices.

Higher variation of an asset price leads to higher call option


prices.
c 2009. Miguel A. Arcones. All rights reserved. Manual for SOA Exam FM/CAS Exam 2.
102/112
Chapter 7. Derivatives markets. Section 7.4. Call options.
Since the call option buyers payo decreases as the strike
increases, the (price) premium of a call option decrease as the
strikes increases. Hence, between two call options with dierent
strike prices:
(i) The call option with smaller strike price has a bigger premium.
(ii) If the spot price is low enough, both call options substain a loss.
The loss is bigger for the call option with the smaller strike price.
(iii) If the spot price is high enough, both call options have a
positive prot. The prot is bigger for the call option with the
smaller strike price.
We can check the previous assertions analytically using Theorem 6.
c 2009. Miguel A. Arcones. All rights reserved. Manual for SOA Exam FM/CAS Exam 2.
103/112
Chapter 7. Derivatives markets. Section 7.4. Call options.
The strike price is paid at the expiration time, as higher the
interest rate is as higher the call option premium is. As higher the
expiration time as higher the call option premium is. The greater
the past variability of the price of an asset is as more likely is that
the option will be exercised. So, higher variation of an asset price
leads to higher call option prices.
c 2009. Miguel A. Arcones. All rights reserved. Manual for SOA Exam FM/CAS Exam 2.
104/112
Chapter 7. Derivatives markets. Section 7.4. Call options.
The common method to nd the price of a call option of a stock is
to use the BlackScholes formula
1
Call(K, T) = S
0
e
T
(d
1
) Ke
rT
(d
2
)
where
d
1
=
log(S
0
/K) + (r +
2
/2)T

T
;
d
2
= d
1

T;
S
0
is the current price of the stock; K is the strike price; r is the
risk free continuously compounded annual interest rate; is the
continuous rate of dividend payments; T is the expiration time in
years of the option; is the implied volatility for the underlying
asset and the cumulative distribution function of a standard
normal distribution.
1
In 1973, Fischer Black and Myron Scholes published a paper presenting the
pricing formula for call and put options.
c 2009. Miguel A. Arcones. All rights reserved. Manual for SOA Exam FM/CAS Exam 2.
105/112
Chapter 7. Derivatives markets. Section 7.4. Call options.
Table 1 shows the premium of a call option for dierent strike
prices. We have used S
0
= 75, T = 1, = 0.20, = 0,
r = ln(1.05).
Table 1:
K 65 70 75 80 85
Call(K, T) 14.31722 10.75552 7.78971 5.444947 3.680736
c 2009. Miguel A. Arcones. All rights reserved. Manual for SOA Exam FM/CAS Exam 2.
106/112
Chapter 7. Derivatives markets. Section 7.4. Call options.
Example 15
The current price of XYZ stock is $75 per share. The annual
eective rate of interest is 5%. The redemption time is one year
from now. The price of stock one year from now is $73.5.
Calculate the prot per share at expiration for the holder of each
one of the call options in Table 1.
c 2009. Miguel A. Arcones. All rights reserved. Manual for SOA Exam FM/CAS Exam 2.
107/112
Chapter 7. Derivatives markets. Section 7.4. Call options.
Solution: The prot is
max(S
T
K, 0) (1.05)Call(K, T)
=max(73.5 K, 0) (1.05)Call(K, T).
The corresponding prots are:
if K = 65, max(73.5 65, 0) (1.05)(14.31722) = 6.533081,
if K = 70, max(73.5 70, 0) (1.05)(10.75552) = 7.793296,
if K = 75, max(73.5 75, 0) (1.05)(7.78971) = 8.1791955,
if K = 80, max(73.5 80, 0) (1.05)(5.444947) = 5.71719435,
if K = 85, max(73.5 85, 0) (1.05)(3.680736) = 3.8647728.
c 2009. Miguel A. Arcones. All rights reserved. Manual for SOA Exam FM/CAS Exam 2.
108/112
Chapter 7. Derivatives markets. Section 7.4. Call options.
If K is very small, the call option will almost certainly be executed.
Hence, if K is very small, Call(K, T) = F
0,T
, i.e.
lim
K0+
Call(K, T) = F
0,T
. If K is very large, the call option will
almost certainly not be executed. Hence, lim
K
Call(K, T) = 0. As
a function on K, Call(K, T) is a decreasing function with
lim
K0+
Call(K, T) = F
0,T
and lim
K
Call(K, T) = 0. Figure 6
shows the graph of Call(K, T) as a function of T.
c 2009. Miguel A. Arcones. All rights reserved. Manual for SOA Exam FM/CAS Exam 2.
109/112
Chapter 7. Derivatives markets. Section 7.4. Call options.
Figure 6: Example 16. Graph of Call(K, T) as a function of K.
c 2009. Miguel A. Arcones. All rights reserved. Manual for SOA Exam FM/CAS Exam 2.
110/112
Chapter 7. Derivatives markets. Section 7.4. Call options.
Example 16
Using the BlackScholes formula with T = 1, S
0
= 100, T = 1,
= 0.25, r = ln(1.06) and = 0.0, the following table of call
option premiums was obtained:
Call(K, T) 76.4150 52.8366 30.0399 12.7562 4.1341 0.8417 0.2672 0.0605
K 25 50 75 100 125 150 175 200
Figure 6 shows the graph of this function.
When we consider Call(K, T) as function of T. If T is small
enough, then the option will be exercised if S
0
> K with a prot of
S
0
K. Hence, if S
0
> K, lim
T0+
Call(K, T) = S
0
K. Notice
that by buying the call option for Call(K, T), we buy an asset
worth S
0
for K. If T is small enough and S
0
< K, the option is
not exercised and his value is zero, i.e. lim
T0+
Call(K, T) = 0.
c 2009. Miguel A. Arcones. All rights reserved. Manual for SOA Exam FM/CAS Exam 2.
111/112
Chapter 7. Derivatives markets. Section 7.4. Call options.
An option is inthemoney option if it would have a positive
payo if exercised immediately. An option is outthemoney
option if it would have a negative payo if exercised immediately.
An option is atthemoney option if it would have a zero payo if
exercised immediately. The previous denition hold for both call
and put options. Put options will considered shortly. For a
purchased call option, we have
c 2009. Miguel A. Arcones. All rights reserved. Manual for SOA Exam FM/CAS Exam 2.
112/112
Chapter 7. Derivatives markets. Section 7.4. Call options.

The purchased call option is inthemoney, if S


0
> K.

The purchased call option is outthemoney, if S


0
< K.

The purchased call option is atthemoney, if S


0
= K.
c 2009. Miguel A. Arcones. All rights reserved. Manual for SOA Exam FM/CAS Exam 2.
1/83
Chapter 7. Derivatives markets.
Manual for SOA Exam FM/CAS Exam 2.
Chapter 7. Derivatives markets.
Section 7.5. Put options.
c 2009. Miguel A. Arcones. All rights reserved.
Extract from:
Arcones Manual for the SOA Exam FM/CAS Exam 2,
Financial Mathematics. Fall 2009 Edition,
available at http://www.actexmadriver.com/
c 2009. Miguel A. Arcones. All rights reserved. Manual for SOA Exam FM/CAS Exam 2.
2/83
Chapter 7. Derivatives markets. Section 7.5. Put options.
Put options
Denition 1
A put option is a nancial contract which gives the (holder)
owner the right, but not the obligation, to sell a specied amount
of a given security at a specied price at a specied time.
The put option owner exercises the option by selling the asset at
the specied call price to the put writer. A put option is executed
only if the put owner decides to do so. A put option owner
executes a put option only when it benets him, i.e. when the
specied call price is bigger than the current (market value) spot
price. Since the owner of a put option can make money if the
option is exercised, put options are sold. The owner of the put
option must pay to its counterpart for holding a put option. The
price of a put option is called its premium.
c 2009. Miguel A. Arcones. All rights reserved. Manual for SOA Exam FM/CAS Exam 2.
3/83
Chapter 7. Derivatives markets. Section 7.5. Put options.
Put options
Denition 1
A put option is a nancial contract which gives the (holder)
owner the right, but not the obligation, to sell a specied amount
of a given security at a specied price at a specied time.
The put option owner exercises the option by selling the asset at
the specied call price to the put writer. A put option is executed
only if the put owner decides to do so. A put option owner
executes a put option only when it benets him, i.e. when the
specied call price is bigger than the current (market value) spot
price. Since the owner of a put option can make money if the
option is exercised, put options are sold. The owner of the put
option must pay to its counterpart for holding a put option. The
price of a put option is called its premium.
c 2009. Miguel A. Arcones. All rights reserved. Manual for SOA Exam FM/CAS Exam 2.
4/83
Chapter 7. Derivatives markets. Section 7.5. Put options.

The (owner) buyer of a put option is called the option put


holder. The holder of a put option is said to have a long put
position.

The seller of a put option is called the option put writer.


The writer of a put is said to have a short put position.

Assets used in put options are in commodities, currency


exchange, stock shares and stock indices.

A put option needs to specify the type and quality of the


underlying.

The asset used in the put option is called the underlier or


underlying asset.

The amount of the underlying asset to which the put option


applies is called the notional amount.

The specied price of an asset in a put option is called the


strike price, or exercise price.

A forward contract forces the buyer and seller to execute the


sale. A put option is executed only if the put holder decides to
do so.
c 2009. Miguel A. Arcones. All rights reserved. Manual for SOA Exam FM/CAS Exam 2.
5/83
Chapter 7. Derivatives markets. Section 7.5. Put options.

For an European option, the exercise of the option must


occur at a certain time (the expiration date).

For an American option, the exercise of the option must


occur any time by the expiration date.

For a Bermudan option, the buyer can exercise the call


option during specied periods.
Unless say otherwise, we will assume that an option is an European
option. European options are simpler and easier to study.
c 2009. Miguel A. Arcones. All rights reserved. Manual for SOA Exam FM/CAS Exam 2.
6/83
Chapter 7. Derivatives markets. Section 7.5. Put options.
Example 1
John buys a sixmonth put option for 150 shares with a strike price
of $45 per share.
(i) If the price per share six months from now is $40, John sells
150 shares to the put option writer for (150)(45) = 6750. Since
the market value of these 150 shares is (150)(40) = 6000. John
makes (before expenses) 6750 6000 = 750 on this contract.
(ii) If the price per share six months from now is $50, John does
not exercise the put option.
c 2009. Miguel A. Arcones. All rights reserved. Manual for SOA Exam FM/CAS Exam 2.
7/83
Chapter 7. Derivatives markets. Section 7.5. Put options.

The put option buyers payo per share is


max(K S
T
, 0) =

K S
T
if S
T
< K,
0 if S
T
K,
where K is the strike price and S
T
is the spot price at
redemption.

The put option writers payo per share is


max(K S
T
, 0) =

(K S
T
) if S
T
< K,
0 if S
T
K,
Figure 1 shows the graph of the payo of a put option. A put
option is a zerosum game.
c 2009. Miguel A. Arcones. All rights reserved. Manual for SOA Exam FM/CAS Exam 2.
8/83
Chapter 7. Derivatives markets. Section 7.5. Put options.
T
E
d
d
d
d
max(0, K S
T
)
S
T
K
Payo for the put option holder
T
E

max(0, K S
T
)
S
T
K
Payo for the put option writer
Figure 1: Payos of a put option
c 2009. Miguel A. Arcones. All rights reserved. Manual for SOA Exam FM/CAS Exam 2.
9/83
Chapter 7. Derivatives markets. Section 7.5. Put options.
Recall:

The put option holders payo is


max(0, K S
T
).

The put option writers payo is


max(0, K S
T
).
We get from Figure 1 that:

The minimum payo for the put option holder is 0. The


maximum payo for the put option holder is K.

The minimum payo for the put option writer is K. The


maximum payo for the put option writer is 0.
minimum payo maximum payo
put option holder 0 K
put option writer K 0
c 2009. Miguel A. Arcones. All rights reserved. Manual for SOA Exam FM/CAS Exam 2.
10/83
Chapter 7. Derivatives markets. Section 7.5. Put options.
Example 2
Daniel buys a 55strike put option on XYZ stock with a nominal
amount of 5000 shares. The expiration date is 6 months from now.
The nominal amount of the put option is 5000 shares of XYZ
stock.
(i) Calculate Daniels payo for the following spot prices per share
at expiration: 40, 45, 55, 60, 60.
(ii) Calculate Daniels minimum and maximum payos.
Solution: (i) Daniels payo is 5000 max(55 S
T
, 0). The
corresponding payos are:
if S
T
= 40, payo = (5000) max(55 40, 0) = 75000,
if S
T
= 45, payo = (5000) max(55 45, 0) = 50000,
if S
T
= 50, payo = (5000) max(55 50, 0) = 25000,
if S
T
= 55, payo = (5000) max(55 55, 0) = 0,
if S
T
= 60, payo = (5000) max(55 60, 0) = 0.
(ii) Daniels minimum payo is zero. Daniels maximum payo is
(5000)(55) = 275000.
c 2009. Miguel A. Arcones. All rights reserved. Manual for SOA Exam FM/CAS Exam 2.
11/83
Chapter 7. Derivatives markets. Section 7.5. Put options.
Example 2
Daniel buys a 55strike put option on XYZ stock with a nominal
amount of 5000 shares. The expiration date is 6 months from now.
The nominal amount of the put option is 5000 shares of XYZ
stock.
(i) Calculate Daniels payo for the following spot prices per share
at expiration: 40, 45, 55, 60, 60.
(ii) Calculate Daniels minimum and maximum payos.
Solution: (i) Daniels payo is 5000 max(55 S
T
, 0). The
corresponding payos are:
if S
T
= 40, payo = (5000) max(55 40, 0) = 75000,
if S
T
= 45, payo = (5000) max(55 45, 0) = 50000,
if S
T
= 50, payo = (5000) max(55 50, 0) = 25000,
if S
T
= 55, payo = (5000) max(55 55, 0) = 0,
if S
T
= 60, payo = (5000) max(55 60, 0) = 0.
(ii) Daniels minimum payo is zero. Daniels maximum payo is
(5000)(55) = 275000.
c 2009. Miguel A. Arcones. All rights reserved. Manual for SOA Exam FM/CAS Exam 2.
12/83
Chapter 7. Derivatives markets. Section 7.5. Put options.
Example 3
Isabella sells a 55strike put option on XYZ stock. The expiration
date is 18 months from now. The nominal amount of the put
option is 10000 shares of XYZ stock.
(i) Calculate Isabellas payo for the following spot prices per share
at expiration: 40, 45, 55, 60, 60.
(ii) Calculate Isabellas minimum and maximum payos.
Solution: (i) Isabellas payo is 10000 max(55 S
T
, 0). The
corresponding payos are:
if S
T
= 40, payo = (10000) max(55 40, 0) = 150000,
if S
T
= 45, payo = (10000) max(55 45, 0) = 100000,
if S
T
= 50, payo = (10000) max(55 50, 0) = 50000,
if S
T
= 55, payo = (10000) max(55 55, 0) = 0,
if S
T
= 60, payo = (10000) max(55 60, 0) = 0.
(ii) Isabellas minimum payo is (10000)(55) = 550000.
Isabellas maximum payo is zero.
c 2009. Miguel A. Arcones. All rights reserved. Manual for SOA Exam FM/CAS Exam 2.
13/83
Chapter 7. Derivatives markets. Section 7.5. Put options.
Example 3
Isabella sells a 55strike put option on XYZ stock. The expiration
date is 18 months from now. The nominal amount of the put
option is 10000 shares of XYZ stock.
(i) Calculate Isabellas payo for the following spot prices per share
at expiration: 40, 45, 55, 60, 60.
(ii) Calculate Isabellas minimum and maximum payos.
Solution: (i) Isabellas payo is 10000 max(55 S
T
, 0). The
corresponding payos are:
if S
T
= 40, payo = (10000) max(55 40, 0) = 150000,
if S
T
= 45, payo = (10000) max(55 45, 0) = 100000,
if S
T
= 50, payo = (10000) max(55 50, 0) = 50000,
if S
T
= 55, payo = (10000) max(55 55, 0) = 0,
if S
T
= 60, payo = (10000) max(55 60, 0) = 0.
(ii) Isabellas minimum payo is (10000)(55) = 550000.
Isabellas maximum payo is zero.
c 2009. Miguel A. Arcones. All rights reserved. Manual for SOA Exam FM/CAS Exam 2.
14/83
Chapter 7. Derivatives markets. Section 7.5. Put options.
Let Put(K, T) be the premium per unit paid of a put option with
strike price K and expiration time T years. Notice that
Put(K, T) > 0. Let i be the risk free annual eective rate of
interest. The put option holders prot is
max(K S
T
, 0) Put(K, T)(1 + i )
T
=

K S
T
Put(K, T)(1 + i )
T
if S
T
< K,
Put(K, T)(1 + i )
T
if S
T
K.
Put(K, T)(1 + i )
T
is the future value at time T of the purchase
price. The put option writers prot is
max(K S
T
, 0) + Put(K, T)(1 + i )
T
=

K + S
T
+ Put(K, T)(1 + i )
T
if S
T
< K,
Put(K, T)(1 + i )
T
if S
T
K.
Figure 2 shows a graph of the put prot.
c 2009. Miguel A. Arcones. All rights reserved. Manual for SOA Exam FM/CAS Exam 2.
15/83
Chapter 7. Derivatives markets. Section 7.5. Put options.
prot
put holder max(K S
T
, 0) Put(K, T)e
rT
put writer max(K S
T
, 0) + Put(K, T)e
rT
minimum prot maximum prot
put holder Put(K, T)e
rT
K Put(K, T)e
rT
put writer K + Put(K, T)e
rT
Put(K, T)e
rT
Figure 2 shows a graph of the put prot as a function of S
T
.
c 2009. Miguel A. Arcones. All rights reserved. Manual for SOA Exam FM/CAS Exam 2.
16/83
Chapter 7. Derivatives markets. Section 7.5. Put options.
T
E
d
d
d
d
K
max(K S
T
, 0) P(1 + i )
T
S
T
K P(1 + i )
T
P(1 + i )
T
Prot for the put option holder
T
E

K
P(1 + i )
T
max(K S
T
, 0)
S
T
P(1 + i )
T
K
P(1 + i )
T
Prot for the put option writer
Figure 2: Prot of a put option
c 2009. Miguel A. Arcones. All rights reserved. Manual for SOA Exam FM/CAS Exam 2.
17/83
Chapter 7. Derivatives markets. Section 7.5. Put options.
Notice that the put option holders prot as a function of S
T
is
nonincreasing. The put option holder benets from a decrease on
the spot price. The minimum of the put option holders prot is
Put(K, T)(1 + i )
T
.
The maximum of the put options holder prot is
K Put(K, T)(1 + i )
T
. If there exists no arbitrage
Put(K, T)(1 + i )
T
< 0 < K Put(K, T)(1 + i )
T
,
which is equivalent to
0 < Put(K, T) < K(1 + i )
T
.
c 2009. Miguel A. Arcones. All rights reserved. Manual for SOA Exam FM/CAS Exam 2.
18/83
Chapter 7. Derivatives markets. Section 7.5. Put options.
Theorem 1
If there exists no arbitrage, then
max((1 + i )
T
K S
0
, 0) < Put(K, T) < K(1 + i )
T
.
Proof.
Consider the portfolio consisting of buying an asset and a put
option on this asset, both for the same notional amount. The
prot at expiration is
S
T
+ max(K S
T
, 0) S
0
(1 + i )
T
Put(K, T)(1 + i )
T
=max(K, S
T
) (Put(K, T) + S
0
)(1 + i )
T
.
The maximum prot is . The minimum prot is
K (Put(K, T) + S
0
)(1 + i )
T
. If there exists no arbitrage
K (Put(K, T) + S
0
)(1 + i )
T
< 0, which is equivalent to
(1 + i )
T
K S
0
< Put(K, T). From this bound and the bounds
before the theorem, the claim follows.
c 2009. Miguel A. Arcones. All rights reserved. Manual for SOA Exam FM/CAS Exam 2.
19/83
Chapter 7. Derivatives markets. Section 7.5. Put options.
Example 4
The current price of XYZ stock is 160 per share. The annual
eective interest rate is 7%. The price of a oneyear European
200strike put option for XYZ stock is $20 per share. Find an
arbitrage strategy and the minimum prot per share.
Solution: We have that
Put(K, T) + S
0
(1 + i )
T
K = 20 + 160 (200)(1.07)
1
=6.91588785 < 0.
The put premium is too low. Consider the portfolio consisting of
buying the put and the stock, both for the same nominal amount.
The prot per share is
max(200S
T
, 0)20(1.07)+S
T
160(1.07) = max(200, S
T
)192.6.
The minimum prot per share is 200 192.6 = 7.4.
c 2009. Miguel A. Arcones. All rights reserved. Manual for SOA Exam FM/CAS Exam 2.
20/83
Chapter 7. Derivatives markets. Section 7.5. Put options.
Example 4
The current price of XYZ stock is 160 per share. The annual
eective interest rate is 7%. The price of a oneyear European
200strike put option for XYZ stock is $20 per share. Find an
arbitrage strategy and the minimum prot per share.
Solution: We have that
Put(K, T) + S
0
(1 + i )
T
K = 20 + 160 (200)(1.07)
1
=6.91588785 < 0.
The put premium is too low. Consider the portfolio consisting of
buying the put and the stock, both for the same nominal amount.
The prot per share is
max(200S
T
, 0)20(1.07)+S
T
160(1.07) = max(200, S
T
)192.6.
The minimum prot per share is 200 192.6 = 7.4.
c 2009. Miguel A. Arcones. All rights reserved. Manual for SOA Exam FM/CAS Exam 2.
21/83
Chapter 7. Derivatives markets. Section 7.5. Put options.
Example 5
The current price of XYZ stock is 160 per share. The annual
eective interest rate is 7%. The price of a oneyear European
200strike put option for XYZ stock is $190 per share. Find an
arbitrage strategy and the minimum prot per share.
Solution: We have that
(1 + i )
T
K Put(K, T) = (200)(1.07)
1
190
=3.08411215 < 0.
The put is overpriced. Consider the portfolio consisting of selling
the put. The prot per share is 190(1.07) max(200 S
T
, 0).
The minimum prot per share is 190(1.07) 200 = 3.3.
c 2009. Miguel A. Arcones. All rights reserved. Manual for SOA Exam FM/CAS Exam 2.
22/83
Chapter 7. Derivatives markets. Section 7.5. Put options.
Example 5
The current price of XYZ stock is 160 per share. The annual
eective interest rate is 7%. The price of a oneyear European
200strike put option for XYZ stock is $190 per share. Find an
arbitrage strategy and the minimum prot per share.
Solution: We have that
(1 + i )
T
K Put(K, T) = (200)(1.07)
1
190
=3.08411215 < 0.
The put is overpriced. Consider the portfolio consisting of selling
the put. The prot per share is 190(1.07) max(200 S
T
, 0).
The minimum prot per share is 190(1.07) 200 = 3.3.
c 2009. Miguel A. Arcones. All rights reserved. Manual for SOA Exam FM/CAS Exam 2.
23/83
Chapter 7. Derivatives markets. Section 7.5. Put options.
The prot of a put option holder is positive if
max(K S
T
, 0) Put(K, T)(1 + i )
T
> 0,
which is equivalent to
K Put(K, T)(1 + i )
T
> S
T
.
If K Put(K, T)(1 + i )
T
< S
T
, the put option holders prot is
negative.
c 2009. Miguel A. Arcones. All rights reserved. Manual for SOA Exam FM/CAS Exam 2.
24/83
Chapter 7. Derivatives markets. Section 7.5. Put options.
Example 6
Ashley buys a 85strike put option for 4.3185816 per share. The
nominal amount of the put option is 2500 shares of XYZ stock.
The expiration date of this option is one year. The annual interest
rate compounded continuously is 5%.
c 2009. Miguel A. Arcones. All rights reserved. Manual for SOA Exam FM/CAS Exam 2.
25/83
Chapter 7. Derivatives markets. Section 7.5. Put options.
Example 6
Ashley buys a 85strike put option for 4.3185816 per share. The
nominal amount of the put option is 2500 shares of XYZ stock.
The expiration date of this option is one year. The annual interest
rate compounded continuously is 5%.
(i) Calculate Ashleys prot function.
c 2009. Miguel A. Arcones. All rights reserved. Manual for SOA Exam FM/CAS Exam 2.
26/83
Chapter 7. Derivatives markets. Section 7.5. Put options.
Example 6
Ashley buys a 85strike put option for 4.3185816 per share. The
nominal amount of the put option is 2500 shares of XYZ stock.
The expiration date of this option is one year. The annual interest
rate compounded continuously is 5%.
(i) Calculate Ashleys prot function.
Solution: (i) Ashleys prot is
(2500)(max(85 S
T
, 0) 4.3185816e
0.05
)
=(2500) max(85 S
T
, 0) 11350.
c 2009. Miguel A. Arcones. All rights reserved. Manual for SOA Exam FM/CAS Exam 2.
27/83
Chapter 7. Derivatives markets. Section 7.5. Put options.
Example 6
Ashley buys a 85strike put option for 4.3185816 per share. The
nominal amount of the put option is 2500 shares of XYZ stock.
The expiration date of this option is one year. The annual interest
rate compounded continuously is 5%.
(ii) Calculate Ashleys prot for the following spot prices at expira-
tion: 75, 80, 85, 90, 95.
c 2009. Miguel A. Arcones. All rights reserved. Manual for SOA Exam FM/CAS Exam 2.
28/83
Chapter 7. Derivatives markets. Section 7.5. Put options.
Example 6
Ashley buys a 85strike put option for 4.3185816 per share. The
nominal amount of the put option is 2500 shares of XYZ stock.
The expiration date of this option is one year. The annual interest
rate compounded continuously is 5%.
(ii) Calculate Ashleys prot for the following spot prices at expira-
tion: 75, 80, 85, 90, 95.
Solution: (ii) The prots corresponding to the considered spot
prices are:
if S
T
= 75, prot = (2500) max(85 75, 0) 11350 = 13650,
if S
T
= 80, prot = (2500) max(85 80, 0) 11350 = 1150,
if S
T
= 85, prot = (2500) max(85 85, 0) 11350 = 11350,
if S
T
= 90, prot = (2500) max(85 90, 0) 11350 = 11350,
if S
T
= 95, prot = (2500) max(85 95, 0) 11350 = 11350.
c 2009. Miguel A. Arcones. All rights reserved. Manual for SOA Exam FM/CAS Exam 2.
29/83
Chapter 7. Derivatives markets. Section 7.5. Put options.
Example 6
Ashley buys a 85strike put option for 4.3185816 per share. The
nominal amount of the put option is 2500 shares of XYZ stock.
The expiration date of this option is one year. The annual interest
rate compounded continuously is 5%.
(iii) Calculate Ashleys minimum and maximum prots.
c 2009. Miguel A. Arcones. All rights reserved. Manual for SOA Exam FM/CAS Exam 2.
30/83
Chapter 7. Derivatives markets. Section 7.5. Put options.
Example 6
Ashley buys a 85strike put option for 4.3185816 per share. The
nominal amount of the put option is 2500 shares of XYZ stock.
The expiration date of this option is one year. The annual interest
rate compounded continuously is 5%.
(iii) Calculate Ashleys minimum and maximum prots.
Solution: (iii) Ashleys minimum prot is 11350. Ashleys maxi-
mum prot is (2500)(85) 11350 = 201150.
c 2009. Miguel A. Arcones. All rights reserved. Manual for SOA Exam FM/CAS Exam 2.
31/83
Chapter 7. Derivatives markets. Section 7.5. Put options.
Example 6
Ashley buys a 85strike put option for 4.3185816 per share. The
nominal amount of the put option is 2500 shares of XYZ stock.
The expiration date of this option is one year. The annual interest
rate compounded continuously is 5%.
(iv) Calculate the spot prices at which Ashleys prot is positive.
c 2009. Miguel A. Arcones. All rights reserved. Manual for SOA Exam FM/CAS Exam 2.
32/83
Chapter 7. Derivatives markets. Section 7.5. Put options.
Example 6
Ashley buys a 85strike put option for 4.3185816 per share. The
nominal amount of the put option is 2500 shares of XYZ stock.
The expiration date of this option is one year. The annual interest
rate compounded continuously is 5%.
(iv) Calculate the spot prices at which Ashleys prot is positive.
Solution: (iv) Ashleys prot is positive if (2500) max(85S
T
, 0)
11350 > 0, which is equivalent to S
T
< 85
11350
2500
= 80.46.
c 2009. Miguel A. Arcones. All rights reserved. Manual for SOA Exam FM/CAS Exam 2.
33/83
Chapter 7. Derivatives markets. Section 7.5. Put options.
Example 6
Ashley buys a 85strike put option for 4.3185816 per share. The
nominal amount of the put option is 2500 shares of XYZ stock.
The expiration date of this option is one year. The annual interest
rate compounded continuously is 5%.
(v) Calculate the spot price at expiration at which Ashley breaks
even.
c 2009. Miguel A. Arcones. All rights reserved. Manual for SOA Exam FM/CAS Exam 2.
34/83
Chapter 7. Derivatives markets. Section 7.5. Put options.
Example 6
Ashley buys a 85strike put option for 4.3185816 per share. The
nominal amount of the put option is 2500 shares of XYZ stock.
The expiration date of this option is one year. The annual interest
rate compounded continuously is 5%.
(v) Calculate the spot price at expiration at which Ashley breaks
even.
Solution: (v) Ashley breaks even if (2500) max(85 S
T
, 0)
11350 = 0, i.e. if S
T
= 85
11350
2500
= 80.46.
c 2009. Miguel A. Arcones. All rights reserved. Manual for SOA Exam FM/CAS Exam 2.
35/83
Chapter 7. Derivatives markets. Section 7.5. Put options.
Example 6
Ashley buys a 85strike put option for 4.3185816 per share. The
nominal amount of the put option is 2500 shares of XYZ stock.
The expiration date of this option is one year. The annual interest
rate compounded continuously is 5%.
(vi) Calculate Ashleys annual yield in her investment for the spot
prices at expiration in (ii).
c 2009. Miguel A. Arcones. All rights reserved. Manual for SOA Exam FM/CAS Exam 2.
36/83
Chapter 7. Derivatives markets. Section 7.5. Put options.
Example 6
Ashley buys a 85strike put option for 4.3185816 per share. The
nominal amount of the put option is 2500 shares of XYZ stock.
The expiration date of this option is one year. The annual interest
rate compounded continuously is 5%.
(vi) Calculate Ashleys annual yield in her investment for the spot
prices at expiration in (ii).
Solution: (vi) Ashley invests (2500)(4.3185816) = 10796.454.
Ashleys return is (2500) max(85 S
T
, 0). Let j be Ashleys an-
nual yield. Then, 10796.454(1 + j )
0.5
= (2500) max(85 S
T
, 0).
Hence, j =

(2500) max(85S
T
,0)
10796.454

2
1. Therefore,
if S
T
= 75, j =

(2500) max(8575,0)
10796.454

2
1 = 436.1888022%,
if S
T
= 80, j =

(2500) max(8580,0)
10796.454

2
1 = 34.04720055%,
if S
T
= 85, or 90, or 95, j = 100%.
c 2009. Miguel A. Arcones. All rights reserved. Manual for SOA Exam FM/CAS Exam 2.
37/83
Chapter 7. Derivatives markets. Section 7.5. Put options.
Example 7
William sells a 85strike put option for 4.3185816 per share. The
nominal amount of the put option is 2500 shares of XYZ stock.
The expiration date of the option is one year. The annual interest
rate compounded continuously is 5%.
c 2009. Miguel A. Arcones. All rights reserved. Manual for SOA Exam FM/CAS Exam 2.
38/83
Chapter 7. Derivatives markets. Section 7.5. Put options.
Example 7
William sells a 85strike put option for 4.3185816 per share. The
nominal amount of the put option is 2500 shares of XYZ stock.
The expiration date of the option is one year. The annual interest
rate compounded continuously is 5%.
(i) Calculate Williams prot function.
c 2009. Miguel A. Arcones. All rights reserved. Manual for SOA Exam FM/CAS Exam 2.
39/83
Chapter 7. Derivatives markets. Section 7.5. Put options.
Example 7
William sells a 85strike put option for 4.3185816 per share. The
nominal amount of the put option is 2500 shares of XYZ stock.
The expiration date of the option is one year. The annual interest
rate compounded continuously is 5%.
(i) Calculate Williams prot function.
Solution: (i) Williams prot is
(2500)(4.3185816e
0.05
max(85 S
T
, 0))
=11350 (2500) max(85 S
T
, 0).
c 2009. Miguel A. Arcones. All rights reserved. Manual for SOA Exam FM/CAS Exam 2.
40/83
Chapter 7. Derivatives markets. Section 7.5. Put options.
Example 7
William sells a 85strike put option for 4.3185816 per share. The
nominal amount of the put option is 2500 shares of XYZ stock.
The expiration date of the option is one year. The annual interest
rate compounded continuously is 5%.
(ii) Calculate Williams prot for the following spot prices at expi-
ration: 75, 80, 85, 90, 95.
c 2009. Miguel A. Arcones. All rights reserved. Manual for SOA Exam FM/CAS Exam 2.
41/83
Chapter 7. Derivatives markets. Section 7.5. Put options.
Example 7
William sells a 85strike put option for 4.3185816 per share. The
nominal amount of the put option is 2500 shares of XYZ stock.
The expiration date of the option is one year. The annual interest
rate compounded continuously is 5%.
(ii) Calculate Williams prot for the following spot prices at expi-
ration: 75, 80, 85, 90, 95.
Solution: (ii) Williams prots for the considered spot prices are
if S
T
= 75, prot = 11350 (2500) max(85 75, 0) = 13650,
if S
T
= 80, prot = 11350 (2500) max(85 80, 0) = 1150,
if S
T
= 85, prot = 11350 (2500) max(85 85, 0) = 11350,
if S
T
= 90, prot = 11350 (2500) max(85 90, 0) = 11350,
if S
T
= 95, prot = 11350 (2500) max(85 95, 0) = 11350.
c 2009. Miguel A. Arcones. All rights reserved. Manual for SOA Exam FM/CAS Exam 2.
42/83
Chapter 7. Derivatives markets. Section 7.5. Put options.
Example 7
William sells a 85strike put option for 4.3185816 per share. The
nominal amount of the put option is 2500 shares of XYZ stock.
The expiration date of the option is one year. The annual interest
rate compounded continuously is 5%.
(iii) Calculate Williams minimum and maximum prots.
c 2009. Miguel A. Arcones. All rights reserved. Manual for SOA Exam FM/CAS Exam 2.
43/83
Chapter 7. Derivatives markets. Section 7.5. Put options.
Example 7
William sells a 85strike put option for 4.3185816 per share. The
nominal amount of the put option is 2500 shares of XYZ stock.
The expiration date of the option is one year. The annual interest
rate compounded continuously is 5%.
(iii) Calculate Williams minimum and maximum prots.
Solution: (iii) Williams minimum prot is 11350 (2500)(85) =
201150. Williams maximum prot is 11350.
c 2009. Miguel A. Arcones. All rights reserved. Manual for SOA Exam FM/CAS Exam 2.
44/83
Chapter 7. Derivatives markets. Section 7.5. Put options.
Example 7
William sells a 85strike put option for 4.3185816 per share. The
nominal amount of the put option is 2500 shares of XYZ stock.
The expiration date of the option is one year. The annual interest
rate compounded continuously is 5%.
(iv) Calculate the spot prices at expiration at which William makes
a positive prot.
c 2009. Miguel A. Arcones. All rights reserved. Manual for SOA Exam FM/CAS Exam 2.
45/83
Chapter 7. Derivatives markets. Section 7.5. Put options.
Example 7
William sells a 85strike put option for 4.3185816 per share. The
nominal amount of the put option is 2500 shares of XYZ stock.
The expiration date of the option is one year. The annual interest
rate compounded continuously is 5%.
(iv) Calculate the spot prices at expiration at which William makes
a positive prot.
Solution: (iv) William makes a positive prot if 11350
(2500) max(85 S
T
, 0) > 0, i.e. if S
T
> 85
11350
2500
= 80.46.
c 2009. Miguel A. Arcones. All rights reserved. Manual for SOA Exam FM/CAS Exam 2.
46/83
Chapter 7. Derivatives markets. Section 7.5. Put options.
A put option is a way to sell an asset in the future. A short
forward is another way to sell an asset in the future.
c 2009. Miguel A. Arcones. All rights reserved. Manual for SOA Exam FM/CAS Exam 2.
47/83
Chapter 7. Derivatives markets. Section 7.5. Put options.
Example 8
Rachel enters into a short forward contract for 500 shares of XYZ
stock for $26.35035 per share. The exercise date is three months
from now. The risk free eective annual interest rate is 5.5%. The
current price of XYZ stock is $26 per share. Dan buys a put
option of 500 shares of XYZ stock with a strike price of $26 per
share. The exercise date is three months from now. The premium
of this put option is $1.377368 per share.
c 2009. Miguel A. Arcones. All rights reserved. Manual for SOA Exam FM/CAS Exam 2.
48/83
Chapter 7. Derivatives markets. Section 7.5. Put options.
Example 8
Rachel enters into a short forward contract for 500 shares of XYZ
stock for $26.35035 per share. The exercise date is three months
from now. The risk free eective annual interest rate is 5.5%. The
current price of XYZ stock is $26 per share. Dan buys a put
option of 500 shares of XYZ stock with a strike price of $26 per
share. The exercise date is three months from now. The premium
of this put option is $1.377368 per share.
(i) Find Rachels and Dans prots as a function of the spot price
at expiration.
c 2009. Miguel A. Arcones. All rights reserved. Manual for SOA Exam FM/CAS Exam 2.
49/83
Chapter 7. Derivatives markets. Section 7.5. Put options.
Example 8
Rachel enters into a short forward contract for 500 shares of XYZ
stock for $26.35035 per share. The exercise date is three months
from now. The risk free eective annual interest rate is 5.5%. The
current price of XYZ stock is $26 per share. Dan buys a put
option of 500 shares of XYZ stock with a strike price of $26 per
share. The exercise date is three months from now. The premium
of this put option is $1.377368 per share.
(i) Find Rachels and Dans prots as a function of the spot price
at expiration.
Solution: (i) The no arbitrage price of XYZ stock is
(26)(1.055)
0.25
= 26.35035454. Rachels prot is
(500)(26.35035 S
T
) = 13175.17727 500S
T
.
c 2009. Miguel A. Arcones. All rights reserved. Manual for SOA Exam FM/CAS Exam 2.
50/83
Chapter 7. Derivatives markets. Section 7.5. Put options.
Example 8
Rachel enters into a short forward contract for 500 shares of XYZ
stock for $26.35035 per share. The exercise date is three months
from now. The risk free eective annual interest rate is 5.5%. The
current price of XYZ stock is $26 per share. Dan buys a put
option of 500 shares of XYZ stock with a strike price of $26 per
share. The exercise date is three months from now. The premium
of this put option is $1.377368 per share.
(i) Find Rachels and Dans prots as a function of the spot price
at expiration.
Solution: (i) (continuation) Dans prot is
500 max(0, K S
T
) 500Put(K, T)(1 + i )
T
= 500 max(0, 26 S
T
) (500)(1.377368)(1.055)
0.25
= 500 max(0, 26 S
T
) 697.9641.
c 2009. Miguel A. Arcones. All rights reserved. Manual for SOA Exam FM/CAS Exam 2.
51/83
Chapter 7. Derivatives markets. Section 7.5. Put options.
Example 8
Rachel enters into a short forward contract for 500 shares of XYZ
stock for $26.35035 per share. The exercise date is three months
from now. The risk free eective annual interest rate is 5.5%. The
current price of XYZ stock is $26 per share. Dan buys a put
option of 500 shares of XYZ stock with a strike price of $26 per
share. The exercise date is three months from now. The premium
of this put option is $1.377368 per share.
(ii) Make a table with Rachels and Dans prots when the spot price
at expiration is $18, $20, $22, $24, $26, $28, $30.
c 2009. Miguel A. Arcones. All rights reserved. Manual for SOA Exam FM/CAS Exam 2.
52/83
Chapter 7. Derivatives markets. Section 7.5. Put options.
Example 8
Rachel enters into a short forward contract for 500 shares of XYZ
stock for $26.35035 per share. The exercise date is three months
from now. The risk free eective annual interest rate is 5.5%. The
current price of XYZ stock is $26 per share. Dan buys a put
option of 500 shares of XYZ stock with a strike price of $26 per
share. The exercise date is three months from now. The premium
of this put option is $1.377368 per share.
(ii) Make a table with Rachels and Dans prots when the spot price
at expiration is $18, $20, $22, $24, $26, $28, $30.
Solution: (ii)
Rachels prot 4175.18 3175.18 2175.18 1175.18 175.18 824.82 1824.82
Dans prot 3302.04 2302.04 1302.04 302.04 697.96 697.96 697.96
Spot Price 18 20 22 24 26 28 30
c 2009. Miguel A. Arcones. All rights reserved. Manual for SOA Exam FM/CAS Exam 2.
53/83
Chapter 7. Derivatives markets. Section 7.5. Put options.
Example 8
Rachel enters into a short forward contract for 500 shares of XYZ
stock for $26.35035 per share. The exercise date is three months
from now. The risk free eective annual interest rate is 5.5%. The
current price of XYZ stock is $26 per share. Dan buys a put
option of 500 shares of XYZ stock with a strike price of $26 per
share. The exercise date is three months from now. The premium
of this put option is $1.377368 per share.
(iii) Calculate Rachels and Dans minimum and maximum payos.
c 2009. Miguel A. Arcones. All rights reserved. Manual for SOA Exam FM/CAS Exam 2.
54/83
Chapter 7. Derivatives markets. Section 7.5. Put options.
Example 8
Rachel enters into a short forward contract for 500 shares of XYZ
stock for $26.35035 per share. The exercise date is three months
from now. The risk free eective annual interest rate is 5.5%. The
current price of XYZ stock is $26 per share. Dan buys a put
option of 500 shares of XYZ stock with a strike price of $26 per
share. The exercise date is three months from now. The premium
of this put option is $1.377368 per share.
(iii) Calculate Rachels and Dans minimum and maximum payos.
Solution: (iii) Rachels minimum prot is . Rachels maximum
prot is 13175.18. Dans minimum prot is 697.96. Dans maxi-
mum prot is (500)(26) 697.9641 = 12302.04.
c 2009. Miguel A. Arcones. All rights reserved. Manual for SOA Exam FM/CAS Exam 2.
55/83
Chapter 7. Derivatives markets. Section 7.5. Put options.
Example 8
Rachel enters into a short forward contract for 500 shares of XYZ
stock for $26.35035 per share. The exercise date is three months
from now. The risk free eective annual interest rate is 5.5%. The
current price of XYZ stock is $26 per share. Dan buys a put
option of 500 shares of XYZ stock with a strike price of $26 per
share. The exercise date is three months from now. The premium
of this put option is $1.377368 per share.
(iv) Calculate the spot price at expiration at which Dan and Rachel
make the same prot.
c 2009. Miguel A. Arcones. All rights reserved. Manual for SOA Exam FM/CAS Exam 2.
56/83
Chapter 7. Derivatives markets. Section 7.5. Put options.
Example 8
Rachel enters into a short forward contract for 500 shares of XYZ
stock for $26.35035 per share. The exercise date is three months
from now. The risk free eective annual interest rate is 5.5%. The
current price of XYZ stock is $26 per share. Dan buys a put
option of 500 shares of XYZ stock with a strike price of $26 per
share. The exercise date is three months from now. The premium
of this put option is $1.377368 per share.
(iv) Calculate the spot price at expiration at which Dan and Rachel
make the same prot.
Solution: (iv) Since the prots are equal for some S
T
> 26, we
solve 697.9641 = (500)(26.35035S
T
) and get S
T
= 26.35035+
697.9641/500 = 27.74628.
c 2009. Miguel A. Arcones. All rights reserved. Manual for SOA Exam FM/CAS Exam 2.
57/83
Chapter 7. Derivatives markets. Section 7.5. Put options.
Example 8
Rachel enters into a short forward contract for 500 shares of XYZ
stock for $26.35035 per share. The exercise date is three months
from now. The risk free eective annual interest rate is 5.5%. The
current price of XYZ stock is $26 per share. Dan buys a put
option of 500 shares of XYZ stock with a strike price of $26 per
share. The exercise date is three months from now. The premium
of this put option is $1.377368 per share.
(v) Draw the graphs of the prot versus the spot price at expiration
for Dan and Rachel.
c 2009. Miguel A. Arcones. All rights reserved. Manual for SOA Exam FM/CAS Exam 2.
58/83
Chapter 7. Derivatives markets. Section 7.5. Put options.
Example 8
Rachel enters into a short forward contract for 500 shares of XYZ
stock for $26.35035 per share. The exercise date is three months
from now. The risk free eective annual interest rate is 5.5%. The
current price of XYZ stock is $26 per share. Dan buys a put
option of 500 shares of XYZ stock with a strike price of $26 per
share. The exercise date is three months from now. The premium
of this put option is $1.377368 per share.
(v) Draw the graphs of the prot versus the spot price at expiration
for Dan and Rachel.
Solution: (v) The graphs of (short forward) Rachels prot and
(purchased put option) Dans prot are in Figure 3.
c 2009. Miguel A. Arcones. All rights reserved. Manual for SOA Exam FM/CAS Exam 2.
59/83
Chapter 7. Derivatives markets. Section 7.5. Put options.
Figure 3: Prot for short forward and purchased put option.
c 2009. Miguel A. Arcones. All rights reserved. Manual for SOA Exam FM/CAS Exam 2.
60/83
Chapter 7. Derivatives markets. Section 7.5. Put options.
A purchased put option reduces losses over a short forward. The
prot per unit of a short forward contract is F
0,T
S
T
. The
minimum prot for a short forward contract is . The maximum
prot for a short forward contract is F
0,T
. The prot for the put
option holder is max(K S
T
, 0) Put(K, T)(1 + i )
T
. The
minimum of the put option holders prot is Put(K, T)(1 + i )
T
.
The maximum of the put option holders prot is
K Put(K, T)(1 + i )
T
. A put option is an insured position in an
asset. In return for not having large losses, the possible returns for
a put option are smaller than those for a short forward.
c 2009. Miguel A. Arcones. All rights reserved. Manual for SOA Exam FM/CAS Exam 2.
61/83
Chapter 7. Derivatives markets. Section 7.5. Put options.
Theorem 2
If there exists no arbitrage, then
max((1 + i )
T
(K F
0,t
), 0) Put(K, T) K(1 + i )
T
.
Proof.
Consider the portfolio consisting of entering a long forward
contract and buying a put option. The prot at expiration is
S
T
F
0,T
+ max(K S
T
, 0) Put(K, T)(1 + i )
T
=max(K, S
T
) F
0,T
(Put(K, T) + S
0
)(1 + i )
T
.
The maximum prot is . The minimum prot is
K F
0,T
(Put(K, T) + S
0
)(1 + i )
T
. If there exists no arbitrage
K F
0,T
Put(K, T)(1 + i )
T
< 0. The claim follows from this
bound and the bounds in Theorem 1.
c 2009. Miguel A. Arcones. All rights reserved. Manual for SOA Exam FM/CAS Exam 2.
62/83
Chapter 7. Derivatives markets. Section 7.5. Put options.
Theorem 2
If there exists no arbitrage, then
max((1 + i )
T
(K F
0,t
), 0) Put(K, T) K(1 + i )
T
.
Proof.
Consider the portfolio consisting of entering a long forward
contract and buying a put option. The prot at expiration is
S
T
F
0,T
+ max(K S
T
, 0) Put(K, T)(1 + i )
T
=max(K, S
T
) F
0,T
(Put(K, T) + S
0
)(1 + i )
T
.
The maximum prot is . The minimum prot is
K F
0,T
(Put(K, T) + S
0
)(1 + i )
T
. If there exists no arbitrage
K F
0,T
Put(K, T)(1 + i )
T
< 0. The claim follows from this
bound and the bounds in Theorem 1.
c 2009. Miguel A. Arcones. All rights reserved. Manual for SOA Exam FM/CAS Exam 2.
63/83
Chapter 7. Derivatives markets. Section 7.5. Put options.
Example 9
The current price of a forward of corn is $3.3 per bushel. The
annual eective interest rate is 7.5%. The price of a oneyear
European 3.5strike put option for corn is $0.18 per bushel. Find
an arbitrage strategy and its minimum prot per bushel.
Solution: We have that
Put(K, T) ((1 + i )
T
(K F
0,t
)
=0.18 (1.075)(3.5 3.3) = 0.035 < 0.
The put premium is too low. Consider the portfolio consisting of
entering into a long forward contract and buying a put option,
both for the same nominal amount. The prot is
S
T
3.3 + max(3.5 S
T
, 0) (0.18)(1.075)
=max(3.5, S
T
) 3.3 (0.18)(1.075)
The minimum prot per share is
3.5 3.3 (0.18)(1.075) = 0.0065.
c 2009. Miguel A. Arcones. All rights reserved. Manual for SOA Exam FM/CAS Exam 2.
64/83
Chapter 7. Derivatives markets. Section 7.5. Put options.
Example 9
The current price of a forward of corn is $3.3 per bushel. The
annual eective interest rate is 7.5%. The price of a oneyear
European 3.5strike put option for corn is $0.18 per bushel. Find
an arbitrage strategy and its minimum prot per bushel.
Solution: We have that
Put(K, T) ((1 + i )
T
(K F
0,t
)
=0.18 (1.075)(3.5 3.3) = 0.035 < 0.
The put premium is too low. Consider the portfolio consisting of
entering into a long forward contract and buying a put option,
both for the same nominal amount. The prot is
S
T
3.3 + max(3.5 S
T
, 0) (0.18)(1.075)
=max(3.5, S
T
) 3.3 (0.18)(1.075)
The minimum prot per share is
3.5 3.3 (0.18)(1.075) = 0.0065.
c 2009. Miguel A. Arcones. All rights reserved. Manual for SOA Exam FM/CAS Exam 2.
65/83
Chapter 7. Derivatives markets. Section 7.5. Put options.
Example 10
The payo of a purchase put option is similar to the one on a
policy insurance of some asset. Suppose that you car is worth
$20000. In the case of an accident, the insurance company pays
you max(20000 S
T
, 0), where S
T
is the price of the car after the
accident. If you own stock valued at K and buy a put option with
strike price K, the payo of the put option at expiration time is
max(K S
T
, 0). The payo is precisely the loss in value of the
stock.
There are minor dierences between these two examples. In the
case of the insurance of car, usually a deductible is applied. If the
deductible in your car insurance is $500, the payment by the
insurance company is max(20000 500 S
T
, 0). Since cost of an
accident is always positive, S
T
< 20000.
c 2009. Miguel A. Arcones. All rights reserved. Manual for SOA Exam FM/CAS Exam 2.
66/83
Chapter 7. Derivatives markets. Section 7.5. Put options.
Theorem 3
If 0 < K
1
< K
2
, then
Put(K
1
, T) Put(K
2
, T) Put(K
1
, T) + (K
2
K
1
)e
rT
.
c 2009. Miguel A. Arcones. All rights reserved. Manual for SOA Exam FM/CAS Exam 2.
67/83
Chapter 7. Derivatives markets. Section 7.5. Put options.
Proof.
We have that
max(K
1
S
T
, 0)
max(K
2
S
T
, 0) = K
2
K
1
+ max(K
1
S
T
, K
1
K
2
)
K
2
K
1
+ max(K
1
S
T
, 0).
In other words,
(i) The payo for a K
1
strike put is smaller than or equal to the
payo for a K
2
strike put.
(ii) The payo for a K
2
strike put is smaller than or equal to
(K
2
K
1
) plus the payo for K
1
strike put.
Hence, if there exist no arbitrage, then
Put(K
1
, T) Put(K
2
, T) Put(K
1
, T) + (K
2
K
1
)e
rT
.
c 2009. Miguel A. Arcones. All rights reserved. Manual for SOA Exam FM/CAS Exam 2.
68/83
Chapter 7. Derivatives markets. Section 7.5. Put options.
Example 11
The price of a oneyear European 3.5strike put option for corn is
$0.18 per bushel. The price of a oneyear European 3.75strike put
option for corn is $0.15 per bushel. The annual eective interest
rate is 7.5%. Find an arbitrage strategy and it minimum prot.
Solution: In this case, Put(K
1
, T) Put(K
2
, T) does not hold.
We can do arbitrage by a buying a 3.75strike put option and
selling 3.5strike put option, both for the same nominal amount.
The prot per share is
max(3.75 S
T
, 0) max(3.5 S
T
, 0) (0.15 0.18)(1.075)
max(3.75, S
T
) max(3.5, S
T
) (0.15 0.18)(1.075)
(0.18 0.15)(1.075) = 0.03225.
c 2009. Miguel A. Arcones. All rights reserved. Manual for SOA Exam FM/CAS Exam 2.
69/83
Chapter 7. Derivatives markets. Section 7.5. Put options.
Example 11
The price of a oneyear European 3.5strike put option for corn is
$0.18 per bushel. The price of a oneyear European 3.75strike put
option for corn is $0.15 per bushel. The annual eective interest
rate is 7.5%. Find an arbitrage strategy and it minimum prot.
Solution: In this case, Put(K
1
, T) Put(K
2
, T) does not hold.
We can do arbitrage by a buying a 3.75strike put option and
selling 3.5strike put option, both for the same nominal amount.
The prot per share is
max(3.75 S
T
, 0) max(3.5 S
T
, 0) (0.15 0.18)(1.075)
max(3.75, S
T
) max(3.5, S
T
) (0.15 0.18)(1.075)
(0.18 0.15)(1.075) = 0.03225.
c 2009. Miguel A. Arcones. All rights reserved. Manual for SOA Exam FM/CAS Exam 2.
70/83
Chapter 7. Derivatives markets. Section 7.5. Put options.
Example 12
Consider two European put options on a stock, both with
expiration date exactly two years from now. One put option has
strike price $85 and the other one $95. The price of the 85strike
put is 8. The price of the 95strike put option is 20. The riskfree
annual rate of interest compounded continuously is 5%. Find an
arbitrage portfolio and its minimum prot.
Solution: In this case Put(K
2
, T) Put(K
1
, T) + (K
2
K
1
)e
rT
does not hold. Notice that
Put(K
1
, T)+(K
2
K
1
)e
rT
= 8+(9585)e
(2)(0.05)
= 17.04837418.
We can do arbitrage by buying a 85strike put option and selling a
95strike put option. The prot per share is
max(85 S
T
, 0) max(95 S
T
, 0) + (20 8)e
(2)(0.05)
=max(85 S
T
, 0) 10 max(85 S
T
, 10) + 13.26205102
10 + 13.26205102 = 3.26205102.
c 2009. Miguel A. Arcones. All rights reserved. Manual for SOA Exam FM/CAS Exam 2.
71/83
Chapter 7. Derivatives markets. Section 7.5. Put options.
Example 12
Consider two European put options on a stock, both with
expiration date exactly two years from now. One put option has
strike price $85 and the other one $95. The price of the 85strike
put is 8. The price of the 95strike put option is 20. The riskfree
annual rate of interest compounded continuously is 5%. Find an
arbitrage portfolio and its minimum prot.
Solution: In this case Put(K
2
, T) Put(K
1
, T) + (K
2
K
1
)e
rT
does not hold. Notice that
Put(K
1
, T)+(K
2
K
1
)e
rT
= 8+(9585)e
(2)(0.05)
= 17.04837418.
We can do arbitrage by buying a 85strike put option and selling a
95strike put option. The prot per share is
max(85 S
T
, 0) max(95 S
T
, 0) + (20 8)e
(2)(0.05)
=max(85 S
T
, 0) 10 max(85 S
T
, 10) + 13.26205102
10 + 13.26205102 = 3.26205102.
c 2009. Miguel A. Arcones. All rights reserved. Manual for SOA Exam FM/CAS Exam 2.
72/83
Chapter 7. Derivatives markets. Section 7.5. Put options.
We have the following rules of thump for price of a put option of
an asset:

Higher asset prices lead to higher put option prices.

Higher strike prices lead to higher put option prices.

Higher interest rates lead to higher put option prices.

Higher expiration time lead to higher put option prices.

Higher variation of an asset price lead to higher put option


prices.
c 2009. Miguel A. Arcones. All rights reserved. Manual for SOA Exam FM/CAS Exam 2.
73/83
Chapter 7. Derivatives markets. Section 7.5. Put options.
Since the put option buyers payo increases as the strike increases,
the premium of a put option increases as the strike price increases.
Hence, between two put options with dierent strike prices:
(i) The put option with smaller strike price has a smaller price.
(ii) If the spot price is low enough, both put options have a positive
prot. The loss is bigger for put option with the higher strike price.
(iii) If the spot price is high enough, both put options have a loss.
The prot is bigger for put option with the bigger strike price.
c 2009. Miguel A. Arcones. All rights reserved. Manual for SOA Exam FM/CAS Exam 2.
74/83
Chapter 7. Derivatives markets. Section 7.5. Put options.
Since the strike price is paid at the expiration time, as higher the
interest rate is as higher the put option price is. As higher the
expiration time is as higher the put option price is.
Usually, the price of a put option is found using the BlackScholes
formula. The BlackScholes formula for the price of a put option is
Put(K, T) = Ke
rT
(d
2
) S
0
e
T
(d
1
).
c 2009. Miguel A. Arcones. All rights reserved. Manual for SOA Exam FM/CAS Exam 2.
75/83
Chapter 7. Derivatives markets. Section 7.5. Put options.
Table 1 shows the premium of a put option for dierent strike
prices. We have used the BlackScholes formula with S
0
= 26,
t = 0.25, = 0.3, = 0 and r = ln(1.055).
Table 1:
Spot Price 22 24 26 28 30
Premium of
a put option
0.1983495 0.6038701 1.377368 2.546227 4.045966
c 2009. Miguel A. Arcones. All rights reserved. Manual for SOA Exam FM/CAS Exam 2.
76/83
Chapter 7. Derivatives markets. Section 7.5. Put options.
Example 13
Use the put premiums from Table 1, i = 5.5% and T = 0.25. An
investor buys a put option for 500 shares. Find the prot function
for the buyer of a put option with the following strike prices: $24,
$26, $28.
Solution: Prot = 500 max(0, K S
T
) 500Put(K, T)(1 + i )
T
.
If K = 24, the prot is
500 max(0, 24 S
T
) (500)(0.6038701)(1.055)
0.25
=500 max(0, 24 S
T
) 306.0036775.
If K = 26, the prot is
500 max(0, 26 S
T
) (500)(1.377368)(1.055)
0.25
=500 max(0, 26 S
T
) 697.9641.
If K = 28, the prot is
500 max(0, 28 S
T
) (500)(2.546227)(1.055)
0.25
=500 max(0, 28 S
T
) 1290.268926.
c 2009. Miguel A. Arcones. All rights reserved. Manual for SOA Exam FM/CAS Exam 2.
77/83
Chapter 7. Derivatives markets. Section 7.5. Put options.
Example 13
Use the put premiums from Table 1, i = 5.5% and T = 0.25. An
investor buys a put option for 500 shares. Find the prot function
for the buyer of a put option with the following strike prices: $24,
$26, $28.
Solution: Prot = 500 max(0, K S
T
) 500Put(K, T)(1 + i )
T
.
If K = 24, the prot is
500 max(0, 24 S
T
) (500)(0.6038701)(1.055)
0.25
=500 max(0, 24 S
T
) 306.0036775.
If K = 26, the prot is
500 max(0, 26 S
T
) (500)(1.377368)(1.055)
0.25
=500 max(0, 26 S
T
) 697.9641.
If K = 28, the prot is
500 max(0, 28 S
T
) (500)(2.546227)(1.055)
0.25
=500 max(0, 28 S
T
) 1290.268926.
c 2009. Miguel A. Arcones. All rights reserved. Manual for SOA Exam FM/CAS Exam 2.
78/83
Chapter 7. Derivatives markets. Section 7.5. Put options.
When S
T
is low, the higher the strike price is, the higher the prot
is. When S
T
is high, the higher the strike price is, the higher the
loss is.
Figure 4: Prot for three puts.
c 2009. Miguel A. Arcones. All rights reserved. Manual for SOA Exam FM/CAS Exam 2.
79/83
Chapter 7. Derivatives markets. Section 7.5. Put options.

A purchased put option is inthemoney, if S


T
< K.

A purchased put option is outthemoney, if S


T
> K.

A purchased put option is atthemoney, if S


T
= K.
c 2009. Miguel A. Arcones. All rights reserved. Manual for SOA Exam FM/CAS Exam 2.
80/83
Chapter 7. Derivatives markets. Section 7.5. Put options.
If K is very small, the put option will almost certainly not be
executed. Hence, if K is very small, Put(K, T) = 0, i.e.
lim
K0+
Put(K, T) = 0. If K is very large, the put option will almost
certainly be executed. If a put is executed its prot is K S
T
.
Hence, lim
K
Put(K, T) = . As a function on K, Put(K, T) is
an increasing function with lim
K0+
Put(K, T) = 0 and
lim
K
Put(K, T) = .
Figure 5 shows the graph of Put(K, T) as a function of K.
Put(K, T) was found using the BlackScholes formula with
T = 1, S
0
= 100, T = 1, = 0.25, r = ln(1.06) and = 0.0.
c 2009. Miguel A. Arcones. All rights reserved. Manual for SOA Exam FM/CAS Exam 2.
81/83
Chapter 7. Derivatives markets. Section 7.5. Put options.
Figure 5: Graph of Put(K, T) as a function of K.
c 2009. Miguel A. Arcones. All rights reserved. Manual for SOA Exam FM/CAS Exam 2.
82/83
Chapter 7. Derivatives markets. Section 7.5. Put options.
We have the following strategies to speculate on the change of an
asset
volatility
will decrease
no volatility info
volatility
will increase
price
will decrease
sell a call sell asset buy a put
price
will increase
sell a put buy asset buy a call
c 2009. Miguel A. Arcones. All rights reserved. Manual for SOA Exam FM/CAS Exam 2.
83/83
Chapter 7. Derivatives markets. Section 7.5. Put options.
If the price of asset will decrease, we can make a prot by either
selling a call, or selling asset, or buying a put. If the volatility will
decrease, the chances than option is executed decrease. Hence, if
the price of asset and volatility will decrease, then the preferred
strategy is to sell a call.
By a similar argument, we have that:
If the price of asset will decrease and the volatility will increase,
then the preferred strategy is to buy a put.
If the price of asset will increase and the volatility will decrease,
then the preferred strategy is to sell a put.
If the price of asset and volatility will increase, then the preferred
strategy is to buy a call.
c 2009. Miguel A. Arcones. All rights reserved. Manual for SOA Exam FM/CAS Exam 2.
1/51
Chapter 7. Derivatives markets.
Manual for SOA Exam FM/CAS Exam 2.
Chapter 7. Derivatives markets.
Section 7.6. Putcall parity.
c 2009. Miguel A. Arcones. All rights reserved.
Extract from:
Arcones Manual for the SOA Exam FM/CAS Exam 2,
Financial Mathematics. Fall 2009 Edition,
available at http://www.actexmadriver.com/
c 2009. Miguel A. Arcones. All rights reserved. Manual for SOA Exam FM/CAS Exam 2.
2/51
Chapter 7. Derivatives markets. Section 7.6. Putcall parity.
Putcall parity
Recall that the actions and payos corresponding to a call/put are:
If S
T
< K If K < S
T
long call no action buy the stock
short call no action sell the stock
long put sell the stock no action
short put buy the stock no action
If S
T
< K If K < S
T
long call 0 S
T
K
short call 0 (S
T
K)
long put K S
T
0
short put (K S
T
) 0
c 2009. Miguel A. Arcones. All rights reserved. Manual for SOA Exam FM/CAS Exam 2.
3/51
Chapter 7. Derivatives markets. Section 7.6. Putcall parity.

If we have a Kstrike long call and a Kstrike short put, we


are able to buy the asset at time T for K. Hence, having
both a Kstrike long call and a Kstrike short put is
equivalent to have a Kstrike long forward contract with
price K.

Entering into both a Kstrike long call and a Kstrike short


put is called a synthetic long forward.

Reciprocally, if we have a Kstrike short call and a Kstrike


long put, we are able to sell the asset at time T for K.
Having both a Kstrike short call and a Kstrike long
put is equivalent to have a short forward contract with
price K.

Entering into both a Kstrike short call and a Kstrike long


put is called a synthetic short forward.
c 2009. Miguel A. Arcones. All rights reserved. Manual for SOA Exam FM/CAS Exam 2.
4/51
Chapter 7. Derivatives markets. Section 7.6. Putcall parity.
The no arbitrage cost at time T of buying an asset using a long
forward contract is F
0,T
. The cost at time T for buying an asset
using a Kstrike long call and a Kstrike short put is
(Call(K, T) Put(K, T))e
rT
+ K.
If there exists no arbitrage, then:
Theorem 1
(Putcall parity formula)
(Call(K, T) Put(K, T))e
rT
+ K = F
0,T
.
If we use eective interest, the putcall parity formula becomes:
(Call(K, T) Put(K, T))(1 + i )
T
+ K = F
0,T
.
c 2009. Miguel A. Arcones. All rights reserved. Manual for SOA Exam FM/CAS Exam 2.
5/51
Chapter 7. Derivatives markets. Section 7.6. Putcall parity.
Often, F
0,T
= S
0
(1 + i )
T
. This forward price applies to assets
which have neither cost nor benet associated with owning them.
In the absence of arbitrage, we have the following relation between
call and put prices:
Theorem 2
(Putcall parity formula) For a stock which does not pay any
dividends,
(Call(K, T) Put(K, T))e
rT
+ K = S
0
e
rT
.
c 2009. Miguel A. Arcones. All rights reserved. Manual for SOA Exam FM/CAS Exam 2.
6/51
Chapter 7. Derivatives markets. Section 7.6. Putcall parity.
Proof.
Consider the portfolio consisting of buying one share of stock and
a Kstrike put for one share; selling a Kstrike call for one share;
and borrowing S
0
Call(K, T) + Put(K, T). At time T, we have
the following possibilities:
1. If S
T
< K, then the put is exercised and the call is not. We
nish without stock and with a payo for the put of K.
2. If S
T
> K, then the call is exercised and the put is not. We
nish without stock and with a payo for the call of K.
In any case, the payo of this portfolio is K. Hence, K should be
equal to the return in an investment of
S
0
+ Put(K, T) Call(K, T) in a zerocoupon bond, i.e.
K = (S
0
+ Put(K, T) Call(K, T))e
rT
.
c 2009. Miguel A. Arcones. All rights reserved. Manual for SOA Exam FM/CAS Exam 2.
7/51
Chapter 7. Derivatives markets. Section 7.6. Putcall parity.
Example 1
The current value of XYZ stock is 75.38 per share. XYZ stock
does not pay any dividends. The premium of a ninemonth
80strike call is 5.737192 per share. The premium of a ninemonth
80strike put is 7.482695 per share. Find the annual eective rate
of interest.
Solution: The putcall parity formula states that
(Call(K, T) Put(K, T))(1 + i )
T
+ K = S
0
(1 + i )
T
.
So,
(5.737192 7.482695)(1 + i )
3/4
+ 80 = 75.38(1 + i )
T
.
80 = (75.38 (5.737192 7.482695))(1 + i )
3/4
=
(77.125503)(1 + i )
3/4
, and i = 5%.
c 2009. Miguel A. Arcones. All rights reserved. Manual for SOA Exam FM/CAS Exam 2.
8/51
Chapter 7. Derivatives markets. Section 7.6. Putcall parity.
Example 1
The current value of XYZ stock is 75.38 per share. XYZ stock
does not pay any dividends. The premium of a ninemonth
80strike call is 5.737192 per share. The premium of a ninemonth
80strike put is 7.482695 per share. Find the annual eective rate
of interest.
Solution: The putcall parity formula states that
(Call(K, T) Put(K, T))(1 + i )
T
+ K = S
0
(1 + i )
T
.
So,
(5.737192 7.482695)(1 + i )
3/4
+ 80 = 75.38(1 + i )
T
.
80 = (75.38 (5.737192 7.482695))(1 + i )
3/4
=
(77.125503)(1 + i )
3/4
, and i = 5%.
c 2009. Miguel A. Arcones. All rights reserved. Manual for SOA Exam FM/CAS Exam 2.
9/51
Chapter 7. Derivatives markets. Section 7.6. Putcall parity.
Example 2
The current value of XYZ stock is 85 per share. XYZ stock does
not pay any dividends. The premium of a sixmonth Kstrike call
is 3.329264 per share and the premium of a one year Kstrike put
is 10.384565 per share. The annual eective rate of interest is
6.5%. Find K.
Solution: The putcall parity formula states that
(Call(K, T) Put(K, T))(1 + i )
T
+ K = S
0
(1 + i )
T
.
So, (3.329264 10.384565)(1.065)
0.5
+ K = 85(1.065)
0.5
and
K = (85 3.329264 + 10.384565)(1.065)
0.5
= 95.
c 2009. Miguel A. Arcones. All rights reserved. Manual for SOA Exam FM/CAS Exam 2.
10/51
Chapter 7. Derivatives markets. Section 7.6. Putcall parity.
Example 2
The current value of XYZ stock is 85 per share. XYZ stock does
not pay any dividends. The premium of a sixmonth Kstrike call
is 3.329264 per share and the premium of a one year Kstrike put
is 10.384565 per share. The annual eective rate of interest is
6.5%. Find K.
Solution: The putcall parity formula states that
(Call(K, T) Put(K, T))(1 + i )
T
+ K = S
0
(1 + i )
T
.
So, (3.329264 10.384565)(1.065)
0.5
+ K = 85(1.065)
0.5
and
K = (85 3.329264 + 10.384565)(1.065)
0.5
= 95.
c 2009. Miguel A. Arcones. All rights reserved. Manual for SOA Exam FM/CAS Exam 2.
11/51
Chapter 7. Derivatives markets. Section 7.6. Putcall parity.
Example 3
XYZ stock does not pay any dividends. The price of a one year
forward for one share of XYZ stock is 47.475. The premium of a
one year 55strike put option of XYZ stock is 9.204838 per share.
The annual eective rate of interest is 5.5%. Calculate the price of
a one year 55strike call option for one share of XYZ stock.
Solution: The putcall parity formula states that
(Call(K, T) Put(K, T))(1 + i )
T
+ K = F
0,T
.
So, (Call(55, 1) 9.204838)(1.055) + 55 = 47.475 and
Call(55, 1) = 9.204838 + (47.475 55)(1.055)
1
= 2.072136578.
c 2009. Miguel A. Arcones. All rights reserved. Manual for SOA Exam FM/CAS Exam 2.
12/51
Chapter 7. Derivatives markets. Section 7.6. Putcall parity.
Example 3
XYZ stock does not pay any dividends. The price of a one year
forward for one share of XYZ stock is 47.475. The premium of a
one year 55strike put option of XYZ stock is 9.204838 per share.
The annual eective rate of interest is 5.5%. Calculate the price of
a one year 55strike call option for one share of XYZ stock.
Solution: The putcall parity formula states that
(Call(K, T) Put(K, T))(1 + i )
T
+ K = F
0,T
.
So, (Call(55, 1) 9.204838)(1.055) + 55 = 47.475 and
Call(55, 1) = 9.204838 + (47.475 55)(1.055)
1
= 2.072136578.
c 2009. Miguel A. Arcones. All rights reserved. Manual for SOA Exam FM/CAS Exam 2.
13/51
Chapter 7. Derivatives markets. Section 7.6. Putcall parity.
If prices of put options and call options do not satisfy the putcall
parity, it is possible to do arbitrage.

If
(S
0
Call(K, T) + Put(K, T))e
rT
> K,
we can make a prot by buying a call option, selling a put
option and shorting stock. The prot of this strategy is
=K + (S
0
Call(K, T) + Put(K, T))e
rT
.

If
(S
0
Call(K, T) + Put(K, T))e
rT
< K,
we can do arbitrage by selling a call option, buying a put
option and buying stock. At expiration time, we get rid of the
stock by satisfying the options and make
K (S
0
Call(K, T) + Put(K, T))e
rT
.
c 2009. Miguel A. Arcones. All rights reserved. Manual for SOA Exam FM/CAS Exam 2.
14/51
Chapter 7. Derivatives markets. Section 7.6. Putcall parity.
Example 4
XYZ stock trades at $54 per share. XYZ stock does not pay any
dividends. The cost of an European call option with strike price
$50 and expiration date in three months is $8 per share. The risk
free annual interest rate continuously compounded is 4%.
c 2009. Miguel A. Arcones. All rights reserved. Manual for SOA Exam FM/CAS Exam 2.
15/51
Chapter 7. Derivatives markets. Section 7.6. Putcall parity.
Example 4
XYZ stock trades at $54 per share. XYZ stock does not pay any
dividends. The cost of an European call option with strike price
$50 and expiration date in three months is $8 per share. The risk
free annual interest rate continuously compounded is 4%.
(i) Find the noarbitrage price of a European put option with the
same strike price and expiration time.
c 2009. Miguel A. Arcones. All rights reserved. Manual for SOA Exam FM/CAS Exam 2.
16/51
Chapter 7. Derivatives markets. Section 7.6. Putcall parity.
Example 4
XYZ stock trades at $54 per share. XYZ stock does not pay any
dividends. The cost of an European call option with strike price
$50 and expiration date in three months is $8 per share. The risk
free annual interest rate continuously compounded is 4%.
(i) Find the noarbitrage price of a European put option with the
same strike price and expiration time.
Solution: (i) With continuous interest, the putcall parity formula
is
(Call(K, T) Put(K, T))e
rT
+ K = S
0
e
rT
.
Hence,
(8 Put(50, 0.25))e
0.04(0.25)
+ 50 = 54e
0.04(0.25)
and Put(50, 0.25) = 8 54 + 50e
0.04(0.25)
= 3.502491687.
c 2009. Miguel A. Arcones. All rights reserved. Manual for SOA Exam FM/CAS Exam 2.
17/51
Chapter 7. Derivatives markets. Section 7.6. Putcall parity.
Example 4
XYZ stock trades at $54 per share. XYZ stock does not pay any
dividends. The cost of an European call option with strike price
$50 and expiration date in three months is $8 per share. The risk
free annual interest rate continuously compounded is 4%.
(ii) Suppose that the price of an European put option with the same
strike price and expiration time is $3, nd an arbitrage strategy and
its prot per share.
c 2009. Miguel A. Arcones. All rights reserved. Manual for SOA Exam FM/CAS Exam 2.
18/51
Chapter 7. Derivatives markets. Section 7.6. Putcall parity.
Example 4
XYZ stock trades at $54 per share. XYZ stock does not pay any
dividends. The cost of an European call option with strike price
$50 and expiration date in three months is $8 per share. The risk
free annual interest rate continuously compounded is 4%.
(ii) Suppose that the price of an European put option with the same
strike price and expiration time is $3, nd an arbitrage strategy and
its prot per share.
Solution: (ii) A put option for $3 per share is undervalued. An ar-
bitrage portfolio consists in selling a call option, buying a put option
and stock and borrowing $8 + 3 + 54 =$49, with all derivatives
for one share of stock. At redemption time, we sell the stock and
use it to execute the option which will be executed. We also repaid
the loan. The prot is 50 (49)e
(0.04)(0.25)
= 50 49.49245819 =
0.50754181.
c 2009. Miguel A. Arcones. All rights reserved. Manual for SOA Exam FM/CAS Exam 2.
19/51
Chapter 7. Derivatives markets. Section 7.6. Putcall parity.
Example 5
Suppose that the current price of XYZ stock is 31. XYZ stock does
not give any dividends. The risk free annual eective interest rate
is 10%. The price of a threemonth 30strike European call option
is $3. The price of a threemonth 30strike European put option is
$2.25. Find an arbitrage opportunity and its prot per share.
Solution: We have that
(S
0
+ Put(K, T) Call(K, T))(1 + i )
T
=(31 + 2.25 3)(1.1)
0.25
= 30.97943909 > 30.
We conclude that the put is overpriced relatively to the call. We
can sell a put, buy a call and short stock. The prot per share is
(2.25 3 + 31)(1.1)
0.25
30 = 0.9794390948.
Notice that at expiration time one of the options is executed and
we get back the stock which we sold.
c 2009. Miguel A. Arcones. All rights reserved. Manual for SOA Exam FM/CAS Exam 2.
20/51
Chapter 7. Derivatives markets. Section 7.6. Putcall parity.
Example 5
Suppose that the current price of XYZ stock is 31. XYZ stock does
not give any dividends. The risk free annual eective interest rate
is 10%. The price of a threemonth 30strike European call option
is $3. The price of a threemonth 30strike European put option is
$2.25. Find an arbitrage opportunity and its prot per share.
Solution: We have that
(S
0
+ Put(K, T) Call(K, T))(1 + i )
T
=(31 + 2.25 3)(1.1)
0.25
= 30.97943909 > 30.
We conclude that the put is overpriced relatively to the call. We
can sell a put, buy a call and short stock. The prot per share is
(2.25 3 + 31)(1.1)
0.25
30 = 0.9794390948.
Notice that at expiration time one of the options is executed and
we get back the stock which we sold.
c 2009. Miguel A. Arcones. All rights reserved. Manual for SOA Exam FM/CAS Exam 2.
21/51
Chapter 7. Derivatives markets. Section 7.6. Putcall parity.
Synthetic forward.
Denition 1
A synthetic long forward is the combination of buying a call and
selling a put, both with the same strike price, amount of the asset
and expiration date.
The payments to get a synthetic long forward are

(Call(K, T) Put(K, T)) paid at time zero.

K paid at time T.
The future value of these payments at time T is
K + (Call(K, T) Put(K, T))(1 + i )
T
.
The payment of long forward is F
0,T
paid at time T.
In the absence of arbitrage (putcall parity)
F
0,T
= K + (Call(K, T) Put(K, T))(1 + i )
T
.
c 2009. Miguel A. Arcones. All rights reserved. Manual for SOA Exam FM/CAS Exam 2.
22/51
Chapter 7. Derivatives markets. Section 7.6. Putcall parity.
The premium of a synthetic long forward, i.e. the cost of entering
this position, is
(Call(K, T) Put(K, T)).

If F
0,T
= K, the premium of a synthetic long forward is zero.
You will buying the asset at the estimated future value of the
asset.

If F
0,T
> K, the premium of a synthetic long forward is
positive. You will buying the asset lower than the estimated
future value of the asset.

If F
0,T
< K, the premium of a synthetic long forward is
negative. You will buying the asset higher than the estimated
future value of the asset.
c 2009. Miguel A. Arcones. All rights reserved. Manual for SOA Exam FM/CAS Exam 2.
23/51
Chapter 7. Derivatives markets. Section 7.6. Putcall parity.
Constructive sale.
An investor owns stock. He would like to sell his stock. But, he
does not want to report capital gains to the IRS this year. So,
instead of selling this stock, he holds the stock, buys a Kstrike
put, sells a Kstrike call, and borrows K(1 + i )
T
. The payo
which he gets at time T is
S
T
+ max(K S
T
, 0) max(S
T
K, 0) K
=max(S
T
, K) max(S
T
, K) = 0.
At time zero, the investor gets
Call(K, T) Put(K, T) + K(1 + i )
T
= F
0,T
(1 + i )
T
.
At expiration time, the investor can use the stock to meet the
option which will be executed. Practically, the investor sold his
stock at time zero for F
0,T
(1 + i )
T
. According with current USA
tax laws, this is considered a constructive sale. He will have to
declare capital gains when the options are bought.
c 2009. Miguel A. Arcones. All rights reserved. Manual for SOA Exam FM/CAS Exam 2.
24/51
Chapter 7. Derivatives markets. Section 7.6. Putcall parity.
Floor.
Suppose that you own some asset. If the asset losses value in the
future, you lose money. A way to insure this long position is to buy
a put position. The purchase of a put option is called a oor. A
oor guarantees a minimum sale price of the value of an asset.

The prot of buying an asset is S


T
S
0
(1 + i )
T
, which is the
same as the prot of a long forward. The minimum prot of
buying an asset is S
0
(1 + i )
T
.

The prot for buying an asset and a put option is


S
T
S
0
(1 + i )
T
+ max(K S
T
, 0) Put(K, T)(1 + i )
T
=max(S
T
, K) (S
0
+ Put(K, T))(1 + i )
T
.
The minimum prot for buying an asset and a put option is
K (S
0
+ Put(K, T))(1 + i )
T
.
We know that S
0
(1 + i )
T
< K (S
0
+ Put(K, T))(1 + i )
T
.
c 2009. Miguel A. Arcones. All rights reserved. Manual for SOA Exam FM/CAS Exam 2.
25/51
Chapter 7. Derivatives markets. Section 7.6. Putcall parity.
Here is the graph of the prot for buying an asset and a put
option:
Figure 1: Prot for a long position and a long put.
Notice that this is the graph of the prot of a purchased call.
c 2009. Miguel A. Arcones. All rights reserved. Manual for SOA Exam FM/CAS Exam 2.
26/51
Chapter 7. Derivatives markets. Section 7.6. Putcall parity.
Here is a joint graph for the prots of the strategies: (i) buying an
asset, (ii) buying an asset and a put.
Figure 2: Prot for long forward and buying an asset and a put.
c 2009. Miguel A. Arcones. All rights reserved. Manual for SOA Exam FM/CAS Exam 2.
27/51
Chapter 7. Derivatives markets. Section 7.6. Putcall parity.
Table 1 was found using the BlackScholes formula with S
0
= 75,
t = 1, = 0.20, = 0, r = log(1.05).
Table 1: Prices of some calls and some puts.
K 65 70 75 80 85
Call(K, T) 14.31722 10.75552 7.78971 5.444947 3.680736
Put(K, T) 1.221977 2.422184 4.218281 6.635423 9.633117
c 2009. Miguel A. Arcones. All rights reserved. Manual for SOA Exam FM/CAS Exam 2.
28/51
Chapter 7. Derivatives markets. Section 7.6. Putcall parity.
Example 6
The current price of one share of XYZ stock is $75. Use the Table
1 for prices of puts and calls. Steve buys 500 shares of XYZ stock.
Nicole buys 500 shares of XYZ stock and a put option on 500
shares of XYZ stock with a strike price 65 and an expiration date
one year from now. The premium of a 65strike put option is
$1.221977. The risk free annual eective rate of interest is 5%.
c 2009. Miguel A. Arcones. All rights reserved. Manual for SOA Exam FM/CAS Exam 2.
29/51
Chapter 7. Derivatives markets. Section 7.6. Putcall parity.
Example 6
The current price of one share of XYZ stock is $75. Use the Table
1 for prices of puts and calls. Steve buys 500 shares of XYZ stock.
Nicole buys 500 shares of XYZ stock and a put option on 500
shares of XYZ stock with a strike price 65 and an expiration date
one year from now. The premium of a 65strike put option is
$1.221977. The risk free annual eective rate of interest is 5%.
(i) Calculate the prots for Steve and Nicole.
c 2009. Miguel A. Arcones. All rights reserved. Manual for SOA Exam FM/CAS Exam 2.
30/51
Chapter 7. Derivatives markets. Section 7.6. Putcall parity.
Example 6
The current price of one share of XYZ stock is $75. Use the Table
1 for prices of puts and calls. Steve buys 500 shares of XYZ stock.
Nicole buys 500 shares of XYZ stock and a put option on 500
shares of XYZ stock with a strike price 65 and an expiration date
one year from now. The premium of a 65strike put option is
$1.221977. The risk free annual eective rate of interest is 5%.
(i) Calculate the prots for Steve and Nicole.
Solution: (i) Steves prot is (500)(S
T
75). Nicoles prot is
(500)(S
T
75 + max(65 S
T
, 0) 1.221977(1.05))
=(500)(max(65, S
T
) 76.28307585).
c 2009. Miguel A. Arcones. All rights reserved. Manual for SOA Exam FM/CAS Exam 2.
31/51
Chapter 7. Derivatives markets. Section 7.6. Putcall parity.
Example 6
The current price of one share of XYZ stock is $75. Use the Table
1 for prices of puts and calls. Steve buys 500 shares of XYZ stock.
Nicole buys 500 shares of XYZ stock and a put option on 500
shares of XYZ stock with a strike price 65 and an expiration date
one year from now. The premium of a 65strike put option is
$1.221977. The risk free annual eective rate of interest is 5%.
(ii) Find a table with Steves and Nicoles prots for the following
spot prices at expiration: 60, 65, 70, 75, 80 and 85.
c 2009. Miguel A. Arcones. All rights reserved. Manual for SOA Exam FM/CAS Exam 2.
32/51
Chapter 7. Derivatives markets. Section 7.6. Putcall parity.
Example 6
The current price of one share of XYZ stock is $75. Use the Table
1 for prices of puts and calls. Steve buys 500 shares of XYZ stock.
Nicole buys 500 shares of XYZ stock and a put option on 500
shares of XYZ stock with a strike price 65 and an expiration date
one year from now. The premium of a 65strike put option is
$1.221977. The risk free annual eective rate of interest is 5%.
(ii) Find a table with Steves and Nicoles prots for the following
spot prices at expiration: 60, 65, 70, 75, 80 and 85.
Solution: (ii)
Spot Price 60 65 70 75 80 85
Steves prof 7500 5000 2500 0 2500 5000
Nicoles prof 5641.54 5641.54 3141.54 641.54 1858.46 4358.46
c 2009. Miguel A. Arcones. All rights reserved. Manual for SOA Exam FM/CAS Exam 2.
33/51
Chapter 7. Derivatives markets. Section 7.6. Putcall parity.
Example 6
The current price of one share of XYZ stock is $75. Use the Table
1 for prices of puts and calls. Steve buys 500 shares of XYZ stock.
Nicole buys 500 shares of XYZ stock and a put option on 500
shares of XYZ stock with a strike price 65 and an expiration date
one year from now. The premium of a 65strike put option is
$1.221977. The risk free annual eective rate of interest is 5%.
(iii) Calculate the minimum and maximum prots for Steve and
Nicole.
c 2009. Miguel A. Arcones. All rights reserved. Manual for SOA Exam FM/CAS Exam 2.
34/51
Chapter 7. Derivatives markets. Section 7.6. Putcall parity.
Example 6
The current price of one share of XYZ stock is $75. Use the Table
1 for prices of puts and calls. Steve buys 500 shares of XYZ stock.
Nicole buys 500 shares of XYZ stock and a put option on 500
shares of XYZ stock with a strike price 65 and an expiration date
one year from now. The premium of a 65strike put option is
$1.221977. The risk free annual eective rate of interest is 5%.
(iii) Calculate the minimum and maximum prots for Steve and
Nicole.
Solution: (iii) The minimum and maximum of Steves prot are
(500)(75) = 37500 and , respectively. The minimum Nicoles
prot is (500)(65 76.28307585) = 5641.537925. The maximum
Nicoles prot is .
c 2009. Miguel A. Arcones. All rights reserved. Manual for SOA Exam FM/CAS Exam 2.
35/51
Chapter 7. Derivatives markets. Section 7.6. Putcall parity.
Example 6
The current price of one share of XYZ stock is $75. Use the Table
1 for prices of puts and calls. Steve buys 500 shares of XYZ stock.
Nicole buys 500 shares of XYZ stock and a put option on 500
shares of XYZ stock with a strike price 65 and an expiration date
one year from now. The premium of a 65strike put option is
$1.221977. The risk free annual eective rate of interest is 5%.
(iv) Find spot prices at expiration at which each of Steve makes a
prot. Answer the previous question for Nicole.
c 2009. Miguel A. Arcones. All rights reserved. Manual for SOA Exam FM/CAS Exam 2.
36/51
Chapter 7. Derivatives markets. Section 7.6. Putcall parity.
Example 6
The current price of one share of XYZ stock is $75. Use the Table
1 for prices of puts and calls. Steve buys 500 shares of XYZ stock.
Nicole buys 500 shares of XYZ stock and a put option on 500
shares of XYZ stock with a strike price 65 and an expiration date
one year from now. The premium of a 65strike put option is
$1.221977. The risk free annual eective rate of interest is 5%.
(iv) Find spot prices at expiration at which each of Steve makes a
prot. Answer the previous question for Nicole.
Solution: (iv) Steves prot is positive if (500)(S
T
75) > 0,
i.e. if S
T
> 75. Nicoles prot is positive if (500)(max(65, S
T
)
76.28307585) > 0, ie. if S
T
> 76.28307585.
c 2009. Miguel A. Arcones. All rights reserved. Manual for SOA Exam FM/CAS Exam 2.
37/51
Chapter 7. Derivatives markets. Section 7.6. Putcall parity.
Next, we proof that the prot of buying an asset and a put is the
same as the prot of buying a call option. The payo for buying a
call option and a zerocoupon bond which pays the strike price at
expiration date is
max(0, S
T
K) + K = max(S
T
, K)
The payo for buying stock and a put option is
S
T
+ max(K S
T
, 0) = max(S
T
, K).
Since the two strategies have the same payo in the absence of
arbitrage, they have the same prot, i.e.
max(S
T
, K) K (Call(K, T))(1 + i )
T
=max(S
T
, K) (S
0
+ Put(K, T))(1 + i )
T
.
This equation is equivalent to the putcall parity:
(S
0
+ Put(K, T))(1 + i )
T
= K + Call(K, T)(1 + i )
T
.
c 2009. Miguel A. Arcones. All rights reserved. Manual for SOA Exam FM/CAS Exam 2.
38/51
Chapter 7. Derivatives markets. Section 7.6. Putcall parity.
Example 7
Michael buys 500 shares of XYZ stock and a 45strike fouryear
put for 500 shares of XYZ stock. Rita buys a 45strike fouryear
call for 500 shares of XYZ stock and invests P into a zerocoupon
bond. The annual rate of interest continuously compounded is
4.5%. Find P so that Michael and Rita have the same payo at
expiration.
Solution: Michaels payo at expiration is
500(S
4
+ max(45 S
4
, 0)) = 500 max(45, S
4
).
Ritas payo at expiration is
500 max(S
4
45, 0) + Pe
(0.045)(4)
=500 max(45, S
4
) (500)(45) + Pe
(0.045)(4)
.
Hence, P = (500)(45)e
(0.045)(4)
= 18793.57976.
c 2009. Miguel A. Arcones. All rights reserved. Manual for SOA Exam FM/CAS Exam 2.
39/51
Chapter 7. Derivatives markets. Section 7.6. Putcall parity.
Example 7
Michael buys 500 shares of XYZ stock and a 45strike fouryear
put for 500 shares of XYZ stock. Rita buys a 45strike fouryear
call for 500 shares of XYZ stock and invests P into a zerocoupon
bond. The annual rate of interest continuously compounded is
4.5%. Find P so that Michael and Rita have the same payo at
expiration.
Solution: Michaels payo at expiration is
500(S
4
+ max(45 S
4
, 0)) = 500 max(45, S
4
).
Ritas payo at expiration is
500 max(S
4
45, 0) + Pe
(0.045)(4)
=500 max(45, S
4
) (500)(45) + Pe
(0.045)(4)
.
Hence, P = (500)(45)e
(0.045)(4)
= 18793.57976.
c 2009. Miguel A. Arcones. All rights reserved. Manual for SOA Exam FM/CAS Exam 2.
40/51
Chapter 7. Derivatives markets. Section 7.6. Putcall parity.
Cap.
If you have an obligation to buy stock in the future, you have a
short position on the stock. You will experience a loss, when the
price of the stock price rises. You can insure a short position by
purchasing a call option. Buying a call option when you are in a
short position is called a cap. The payo of having a short position
and buying a call option is
S
T
+ max(S
T
K, 0) = max(S
T
, K) = min(S
T
, K).
The payo of having a purchased put combined with borrowing the
strike price at closing is
max(K S
T
, 0) K = max(S
T
, K) = min(S
T
, K),
which is the same as before.
c 2009. Miguel A. Arcones. All rights reserved. Manual for SOA Exam FM/CAS Exam 2.
41/51
Chapter 7. Derivatives markets. Section 7.6. Putcall parity.
Suppose that you have a short position on a stock. Consider the
following two strategies:
1. Buy a call option. The payo at expiration is
S
T
+ max(S
T
K, 0) = max(S
T
, K).
The prot at expiration is
max(S
T
, K) Call(K, T)(1 + i )
T
.
2. Buy stock (to cover the short) and a put option. The payo at
expiration is
max(K S
T
, 0) = K + max(S
T
, K).
The prot at expiration is
K + max(S
T
, K) (S
0
+ Put(K, T))(1 + i )
T
.
By the putcall parity formula, both strategies have the same
prot.
c 2009. Miguel A. Arcones. All rights reserved. Manual for SOA Exam FM/CAS Exam 2.
42/51
Chapter 7. Derivatives markets. Section 7.6. Putcall parity.
Example 8
Heather has a short position in 1200 shares of stock XYZ. She is
supposed to return this stock one year from now. To insure her
position, she buys a $65strike call. The premium of a $65strike
call is $14.31722. The current price of one share of XYZ stock is
$75. The risk free annual eective rate of interest is 5%.
c 2009. Miguel A. Arcones. All rights reserved. Manual for SOA Exam FM/CAS Exam 2.
43/51
Chapter 7. Derivatives markets. Section 7.6. Putcall parity.
Example 8
Heather has a short position in 1200 shares of stock XYZ. She is
supposed to return this stock one year from now. To insure her
position, she buys a $65strike call. The premium of a $65strike
call is $14.31722. The current price of one share of XYZ stock is
$75. The risk free annual eective rate of interest is 5%.
(i) Make a table with Heathers prot when the spot price at expi-
ration is $40, $50, $60, $70, $80, $90.
c 2009. Miguel A. Arcones. All rights reserved. Manual for SOA Exam FM/CAS Exam 2.
44/51
Chapter 7. Derivatives markets. Section 7.6. Putcall parity.
Example 8
Heather has a short position in 1200 shares of stock XYZ. She is
supposed to return this stock one year from now. To insure her
position, she buys a $65strike call. The premium of a $65strike
call is $14.31722. The current price of one share of XYZ stock is
$75. The risk free annual eective rate of interest is 5%.
(i) Make a table with Heathers prot when the spot price at expi-
ration is $40, $50, $60, $70, $80, $90.
Solution: (i) Heathers prot is
(1200)

max(S
T
, K) Call(K, T)(1 + i )
T

=1200(max(S
T
, 65) 14.31722(1.05)
1
)
=1200(max(S
T
, 65) 15.033081200).
Spot Price 40 50 60 70 80 90
Hs prot 66039.70 78039.70 90039.70 96039.70 96039.70 96039.70
c 2009. Miguel A. Arcones. All rights reserved. Manual for SOA Exam FM/CAS Exam 2.
45/51
Chapter 7. Derivatives markets. Section 7.6. Putcall parity.
Example 8
Heather has a short position in 1200 shares of stock XYZ. She is
supposed to return this stock one year from now. To insure her
position, she buys a $65strike call. The premium of a $65strike
call is $14.31722. The current price of one share of XYZ stock is
$75. The risk free annual eective rate of interest is 5%.
(ii) Assuming that she does not buy the call, make a table with her
prot when the spot price at expiration is $40, $50, $60, $70, $80,
$90.
c 2009. Miguel A. Arcones. All rights reserved. Manual for SOA Exam FM/CAS Exam 2.
46/51
Chapter 7. Derivatives markets. Section 7.6. Putcall parity.
Example 8
Heather has a short position in 1200 shares of stock XYZ. She is
supposed to return this stock one year from now. To insure her
position, she buys a $65strike call. The premium of a $65strike
call is $14.31722. The current price of one share of XYZ stock is
$75. The risk free annual eective rate of interest is 5%.
(ii) Assuming that she does not buy the call, make a table with her
prot when the spot price at expiration is $40, $50, $60, $70, $80,
$90.
Solution: (ii) Heathers prot is (1200)S
T
.
Spot Price 40 50 60 70 80 90
Hs prot 48000 60000 72000 84000 96000 108000
c 2009. Miguel A. Arcones. All rights reserved. Manual for SOA Exam FM/CAS Exam 2.
47/51
Chapter 7. Derivatives markets. Section 7.6. Putcall parity.
Example 8
Heather has a short position in 1200 shares of stock XYZ. She is
supposed to return this stock one year from now. To insure her
position, she buys a $65strike call. The premium of a $65strike
call is $14.31722. The current price of one share of XYZ stock is
$75. The risk free annual eective rate of interest is 5%.
(iii) Draw the graphs of the prot versus the spot price at expiration
for the strategies in (i) and in (ii).
c 2009. Miguel A. Arcones. All rights reserved. Manual for SOA Exam FM/CAS Exam 2.
48/51
Chapter 7. Derivatives markets. Section 7.6. Putcall parity.
Example 8
Heather has a short position in 1200 shares of stock XYZ. She is
supposed to return this stock one year from now. To insure her
position, she buys a $65strike call. The premium of a $65strike
call is $14.31722. The current price of one share of XYZ stock is
$75. The risk free annual eective rate of interest is 5%.
(iii) Draw the graphs of the prot versus the spot price at expiration
for the strategies in (i) and in (ii).
Solution: (iii) The graph of prots is on Figure 3.
c 2009. Miguel A. Arcones. All rights reserved. Manual for SOA Exam FM/CAS Exam 2.
49/51
Chapter 7. Derivatives markets. Section 7.6. Putcall parity.
Notice that the possible loss for the short position can be arbitrarily
large. By buying the call Heather has limited her possible losses.
Figure 3: Prot for a long position and a long put.
c 2009. Miguel A. Arcones. All rights reserved. Manual for SOA Exam FM/CAS Exam 2.
50/51
Chapter 7. Derivatives markets. Section 7.6. Putcall parity.
Selling calls and puts.
Selling a option when there is a corresponding long position in the
underlying asset is called covered writing or option overwriting.
Naked writing occurs when the writer of an option does not have
a position in the asset.

A covered call is achieved by writing a call against a long


position on the stock. An investor holding a long position in
an asset may write a call to generate some income from the
asset.

A covered put is achieved by writing a put against a short


position on the stock.
c 2009. Miguel A. Arcones. All rights reserved. Manual for SOA Exam FM/CAS Exam 2.
51/51
Chapter 7. Derivatives markets. Section 7.6. Putcall parity.
An arbitrageur buys and sells call and puts. A way to limit risks in
the sales of options is to cover these positions. He also can match
opposite options. For example,

A purchase of Kstrike call option, a sale of a Kstrike put


option and a short forward cancel each other.

A sale of Kstrike call option, a purchase of a Kstrike put


option and a long forward cancel each other.
c 2009. Miguel A. Arcones. All rights reserved. Manual for SOA Exam FM/CAS Exam 2.
1/21
Chapter 7. Derivatives markets.
Manual for SOA Exam FM/CAS Exam 2.
Chapter 7. Derivatives markets.
7.7. Equity linked certicates of deposit.
c 2009. Miguel A. Arcones. All rights reserved.
Extract from:
Arcones Manual for the SOA Exam FM/CAS Exam 2,
Financial Mathematics. Fall 2009 Edition,
available at http://www.actexmadriver.com/
c 2009. Miguel A. Arcones. All rights reserved. Manual for SOA Exam FM/CAS Exam 2.
2/21
Chapter 7. Derivatives markets. Section 7.7. Equity linked certicates of deposit.
Equity linked certicates of deposit
To estimate the evolution of the stock market, stock indexes are
used. A stock market index is a listing of stocks and a way to
obtain the composite value of its components. The three most
used stock indexes are the (Dow Jones) Dow Jones Industrial
Average, (S & P 500) Standard & Poors 500 index, and the
NASDAQ Composite Index. Usually the composite value of an
index is a sort of average of the stocks in the index. However,
there are dierent ways to nd this average. Every stock market
index has its rules to nd its composite value.
c 2009. Miguel A. Arcones. All rights reserved. Manual for SOA Exam FM/CAS Exam 2.
3/21
Chapter 7. Derivatives markets. Section 7.7. Equity linked certicates of deposit.
The S & P 500 index consisting of 500 stocks of large corporations
selected by Standard & Poor. Standard & Poor is a nancial
company which specializes in providing independent credit rating
and index evaluation. The stocks on the S & P 500 trade in stock
markets, like the (NYSE) NYSE New York Stock Exchange and
(National Association of Securities Dealers Automated Quotations
system) NASDAQ. The New York Stock Exchange is the largest
equities marketplace in the world. NASDAQ is an electronic stock
exchange.
c 2009. Miguel A. Arcones. All rights reserved. Manual for SOA Exam FM/CAS Exam 2.
4/21
Chapter 7. Derivatives markets. Section 7.7. Equity linked certicates of deposit.
The Dow Jones Industrial Average is obtained averaging the
value of 30 stocks selected by the Dow Jones & Company. These
30 stocks are selected from largest and the most widely held public
companies in the USA across a range of industries except for
transport and utilities. Dow Jones & Company publishes the The
Wall Street Journal. The Wall Street Journal editors have a lot
input on the selection of the stocks in the Dow Jones Industrial
Average.
The NASDAQ Composite Index consists of all securities listed on
NASDAQ. It contains mainly stocks of technology and growth
companies.
Roughly, the dierence between the three indexes is on the type of
stocks which they represent. The S & P 500 focuses on all
largecap stocks in the market. The Dow Jones Industrial Average
focuses on a very selected group of large companies. NASDAQ
focuses on technology and fast growing companies.
c 2009. Miguel A. Arcones. All rights reserved. Manual for SOA Exam FM/CAS Exam 2.
5/21
Chapter 7. Derivatives markets. Section 7.7. Equity linked certicates of deposit.
An (ELCD) equity linked CD (or equity linked note, or equity
indexed CD, or market index linked CD) is an FDICinsured
certicate of deposit that ties the rate of return to the
performance of a stock index such as the S & P 500 and guarantee
a certain payment. Chase Manhattan Bank rst introduced
ELCDs in 1987. But, now many nancial institutions oer
ELCDs. Usually, the guarantee payment is the original principal.
The investor at expiration also gets a payment depending on the
performance of the stock index. Usually, there exists a
participation rate r , 0 < r 1, such that the investor gets the
guarantee payment plus rP max

S
T
S
0
1, 0

, where P is the
principal invested, S
T
is the index price at expiration, S
0
is the
spot price. Hence, usually, the payo of an ELCD is
P

1 + r max

S
T
S
0
1, 0

.
c 2009. Miguel A. Arcones. All rights reserved. Manual for SOA Exam FM/CAS Exam 2.
6/21
Chapter 7. Derivatives markets. Section 7.7. Equity linked certicates of deposit.
An investor is attracted to ELCDs because it has the potential for
market appreciation and diversication without risking capital.
Diversication is attained by using market indices, which combine
several stocks. Usually the return of the capital is FDIC insured.
One disadvantage is the possible loss of interest on the invested
principal. Notice that the smallest payo which the investor may
get is his invested principal, i.e. he does not get any interest. The
instrument is appropriate for conservative equity investors or xed
income investors who desire equity exposure with controlled risk.
c 2009. Miguel A. Arcones. All rights reserved. Manual for SOA Exam FM/CAS Exam 2.
7/21
Chapter 7. Derivatives markets. Section 7.7. Equity linked certicates of deposit.
Example 1
Aaron deposits $15000 in an ELCD, which provides 100% principal
protection and pays 80% of the appreciation of the S & P 500
three years from now. The index closes at 1500 on the day the
ELCD is issued.
c 2009. Miguel A. Arcones. All rights reserved. Manual for SOA Exam FM/CAS Exam 2.
8/21
Chapter 7. Derivatives markets. Section 7.7. Equity linked certicates of deposit.
Example 1
Aaron deposits $15000 in an ELCD, which provides 100% principal
protection and pays 80% of the appreciation of the S & P 500
three years from now. The index closes at 1500 on the day the
ELCD is issued.
(i) Find Aarons payo as a function of S
3
.
c 2009. Miguel A. Arcones. All rights reserved. Manual for SOA Exam FM/CAS Exam 2.
9/21
Chapter 7. Derivatives markets. Section 7.7. Equity linked certicates of deposit.
Example 1
Aaron deposits $15000 in an ELCD, which provides 100% principal
protection and pays 80% of the appreciation of the S & P 500
three years from now. The index closes at 1500 on the day the
ELCD is issued.
(i) Find Aarons payo as a function of S
3
.
Solution: (i) Aarons payo is
P

1 + r max

S
T
S
0
1, 0

=(15000)

1 + (0.80) max

S
T
1500
1, 0

.
c 2009. Miguel A. Arcones. All rights reserved. Manual for SOA Exam FM/CAS Exam 2.
10/21
Chapter 7. Derivatives markets. Section 7.7. Equity linked certicates of deposit.
Example 1
Aaron deposits $15000 in an ELCD, which provides 100% principal
protection and pays 80% of the appreciation of the S & P 500
three years from now. The index closes at 1500 on the day the
ELCD is issued.
(ii) Find the Aarons payo in the forward contract if S
3
is $1300,
$1400, $1500, $1600, $1700, $1800.
c 2009. Miguel A. Arcones. All rights reserved. Manual for SOA Exam FM/CAS Exam 2.
11/21
Chapter 7. Derivatives markets. Section 7.7. Equity linked certicates of deposit.
Example 1
Aaron deposits $15000 in an ELCD, which provides 100% principal
protection and pays 80% of the appreciation of the S & P 500
three years from now. The index closes at 1500 on the day the
ELCD is issued.
(ii) Find the Aarons payo in the forward contract if S
3
is $1300,
$1400, $1500, $1600, $1700, $1800.
Solution: (ii) Using that Aarons payo is
(15000)

1 + (0.80) max

S
T
1500
1, 0

,
Payo 15000 15000 15000 15800 16600 17400
S
3
1300 1400 1500 1600 1700 1800
c 2009. Miguel A. Arcones. All rights reserved. Manual for SOA Exam FM/CAS Exam 2.
12/21
Chapter 7. Derivatives markets. Section 7.7. Equity linked certicates of deposit.
Example 1
Aaron deposits $15000 in an ELCD, which provides 100% principal
protection and pays 80% of the appreciation of the S & P 500
three years from now. The index closes at 1500 on the day the
ELCD is issued.
(iii) Graph Aarons payo as a function of S
3
.
c 2009. Miguel A. Arcones. All rights reserved. Manual for SOA Exam FM/CAS Exam 2.
13/21
Chapter 7. Derivatives markets. Section 7.7. Equity linked certicates of deposit.
Example 1
Aaron deposits $15000 in an ELCD, which provides 100% principal
protection and pays 80% of the appreciation of the S & P 500
three years from now. The index closes at 1500 on the day the
ELCD is issued.
(iii) Graph Aarons payo as a function of S
3
.
Solution: (iii) Aarons payo is Figure 1.
c 2009. Miguel A. Arcones. All rights reserved. Manual for SOA Exam FM/CAS Exam 2.
14/21
Chapter 7. Derivatives markets. Section 7.7. Equity linked certicates of deposit.
Figure 1: Example 1. ELCD payo.
c 2009. Miguel A. Arcones. All rights reserved. Manual for SOA Exam FM/CAS Exam 2.
15/21
Chapter 7. Derivatives markets. Section 7.7. Equity linked certicates of deposit.
The payo of an ELCD is a combination of the payo of a long call
option and a long bond position. It is possible to create a synthetic
ELCD by buying a long call option and a zerocoupon bond.
c 2009. Miguel A. Arcones. All rights reserved. Manual for SOA Exam FM/CAS Exam 2.
16/21
Chapter 7. Derivatives markets. Section 7.7. Equity linked certicates of deposit.
The return on a ELCD is
P

1 + r max

S
T
S
0
1, 0

= P +
Pr
S
0
max(S
T
S
0
, 0),
Suppose that an investor buys a zerocoupon bond with face value
P and a S
0
strike call option for
Pr
S
0
shares. His payo at
expiration date is
P +
Pr
S
0
max(S
T
S
0
, 0).
The cost of making this investment is
P(1 + i )
T
+
Pr
S
0
Call(S
0
, T).
If there exist no arbitrage,
P = P(1 + i )
T
+
Pr
S
0
Call(S
0
, T).
Hence, r =
(1(1+i )
T
)S
0
Call(S
0
,T)
.
c 2009. Miguel A. Arcones. All rights reserved. Manual for SOA Exam FM/CAS Exam 2.
17/21
Chapter 7. Derivatives markets. Section 7.7. Equity linked certicates of deposit.
Example 2
The riskfree eective rate of interest is 7%. The current price of
the S & P 500 is 1500. The price of an European call with strike
1500 and expiration date in 3 years is 400. Find the participation
rate of a threeyear ELCD which provides 100% principal
protection.
Solution: We have that
r =
(1 (1 + i )
T
)S
0
Call(S
0
, T)
=
(1 (1.07)
3
)1500
400
= 0.6888829617 = 68.88829617%.
c 2009. Miguel A. Arcones. All rights reserved. Manual for SOA Exam FM/CAS Exam 2.
18/21
Chapter 7. Derivatives markets. Section 7.7. Equity linked certicates of deposit.
Example 2
The riskfree eective rate of interest is 7%. The current price of
the S & P 500 is 1500. The price of an European call with strike
1500 and expiration date in 3 years is 400. Find the participation
rate of a threeyear ELCD which provides 100% principal
protection.
Solution: We have that
r =
(1 (1 + i )
T
)S
0
Call(S
0
, T)
=
(1 (1.07)
3
)1500
400
= 0.6888829617 = 68.88829617%.
c 2009. Miguel A. Arcones. All rights reserved. Manual for SOA Exam FM/CAS Exam 2.
19/21
Chapter 7. Derivatives markets. Section 7.7. Equity linked certicates of deposit.
Suppose that an ELCD return all the principal invested at
expiration, oers a guaranteed rate of return g, where g < i , and a
participation rate r on the S & P 500 stock index. The payo of
this ELCD is
P(1+g)
T
+Pr max

S
T
S
0
S
0
, 0

= P(1+g)
T
+
Pr
S
0
max(S
T
S
0
, 0).
We can get this payo by buying a zerocoupon bond with face
value P(1 + g)
T
and a S
0
strike call option for
Pr
S
0
shares. The
cost of this portfolio is
P(1 + g)
T
(1 + i )
T
+
Pr
S
0
Call(S
0
, T).
If there exist no arbitrage,
P = P(1 + g)
T
(1 + i )
T
+
Pr
S
0
Call(S
0
, T)
and
r =
(1 (1 + g)
T
(1 + i )
T
)S
0
Call(S
0
, T)
.
c 2009. Miguel A. Arcones. All rights reserved. Manual for SOA Exam FM/CAS Exam 2.
20/21
Chapter 7. Derivatives markets. Section 7.7. Equity linked certicates of deposit.
Example 3
The riskfree eective rate of interest is 5%. The current price of
the S & P 500 is 1500. The price of an European call with strike
1500 and expiration date in six month is 125. Find the
participation rate of a ELCD which provides 100% principal
protection and a guaranteed annual interest rate of 1.5%.
Solution:
r =
(1 (1 + g)
T
(1 + i )
T
)S
0
Call(S
0
, T)
=
(1 (1.015)
0.5
(1.05)
0.5
)1500
125
= 0.201695 = 20.1695%.
c 2009. Miguel A. Arcones. All rights reserved. Manual for SOA Exam FM/CAS Exam 2.
21/21
Chapter 7. Derivatives markets. Section 7.7. Equity linked certicates of deposit.
Example 3
The riskfree eective rate of interest is 5%. The current price of
the S & P 500 is 1500. The price of an European call with strike
1500 and expiration date in six month is 125. Find the
participation rate of a ELCD which provides 100% principal
protection and a guaranteed annual interest rate of 1.5%.
Solution:
r =
(1 (1 + g)
T
(1 + i )
T
)S
0
Call(S
0
, T)
=
(1 (1.015)
0.5
(1.05)
0.5
)1500
125
= 0.201695 = 20.1695%.
c 2009. Miguel A. Arcones. All rights reserved. Manual for SOA Exam FM/CAS Exam 2.
1/97
Chapter 7. Derivatives markets.
Manual for SOA Exam FM/CAS Exam 2.
Chapter 7. Derivatives markets.
Section 7.8. Spreads.
c 2009. Miguel A. Arcones. All rights reserved.
Extract from:
Arcones Manual for the SOA Exam FM/CAS Exam 2,
Financial Mathematics. Fall 2009 Edition,
available at http://www.actexmadriver.com/
c 2009. Miguel A. Arcones. All rights reserved. Manual for SOA Exam FM/CAS Exam 2.
2/97
Chapter 7. Derivatives markets. Section 7.8. Spreads.
Spreads
An option spread (or a vertical spread) is a combination of only
calls or only puts, in which some options are bought and some
others are sold. By buying/selling several call/puts we can create
portfolios useful for many dierent objectives. A ratio spread is a
combination of buying m calls at one strike price and selling n calls
at a dierent strike price.
c 2009. Miguel A. Arcones. All rights reserved. Manual for SOA Exam FM/CAS Exam 2.
3/97
Chapter 7. Derivatives markets. Section 7.8. Spreads.
Speculating on the increase of an asset price. Bull spread.
Denition 1
A bull spread consists on buying a K
1
strike call and selling a
K
2
strike call, both with the same expiration date T and nominal
amount, where 0 < K
1
< K
2
.
A way to speculate on the increase of an asset price is buying the
asset. This position needs a lot of investment. Another way to
speculate on the increase of an asset price is to buy a call option.
A bull spread allows to speculate on increase of an asset price by
making a limited investment.
c 2009. Miguel A. Arcones. All rights reserved. Manual for SOA Exam FM/CAS Exam 2.
4/97
Chapter 7. Derivatives markets. Section 7.8. Spreads.
The payo for buying a K
1
strike call is max(S
T
K
1
, 0).
The payo for selling a K
2
strike call is max(S
T
K
2
, 0).
T
E

max(S
T
K
1
, 0)
S
T
K
1
K
2
T
E
d
d
d
d
d
max(S
T
K
2
, 0)
S
T
K
1
K
2
Figure 1: Payo of the calls in a bull spread
c 2009. Miguel A. Arcones. All rights reserved. Manual for SOA Exam FM/CAS Exam 2.
5/97
Chapter 7. Derivatives markets. Section 7.8. Spreads.
The bull spread payo is
max(S
T
K
1
, 0) max(S
T
K
2
, 0)
=max(S
T
K
1
, 0) + K
2
K
1
max(S
T
K
1
, K
2
K
1
)
=max(S
T
K
1
, 0) + K
2
K
1
max(S
T
K
1
, 0, K
2
K
1
)
=max(S
T
K
1
, 0) + K
2
K
1
max(max(S
T
K
1
, 0), K
2
K
1
)
=min(max(S
T
K
1
, 0), K
2
K
1
)
=

0 if S
T
< K
1
,
S
T
K
1
if K
1
S
T
< K
2
,
K
2
K
1
if K
2
S
T
.
c 2009. Miguel A. Arcones. All rights reserved. Manual for SOA Exam FM/CAS Exam 2.
6/97
Chapter 7. Derivatives markets. Section 7.8. Spreads.
T
E

max(S
T
K
1
, 0) max(S
T
K
2
, 0)
S
T
K
1
K
2
Figure 2: Payo of a bull spread
c 2009. Miguel A. Arcones. All rights reserved. Manual for SOA Exam FM/CAS Exam 2.
7/97
Chapter 7. Derivatives markets. Section 7.8. Spreads.
The prot of a bull spread is
min(max(S
T
K
1
, 0), K
2
K
1
)
(Call(K
1
, T) Call(K
2
, T))(1 + i )
T
=

(Call(K
1
, T) Call(K
2
, T))(1 + i )
T
if S
T
< K
1
,
S
T
K
1
(Call(K
1
, T) Call(K
2
, T))(1 + i )
T
if K
1
S
T
< K
2
,
K
2
K
1
(Call(K
1
, T) Call(K
2
, T))(1 + i )
T
if K
2
S
T
.
Figure 3 shows a graph of the prot of a bull spread. Notice that
the prot is positive for values of S
T
large enough.
c 2009. Miguel A. Arcones. All rights reserved. Manual for SOA Exam FM/CAS Exam 2.
8/97
Chapter 7. Derivatives markets. Section 7.8. Spreads.
Figure 3: Prot for a bull spread.
c 2009. Miguel A. Arcones. All rights reserved. Manual for SOA Exam FM/CAS Exam 2.
9/97
Chapter 7. Derivatives markets. Section 7.8. Spreads.
In this section, we will use the values of calls/puts in Table 1.
Table 1: Prices of some calls and some puts.
K 65 70 75 80 85
Call(K, T) 14.31722 10.75552 7.78971 5.444947 3.680736
Put(K, T) 1.221977 2.422184 4.218281 6.635423 9.633117
c 2009. Miguel A. Arcones. All rights reserved. Manual for SOA Exam FM/CAS Exam 2.
10/97
Chapter 7. Derivatives markets. Section 7.8. Spreads.
Example 1
(Use Table 1) Ronald buys a $70strike call and sells a $85strike
call for 100 shares of XYZ stock. Both have expiration date one
year from now. The current price of one share of XYZ stock is
$75. The risk free annual eective rate of interest is 5%. The
premium per share of $70strike and $85strike calls are 10.75552
and 3.680736 respectively.
c 2009. Miguel A. Arcones. All rights reserved. Manual for SOA Exam FM/CAS Exam 2.
11/97
Chapter 7. Derivatives markets. Section 7.8. Spreads.
Example 1
(Use Table 1) Ronald buys a $70strike call and sells a $85strike
call for 100 shares of XYZ stock. Both have expiration date one
year from now. The current price of one share of XYZ stock is
$75. The risk free annual eective rate of interest is 5%. The
premium per share of $70strike and $85strike calls are 10.75552
and 3.680736 respectively.
(i) Find the prot at expiration as a function of the strike price.
c 2009. Miguel A. Arcones. All rights reserved. Manual for SOA Exam FM/CAS Exam 2.
12/97
Chapter 7. Derivatives markets. Section 7.8. Spreads.
Example 1
(Use Table 1) Ronald buys a $70strike call and sells a $85strike
call for 100 shares of XYZ stock. Both have expiration date one
year from now. The current price of one share of XYZ stock is
$75. The risk free annual eective rate of interest is 5%. The
premium per share of $70strike and $85strike calls are 10.75552
and 3.680736 respectively.
(i) Find the prot at expiration as a function of the strike price.
Solution: (i) Ronalds prot is
(100) (min(max(S
T
70, 0), 85 70) (10.75552 3.680736)(1.05))
=(100) (min(max(S
T
70, 0), 85 70) 7.428523)
=

(100)(7.428523) if S
T
< 70,
(100)(S
T
70 7.428523) if 70 S
T
< 85,
(100)(85 70 7.428523) if 85 S
T
.
c 2009. Miguel A. Arcones. All rights reserved. Manual for SOA Exam FM/CAS Exam 2.
13/97
Chapter 7. Derivatives markets. Section 7.8. Spreads.
Example 1
(Use Table 1) Ronald buys a $70strike call and sells a $85strike
call for 100 shares of XYZ stock. Both have expiration date one
year from now. The current price of one share of XYZ stock is
$75. The risk free annual eective rate of interest is 5%. The
premium per share of $70strike and $85strike calls are 10.75552
and 3.680736 respectively.
(ii) Make a table with Ronalds prot when the spot price at expi-
ration is $65, $70, $75, $80, $85, $90.
c 2009. Miguel A. Arcones. All rights reserved. Manual for SOA Exam FM/CAS Exam 2.
14/97
Chapter 7. Derivatives markets. Section 7.8. Spreads.
Example 1
(Use Table 1) Ronald buys a $70strike call and sells a $85strike
call for 100 shares of XYZ stock. Both have expiration date one
year from now. The current price of one share of XYZ stock is
$75. The risk free annual eective rate of interest is 5%. The
premium per share of $70strike and $85strike calls are 10.75552
and 3.680736 respectively.
(ii) Make a table with Ronalds prot when the spot price at expi-
ration is $65, $70, $75, $80, $85, $90.
Solution: (ii)
Spot Price 65 70 75
Ronalds prot 742.8523 742.8523 242.8523
Spot Price 80 85 90
Ronalds prot 257.1477 757.1477 757.1477
c 2009. Miguel A. Arcones. All rights reserved. Manual for SOA Exam FM/CAS Exam 2.
15/97
Chapter 7. Derivatives markets. Section 7.8. Spreads.
Example 1
(Use Table 1) Ronald buys a $70strike call and sells a $85strike
call for 100 shares of XYZ stock. Both have expiration date one
year from now. The current price of one share of XYZ stock is
$75. The risk free annual eective rate of interest is 5%. The
premium per share of $70strike and $85strike calls are 10.75552
and 3.680736 respectively.
(iii) Draw the graph of Ronalds prot.
c 2009. Miguel A. Arcones. All rights reserved. Manual for SOA Exam FM/CAS Exam 2.
16/97
Chapter 7. Derivatives markets. Section 7.8. Spreads.
Example 1
(Use Table 1) Ronald buys a $70strike call and sells a $85strike
call for 100 shares of XYZ stock. Both have expiration date one
year from now. The current price of one share of XYZ stock is
$75. The risk free annual eective rate of interest is 5%. The
premium per share of $70strike and $85strike calls are 10.75552
and 3.680736 respectively.
(iii) Draw the graph of Ronalds prot.
Solution: (ii) The graph of the prot is Figure 3.
c 2009. Miguel A. Arcones. All rights reserved. Manual for SOA Exam FM/CAS Exam 2.
17/97
Chapter 7. Derivatives markets. Section 7.8. Spreads.
Let 0 < K
1
< K
2
. We have that
[long K
1
strike call ] + [short K
1
strike put ]
[buying asset for K
1
at time T],
[long K
2
strike call ] + [short K
2
strike put ]
[buying asset for K
2
at time T].
Since the two investment strategies dier by a constant, they have
the same prot. Hence, so do the following strategies,
[long K
1
strike call ] + [short K
2
strike call ],
[long K
1
strike put ] + [short K
2
strike put ].
In other words, buying a K
1
strike call and selling a K
2
strike call
has the same prot as buying a K
1
strike put and selling a
K
2
strike put.
We can form a bull spread either buying a K
1
strike call and selling
a K
2
strike call, or buying a K
1
strike put and selling a K
2
strike
put.
c 2009. Miguel A. Arcones. All rights reserved. Manual for SOA Exam FM/CAS Exam 2.
18/97
Chapter 7. Derivatives markets. Section 7.8. Spreads.
Example 2
The current price of XYZ stock is $75 per share. The eective
annual interest rate is 5%. Elizabeth, Daniel and Catherine believe
that the price of XYZ stock is going to appreciate signicantly in
the next year. Each person has $10000 to invest. The premium of
a oneyear 85strike call option is 3.680736 per share. The
premium of a oneyear 75strike call option is 7.78971 per share.
Elizabeth buys a oneyear zerocoupon bond for $10000. She also
enters into a oneyear forward contract on XYZ stock worth equal
to the her bond payo at redemption. Daniel buys a oneyear
85strike call option which costs $10000. Catherine buys a
oneyear 75strike call option and sells a oneyear 85strike call
option. The nominal amounts on both calls are the same. The
dierence between the cost of the 85strike call option and the
75strike call option is 10000. Suppose that the stock price at
redemption is 90 per share. Calculate the prots and the yield
rates for Elizabeth, Daniel and Catherine. Which one makes a
bigger prot?
c 2009. Miguel A. Arcones. All rights reserved. Manual for SOA Exam FM/CAS Exam 2.
19/97
Chapter 7. Derivatives markets. Section 7.8. Spreads.
Solution: The price of a long forward is 75(1.05) = 78.75. So,
Elizabeth long forward is for
(10000)(1.05)
78.75
= 133.333333 shares.
Elizabeths prot is 133.333333(90 78.75) = 1500. Her yield rate
is
1500
10000
= 15%.
The nominal amount in Daniels call is
10000
3.680736
= 2716.847935
shares. Daniels prot is 2716.847935(90 85) = 13584.24.
Daniels yield of return is
13584.24
10000
= 135.8424%.
The nominal amount in Catherines calls is
10000
7.789713.680736
= 2433.697561 shares. Catherines prot is
2433.697561(85 75) = 24336.97561. Catherines yield of return
is
24336.97561
10000
= 243.3697561%.
Catherines prot is biggest.
c 2009. Miguel A. Arcones. All rights reserved. Manual for SOA Exam FM/CAS Exam 2.
20/97
Chapter 7. Derivatives markets. Section 7.8. Spreads.
Speculating on the decrease of an asset price. Bear spread.
A bear spread is precisely the opposite of a bull spread. Suppose
that you want to speculate on the price of an asset decreasing. Let
0 < K
1
< K
2
. Consider selling a K
1
strike call and buying a
K
2
strike call, both with the same expiration date T. The prot is
min(max(S
T
K
1
, 0), K
2
K
1
)+(Call(K
1
, T)Call(K
2
, T))(1+i )
T
or

(Call(K
1
, T) Call(K
2
, T))(1 + i )
T
if S
T
< K
1
,
K
1
S
T
+ (Call(K
1
, T) Call(K
2
, T))(1 + i )
T
if K
1
S
T
< K
2
,
K
1
K
2
+ (Call(K
1
, T) Call(K
2
, T))(1 + i )
T
if K
2
S
T
.
A graph of this prot is in Figure 4. Notice that the prot is
positive for values of S
T
small enough.
c 2009. Miguel A. Arcones. All rights reserved. Manual for SOA Exam FM/CAS Exam 2.
21/97
Chapter 7. Derivatives markets. Section 7.8. Spreads.
Figure 4: Prot for a bear spread.
c 2009. Miguel A. Arcones. All rights reserved. Manual for SOA Exam FM/CAS Exam 2.
22/97
Chapter 7. Derivatives markets. Section 7.8. Spreads.
Example 3
(Use Table 1) Rebecca sells a $65strike call and buys a $80strike
call for 1000 shares of XYZ stock. Both have expiration date one
year from now. The current price of one share of XYZ stock is
$75. The risk free annual eective rate of interest is 5%. The
premium per share of $65strike and $80strike calls are 14.31722
and 5.444947 respectively.
c 2009. Miguel A. Arcones. All rights reserved. Manual for SOA Exam FM/CAS Exam 2.
23/97
Chapter 7. Derivatives markets. Section 7.8. Spreads.
Example 3
(Use Table 1) Rebecca sells a $65strike call and buys a $80strike
call for 1000 shares of XYZ stock. Both have expiration date one
year from now. The current price of one share of XYZ stock is
$75. The risk free annual eective rate of interest is 5%. The
premium per share of $65strike and $80strike calls are 14.31722
and 5.444947 respectively.
(i) Find Rebeccas prot as a function of the spot price at expiration.
c 2009. Miguel A. Arcones. All rights reserved. Manual for SOA Exam FM/CAS Exam 2.
24/97
Chapter 7. Derivatives markets. Section 7.8. Spreads.
Example 3
(Use Table 1) Rebecca sells a $65strike call and buys a $80strike
call for 1000 shares of XYZ stock. Both have expiration date one
year from now. The current price of one share of XYZ stock is
$75. The risk free annual eective rate of interest is 5%. The
premium per share of $65strike and $80strike calls are 14.31722
and 5.444947 respectively.
(i) Find Rebeccas prot as a function of the spot price at expiration.
Solution: (i) Rebeccas prot is
(100) max(S
T
65, 0) + (100) max(S
T
80, 0)
+ (100)(14.31722 5.444947)(1.05)
=(100) min(max(S
T
65, 0), 80 65) + (100)(9.31588665)
=

(100)(9.31588665) if S
T
< 65
(100)(65 S
T
+ 9.31588665) if 65 S
T
< 80
(100)(65 80 + 9.31588665) if 80 S
T
.
c 2009. Miguel A. Arcones. All rights reserved. Manual for SOA Exam FM/CAS Exam 2.
25/97
Chapter 7. Derivatives markets. Section 7.8. Spreads.
Example 3
(Use Table 1) Rebecca sells a $65strike call and buys a $80strike
call for 1000 shares of XYZ stock. Both have expiration date one
year from now. The current price of one share of XYZ stock is
$75. The risk free annual eective rate of interest is 5%. The
premium per share of $65strike and $80strike calls are 14.31722
and 5.444947 respectively.
(ii) Make a table with Rebeccas prot when the spot price at expi-
ration is $60, $65, $70, $75, $80, $85, $90.
c 2009. Miguel A. Arcones. All rights reserved. Manual for SOA Exam FM/CAS Exam 2.
26/97
Chapter 7. Derivatives markets. Section 7.8. Spreads.
Example 3
(Use Table 1) Rebecca sells a $65strike call and buys a $80strike
call for 1000 shares of XYZ stock. Both have expiration date one
year from now. The current price of one share of XYZ stock is
$75. The risk free annual eective rate of interest is 5%. The
premium per share of $65strike and $80strike calls are 14.31722
and 5.444947 respectively.
(ii) Make a table with Rebeccas prot when the spot price at expi-
ration is $60, $65, $70, $75, $80, $85, $90.
Solution: (ii)
Spot Price 60 65 70 75
Rebeccas prot 931.59 931.59 431.59 68.41
Spot Price 75 80 85 90
Rebeccas prot 68.41 568.41 568.41 568.41
c 2009. Miguel A. Arcones. All rights reserved. Manual for SOA Exam FM/CAS Exam 2.
27/97
Chapter 7. Derivatives markets. Section 7.8. Spreads.
Example 3
(Use Table 1) Rebecca sells a $65strike call and buys a $80strike
call for 1000 shares of XYZ stock. Both have expiration date one
year from now. The current price of one share of XYZ stock is
$75. The risk free annual eective rate of interest is 5%. The
premium per share of $65strike and $80strike calls are 14.31722
and 5.444947 respectively.
(iii) Draw the graph of Rebeccas prot.
c 2009. Miguel A. Arcones. All rights reserved. Manual for SOA Exam FM/CAS Exam 2.
28/97
Chapter 7. Derivatives markets. Section 7.8. Spreads.
Example 3
(Use Table 1) Rebecca sells a $65strike call and buys a $80strike
call for 1000 shares of XYZ stock. Both have expiration date one
year from now. The current price of one share of XYZ stock is
$75. The risk free annual eective rate of interest is 5%. The
premium per share of $65strike and $80strike calls are 14.31722
and 5.444947 respectively.
(iii) Draw the graph of Rebeccas prot.
Solution: (iii) Figure 4 shows the graph of the prot.
c 2009. Miguel A. Arcones. All rights reserved. Manual for SOA Exam FM/CAS Exam 2.
29/97
Chapter 7. Derivatives markets. Section 7.8. Spreads.
Collar
A collar is the purchase of a put option at a strike price and the
sale of a call option at a higher strike price. Let K
1
be the strike
price of the put option. Let K
2
be the strike price of the call
option. Assume that K
1
< K
2
. The prot of this strategy is
max(K
1
S
T
, 0) max(S
T
K
2
, 0)
(Put(K
1
, T) Call(K
2
, T))(1 + i )
T
=

K
1
S
T
(Put(K
1
, T) Call(K
2
, T))(1 + i )
T
if S
T
< K
1
,
(Put(K
1
, T) Call(K
2
, T))(1 + i )
T
if K
1
S
T
< K
2
,
K
2
S
T
(Put(K
1
, T) Call(K
2
, T))(1 + i )
T
if K
2
S
T
.
A collar can be use to speculate on the decrease of price of an
asset.
The collar width is the dierence between the call strike and the
put strike.
c 2009. Miguel A. Arcones. All rights reserved. Manual for SOA Exam FM/CAS Exam 2.
30/97
Chapter 7. Derivatives markets. Section 7.8. Spreads.
Example 4
(Use Table 1) Toto buys a $65strike put option and sells a
$80strike call for 100 shares of XYZ stock. Both have expiration
date one year from now. The current price of one share of XYZ
stock is $75. The risk free annual eective rate of interest is 5%.
The premium of a $65strike put option is 1.221977 per share.
The premium of a $80strike call option is 5.444947 per share.
c 2009. Miguel A. Arcones. All rights reserved. Manual for SOA Exam FM/CAS Exam 2.
31/97
Chapter 7. Derivatives markets. Section 7.8. Spreads.
Example 4
(Use Table 1) Toto buys a $65strike put option and sells a
$80strike call for 100 shares of XYZ stock. Both have expiration
date one year from now. The current price of one share of XYZ
stock is $75. The risk free annual eective rate of interest is 5%.
The premium of a $65strike put option is 1.221977 per share.
The premium of a $80strike call option is 5.444947 per share.
(i) Find Totos prot as a function of the spot price at expiration.
c 2009. Miguel A. Arcones. All rights reserved. Manual for SOA Exam FM/CAS Exam 2.
32/97
Chapter 7. Derivatives markets. Section 7.8. Spreads.
Example 4
(Use Table 1) Toto buys a $65strike put option and sells a
$80strike call for 100 shares of XYZ stock. Both have expiration
date one year from now. The current price of one share of XYZ
stock is $75. The risk free annual eective rate of interest is 5%.
The premium of a $65strike put option is 1.221977 per share.
The premium of a $80strike call option is 5.444947 per share.
(i) Find Totos prot as a function of the spot price at expiration.
Solution: (i) Totos prot is
(100) (max(65 S
T
, 0) max(S
T
80, 0)
(1.221977 5.444947)(1.05))
=(100) (max(65 S
T
, 0) max(S
T
80, 0) + 4.4341185)
=

(100)(65 S
T
+ 4.4341185) if S
T
< 65,
100(4.4341185) if 65 S
T
< 80,
100(80 S
T
+ 4.4341185) if 80 S
T
.
c 2009. Miguel A. Arcones. All rights reserved. Manual for SOA Exam FM/CAS Exam 2.
33/97
Chapter 7. Derivatives markets. Section 7.8. Spreads.
Example 4
(Use Table 1) Toto buys a $65strike put option and sells a
$80strike call for 100 shares of XYZ stock. Both have expiration
date one year from now. The current price of one share of XYZ
stock is $75. The risk free annual eective rate of interest is 5%.
The premium of a $65strike put option is 1.221977 per share.
The premium of a $80strike call option is 5.444947 per share.
(ii) Make a table with Totos prot when the spot price at expiration
is $55, $60, $65, $70, $75, $80, $85.
c 2009. Miguel A. Arcones. All rights reserved. Manual for SOA Exam FM/CAS Exam 2.
34/97
Chapter 7. Derivatives markets. Section 7.8. Spreads.
Example 4
(Use Table 1) Toto buys a $65strike put option and sells a
$80strike call for 100 shares of XYZ stock. Both have expiration
date one year from now. The current price of one share of XYZ
stock is $75. The risk free annual eective rate of interest is 5%.
The premium of a $65strike put option is 1.221977 per share.
The premium of a $80strike call option is 5.444947 per share.
(ii) Make a table with Totos prot when the spot price at expiration
is $55, $60, $65, $70, $75, $80, $85.
Solution: (ii) A table with Totos prot is
Spot Price 55 60 65 70
Totos prot 1443.41 943.41 443.41 443.41
Spot Price 75 80 85 90
Totos prot 443.41 443.41 56.59 556.59
c 2009. Miguel A. Arcones. All rights reserved. Manual for SOA Exam FM/CAS Exam 2.
35/97
Chapter 7. Derivatives markets. Section 7.8. Spreads.
Example 4
(Use Table 1) Toto buys a $65strike put option and sells a
$80strike call for 100 shares of XYZ stock. Both have expiration
date one year from now. The current price of one share of XYZ
stock is $75. The risk free annual eective rate of interest is 5%.
The premium of a $65strike put option is 1.221977 per share.
The premium of a $80strike call option is 5.444947 per share.
(iii) Draw the graph of Totos prot.
c 2009. Miguel A. Arcones. All rights reserved. Manual for SOA Exam FM/CAS Exam 2.
36/97
Chapter 7. Derivatives markets. Section 7.8. Spreads.
Example 4
(Use Table 1) Toto buys a $65strike put option and sells a
$80strike call for 100 shares of XYZ stock. Both have expiration
date one year from now. The current price of one share of XYZ
stock is $75. The risk free annual eective rate of interest is 5%.
The premium of a $65strike put option is 1.221977 per share.
The premium of a $80strike call option is 5.444947 per share.
(iii) Draw the graph of Totos prot.
Solution: (iii) The graph of the prot is on Figure 5.
c 2009. Miguel A. Arcones. All rights reserved. Manual for SOA Exam FM/CAS Exam 2.
37/97
Chapter 7. Derivatives markets. Section 7.8. Spreads.
Figure 5: Prot for a collar.
c 2009. Miguel A. Arcones. All rights reserved. Manual for SOA Exam FM/CAS Exam 2.
38/97
Chapter 7. Derivatives markets. Section 7.8. Spreads.
Written collar
A written collar is a reverse collar.
c 2009. Miguel A. Arcones. All rights reserved. Manual for SOA Exam FM/CAS Exam 2.
39/97
Chapter 7. Derivatives markets. Section 7.8. Spreads.
Collars are used to insure a long position on a stock. This position
is called a collared stock. A collared stock involves buying the
index, buy a K
1
strike put option and selling a K
2
strike call
option, where K
1
< K
2
. The payo per share of this strategy is
S
T
+ max(K
1
S
T
, 0) max(S
T
K
2
, 0)
=max(K
1
, S
T
) max(S
T
, K
2
) + K
2
=max(K
1
, S
T
) max(S
T
, K
1
, K
2
) + K
2
=min(max(K
1
, S
T
), K
2
)
The prot per share of this portfolio is
min(max(K
1
, S
T
), K
2
) (S
0
+ Put(K
1
, T) Call(K
2
, T))(1 + i )
T
=

K
1
(S
0
+ Put(K
1
, T) Call(K
2
, T))(1 + i )
T
if S
T
< K
1
,
S
T
(S
0
+ Put(K
1
, T) Call(K
2
, T))(1 + i )
T
if K
1
S
T
< K
2
,
K
2
(S
0
+ Put(K
1
, T) Call(K
2
, T))(1 + i )
T
if K
2
S
T
.
c 2009. Miguel A. Arcones. All rights reserved. Manual for SOA Exam FM/CAS Exam 2.
40/97
Chapter 7. Derivatives markets. Section 7.8. Spreads.
Example 5
(Use Table 1) Maggie buys 100 shares of XYZ stock, a $65strike
put option and sells a $80strike call. Both options are for 100
shares of XYZ stock and have expiration date one year from now.
The current price of one share of XYZ stock is $75. The risk free
annual eective rate of interest is 5%. The premium per share of a
$65strike put option is 1.221977. The premium per share of a
$80strike call is 5.444947.
c 2009. Miguel A. Arcones. All rights reserved. Manual for SOA Exam FM/CAS Exam 2.
41/97
Chapter 7. Derivatives markets. Section 7.8. Spreads.
Example 5
(Use Table 1) Maggie buys 100 shares of XYZ stock, a $65strike
put option and sells a $80strike call. Both options are for 100
shares of XYZ stock and have expiration date one year from now.
The current price of one share of XYZ stock is $75. The risk free
annual eective rate of interest is 5%. The premium per share of a
$65strike put option is 1.221977. The premium per share of a
$80strike call is 5.444947.
(i) Find Maggies prot as a function of the spot price at expiration.
c 2009. Miguel A. Arcones. All rights reserved. Manual for SOA Exam FM/CAS Exam 2.
42/97
Chapter 7. Derivatives markets. Section 7.8. Spreads.
Example 5
(Use Table 1) Maggie buys 100 shares of XYZ stock, a $65strike
put option and sells a $80strike call. Both options are for 100
shares of XYZ stock and have expiration date one year from now.
The current price of one share of XYZ stock is $75. The risk free
annual eective rate of interest is 5%. The premium per share of a
$65strike put option is 1.221977. The premium per share of a
$80strike call is 5.444947.
(i) Find Maggies prot as a function of the spot price at expiration.
Solution: (i) Maggies prot is
(100)

min(max(65, S
T
), 80) (75 + 1.221977 5.444947)(1.05)
1

=(100) (min(max(65, S
T
), 80) 74.315882)
=

100(65 74.315882) if S
T
< 65,
100(S
T
74.315882) if 65 S
T
< 80,
100(80 74.315882) if 80 S
T
.
c 2009. Miguel A. Arcones. All rights reserved. Manual for SOA Exam FM/CAS Exam 2.
43/97
Chapter 7. Derivatives markets. Section 7.8. Spreads.
Example 5
(Use Table 1) Maggie buys 100 shares of XYZ stock, a $65strike
put option and sells a $80strike call. Both options are for 100
shares of XYZ stock and have expiration date one year from now.
The current price of one share of XYZ stock is $75. The risk free
annual eective rate of interest is 5%. The premium per share of a
$65strike put option is 1.221977. The premium per share of a
$80strike call is 5.444947.
(ii) Make a table with Maggies prot when the spot price at expi-
ration is $60, $65, $70, $75, $80, $85.
c 2009. Miguel A. Arcones. All rights reserved. Manual for SOA Exam FM/CAS Exam 2.
44/97
Chapter 7. Derivatives markets. Section 7.8. Spreads.
Example 5
(Use Table 1) Maggie buys 100 shares of XYZ stock, a $65strike
put option and sells a $80strike call. Both options are for 100
shares of XYZ stock and have expiration date one year from now.
The current price of one share of XYZ stock is $75. The risk free
annual eective rate of interest is 5%. The premium per share of a
$65strike put option is 1.221977. The premium per share of a
$80strike call is 5.444947.
(ii) Make a table with Maggies prot when the spot price at expi-
ration is $60, $65, $70, $75, $80, $85.
Solution: (ii) A table with Maggies prot for the considered spot
prices is
Spot Price 60 65 70
Maggies prot 931.59 931.59 431.59
Spot Price 75 80 85
Maggies prot 68.41 568.41 568.41
c 2009. Miguel A. Arcones. All rights reserved. Manual for SOA Exam FM/CAS Exam 2.
45/97
Chapter 7. Derivatives markets. Section 7.8. Spreads.
Example 5
(Use Table 1) Maggie buys 100 shares of XYZ stock, a $65strike
put option and sells a $80strike call. Both options are for 100
shares of XYZ stock and have expiration date one year from now.
The current price of one share of XYZ stock is $75. The risk free
annual eective rate of interest is 5%. The premium per share of a
$65strike put option is 1.221977. The premium per share of a
$80strike call is 5.444947.
(iii) Draw the graph of Maggies prot.
c 2009. Miguel A. Arcones. All rights reserved. Manual for SOA Exam FM/CAS Exam 2.
46/97
Chapter 7. Derivatives markets. Section 7.8. Spreads.
Example 5
(Use Table 1) Maggie buys 100 shares of XYZ stock, a $65strike
put option and sells a $80strike call. Both options are for 100
shares of XYZ stock and have expiration date one year from now.
The current price of one share of XYZ stock is $75. The risk free
annual eective rate of interest is 5%. The premium per share of a
$65strike put option is 1.221977. The premium per share of a
$80strike call is 5.444947.
(iii) Draw the graph of Maggies prot.
Solution: (iii) The graph of the prot is on Figure 6.
c 2009. Miguel A. Arcones. All rights reserved. Manual for SOA Exam FM/CAS Exam 2.
47/97
Chapter 7. Derivatives markets. Section 7.8. Spreads.
Figure 6: Prot for a collar plus owning the stock.
c 2009. Miguel A. Arcones. All rights reserved. Manual for SOA Exam FM/CAS Exam 2.
48/97
Chapter 7. Derivatives markets. Section 7.8. Spreads.
Suppose that you worked ve years for Microsoft and have
$100000 in stock of this company. You would like to insure this
position buying a collar with expiration T years from now. You can
choose K
1
and K
2
so that the combined premium is zero. In this
case, no matter what the price of the stock T years form now, you
will have $100000 or more. The cost of buying this insurance is
zero. Notice that you have not got anything from free, you have
loss interest in the stock. You also can make a collar with premium
zero, by taking K
1
= K
2
= F
0,T
. In this case you will receive F
0,T
at time T, i.e. you are entering into a synthetic forward.
Suppose that a zerocost collar consists of buying a K
1
strike put
option and selling a K
2
strike call option, where K
1
< K
2
. The
prot of a zerocost collar is
max(K
1
S
T
, 0)max(S
T
K
2
, 0) =

K
1
S
T
if S
T
< K
1
,
0 if K
1
S
T
< K
2
,
K
2
S
T
if K
2
S
T
.
c 2009. Miguel A. Arcones. All rights reserved. Manual for SOA Exam FM/CAS Exam 2.
49/97
Chapter 7. Derivatives markets. Section 7.8. Spreads.
Speculating on volatility. Straddle
A straddle consists of buying a Kstrike call and a Kstrike put
with the same time to expiration. The payo of this strategy is
max(K S
T
, 0) + max(S
T
K, 0)
=max(K S
T
, 0) min(K S
T
, 0)
=|S
T
K|.
Its prot is
|S
T
K| (Put(K, T) + Call(K, T))(1 + i )
T
.
A straddle is used to bet that the volatility of the market is higher
than the markets assessment of volatility. Notice that the prices of
the put and call use the markets assessment of volatility.
c 2009. Miguel A. Arcones. All rights reserved. Manual for SOA Exam FM/CAS Exam 2.
50/97
Chapter 7. Derivatives markets. Section 7.8. Spreads.
Example 6
(Use Table 1) Pam buys a $80strike call option and a $80strike
put for 100 shares of XYZ stock. Both have expiration date one
year from now. The current price of one share of XYZ stock is
$75. The risk free annual eective rate of interest is 5%. The
premium per share of a $80strike call option is 5.444947. The
premium per share of a $80strike put option is 6.635423.
c 2009. Miguel A. Arcones. All rights reserved. Manual for SOA Exam FM/CAS Exam 2.
51/97
Chapter 7. Derivatives markets. Section 7.8. Spreads.
Example 6
(Use Table 1) Pam buys a $80strike call option and a $80strike
put for 100 shares of XYZ stock. Both have expiration date one
year from now. The current price of one share of XYZ stock is
$75. The risk free annual eective rate of interest is 5%. The
premium per share of a $80strike call option is 5.444947. The
premium per share of a $80strike put option is 6.635423.
(i) Calculate Pams prot as a function of the spot price at expira-
tion.
c 2009. Miguel A. Arcones. All rights reserved. Manual for SOA Exam FM/CAS Exam 2.
52/97
Chapter 7. Derivatives markets. Section 7.8. Spreads.
Example 6
(Use Table 1) Pam buys a $80strike call option and a $80strike
put for 100 shares of XYZ stock. Both have expiration date one
year from now. The current price of one share of XYZ stock is
$75. The risk free annual eective rate of interest is 5%. The
premium per share of a $80strike call option is 5.444947. The
premium per share of a $80strike put option is 6.635423.
(i) Calculate Pams prot as a function of the spot price at expira-
tion.
Solution: (i) The future value per share of the cost of entering the
option contracts is
(Call(80, T) + Put(80, T))(1 + i )
T
=(5.444947 + 6.635423)(1.05) = 12.6843885.
Pams prot is
(100)(|S
T
80| 12.6843885).
c 2009. Miguel A. Arcones. All rights reserved. Manual for SOA Exam FM/CAS Exam 2.
53/97
Chapter 7. Derivatives markets. Section 7.8. Spreads.
Example 6
(Use Table 1) Pam buys a $80strike call option and a $80strike
put for 100 shares of XYZ stock. Both have expiration date one
year from now. The current price of one share of XYZ stock is
$75. The risk free annual eective rate of interest is 5%. The
premium per share of a $80strike call option is 5.444947. The
premium per share of a $80strike put option is 6.635423.
(ii) Make a table with Pams prot when the spot price at expiration
is $65, $70, $75, $80, $85, $90, $95.
c 2009. Miguel A. Arcones. All rights reserved. Manual for SOA Exam FM/CAS Exam 2.
54/97
Chapter 7. Derivatives markets. Section 7.8. Spreads.
Example 6
(Use Table 1) Pam buys a $80strike call option and a $80strike
put for 100 shares of XYZ stock. Both have expiration date one
year from now. The current price of one share of XYZ stock is
$75. The risk free annual eective rate of interest is 5%. The
premium per share of a $80strike call option is 5.444947. The
premium per share of a $80strike put option is 6.635423.
(ii) Make a table with Pams prot when the spot price at expiration
is $65, $70, $75, $80, $85, $90, $95.
Solution: (ii) Pams prot table is
Spot Price 65 70 75 80
Pams prot 231.56 268.44 768.44 1268.44
Spot Price 80 85 90 95
Pams prot 1268.44 768.44 268.44 231.56
c 2009. Miguel A. Arcones. All rights reserved. Manual for SOA Exam FM/CAS Exam 2.
55/97
Chapter 7. Derivatives markets. Section 7.8. Spreads.
Example 6
(Use Table 1) Pam buys a $80strike call option and a $80strike
put for 100 shares of XYZ stock. Both have expiration date one
year from now. The current price of one share of XYZ stock is
$75. The risk free annual eective rate of interest is 5%. The
premium per share of a $80strike call option is 5.444947. The
premium per share of a $80strike put option is 6.635423.
(iii) Draw the graph of Pams prot.
c 2009. Miguel A. Arcones. All rights reserved. Manual for SOA Exam FM/CAS Exam 2.
56/97
Chapter 7. Derivatives markets. Section 7.8. Spreads.
Example 6
(Use Table 1) Pam buys a $80strike call option and a $80strike
put for 100 shares of XYZ stock. Both have expiration date one
year from now. The current price of one share of XYZ stock is
$75. The risk free annual eective rate of interest is 5%. The
premium per share of a $80strike call option is 5.444947. The
premium per share of a $80strike put option is 6.635423.
(iii) Draw the graph of Pams prot.
Solution: (iii) The graph of the prot is on Figure 7.
c 2009. Miguel A. Arcones. All rights reserved. Manual for SOA Exam FM/CAS Exam 2.
57/97
Chapter 7. Derivatives markets. Section 7.8. Spreads.
Example 6
(Use Table 1) Pam buys a $80strike call option and a $80strike
put for 100 shares of XYZ stock. Both have expiration date one
year from now. The current price of one share of XYZ stock is
$75. The risk free annual eective rate of interest is 5%. The
premium per share of a $80strike call option is 5.444947. The
premium per share of a $80strike put option is 6.635423.
(iv) Find the values of the spot price at expiration at which Pam
makes a prot.
c 2009. Miguel A. Arcones. All rights reserved. Manual for SOA Exam FM/CAS Exam 2.
58/97
Chapter 7. Derivatives markets. Section 7.8. Spreads.
Example 6
(Use Table 1) Pam buys a $80strike call option and a $80strike
put for 100 shares of XYZ stock. Both have expiration date one
year from now. The current price of one share of XYZ stock is
$75. The risk free annual eective rate of interest is 5%. The
premium per share of a $80strike call option is 5.444947. The
premium per share of a $80strike put option is 6.635423.
(iv) Find the values of the spot price at expiration at which Pam
makes a prot.
Solution: (iv) Pam makes a prot if (100)(|S
T
80|
12.6843885) > 0, i.e. if |S
T
80| > 12.6843885. This can hap-
pen if either S
T
80 < 12.6843885, or S
T
80 > 12.6843885.
We have that S
T
80 < 12.6843885 is equivalent to S
T
<
80 12.6843885 = 67.3156115. S
T
80 > 12.6843885 is equiv-
alent to S
T
> 92.6843885. Hence, Pam makes a prot if either
S
T
< 67.3156115 or S
T
> 92.6843885.
c 2009. Miguel A. Arcones. All rights reserved. Manual for SOA Exam FM/CAS Exam 2.
59/97
Chapter 7. Derivatives markets. Section 7.8. Spreads.
Figure 7: Prot for a straddle.
c 2009. Miguel A. Arcones. All rights reserved. Manual for SOA Exam FM/CAS Exam 2.
60/97
Chapter 7. Derivatives markets. Section 7.8. Spreads.
Strangle
A strangle consists of buying a K
1
strike put and a K
2
strike call
with the same expiration date, where K
1
< K
2
. The prot of this
portfolio is
max(K
1
S
T
, 0) + max(S
T
K
2
, 0)
((Put(K
1
, T) + Call(K
2
, T))(1 + i )
T
=

K
1
S
T
((Put(K
1
, T) + Call(K
2
, T))(1 + i )
T
if S
T
< K
1
,
((Put(K
1
, T) + Call(K
2
, T))(1 + i )
T
if K
1
S
T
< K
2
,
S
T
K
2
((Put(K
1
, T) + Call(K
2
, T))(1 + i )
T
. if K
2
S
T
.
c 2009. Miguel A. Arcones. All rights reserved. Manual for SOA Exam FM/CAS Exam 2.
61/97
Chapter 7. Derivatives markets. Section 7.8. Spreads.
Example 7
(Use Table 1) Beth buys a $75strike put option and a $85strike
call for 100 shares of XYZ stock. Both have expiration date one
year from now. The current price of one share of XYZ stock is
$75. The risk free annual eective rate of interest is 5%. The
premium per share of a $75strike put option is 4.218281. The
premium per share of a $85strike call option is 3.680736.
c 2009. Miguel A. Arcones. All rights reserved. Manual for SOA Exam FM/CAS Exam 2.
62/97
Chapter 7. Derivatives markets. Section 7.8. Spreads.
Example 7
(Use Table 1) Beth buys a $75strike put option and a $85strike
call for 100 shares of XYZ stock. Both have expiration date one
year from now. The current price of one share of XYZ stock is
$75. The risk free annual eective rate of interest is 5%. The
premium per share of a $75strike put option is 4.218281. The
premium per share of a $85strike call option is 3.680736.
(i) Calculate Beths prot as a function of S
T
.
c 2009. Miguel A. Arcones. All rights reserved. Manual for SOA Exam FM/CAS Exam 2.
63/97
Chapter 7. Derivatives markets. Section 7.8. Spreads.
Example 7
(Use Table 1) Beth buys a $75strike put option and a $85strike
call for 100 shares of XYZ stock. Both have expiration date one
year from now. The current price of one share of XYZ stock is
$75. The risk free annual eective rate of interest is 5%. The
premium per share of a $75strike put option is 4.218281. The
premium per share of a $85strike call option is 3.680736.
(i) Calculate Beths prot as a function of S
T
.
Solution: (i) Beths prot is
100(max(75 S
T
, 0) + max(S
T
85, 0)
(4.218281 + 3.680736)(1.05))
=100(max(75 S
T
, 0) + max(S
T
85, 0) 8.29396785)
=

100(75 S
T
8.29396785) if S
T
< 75,
100(8.29396785) if 75 S
T
< 85,
100(S
T
85 8.29396785) if 85 S
T
.
c 2009. Miguel A. Arcones. All rights reserved. Manual for SOA Exam FM/CAS Exam 2.
64/97
Chapter 7. Derivatives markets. Section 7.8. Spreads.
Example 7
(Use Table 1) Beth buys a $75strike put option and a $85strike
call for 100 shares of XYZ stock. Both have expiration date one
year from now. The current price of one share of XYZ stock is
$75. The risk free annual eective rate of interest is 5%. The
premium per share of a $75strike put option is 4.218281. The
premium per share of a $85strike call option is 3.680736.
(ii) Make a table with Beths prot when the spot price at expiration
is $50, $60, $70, $80, $90, $100, $110.
c 2009. Miguel A. Arcones. All rights reserved. Manual for SOA Exam FM/CAS Exam 2.
65/97
Chapter 7. Derivatives markets. Section 7.8. Spreads.
Example 7
(Use Table 1) Beth buys a $75strike put option and a $85strike
call for 100 shares of XYZ stock. Both have expiration date one
year from now. The current price of one share of XYZ stock is
$75. The risk free annual eective rate of interest is 5%. The
premium per share of a $75strike put option is 4.218281. The
premium per share of a $85strike call option is 3.680736.
(ii) Make a table with Beths prot when the spot price at expiration
is $50, $60, $70, $80, $90, $100, $110.
Solution: (ii)
Spot Price 50 60 70 80
Beths prot 1670.60 670.60 329.40 829.40
Spot Price 90 100 110
Beths prot 329.40 670.60 1670.60
c 2009. Miguel A. Arcones. All rights reserved. Manual for SOA Exam FM/CAS Exam 2.
66/97
Chapter 7. Derivatives markets. Section 7.8. Spreads.
Example 7
(Use Table 1) Beth buys a $75strike put option and a $85strike
call for 100 shares of XYZ stock. Both have expiration date one
year from now. The current price of one share of XYZ stock is
$75. The risk free annual eective rate of interest is 5%. The
premium per share of a $75strike put option is 4.218281. The
premium per share of a $85strike call option is 3.680736.
(iii) Draw the graph of Beths prot.
c 2009. Miguel A. Arcones. All rights reserved. Manual for SOA Exam FM/CAS Exam 2.
67/97
Chapter 7. Derivatives markets. Section 7.8. Spreads.
Example 7
(Use Table 1) Beth buys a $75strike put option and a $85strike
call for 100 shares of XYZ stock. Both have expiration date one
year from now. The current price of one share of XYZ stock is
$75. The risk free annual eective rate of interest is 5%. The
premium per share of a $75strike put option is 4.218281. The
premium per share of a $85strike call option is 3.680736.
(iii) Draw the graph of Beths prot.
Solution: (iii) The graph of the prot is Figure 8.
c 2009. Miguel A. Arcones. All rights reserved. Manual for SOA Exam FM/CAS Exam 2.
68/97
Chapter 7. Derivatives markets. Section 7.8. Spreads.
Figure 8: Prot for a strangle.
c 2009. Miguel A. Arcones. All rights reserved. Manual for SOA Exam FM/CAS Exam 2.
69/97
Chapter 7. Derivatives markets. Section 7.8. Spreads.
The straddle and the strangle bet in volatility of the market in a
similar way. Suppose that a straddle and a strangle are centered
around the same strike price. The maximum loss of the strangle is
smaller than the maximum loss of the straddle. However, the
strangle needs more volatility to attain a prot. The possible prot
of the strangle is smaller than that of the straddle. See Figure 9.
c 2009. Miguel A. Arcones. All rights reserved. Manual for SOA Exam FM/CAS Exam 2.
70/97
Chapter 7. Derivatives markets. Section 7.8. Spreads.
Figure 9: Prot for a straddle and a strangle.
c 2009. Miguel A. Arcones. All rights reserved. Manual for SOA Exam FM/CAS Exam 2.
71/97
Chapter 7. Derivatives markets. Section 7.8. Spreads.
Written strangle
A written strangle consists of selling a K
1
strike call and a
K
2
strike put with the same time to expiration, where
0 < K
1
< K
2
. A written strangle is a bet on low volatility.
c 2009. Miguel A. Arcones. All rights reserved. Manual for SOA Exam FM/CAS Exam 2.
72/97
Chapter 7. Derivatives markets. Section 7.8. Spreads.
Speculating on low volatility: Buttery spread
Given 0 < K
1
< K
2
< K
3
, a buttery spread consists of:
(i) selling a K
2
strike call and a K
2
strike put, all options for the
notional amount.
(ii) buying a K
1
strike call and a K
3
strike put, all options for the
notional amount.
The prot per share of this strategy is
max(S
T
K
1
, 0) + max(K
3
S
T
, 0)
max(S
T
K
2
, 0) max(K
2
S
T
, 0) FVP,
where
FVP = (Call(K
1
, T) Call(K
2
, T)
Put(K
2
, T) + Put(K
3
, T)) (1 + i )
T
.
c 2009. Miguel A. Arcones. All rights reserved. Manual for SOA Exam FM/CAS Exam 2.
73/97
Chapter 7. Derivatives markets. Section 7.8. Spreads.
The prot per share of a buttery spread is
max(S
T
K
1
, 0) + max(K
3
S
T
, 0)
max(S
T
K
2
, 0) max(K
2
S
T
, 0) FVP
=

K
3
K
2
FVP if S
T
< K
1
,
S
T
K
1
K
2
+ K
3
FVP if K
1
S
T
< K
2
,
S
T
K
1
+ K
2
+ K
3
FVP if K
2
S
T
< K
3
,
K
2
K
1
FVP if K
3
S
T
,
The graph of this prot is Figure 10
c 2009. Miguel A. Arcones. All rights reserved. Manual for SOA Exam FM/CAS Exam 2.
74/97
Chapter 7. Derivatives markets. Section 7.8. Spreads.
Figure 10: Prot for a buttery.
c 2009. Miguel A. Arcones. All rights reserved. Manual for SOA Exam FM/CAS Exam 2.
75/97
Chapter 7. Derivatives markets. Section 7.8. Spreads.
Example 8
(Use Table 1) Steve buys a 65strike call and a 85strike put and
sells a 75strike call and a 75strike put. All are for 100 shares and
have expiration date one year from now. The current price of one
share of XYZ stock is $75. The risk free annual eective rate of
interest is 5%.
c 2009. Miguel A. Arcones. All rights reserved. Manual for SOA Exam FM/CAS Exam 2.
76/97
Chapter 7. Derivatives markets. Section 7.8. Spreads.
Example 8
(Use Table 1) Steve buys a 65strike call and a 85strike put and
sells a 75strike call and a 75strike put. All are for 100 shares and
have expiration date one year from now. The current price of one
share of XYZ stock is $75. The risk free annual eective rate of
interest is 5%.
(i) Find Steves prot as a function of the spot price at expiration.
c 2009. Miguel A. Arcones. All rights reserved. Manual for SOA Exam FM/CAS Exam 2.
77/97
Chapter 7. Derivatives markets. Section 7.8. Spreads.
Example 8
(Use Table 1) Steve buys a 65strike call and a 85strike put and
sells a 75strike call and a 75strike put. All are for 100 shares and
have expiration date one year from now. The current price of one
share of XYZ stock is $75. The risk free annual eective rate of
interest is 5%.
(i) Find Steves prot as a function of the spot price at expiration.
Solution: (i) We have that
(Call(K
1
, T) Call(K
2
, T) Put(K
2
, T) + Put(K
3
, T)) (1 + i )
T
=(14.31722 9.633117 7.78971 + 4.218281)(1.05) = 12.539459.
Steves prot is
(100) max(S
T
65, 0) + (100) max(85 S
T
, 0)
(100) max(S
T
75, 0) (100) max(75 S
T
, 0) (100)(12.539459)
c 2009. Miguel A. Arcones. All rights reserved. Manual for SOA Exam FM/CAS Exam 2.
78/97
Chapter 7. Derivatives markets. Section 7.8. Spreads.
Example 8
(Use Table 1) Steve buys a 65strike call and a 85strike put and
sells a 75strike call and a 75strike put. All are for 100 shares and
have expiration date one year from now. The current price of one
share of XYZ stock is $75. The risk free annual eective rate of
interest is 5%.
(i) Find Steves prot as a function of the spot price at expiration.
Solution: (i) (continuation)
(100) max(S
T
65, 0) + (100) max(85 S
T
, 0)
(100) max(S
T
75, 0) (100) max(75 S
T
, 0) (100)(12.539459)
=

100(10 12.539459) if S
T
< 65,
100(S
T
55 12.539459) if 65 S
T
< 75,
100(S
T
+ 95 12.539459) if 75 S
T
< 85,
100(10 12.539459) if 85 S
T
.
c 2009. Miguel A. Arcones. All rights reserved. Manual for SOA Exam FM/CAS Exam 2.
79/97
Chapter 7. Derivatives markets. Section 7.8. Spreads.
Example 8
(Use Table 1) Steve buys a 65strike call and a 85strike put and
sells a 75strike call and a 75strike put. All are for 100 shares and
have expiration date one year from now. The current price of one
share of XYZ stock is $75. The risk free annual eective rate of
interest is 5%.
(ii) Make a table with Steves prot when the spot price at expiration
is $60, $65, $70, $75, $80, $85, $90.
c 2009. Miguel A. Arcones. All rights reserved. Manual for SOA Exam FM/CAS Exam 2.
80/97
Chapter 7. Derivatives markets. Section 7.8. Spreads.
Example 8
(Use Table 1) Steve buys a 65strike call and a 85strike put and
sells a 75strike call and a 75strike put. All are for 100 shares and
have expiration date one year from now. The current price of one
share of XYZ stock is $75. The risk free annual eective rate of
interest is 5%.
(ii) Make a table with Steves prot when the spot price at expiration
is $60, $65, $70, $75, $80, $85, $90.
Solution: (ii) table with Steves prot is
Spot Price 60 65 70 75
Steves prot 253.95 253.95 246.05 746.05
Spot Price 75 80 85 90
Steves prot 746.05 246.05 253.95 253.95
c 2009. Miguel A. Arcones. All rights reserved. Manual for SOA Exam FM/CAS Exam 2.
81/97
Chapter 7. Derivatives markets. Section 7.8. Spreads.
Example 8
(Use Table 1) Steve buys a 65strike call and a 85strike put and
sells a 75strike call and a 75strike put. All are for 100 shares and
have expiration date one year from now. The current price of one
share of XYZ stock is $75. The risk free annual eective rate of
interest is 5%.
(iii) Draw the graph of Steves prot.
c 2009. Miguel A. Arcones. All rights reserved. Manual for SOA Exam FM/CAS Exam 2.
82/97
Chapter 7. Derivatives markets. Section 7.8. Spreads.
Example 8
(Use Table 1) Steve buys a 65strike call and a 85strike put and
sells a 75strike call and a 75strike put. All are for 100 shares and
have expiration date one year from now. The current price of one
share of XYZ stock is $75. The risk free annual eective rate of
interest is 5%.
(iii) Draw the graph of Steves prot.
Solution: (iii) The graph of the prot is Figure 10.
c 2009. Miguel A. Arcones. All rights reserved. Manual for SOA Exam FM/CAS Exam 2.
83/97
Chapter 7. Derivatives markets. Section 7.8. Spreads.
Asymmetric Buttery spread.
Given 0 < K
1
< K
2
< K
3
and 0 < < 1, a buttery spread
consists of: buying K
1
strike calls, buying (1 ) K
3
strike
calls; and selling one K
2
strike call. The prot of this strategy is
() max(S
T
K
1
, 0) + (1 ) max(S
T
K
3
, 0) max(S
T
K
2
, 0)
=

0 FVPrem if S
T
< K
1
,
(S
T
K
1
) FVPrem if K
1
S
T
< K
2
,
K
1
+ K
2
(1 )S
T
FVPrem if K
2
S
T
< K
3
,
K
1
+ K
2
(1 )K
3
FVPrem if K
3
S
T
,
where
FVPrem = (Call(K
1
, T) Call(K
2
, T) + (1 )Call(K
3
, T)) (1+i )
T
.
c 2009. Miguel A. Arcones. All rights reserved. Manual for SOA Exam FM/CAS Exam 2.
84/97
Chapter 7. Derivatives markets. Section 7.8. Spreads.
The prot of an symmetric buttery spread is
() max(S
T
K
1
, 0) + (1 ) max(S
T
K
3
, 0) max(S
T
K
2
, 0)
=

0 FVPrem if S
T
< K
1
,
(S
T
K
1
) FVPrem if K
1
S
T
< K
2
,
K
1
+ K
2
(1 )S
T
FVPrem if K
2
S
T
< K
3
,
K
1
+ K
2
(1 )K
3
FVPrem if K
3
S
T
,
For an asymmetric buttery spread, the prot functions increases
in one interval and decreases in another interval. The rate of
increase is not necessarily equal to the rate of decrease.
c 2009. Miguel A. Arcones. All rights reserved. Manual for SOA Exam FM/CAS Exam 2.
85/97
Chapter 7. Derivatives markets. Section 7.8. Spreads.
Example 9
(Use Table 1) Karen buys seven 65strike calls, buys three
85strike calls and sells ten 75strike calls. Each option involves
100 shares of XYZ stock. Both have expiration date one year from
now. The risk free annual eective rate of interest is 5%.
c 2009. Miguel A. Arcones. All rights reserved. Manual for SOA Exam FM/CAS Exam 2.
86/97
Chapter 7. Derivatives markets. Section 7.8. Spreads.
Example 9
(Use Table 1) Karen buys seven 65strike calls, buys three
85strike calls and sells ten 75strike calls. Each option involves
100 shares of XYZ stock. Both have expiration date one year from
now. The risk free annual eective rate of interest is 5%.
(i) Find Karens prot as a function of the spot price at expiration.
c 2009. Miguel A. Arcones. All rights reserved. Manual for SOA Exam FM/CAS Exam 2.
87/97
Chapter 7. Derivatives markets. Section 7.8. Spreads.
Example 9
(Use Table 1) Karen buys seven 65strike calls, buys three
85strike calls and sells ten 75strike calls. Each option involves
100 shares of XYZ stock. Both have expiration date one year from
now. The risk free annual eective rate of interest is 5%.
(i) Find Karens prot as a function of the spot price at expiration.
Solution: (i) The future value per share of the cost of entering the
option contracts is
(7)(14.31722)(1.05) + (3)(7.78971)(1.05) (3.680736)(1.05)
=35.0339184.
Karens prot is
(100) ((7) max(S
T
65, 0) + (3) max(S
T
85, 0)
(10) max(S
T
75, 0) (35.0339184))
c 2009. Miguel A. Arcones. All rights reserved. Manual for SOA Exam FM/CAS Exam 2.
88/97
Chapter 7. Derivatives markets. Section 7.8. Spreads.
Example 9
(Use Table 1) Karen buys seven 65strike calls, buys three
85strike calls and sells ten 75strike calls. Each option involves
100 shares of XYZ stock. Both have expiration date one year from
now. The risk free annual eective rate of interest is 5%.
(i) Find Karens prot as a function of the spot price at expiration.
Solution: (i) (continuation)
(100) ((7) max(S
T
65, 0) + (3) max(S
T
85, 0)
(10) max(S
T
75, 0) (35.0339184))
=

100(35.0339184) if S
T
< 65,
100((7)(S
T
65) 35.0339184) if 65 S
T
< 75,
100((7)(S
T
65) (10)(S
T
75)
35.0339184) if 75 S
T
< 85,
100((7)(S
T
65) (10)(S
T
75)
+(3)(S
T
85) 35.0339184) if 85 S
T
,
c 2009. Miguel A. Arcones. All rights reserved. Manual for SOA Exam FM/CAS Exam 2.
89/97
Chapter 7. Derivatives markets. Section 7.8. Spreads.
Example 9
(Use Table 1) Karen buys seven 65strike calls, buys three
85strike calls and sells ten 75strike calls. Each option involves
100 shares of XYZ stock. Both have expiration date one year from
now. The risk free annual eective rate of interest is 5%.
(ii) Make a table with Karens prot when the spot price at expiration
is $60, $65, $70, $75, $80, $85, $90.
c 2009. Miguel A. Arcones. All rights reserved. Manual for SOA Exam FM/CAS Exam 2.
90/97
Chapter 7. Derivatives markets. Section 7.8. Spreads.
Example 9
(Use Table 1) Karen buys seven 65strike calls, buys three
85strike calls and sells ten 75strike calls. Each option involves
100 shares of XYZ stock. Both have expiration date one year from
now. The risk free annual eective rate of interest is 5%.
(ii) Make a table with Karens prot when the spot price at expiration
is $60, $65, $70, $75, $80, $85, $90.
Solution: (ii) A table with Karens prot is
Spot Price 60 65 70 75
Karens prot 3503.39 3503.39 3.39 3496.61
Spot Price 75 80 85 90
Karens prot 3496.61 1996.61 496.61 496.61
c 2009. Miguel A. Arcones. All rights reserved. Manual for SOA Exam FM/CAS Exam 2.
91/97
Chapter 7. Derivatives markets. Section 7.8. Spreads.
Example 9
(Use Table 1) Karen buys seven 65strike calls, buys three
85strike calls and sells ten 75strike calls. Each option involves
100 shares of XYZ stock. Both have expiration date one year from
now. The risk free annual eective rate of interest is 5%.
(iii) Draw the graph of Karens prot.
c 2009. Miguel A. Arcones. All rights reserved. Manual for SOA Exam FM/CAS Exam 2.
92/97
Chapter 7. Derivatives markets. Section 7.8. Spreads.
Example 9
(Use Table 1) Karen buys seven 65strike calls, buys three
85strike calls and sells ten 75strike calls. Each option involves
100 shares of XYZ stock. Both have expiration date one year from
now. The risk free annual eective rate of interest is 5%.
(iii) Draw the graph of Karens prot.
Solution: (iii) The graph of the prot is on Figure 11.
c 2009. Miguel A. Arcones. All rights reserved. Manual for SOA Exam FM/CAS Exam 2.
93/97
Chapter 7. Derivatives markets. Section 7.8. Spreads.
Figure 11: Prot for an asymmetric buttery.
c 2009. Miguel A. Arcones. All rights reserved. Manual for SOA Exam FM/CAS Exam 2.
94/97
Chapter 7. Derivatives markets. Section 7.8. Spreads.
Box spreads.
A box spread is a combination of options which create a synthetic
long forward at one price and a synthetic short forward at a
dierent price. Let K
1
be the price of the synthetic long forward.
Let K
2
be the price of the synthetic short forward. With a box
spread, you are able to buy an asset for K
1
at time T and sell it for
K
2
at time T not matter the spot price at expiration. At time T, a
payment of K
2
K
1
per share is obtained. A box spread can be
obtained from:
(i) buy a K
1
strike call and sell a K
1
strike put.
(ii) sell a K
2
strike call and buy a K
2
strike put.
If putcall parity holds, the premium per share to enter these
option contract is
Call(K
1
, T) Put(K
1
, T) (Call(K
2
, T) Put(K
2
, T))
=(K
2
K
1
)(1 + i )
T
.
c 2009. Miguel A. Arcones. All rights reserved. Manual for SOA Exam FM/CAS Exam 2.
95/97
Chapter 7. Derivatives markets. Section 7.8. Spreads.

If K
1
< K
2
, a box spread is a way to lend money. An
investment of (K
2
K
1
)(1 + i )
T
per share is made at time
zero and a return of K
2
K
1
per share is obtained at time T.

If K
1
> K
2
, a box spread is a way to borrow money. A return
of (K
1
K
2
)(1 +i )
T
per share is received at time zero and a
loan payment of K
1
K
2
per share is made at time T.
c 2009. Miguel A. Arcones. All rights reserved. Manual for SOA Exam FM/CAS Exam 2.
96/97
Chapter 7. Derivatives markets. Section 7.8. Spreads.
Example 10
Mario needs $50,000 to open a pizzeria. He can borrow at the
annual eective rate of interest of 8.5%. Mario also can buy/sell
threeyear options on XYZ stock with the following premiums per
share:
Call(K, T) 14.42 7.78
Put(K, T) 7.37 17.29
K 70 90
Mario buys a 90strike call and a 70strike put, and sells a
90strike put and a 70strike call. All the options are for the same
nominal amount. Mario receives a total of $50000 from these
sales. Find Marios cost at expiration time to settle these options.
Find the annual rate of return that Mario gets on this loan.
c 2009. Miguel A. Arcones. All rights reserved. Manual for SOA Exam FM/CAS Exam 2.
97/97
Chapter 7. Derivatives markets. Section 7.8. Spreads.
Solution: The price per share of Marios portfolio is
7.78 + 7.37 17.29 14.42 = 16.56. So, the nominal amount of
each option is
50000
16.56
= 3019.3237. Initially, Mario gets $50000 for
entering these option contracts. In three years, Mario buys at $90
per share and sells at $70 per share. Hence, Mario pays
(3019.3237)(90 70) = 60386.474 for settling the option
contracts. Let i be the annual eective rate of interest on this loan.
We have that 50000(1 + i )
3
= 60386.474 and i = 6.493529844%.
c 2009. Miguel A. Arcones. All rights reserved. Manual for SOA Exam FM/CAS Exam 2.
1/44
Chapter 7. Derivatives markets.
Manual for SOA Exam FM/CAS Exam 2.
Chapter 7. Derivatives markets.
Section 7.9. Risk Management.
c 2009. Miguel A. Arcones. All rights reserved.
Extract from:
Arcones Manual for the SOA Exam FM/CAS Exam 2,
Financial Mathematics. Fall 2009 Edition,
available at http://www.actexmadriver.com/
c 2009. Miguel A. Arcones. All rights reserved. Manual for SOA Exam FM/CAS Exam 2.
2/44
Chapter 7. Derivatives markets. Section 7.9. Risk Management.
Risk Management
The main reasons why rms enter derivatives are: to hedge, to
speculate, and to reduce transactions costs. Managers use
derivatives taking in account their view of the market. So, a
speculative component is added to each decision.
Firms convert inputs, such a labor and raw materials, into goods
and services. A rm makes money if its income exceeds its costs.
A change in the price of raw materials could make the rm
unprotable. A rm can use derivatives to alter its risk and protect
its protability. To do this is to do risk management.
c 2009. Miguel A. Arcones. All rights reserved. Manual for SOA Exam FM/CAS Exam 2.
3/44
Chapter 7. Derivatives markets. Section 7.9. Risk Management.
Firms do risk management to attain nancial stability. There are
many reasons to avoid large losses. After a large loss, interest rates
on loans will be obtained at a higher rate. Large losses can cause
bankruptcy and distress costs. After a large loss, a company can
face low cashows and diculty making xed obligations such as
wages and payments to banks and suppliers. This makes more
costly to nd employees, debtors and suppliers.
Suppose that the prot can be modeled by a random variable X.
Let f (X) be the the prot after taking in account the eects of
this prot. Because of the reasons before, f can be modeled using
a concave function.
c 2009. Miguel A. Arcones. All rights reserved. Manual for SOA Exam FM/CAS Exam 2.
4/44
Chapter 7. Derivatives markets. Section 7.9. Risk Management.
Example 1
Suppose that the nancial total impact of the prot is
f (x) =

1.5x if x 2,
1.1x if 2 < x 0
x if x > 0.
Since
f

(x) =

1.5 if x 2,
1.1 if 2 < x 0
1 if x > 0.
is a nonincreasing function, f is a concave function.
c 2009. Miguel A. Arcones. All rights reserved. Manual for SOA Exam FM/CAS Exam 2.
5/44
Chapter 7. Derivatives markets. Section 7.9. Risk Management.
Example 2
Suppose that the a company prot before taxes can be modeled by
a random variable X. The company pays 35% of its prot in taxes,
if the prot is positive. It does not pay any taxes if its prot is
negative. The companys prot after taxes is f (X), where
f (x) =

(0.65)x if x > 0,
x if x 0.
f (x) is a concave function.
c 2009. Miguel A. Arcones. All rights reserved. Manual for SOA Exam FM/CAS Exam 2.
6/44
Chapter 7. Derivatives markets. Section 7.9. Risk Management.
If the company is able to hedge and attain a constant prot of
E[X], then after taxes its prot is f (E[X]). If the company does
not hedge its prot, its expected prot after taxes is E[f (X)].
Since f is a concave function, by the Jensens inequality,
E[f (X)] f (E[X]).
Theorem 1
(Jensens inequality) Let X be a random variable. Let f : R R
be a function. Then,
(i) If f is convex, then f (E[X]) E[f (X)].
(ii) If f is concave, then E[f (X)] f (E[X]).
By using hedging, the random prot X is changed into a random
prot with less variation (taking big losses with less probability).
The expectation of this new random prot is larger than the
expectation of the original prot.
c 2009. Miguel A. Arcones. All rights reserved. Manual for SOA Exam FM/CAS Exam 2.
7/44
Chapter 7. Derivatives markets. Section 7.9. Risk Management.
Example 3
Hank is a wheat farmer. He will produce 50000 bushels of wheat
at the end of one year. The cost of producing this wheat is $5.7
per bushel. The price of wheat per bushel in one year will be:
S
1
5.5 6.5
Probability 0.5 0.5
A speculator oers short forward contracts for a price equal to 1%
less than the expected value of the price of the wheat. It also
oers a 6strike put option with a price equal to 1% more than the
present value of the expected payo of this put. The annual
eective rate of interest is 5%. Hank pays 30% of its prots on
taxes and has no tax benets for losses.
c 2009. Miguel A. Arcones. All rights reserved. Manual for SOA Exam FM/CAS Exam 2.
8/44
Chapter 7. Derivatives markets. Section 7.9. Risk Management.
(i) Calculate the prices of the forward contract and the put option.
Solution: (i) The expected price of wheat is
E[S
1
] = (0.5)(5.5) + (0.5)(6.5) = 6.00/bush.
The price of a forward contract is 6(0.99) = 5.94/bush.
The expected payo of the put is
E[max(6 S
1
, 0)] = (0.5) max(6 5.5, 0) + (0.5) max(6 6.5, 0)
=(0.5)(6 5.5) + (0.5)(0) = 0.25.
The price of a put contract is
(0.25)(1.01)(1.05)
1
= 0.2404761905/bush.
c 2009. Miguel A. Arcones. All rights reserved. Manual for SOA Exam FM/CAS Exam 2.
9/44
Chapter 7. Derivatives markets. Section 7.9. Risk Management.
(i) Calculate the prices of the forward contract and the put option.
Solution: (i) The expected price of wheat is
E[S
1
] = (0.5)(5.5) + (0.5)(6.5) = 6.00/bush.
The price of a forward contract is 6(0.99) = 5.94/bush.
The expected payo of the put is
E[max(6 S
1
, 0)] = (0.5) max(6 5.5, 0) + (0.5) max(6 6.5, 0)
=(0.5)(6 5.5) + (0.5)(0) = 0.25.
The price of a put contract is
(0.25)(1.01)(1.05)
1
= 0.2404761905/bush.
c 2009. Miguel A. Arcones. All rights reserved. Manual for SOA Exam FM/CAS Exam 2.
10/44
Chapter 7. Derivatives markets. Section 7.9. Risk Management.
(ii) Calculate Hanks expected prot before taxes if (a) he does not
buy any derivative, (b) he enters a short forward contract, (c) he
buys a put option. Which strategy has the biggest expected prot
before taxes?
Solution: (ii) For an uninsured position, Hanks prot before taxes
is 50000(S
1
5.7). Hanks expected prot before taxes is
50000(E[S
1
] 5.7) = 50000(6 5.7) = 15000.
Entering the short forward contract, Hanks prot before taxes is
50000(S
1
5.7 + 5.94 S
1
) = (50000)(5.94 5.7) = 12000.
Entering the short forward contract, Hanks expected prot before
taxes is 12000.
c 2009. Miguel A. Arcones. All rights reserved. Manual for SOA Exam FM/CAS Exam 2.
11/44
Chapter 7. Derivatives markets. Section 7.9. Risk Management.
(ii) Calculate Hanks expected prot before taxes if (a) he does not
buy any derivative, (b) he enters a short forward contract, (c) he
buys a put option. Which strategy has the biggest expected prot
before taxes?
Solution: (ii) For an uninsured position, Hanks prot before taxes
is 50000(S
1
5.7). Hanks expected prot before taxes is
50000(E[S
1
] 5.7) = 50000(6 5.7) = 15000.
Entering the short forward contract, Hanks prot before taxes is
50000(S
1
5.7 + 5.94 S
1
) = (50000)(5.94 5.7) = 12000.
Entering the short forward contract, Hanks expected prot before
taxes is 12000.
c 2009. Miguel A. Arcones. All rights reserved. Manual for SOA Exam FM/CAS Exam 2.
12/44
Chapter 7. Derivatives markets. Section 7.9. Risk Management.
Buying the put option, Hanks prot before taxes is
50000 (S
1
5.7 + max(6 S
1
, 0) (0.2404761905)(1.05))
=50000 (max(6, S
1
) 5.7 (0.2404761905)(1.05))
=50000(max(6, S
1
) 5.9525).
Buying the put option, Hanks expected prot before taxes is
E[50000 (max(6, S
1
) 5.9525)]
=50000 (E[max(6, S
1
)] 5.9525)
=(50000)((0.5)(6) + (0.5)(6.5) 5.9525)) = 14875.
The biggest expected prot before taxes is attained for an
uninsured position.
c 2009. Miguel A. Arcones. All rights reserved. Manual for SOA Exam FM/CAS Exam 2.
13/44
Chapter 7. Derivatives markets. Section 7.9. Risk Management.
(iii) Calculate Hanks expected prot after taxes for each of three
strategies in (ii). Which strategy has the biggest expected prot
after taxes?
Solution: (iii) For an uninsured position, Hanks prot before
taxes is
50000(S
1
5.7) =

50000(5.5 5.7) if S
1
= 5.5,
50000(6.5 5.7) if S
1
= 6.5,
Hanks prot after taxes is
50000(S
1
5.7) =

50000(5.5 5.7) if S
1
= 5.5,
50000(6.5 5.7)(0.7) if S
1
= 6.5,
Hanks expected prot after taxes is
(0.5)(50000)(5.5 5.7) + (0.5)(50000)(6.5 5.7)(0.7) = 9000.
Entering the forward contract, Hanks expected prot after taxes is
12000(0.7) = 8400.
c 2009. Miguel A. Arcones. All rights reserved. Manual for SOA Exam FM/CAS Exam 2.
14/44
Chapter 7. Derivatives markets. Section 7.9. Risk Management.
(iii) Calculate Hanks expected prot after taxes for each of three
strategies in (ii). Which strategy has the biggest expected prot
after taxes?
Solution: (iii) For an uninsured position, Hanks prot before
taxes is
50000(S
1
5.7) =

50000(5.5 5.7) if S
1
= 5.5,
50000(6.5 5.7) if S
1
= 6.5,
Hanks prot after taxes is
50000(S
1
5.7) =

50000(5.5 5.7) if S
1
= 5.5,
50000(6.5 5.7)(0.7) if S
1
= 6.5,
Hanks expected prot after taxes is
(0.5)(50000)(5.5 5.7) + (0.5)(50000)(6.5 5.7)(0.7) = 9000.
Entering the forward contract, Hanks expected prot after taxes is
12000(0.7) = 8400.
c 2009. Miguel A. Arcones. All rights reserved. Manual for SOA Exam FM/CAS Exam 2.
15/44
Chapter 7. Derivatives markets. Section 7.9. Risk Management.
Buying the put option, Hanks prot before taxes is
=50000 (max(6, S
1
) 5.9525)
= =

50000(6 5.9525) if S
1
= 5.5,
50000(6.5 5.9525) if S
1
= 6.5,
.
Hanks prot after taxes is
50000 (max(6, S
1
) 5.9525)
=

50000(6 5.9525)(0.7) if S
1
= 5.5,
50000(6.5 5.9525)(0.7) if S
1
= 6.5,
Hanks expected prot after taxes is
(0.5)(50000)(65.9525)(0.7)+(0.5)(50000)(6.55.9525)(0.7) = 10412.5.
The biggest expected prot after taxes is attained when buying the
put.
c 2009. Miguel A. Arcones. All rights reserved. Manual for SOA Exam FM/CAS Exam 2.
16/44
Chapter 7. Derivatives markets. Section 7.9. Risk Management.
There are several reasons not to hedge:

paying transaction costs.

need expertise to asses costs and benets of a given strategy.

need expertise to do accounting and taxes in derivative


transactions.
In the real world, small companies are discouraged to do
derivatives because the reasons above. However, large companies
have nancial, accounting and legal departments which allow them
to take advantage of the opportunities on market derivatives. The
nancial department of a large company can asses derivatives as
well or better than the market does. Their legal and accounting
departments allow them to take advantage of the current tax laws.
c 2009. Miguel A. Arcones. All rights reserved. Manual for SOA Exam FM/CAS Exam 2.
17/44
Chapter 7. Derivatives markets. Section 7.9. Risk Management.
A producer perspective on risk management.
(URMC) Utah Red Mountain Company is a coppermining
company. It plans to mine and sell 1,000,000 pounds of copper
over the next year. Suppose that it sells all the next years
production, precisely one year from today. Suppose that the rm
incurs in two types of costs: xed costs and variable costs. Fixed
costs are assumed whether its mine is open or closed. Variable
costs are assumed only its mine is open. Suppose that the total
xed costs add to $0.75/lb. and the total variable costs add to
$2.25/lb. Let S
1
be the price of copper per pound in one year.

The net income per pound if the mine is open is


S
1
0.75 2.25 = S
1
3.00.

The net income per pound if the mine is closed is 0.75/lb.


c 2009. Miguel A. Arcones. All rights reserved. Manual for SOA Exam FM/CAS Exam 2.
18/44
Chapter 7. Derivatives markets. Section 7.9. Risk Management.
Utah Red Mountain Company would be better to close its mine if
S
1
3 0.75, which is equivalent to S
1
2.25.
URMC would be better keep its mine open if S
1
2.25. But, if
2.25 S
1
3, URMC assumes the loss 3 S
1
. If S
1
3 and
URMC keeps its mine open, its (positive) net income is S
1
3. The
following table shows the net income of URMC (see also Figure 1).
c 2009. Miguel A. Arcones. All rights reserved. Manual for SOA Exam FM/CAS Exam 2.
19/44
Chapter 7. Derivatives markets. Section 7.9. Risk Management.
Figure 1: Prot according whether the mine is closed, insured open and
open insured with a forward.
c 2009. Miguel A. Arcones. All rights reserved. Manual for SOA Exam FM/CAS Exam 2.
20/44
Chapter 7. Derivatives markets. Section 7.9. Risk Management.
Suppose that URMC can enter a short forward contract agreeing
to sell its copper one year from now. Suppose that F
0,1
= 3.5/lb.
If Utah Red Mountain Company enters this contract, its prot is
$0.5/lb. In this way Utah Red Mountain Company reduces risk.
Figure 1 shows the graph of the prot under the three considered
alternatives. For an uninsured position, the possible losses can be
very high. However, by entering the forward, the company has a
xed benet.
c 2009. Miguel A. Arcones. All rights reserved. Manual for SOA Exam FM/CAS Exam 2.
21/44
Chapter 7. Derivatives markets. Section 7.9. Risk Management.
However, URMC would like to benet if the price of the copper
goes higher than $3.5/lb. It could use options. Suppose that the
continuous rate of interest is 0.05 and the variability is = 0.25.
Using the BlackScholes formula, we have the following table of
premiums of options:
Table 1:
K 3.3 3.4 3.5 3.6 3.7 3.8 3.9
Call(K, T) 0.42567 0.3762 0.33119 0.29048 0.25386 0.22111 0.19196
Put(K, T) 0.23542 0.28107 0.33119 0.3856 0.44411 0.50648 0.57245
c 2009. Miguel A. Arcones. All rights reserved. Manual for SOA Exam FM/CAS Exam 2.
22/44
Chapter 7. Derivatives markets. Section 7.9. Risk Management.
Utah Red Mountain Company could buy a 3.7strike put. URMCs
prot per pound is
S
1
3 + max(3.7 S
1
, 0) (0.44411)e
0.05
=max(3.7, S
1
) 3.466880007
=

0.2331199934 if S
1
< 3.7,
S
1
3.466880007 if 3.7 S
1
.
c 2009. Miguel A. Arcones. All rights reserved. Manual for SOA Exam FM/CAS Exam 2.
23/44
Chapter 7. Derivatives markets. Section 7.9. Risk Management.
Under this strategy, URMC does not benet much if the price of
copper is high. Another strategy is to buy a 3.4strike put. The
prot per pound is
S
1
3 + max(3.4 S
1
, 0) (0.28107)e
0.05
=max(3.4, S
1
) 3.295480767
=

0.104519233 if S
1
< 3.4,
S
1
3.295480767 if 3.4 S
1
.
Under this strategy, the company makes $0.17139924/lb more
than before if the price of copper is over $3.7/oz. However, its
guaranteed prot is 0.104519233, which is smaller than the
guaranteed prot under a 3.7strike put.
c 2009. Miguel A. Arcones. All rights reserved. Manual for SOA Exam FM/CAS Exam 2.
24/44
Chapter 7. Derivatives markets. Section 7.9. Risk Management.
See Figure 2 for the prot under a short forward, a 3.7strike put
and 3.4strike put.
Figure 2: Prot for forward, 3.4strike put and 3.7strike put.
c 2009. Miguel A. Arcones. All rights reserved. Manual for SOA Exam FM/CAS Exam 2.
25/44
Chapter 7. Derivatives markets. Section 7.9. Risk Management.
In other words, buying a put is like buying insurance against small
spot prices. As bigger the strike price as bigger the price of the
insurance. As bigger the strike price as bigger the obtained
payment when the insurance is needed. A producer company needs
to buy a put with a strike large enough strike to cover low spot
prices. If the strike put is too large, the company will be wasting
money in insurance which it does not need.
c 2009. Miguel A. Arcones. All rights reserved. Manual for SOA Exam FM/CAS Exam 2.
26/44
Chapter 7. Derivatives markets. Section 7.9. Risk Management.
Example 4
Utah Red Mountain Company has the following prots:
(i) If it does not insure, S
1
3.00.
(ii) If it enters a short forward, 0.5.
(iii) If it buys a 3.4strike put, max(3.4, S
1
) 3.295480767.
Suppose that an actuary consulting for Utah Red Mountain
Company estimates that the price of cooper in one year will be
either 2.6, or 3.6, or 4.6, with respective probabilities 0.36, 0.33
and 0.31.
(i) Compute the URMCs expected prot before taxes for each of
the above strategies. Find the strategy with the biggest expected
prot before taxes.
(ii) The URMC pays a 40% tax rate, and has no tax benets for
losses. Compute the URMCs expected prot after taxes for each
of the above strategies. Find the strategy with the biggest
expected prot after taxes.
c 2009. Miguel A. Arcones. All rights reserved. Manual for SOA Exam FM/CAS Exam 2.
27/44
Chapter 7. Derivatives markets. Section 7.9. Risk Management.
Solution: (i)
uninsured prot 0.4 0.6 1.6
prot under a short forward 0.5 0.5 0.5
prot under a 3.4strike put 0.104519233 0.304519233 1.304519233
S
T
2.6 3.6 4.6
Probability 0.36 0.33 0.31
The expected prot under an uninsured position is
(0.40)(0.36) + (0.60)(0.33) + (1.60)(0.31) = 0.55.
The expected prot under a short forward is 0.5. The expected
prot under a 3.7strike put is
(0.104519233)(0.36) + (0.304519233)(0.33)(1.304519233)(0.31)
=0.542519233.
The strategy with the biggest expected prot before taxes is the
uninsured position.
c 2009. Miguel A. Arcones. All rights reserved. Manual for SOA Exam FM/CAS Exam 2.
28/44
Chapter 7. Derivatives markets. Section 7.9. Risk Management.
Solution: (ii) After taxes, the prots are given by the following
table:
uninsured prot 0.40 0.36 0.96
prot with a forward 0.3 0.3 0.3
prot with a 3.4strike put 0.0627115398 0.1827115398 0.7827115398
Probability 0.36 0.33 0.31
The expected prot after taxes for an uninsured position is
(0.40)(0.36) + (0.36)(0.33) + (0.96)(0.31) = 0.2724.
The expected prot after taxes under a short forward is
(0.6)(0.5) = 0.3. The expected prot after taxes under a a
3.4strike put is (0.6)(0.542519233) = 0.3255115398.
c 2009. Miguel A. Arcones. All rights reserved. Manual for SOA Exam FM/CAS Exam 2.
29/44
Chapter 7. Derivatives markets. Section 7.9. Risk Management.
Another strategy for URMC is to buy a 3.6strike put and sell a
3.9strike call option. Buying these two options, URMS will sell
cooper at min(max(3.6, S
1
), 3.9). This strategy is called a collar.
URMCs prot per pound is
S
1
3 + max(3.6 S
1
, 0) max(S
1
3.9, 0)
(0.3856 0.19196)e
0.05
=3 + 3.9 + max(3.6, S
1
) max(S
1
, 3.9) (0.3856 0.19196)e
0.05
=3 + 3.9 + max(3.6, S
1
) max(S
1
, 3.9, 3.6) (0.3856 0.19196)e
0.05
=min(max(3.6, S
1
), 3.9) 3
(0.3856 0.19196)e
0.05
=min(max(3.6, S
1
), 3.9) 3.203568135
=

0.396431865 if S
1
< 3.6,
S
1
3.203568135 if 3.6 S
1
< 3.9,
0.696431865 if 3.9 < S
T
.
c 2009. Miguel A. Arcones. All rights reserved. Manual for SOA Exam FM/CAS Exam 2.
30/44
Chapter 7. Derivatives markets. Section 7.9. Risk Management.
Under this strategy the prot of the company is very close to that
of a forward contract. But, instead of winning a constant of
$0.5/lb. in the forward contract, the companys prot varies with
the future price of copper, although not much.
Figure 3: Prot for a 3.63.9 collar.
c 2009. Miguel A. Arcones. All rights reserved. Manual for SOA Exam FM/CAS Exam 2.
31/44
Chapter 7. Derivatives markets. Section 7.9. Risk Management.
Another type of strategies are the ones called paylater strategies.
By a buying a put, URMC hedges against low prices. But, if the
prices are high, its prot is not as high as when it does not hedge.
A paylater strategy allows to have the usual prot for high enough
spot prices. It is like the price of the insurance does not need to be
paid. Consider the strategy of buying two 3.6strike puts and
buying a Kstrike call, such that the cost of the portfolio is zero.
The cost of two 3.6strike puts is (2)(0.3856) = 0.7712. By
numerical methods, we have that the cost of a 4.1763strike put is
0.7712. Hence, K = 4.1763. The prot of this strategy is
S
1
3 + 2 max(3.6 S
1
, 0) max(4.1763 S
1
, 0)
=3 + 2 max(3.6, S
1
) max(4.1763, S
1
)
=

0.0237 if S
1
< 3.6,
2S
1
7.1763 if 3.6 S
1
< 4.1763,
S
1
3 if 4.1763 < S
1
,
c 2009. Miguel A. Arcones. All rights reserved. Manual for SOA Exam FM/CAS Exam 2.
32/44
Chapter 7. Derivatives markets. Section 7.9. Risk Management.
Notice that under the previous strategy Company URMC always
has a positive prot and if the spot price is bigger than 4.1763, its
prot is the same as it would not hedge.
Figure 4: Prot for a paylater.
c 2009. Miguel A. Arcones. All rights reserved. Manual for SOA Exam FM/CAS Exam 2.
33/44
Chapter 7. Derivatives markets. Section 7.9. Risk Management.
The buyers perspective on risk management.
Toughminum makes fridges. Suppose that:
(i) The xed cost per fridge is $100.
(ii) Toughminum sells fridges for $350.
(iii) To manufacture a fridge Toughminum need 5 pounds of
aluminum.
Let S
T
be the price of a pound of aluminum at time T. The prot
one year from now is
350 (5)(S
1
) 100 = 250 5S
1
.
c 2009. Miguel A. Arcones. All rights reserved. Manual for SOA Exam FM/CAS Exam 2.
34/44
Chapter 7. Derivatives markets. Section 7.9. Risk Management.
Figure 5 shows the graph of this prot (uninsured line).
Figure 5: Prot uninsured, long forward and call.
c 2009. Miguel A. Arcones. All rights reserved. Manual for SOA Exam FM/CAS Exam 2.
35/44
Chapter 7. Derivatives markets. Section 7.9. Risk Management.
Toughminum could faces severe loses if the price of the aluminum
goes very high. To hedge risk, it could enter a long forward. If
aluminum is selling at $40 a pound in the forward market, the
prot of entering a long forward is
350 (5)(40) 100 = 50.
Figure 5 shows the graph of this prot (forward line).
c 2009. Miguel A. Arcones. All rights reserved. Manual for SOA Exam FM/CAS Exam 2.
36/44
Chapter 7. Derivatives markets. Section 7.9. Risk Management.
Another alternative is to buy a call. Suppose that Toughminum
buys a 35strike call with an expiration date one year from now
and nominal amount 5 lbs. Assume that = 0.35 and r = 0.06.
Then, Call(35, 1) = 7.609104. The prot is
250 5S
1
+ 5 max(S
1
35, 0) 5Call(35, 1)e
r
=250 + 5 max(35, S
1
) 5(7.609104)e
0.06
=209.6018764 5 min(35, S
1
)
=

209.6018764 5S
1
if S
1
< 35,
34.6018764 if 35 S
1
.
Figure 5 shows the graph of this prot (call line).
c 2009. Miguel A. Arcones. All rights reserved. Manual for SOA Exam FM/CAS Exam 2.
37/44
Chapter 7. Derivatives markets. Section 7.9. Risk Management.
Example 5
Toughminum has the following prots:
(i) If it does not insure, 250 5S
1
.
(ii) If it enters a short forward 50.
(iii) If it buys a 35strike call, 209.6018 5 min(35, S
1
).
Suppose that an actuary consulting for Toughminum estimates
that the price of aluminum in one year will be either 30, 35, or 40,
or 55, with respective probabilities 0.25, 0.25, 0.25, and 0.25.
(i) Compute the Toughminums expected prot before taxes for
each of the above strategies. Find the strategy with the biggest
expected prot before taxes.
(ii) The Toughminum pays a 35% tax rate, and has no tax benets
for losses. Compute the Toughminums expected prot after taxes
for each of the above strategies. Find the strategy with the biggest
expected prot after taxes.
c 2009. Miguel A. Arcones. All rights reserved. Manual for SOA Exam FM/CAS Exam 2.
38/44
Chapter 7. Derivatives markets. Section 7.9. Risk Management.
Solution: (i)
uninsured prot 100 75 50 25
prot under a long forward 50 50 50 50
prot under a 35strike call 59.6019 34.6019 34.6019 34.6019
S
T
30 35 40 55
Probability 0.25 0.25 0.25 0.25
The expected prot under an uninsured position is
(100)(0.25) + (75)(0.25) + (50)(0.25) + (25)(0.25) = 50.
The expected prot under a long forward is 50. The expected
prot under a 35strike call is
(59.6019)(0.25) + (34.6019)(0.25) + (34.6019)(0.25) + (34.6019)(0.25) = 40.8519.
The strategies with the biggest expected payo are the uninsured
position and the long forward position.
c 2009. Miguel A. Arcones. All rights reserved. Manual for SOA Exam FM/CAS Exam 2.
39/44
Chapter 7. Derivatives markets. Section 7.9. Risk Management.
Solution: (ii) After taxes, the prots are given by the following
table:
uninsured prot 65.00 48.75 32.50 25.00
prot with a long forward 32.5 32.5 32.5 32.5
prot under a 35strike call 38.7412 22.4912 22.4912 22.4912
S
T
30 35 40 55
Probability 0.25 0.25 0.25 0.25
The expected prot for an uninsured position is
(65)(0.25)+(48.75)(0.25)+(32.5)(0.25)+(25)(0.25) = 30.3125.
The expected prot under a long forward is 32.5. The expected
prot under a 35strike call is
(38.7412)(0.25) + (22.4912)(0.25) + (22.4912)(0.25) + (22.4912)(0.25) = 26.5537.
The strategy with the biggest expected payo is the long forward
position.
c 2009. Miguel A. Arcones. All rights reserved. Manual for SOA Exam FM/CAS Exam 2.
40/44
Chapter 7. Derivatives markets. Section 7.9. Risk Management.
Toughminum could sell a 30strike put option and buy a 45strike
call option. Both with nominal amount 5 lbs. This position is
called a 3045 reverse collar. Under this position, Toughminum
will buy aluminum at min(max(30, S
1
), 45) per lb. The cost of a
30strike put is $1.308289/lb. The cost of a 45strike call is
$3.514166/lb. The prot per ounce is
350 100 5S
1
5 max(30 S
1
, 0) + 5 max(S
1
45, 0)
+ 5Put(30, 1)e
r
5Call(45, 1)e
r
=250 5 max(30, S
1
) 5(45) + 5 max(S
1
, 45)
+ 5(1.308289)e
0.06
5(3.514166)e
0.06
=238.2886 5 max(30, S
1
) 5(45) + 5 max(S
1
, 45, 30)
=238.2886 5 min(max(30, S
1
), 45)
or
prot =

88.2886 if S
1
< 30,
238.2886 5S
1
if 30 S
1
< 45,
13.2886 if 45 S
1
.
c 2009. Miguel A. Arcones. All rights reserved. Manual for SOA Exam FM/CAS Exam 2.
41/44
Chapter 7. Derivatives markets. Section 7.9. Risk Management.
The graph of this prot in Figure 6.
Figure 6: Prot for a 3045 reverse collar.
c 2009. Miguel A. Arcones. All rights reserved. Manual for SOA Exam FM/CAS Exam 2.
42/44
Chapter 7. Derivatives markets. Section 7.9. Risk Management.
Importer/exporters perspective.
Suppose that Company ABCD imports electronics to Canada. It
gets paid in Canadian dollars. Whenever this company gets a
payment, it needs to exchange Canadian dollars into US dollars.
Suppose that in two months, ABCD expects to get 100,000
Canadian dollars. If the price of a Canadian dollar at time T is S
T
,
the amount of US dollars the company ABCD will get is
100000
S
2/12
.
To hedge against possible changes in exchange rates, company
ABCD can sell a forward on 100000 Canadian dollars at the
current price. It also can buy a put on Canadian dollars.
c 2009. Miguel A. Arcones. All rights reserved. Manual for SOA Exam FM/CAS Exam 2.
43/44
Chapter 7. Derivatives markets. Section 7.9. Risk Management.
Example 6
Suppose that Company ABCD buys a put on (Canadian $s)
CAD100000 with a strike price of (U.S.A. dollar) USD0.85 per
CAD for USD0.01 per CAD. Two months later, ABCD receives
CAD100000. At this moment the exchange rate is USD0.845 per
CAD.
(i) How many US dollars does company ABCD gets in this
transaction?
(ii) How many US dollars would company ABCD have gotten in
the exchange if it would not have signed the put?
Solution: (i) Since the strike price is bigger than the current spot
price, the company exercises the put. It gets
(100000)(0.85 0.01) = 84000 US dollars.
(i) Company ABCD would have got 100000(0.845) = 84500 US
dollars
c 2009. Miguel A. Arcones. All rights reserved. Manual for SOA Exam FM/CAS Exam 2.
44/44
Chapter 7. Derivatives markets. Section 7.9. Risk Management.
Example 6
Suppose that Company ABCD buys a put on (Canadian $s)
CAD100000 with a strike price of (U.S.A. dollar) USD0.85 per
CAD for USD0.01 per CAD. Two months later, ABCD receives
CAD100000. At this moment the exchange rate is USD0.845 per
CAD.
(i) How many US dollars does company ABCD gets in this
transaction?
(ii) How many US dollars would company ABCD have gotten in
the exchange if it would not have signed the put?
Solution: (i) Since the strike price is bigger than the current spot
price, the company exercises the put. It gets
(100000)(0.85 0.01) = 84000 US dollars.
(i) Company ABCD would have got 100000(0.845) = 84500 US
dollars
c 2009. Miguel A. Arcones. All rights reserved. Manual for SOA Exam FM/CAS Exam 2.
1/78
Chapter 7. Derivatives markets.
Manual for SOA Exam FM/CAS Exam 2.
Chapter 7. Derivatives markets.
Section 7.10. Swaps.
c 2009. Miguel A. Arcones. All rights reserved.
Extract from:
Arcones Manual for the SOA Exam FM/CAS Exam 2,
Financial Mathematics. Fall 2009 Edition,
available at http://www.actexmadriver.com/
c 2009. Miguel A. Arcones. All rights reserved. Manual for SOA Exam FM/CAS Exam 2.
2/78
Chapter 7. Derivatives markets. Section 7.10. Swaps.
Swaps
Denition 1
A swap is a contract between two counterparts to exchange two
similar nancial quantities which behave dierently.

The two things exchanged are called the legs of the swap.

A common type of swap involves a commodity. Another


common type of swap is an interest rate swap of a xed
interest rate in return for receiving an adjustable rate.

Usually, one leg involves quantities that are known in advance,


known as the xed leg. The other involves quantities that are
(uncertain) not known in advance, known as the oating leg.

Usually, a swap entails the exchange of payments over time.


c 2009. Miguel A. Arcones. All rights reserved. Manual for SOA Exam FM/CAS Exam 2.
3/78
Chapter 7. Derivatives markets. Section 7.10. Swaps.
LIBOR.
The (London Interbank oce rate) LIBOR is the most widely
used reference rate for short term interest rates worldwide. The
LIBOR is published daily the (British Bankers Association) BBA. It
is based on rates that large international banks in London oer
each other for interbank deposits. Rates are quoted for 1month,
3month, 6month and 12month deposits.
c 2009. Miguel A. Arcones. All rights reserved. Manual for SOA Exam FM/CAS Exam 2.
4/78
Chapter 7. Derivatives markets. Section 7.10. Swaps.
The following table shows the LIBOR interest rates for a loan in
dollars during a week on June, 2007:
Date 1month 3month 6month 9month 12month
6182007 5.3200% 5.3600% 5.4000% 5.4400% 5.4741%
6192007 5.3200% 5.3600% 5.3981% 5.4369% 5.4650%
6202007 5.3200% 5.3600% 5.3931% 5.4194% 5.4387%
6212007 5.3200% 5.3600% 5.3934% 5.4273% 5.4531%
6222007 5.3200% 5.3600% 5.3900% 5.4200% 5.4494%
The previous rates are from http://www.bba.org.uk/bba.
c 2009. Miguel A. Arcones. All rights reserved. Manual for SOA Exam FM/CAS Exam 2.
5/78
Chapter 7. Derivatives markets. Section 7.10. Swaps.
A LIBOR loan is an adjustable loan on which the interest rate is
tied to a specied Libor. The interest rate is the most recent value
of the LIBOR plus a margin, subject to any adjustment cap.
LIBOR is used in determining the price of interest rate futures,
swaps and Eurodollars. The most important nancial derivatives
related to LIBOR are Eurodollar futures. Traded at the Chicago
Mercantile Exchange (CME), Eurodollars are US dollars deposited
at banks outside the United States, primarily in Europe. The
interest rate paid on Eurodollars is largely determined by LIBOR.
Eurodollar futures provide a way of betting on or hedging against
future interest rate changes.
c 2009. Miguel A. Arcones. All rights reserved. Manual for SOA Exam FM/CAS Exam 2.
6/78
Chapter 7. Derivatives markets. Section 7.10. Swaps.
Structure of interest rates.
Let P(0, t) be the price of a $1face value zero coupon bond
maturing on date t. Notice that
1
P(0,t)
is the interest factor from
time zero to time t, i.e. $1 invested at time 0 accumulates to
1
P(0,t)
at time t. P(0, t) is the discount factor from time zero to
time t. The implied interest factor from time t
j 1
to time t
j
is
P(0,t
j 1
)
P(0,t
j
)
. The implied forward rate from time t
j 1
to time t
j
is
r
0
(t
j 1
, t
j
) =
P(0,t
j 1
)
P(0,t
j
)
1. Let s
n
be the nyear spot rate. Then,
(1 + s
n
)
n
=
1
P(0,n)
.
c 2009. Miguel A. Arcones. All rights reserved. Manual for SOA Exam FM/CAS Exam 2.
7/78
Chapter 7. Derivatives markets. Section 7.10. Swaps.
A quantity which appear later is the coupon rate R for a nyear
bond with annual coupons and face value, redemption value and
price all equal to one. The price of this bond is
1 =
n

j =1
RP(0, j ) + P(0, n).
Hence,
R =
1 P(0, n)

n
j =1
P(0, j )
.
c 2009. Miguel A. Arcones. All rights reserved. Manual for SOA Exam FM/CAS Exam 2.
8/78
Chapter 7. Derivatives markets. Section 7.10. Swaps.
Example 1
The following table lists prices of zerocoupon $1face value bonds
with their respective maturities:
Number of years to maturity Price
1 $0.956938
2 $0.907029
3 $0.863838
4 $0.807217
(i) Calculate the 1year, 2year, 3year, and 4year spot rates of
interest.
(ii) Calculate the 1year, 2year, and 3year forward rates of
interest.
(iii) Calculate the coupon rate R for a j year bond with annual
coupons whose face value, redemption value and price are all one.
c 2009. Miguel A. Arcones. All rights reserved. Manual for SOA Exam FM/CAS Exam 2.
9/78
Chapter 7. Derivatives markets. Section 7.10. Swaps.
Solution: (i) Note that the price of j th bond is
P(0, j ) = (1 + s
j
)
j
. Hence, s
j
=
1
P(0,j )
1/j
1. In particular,
s
1
=
1
P(0, 1)
1 =
1
0.956938
1 = 4.499967135%
s
2
=
1
P(0, 2)
1/2
1 =
1
0.907029
1/2
1 = 5.000027694%
s
3
=
1
P(0, 3)
1/3
1 =
1
0.863838
1/3
1 = 4.999983734%
s
4
=
1
P(0, 4)
1/4
1 =
1
0.807217
1/4
1 = 5.499991613%
c 2009. Miguel A. Arcones. All rights reserved. Manual for SOA Exam FM/CAS Exam 2.
10/78
Chapter 7. Derivatives markets. Section 7.10. Swaps.
Solution: (ii) To get the j 1 year forward rate f
j
, we do
f
j
=
(1+s
j
)
j
(1+s
j 1
)
j 1
1 =
P(0,j 1)
P(0,j )
1. We get that:
f
2
=
0.956938
0.907029
1 = 5.502470153%,
f
3
=
0.907029
0.863838
1 = 4.999895814%,
f
4
=
0.863838
0.807217
1 = 7.014346824%.
c 2009. Miguel A. Arcones. All rights reserved. Manual for SOA Exam FM/CAS Exam 2.
11/78
Chapter 7. Derivatives markets. Section 7.10. Swaps.
Solution: (iii) We have that R
j
=
1P(0,j )
P
j
k=1
P(0,k)
. Hence, we get that:
R
1
=
10.956938
0.956938
1 = 4.499978055%,
R
2
=
10.907029
0.956938+0.907029
1 = 4.987802896%,
R
3
=
10.863838
0.956938+0.907029+0.863838
1 = 4.991632466%,
R
4
=
10.807217
0.956938+0.907029+0.863838+0.807217
1 = 5.453516272%.
c 2009. Miguel A. Arcones. All rights reserved. Manual for SOA Exam FM/CAS Exam 2.
12/78
Chapter 7. Derivatives markets. Section 7.10. Swaps.
Example 2
Suppose the current LIBOR discount factors P(0, t
j
) are given by
the table below.
LIBOR
discout rates
P(0, t
j
)
0.986923 0.973921 0.961067 0.948242
time in months 3 6 9 12
Calculate the annual nominal interest rate compounded quarterly
for a loan for the following maturity dates: 3, 6, 9 and 12 months.
c 2009. Miguel A. Arcones. All rights reserved. Manual for SOA Exam FM/CAS Exam 2.
13/78
Chapter 7. Derivatives markets. Section 7.10. Swaps.
Solution: Let s
(4)
j
be the annual nominal interest rate
compounded quarterly for a loan maturing in 3j months,
j = 1, 2, 3, 4. Then, (1 + s
(2)
j
/4)
j
=
1
P(0,j /4)
. So,
s
(4)
1
= 4
_
1
0.986923
1
_
= 5.300109532%,
s
(4)
2
= 4
_
_
1
0.973921
_
1/2
1
_
= 5.320086032%,
s
(4)
3
= 4
_
_
1
0.961067
_
1/3
1
_
= 5.330019497%,
s
(4)
4
= 4
_
_
1
0.948242
_
1/4
1
_
= 5.350015985%.
c 2009. Miguel A. Arcones. All rights reserved. Manual for SOA Exam FM/CAS Exam 2.
14/78
Chapter 7. Derivatives markets. Section 7.10. Swaps.
We denote by r
t
0
(t
1
, t
2
) to the nonannualized interest rate for the
period from t
1
to t
2
using the interest rates at t
0
, i.e. 1 +r
t
0
(t
1
, t
2
)
is the interest factor for the period from t
1
to t
2
using the interest
rates at t
0
. We denote by P
t
0
(t
0
, t
1
) to the price at time t
0
of a
zerocoupon bond with face value $1 and redemption time t
1
. So,
1 + r
t
0
(t
1
, t
2
) =
P
t
0
(t
0
, t
1
)
P
t
0
(t
0
, t
2
)
.
Notice that we abbreviate P
0
(0, t) = P(0, t).
c 2009. Miguel A. Arcones. All rights reserved. Manual for SOA Exam FM/CAS Exam 2.
15/78
Chapter 7. Derivatives markets. Section 7.10. Swaps.
Forward rate agreement.
Suppose that a borrower plans to take a loan of $L at time t
1
,
where t
1
> 0. He will repaid the loan at time t
2
, where t
2
> t
1
.
The amount of the loan payment depends on the term structure of
interest rates at time t
1
. Let r
t
1
(t
1
, t
2
) be interest rate from t
1
to
t
2
with respect to the structure of interest rates at time t
1
, i.e. a
zerocoupon bond with face value F and redemption time t
2
costs
F
1+r
t
1
(t
1
,t
2
)
at time t
1
. To pay the loan, the borrower needs to pay
$L(1 + r
t
1
(t
1
, t
2
)) at time t
2
.
c 2009. Miguel A. Arcones. All rights reserved. Manual for SOA Exam FM/CAS Exam 2.
16/78
Chapter 7. Derivatives markets. Section 7.10. Swaps.
Since r
t
1
(t
1
, t
2
) is unknown at time zero, the borrower does not
know how much it will have to pay for the loan. In order to hedge
against increasing interest rates, the borrower can enter into a
(FRA) forward rate agreement. A FRA is a nancial contract to
exchange interest payments for a notional principal on settlement
date for a specied period from start date to maturity date.
Usually one of the interest payments is relative to a benchmark
such as the LIBOR. This is a oating interest rate, which was
described in Subsection 2. The other interest payment is with
respect to a xed rate of interest. An FRA contract is settled in
cash. The settlement can be made either at the beginning or at
the end of the considered period, i.e. either at the borrowing time
or at the time of repayment of the loan.
c 2009. Miguel A. Arcones. All rights reserved. Manual for SOA Exam FM/CAS Exam 2.
17/78
Chapter 7. Derivatives markets. Section 7.10. Swaps.
The two payments involved in an FRA are called legs. Both
payments are made at time t
2
. Usually FRAs are
oatingagainstxed. One leg consists of an interest payment
with respect to a oating rate. The interest payment of the
oating rate leg is Lr
t
1
(t
1
, t
2
). The side making the oatingrate
leg payment is called either the oatingrate leg party, or the
oatingrate side, or the oatingrate payer. The interest
payment of the xed rate leg is Lr
FRA
, where r
FRA
is an interest
rate specied in the contract. The side making the xedrate leg
payment is called either the xedrate leg party, or the
xedrate side, or the xedrate payer.
c 2009. Miguel A. Arcones. All rights reserved. Manual for SOA Exam FM/CAS Exam 2.
18/78
Chapter 7. Derivatives markets. Section 7.10. Swaps.
If the FRA is settled at the time of the repayment of the loan, we
say that the FRA is settled in arrears.
Suppose that the FRA is settled at time t
2
(in arrears). The FRA
is settled in two dierent ways:
1. If L(r
FRA
r
t
1
(t
1
, t
2
)) > 0, the xedrate side makes a payment
of L(r
FRA
r
t
1
(t
1
, t
2
)) to the oatingrate side.
2. If L(r
FRA
r
t
1
(t
1
, t
2
)) < 0, the oatingrate side makes a
payment of L(r
t
1
(t
1
, t
2
) r
FRA
) to the xedrate side.
c 2009. Miguel A. Arcones. All rights reserved. Manual for SOA Exam FM/CAS Exam 2.
19/78
Chapter 7. Derivatives markets. Section 7.10. Swaps.
Usually an FRA is mentioned as an exchange of (interest
payments) legs. By interchanging their legs, it is meant that:
1. The oatingrate leg party makes a payment of Lr
t
1
(t
1
, t
2
) to
its counterpart.
2. The xedrate leg party makes a payment of Lr
FRA
to its
counterpart.
The combination of these two payments is: the xedrate leg party
makes a payment of L(r
FRA
r
t
1
(t
1
, t
2
)) to the oatingrate leg
party. This means that if L(r
FRA
r
t
1
(t
1
, t
2
)) is a negative
number, the oatingrate side makes a payment of
L(r
t
1
(t
1
, t
2
) r
FRA
) to the xedrate side.
c 2009. Miguel A. Arcones. All rights reserved. Manual for SOA Exam FM/CAS Exam 2.
20/78
Chapter 7. Derivatives markets. Section 7.10. Swaps.
Example 3
Company A pays $75,000 in interest payments at the end of one
year. Company B pays the thencurrent LIBOR plus 50 basis
points on a $1,000,000 loan at the end of the year. Suppose that
the two companies enter into an interest payment swap. Suppose
that in one year the current LIBOR rate is 6.45%. Find which
company is making a payment at the end of year and its amount.
Solution: Company Bs interest payment is
(1000000)(0.0645 + 0.0050) = 69500. To settle the forward
interest agreement, company A must make a payment of
75000 69500 = 5500 to Company B.
c 2009. Miguel A. Arcones. All rights reserved. Manual for SOA Exam FM/CAS Exam 2.
21/78
Chapter 7. Derivatives markets. Section 7.10. Swaps.
Example 3
Company A pays $75,000 in interest payments at the end of one
year. Company B pays the thencurrent LIBOR plus 50 basis
points on a $1,000,000 loan at the end of the year. Suppose that
the two companies enter into an interest payment swap. Suppose
that in one year the current LIBOR rate is 6.45%. Find which
company is making a payment at the end of year and its amount.
Solution: Company Bs interest payment is
(1000000)(0.0645 + 0.0050) = 69500. To settle the forward
interest agreement, company A must make a payment of
75000 69500 = 5500 to Company B.
c 2009. Miguel A. Arcones. All rights reserved. Manual for SOA Exam FM/CAS Exam 2.
22/78
Chapter 7. Derivatives markets. Section 7.10. Swaps.
The xedrate side payment is L(r
FRA
r
t
1
(t
1
, t
2
)). Assuming
that the xedrate side borrows L at time t
1
his total interest
payment at time t
2
is
Lr
t
1
(t
1
, t
2
) + L(r
FRA
r
t
1
(t
1
, t
2
)) = Lr
FRA
.
A borrower can enter into an FRA as a xedrate side to hedge
against increasing interest rates.
If the FRA is settled at time t
1
(at borrowing time), to settle the
FRA the oatingrate side makes a payment of
L(r
t
1
(t
1
,t
2
)r
FRA
)
1+r
t
1
(t
1
,t
2
)
to
the xedrate side. This number could be negative. If
r
FRA
> r
t
1
(t
1
, t
2
), the xedrate side makes a (positive) payment
of
L(r
FRA
r
t
1
(t
1
,t
2
))
1+r
t
1
(t
1
,t
2
)
to the oatingrate side.
c 2009. Miguel A. Arcones. All rights reserved. Manual for SOA Exam FM/CAS Exam 2.
23/78
Chapter 7. Derivatives markets. Section 7.10. Swaps.
Since the interest factor from time t
1
to time t
2
is 1 + r
t
1
(t
1
, t
2
),
the previous payos are equivalent to the ones for an FRA paid in
arrears. In this case, the xedrate side can apply the FRA
payment to the principal he borrows. He takes a loan of
L +
L(r
FRA
r
t
1
(t
1
, t
2
))
1 + r
t
1
(t
1
, t
2
)
=
L(1 + r
FRA
)
1 + r
t
1
(t
1
, t
2
)
at time t
1
. The principal of the loan at time t
2
is
(1 + r
t
1
(t
1
, t
2
))
L(1 + r
FRA
)
1 + r
t
1
(t
1
, t
2
)
= L(1 + r
FRA
).
Again, it is like the xedrate side is able to borrow at the rate
r
FRA
.
c 2009. Miguel A. Arcones. All rights reserved. Manual for SOA Exam FM/CAS Exam 2.
24/78
Chapter 7. Derivatives markets. Section 7.10. Swaps.
Usually the oatingrate side is a market maker. The FRA
agreement transfers interest rate risk from the xedrate side to
the oatingrate side. In order to hedge this interest rate risk, the
market maker could create a synthetic reverse FRA.
c 2009. Miguel A. Arcones. All rights reserved. Manual for SOA Exam FM/CAS Exam 2.
25/78
Chapter 7. Derivatives markets. Section 7.10. Swaps.
Suppose that the FRA is settled in arrears. The scalper buys a
zerocoupon bond maturing at t
1
with face value L and short sells
a zerocoupon bond with face value
LP(0,t
1
)
P(0,t
2
)
maturing at t
2
. The
scalper cashow at time zero is
LP(0, t
1
)
LP(0, t
1
)
P(0, t
2
)
P(0, t
2
) = 0.
At time t
1
, the scalper gets L from the rst bond, which invests at
the current interest rate. He gets L(1 + r
t
1
(t
1
, t
2
)) at time t
2
from
this investment. His total cashow at time t
2
is
L(1 + r
t
1
(t
1
, t
2
)) + L(r
FRA
r
t
1
(t
1
, t
2
))
LP(0, t
1
)
P(0, t
2
)
=L(1 + r
t
1
(t
1
, t
2
)) + L(r
FRA
r
t
1
(t
1
, t
2
)) L(1 + r
0
(t
1
, t
2
))
=L(r
FRA
r
0
(t
1
, t
2
)).
Hence, the no arbitrage rate of an FRA is r
0
(t
1
, t
2
), which is the
current nonannualized interest rate from t
1
to t
2
.
c 2009. Miguel A. Arcones. All rights reserved. Manual for SOA Exam FM/CAS Exam 2.
26/78
Chapter 7. Derivatives markets. Section 7.10. Swaps.
Example 4
Suppose that the current spot rates are given in the following table
(as annual nominal rates convertible semiannually)
spot rate 6% 7.5%
maturity (in months) 6 12
Timothy and David enter into separate forward rate agreements as
xedrate sides for the period of time between 6 months and 12
months. Both FRAs are for a notional amount $10000. Timothys
FRA is settled in 12 months. Davids FRA is settled in 6 months.
In six months, the annual nominal interest rate compounded
semiannually for a six month loan is 7%.
(i) Find the no arbitrage six month rate for an FRA for the period
of time between 6 months and 12 months.
(ii) Calculate Timothys payo from his FRA.
(iii) Calculate Davids payo from his FRA.
c 2009. Miguel A. Arcones. All rights reserved. Manual for SOA Exam FM/CAS Exam 2.
27/78
Chapter 7. Derivatives markets. Section 7.10. Swaps.
Solution: (i) The no arbitrage six month rate for a FRA for the
period of time between 6 months and 12 months is
r
FRA
=
_
1 +
0.075
2
_
2
1 +
0.06
2
1 = 4.5054612%.
(ii) Timothys payo is
(10000)(0.035 4.5054612) = 447.04612
(iii) Davids payo is
(10000)
(0.035 4.5054612)
1 +
0.07
2
)
= 431.9286184.
c 2009. Miguel A. Arcones. All rights reserved. Manual for SOA Exam FM/CAS Exam 2.
28/78
Chapter 7. Derivatives markets. Section 7.10. Swaps.
Interest rate swaps.
An interest rate swap is a contract in which one party exchanges
a stream of interest payments for another partys stream. Interest
rate swaps are normally xedagainstoating. Interest rate
swaps are valued using a notional amount. This nominal amount
can change with time. We only consider constant nominal
amounts. The xed stream of payments are computed with respect
to a rate determined by the contract. The oating stream of
payments are determined using a benchmark, such as the LIBOR.
c 2009. Miguel A. Arcones. All rights reserved. Manual for SOA Exam FM/CAS Exam 2.
29/78
Chapter 7. Derivatives markets. Section 7.10. Swaps.
Suppose that a rm is interested in borrowing a large amount of
money for a long time. One way to borrow is to issue bonds.
Unless its credit rating is good enough, the rm may have trouble
nding buyers. Lenders are unwilling to absorb long term loans
from a rm with a so and so credit rating. So, the rm may have
to borrow short term. Even if a company does not need to borrow
short term, usually short term interest rates are lower than long
term interest rates. As longer the maturity as more likely the
default. Anyhow, suppose that a rm is interested in borrowing
short term, but needs the cash long term. The rm takes a short
term loan. At maturity, the rm pays this loan and takes another
short term loan. This process will be repeated as many times as
needed. Current short term rates are known. But, the short term
interest rates which the rm may need to take in the future are
uncertain. The rm has an interest rate risk. If short term rates
increase, the company may get busted. To hedge this risk, the rm
may enter into an interest rate swap, which we describe next.
c 2009. Miguel A. Arcones. All rights reserved. Manual for SOA Exam FM/CAS Exam 2.
30/78
Chapter 7. Derivatives markets. Section 7.10. Swaps.
Suppose that a borrower takes a loan of L paying a oating
interest rate according a benchmark such as the LIBOR. Suppose
that the interest is paid at times t
1
< t
2
< < t
n
. The principal
owed after each payment is L. This means that at time t
j
, the
borrower pays Lr
t
j 1
(t
j 1
, t
j
) in interest, where 1 + r
t
j 1
(t
j 1
, t
j
) is
the interest factor from time t
j 1
to time t
j
, calculated using the
LIBOR at time t
j 1
. This rate is the rate which the borrower
would pay, if he borrows at time t
j 1
, pays this loan at time t
j
and
takes a new loan for L at time t
j
. The borrower is paying a stream
of oating interest rate payments. The borrower can hedge by
taking several FRAs. If the borrower would like to have a single
contract, he enters into a interest rate swap.
c 2009. Miguel A. Arcones. All rights reserved. Manual for SOA Exam FM/CAS Exam 2.
31/78
Chapter 7. Derivatives markets. Section 7.10. Swaps.
The borrower would like to enter into an interest rate swap so that
the current interest payments plus the payments to the swap will
add to a xed payment. This situation is similar to that of having
several FRAs. The borrower can enter an interest rate swap with
notional amount L. The borrower would like to have a xedrate
leg in its contract:
Payment LR LR LR
Time t
1
t
2
t
n
where R is the swap interest rate in the contract. The borrower
would like that its counterpart has a oatingrate leg:
Payment Lr
0
(0, t
1
) Lr
t
1
(t
1
, t
2
) Lr
t
n1
(t
n1
, t
n
)
Time t
1
t
2
t
n
c 2009. Miguel A. Arcones. All rights reserved. Manual for SOA Exam FM/CAS Exam 2.
32/78
Chapter 7. Derivatives markets. Section 7.10. Swaps.
An interest rate swap consists of an interchange of interest
payments. The total outcome of this interchange is that at every
time t
j
the oatingleg side makes a payment of
L(r
t
j 1
(t
j 1
, t
j
) R) to the xedleg side. Again
L(r
t
j 1
(t
j 1
, t
j
) R) could be negative. If
L(r
t
j 1
(t
j 1
, t
j
) R) < 0, the xedleg side makes a payment at
each time t
j
of L(R r
t
j 1
(t
j 1
, t
j
)) to the oatingleg side.
c 2009. Miguel A. Arcones. All rights reserved. Manual for SOA Exam FM/CAS Exam 2.
33/78
Chapter 7. Derivatives markets. Section 7.10. Swaps.
By exchanging legs, the borrower makes a payment at each time t
j
when L(R r
t
j 1
(t
j 1
, t
j
)) to his counterpart. The borrower is also
making interest payments of Lr
t
j 1
(t
j 1
, t
j
). The total borrowers
interest payments add to LR. By entering a swap, a borrower is
hedging against increasing interest rates.
The total borrowers cashow is that of a company issuing bonds.
Often the borrower has poor credit rating and it is unable to issue
bonds. In some sense, some borrowers enter into an interest rate
swap so that its counterpart issues a bond to them. Sometimes
the borrower uses a interest rate swap to avoid to issue a xed rate
long term loan. Takers of this loan could require a higher interest
to borrow.
c 2009. Miguel A. Arcones. All rights reserved. Manual for SOA Exam FM/CAS Exam 2.
34/78
Chapter 7. Derivatives markets. Section 7.10. Swaps.
Usually, the borrowers counterpart is a marketmaker, which must
hedge its interest rate risk. The (marketmaker) xedrate payer
gets a payment of L(r
t
j 1
(t
j 1
, t
j
) R) at each time t
j
. The
xedrate payer prot by entering the swap is
n

j =1
P(0, t
j
)L(r
t
j 1
(t
j 1
, t
j
) R).
c 2009. Miguel A. Arcones. All rights reserved. Manual for SOA Exam FM/CAS Exam 2.
35/78
Chapter 7. Derivatives markets. Section 7.10. Swaps.
By using bonds, a marketmaker can create a synthetic cashow of
payments equal to the swap payments. The cost of these bonds is
its present value according with the current term structure of
interest rates. Hence, if there is no arbitrage, a marketmaker can
arrange so that the cost of payments he receives is
n

j =1
LP(0, t
j
)(r
0
(t
j 1
, t
j
) R),
i.e. instead of using the uncertain rates r
t
j 1
(t
j 1
, t
j
), the scalper
can use the current forward rates. Therefore, the no arbitrage swap
rate is
R =

n
j =1
P(0, t
j
)r
0
(t
j 1
, t
j
)

n
j =1
P(0, t
j
)
.
The swap rate R when there is no arbitrage is called the par swap
rate. Notice that the par swap rate R is a weighted average of
implied forward rates r
0
(t
j 1
, t
j
). The weights depend on the
present value of a payment made at time t
j
.
c 2009. Miguel A. Arcones. All rights reserved. Manual for SOA Exam FM/CAS Exam 2.
36/78
Chapter 7. Derivatives markets. Section 7.10. Swaps.
Using that
P(0,t
j 1
)
P(0,t
j
)
= 1 + r
0
(t
j 1
, t
j
), we get that
R =

n
j =1
P(0, t
j
)
_
P(0,t
j 1
)
P(0,t
j
)
1
_

n
j =1
P(0, t
j
)
=

n
j =1
P(0, t
j 1
)

n
i =1
P(0, t
j
)

n
j =1
P(0, t
j
)
=
1 P(0, t
n
)

n
j =1
P(0, t
j
)
.
Notice that R is the coupon rate for a bond with price, face value
and redemption all equal, using the current term structure of
interest rates. It is like that the oatingrate party enters the swap
to use the market maker credit rating to issue a bond.
c 2009. Miguel A. Arcones. All rights reserved. Manual for SOA Exam FM/CAS Exam 2.
37/78
Chapter 7. Derivatives markets. Section 7.10. Swaps.
Example 5
Suppose the LIBOR discount factors P(0, t
j
) are given in the table
below. Consider a 3year swap with semiannual payments whose
oating payments are found using the LIBOR rate compiled a
semester before the payment is made. The notional amount of the
swap is 10000.
LIBOR
discount
rates
P(0, t
j
)
0.9748 0.9492 0.9227 0.8960 0.8687 0.8413
time (months) 6 12 18 24 30 36
(i) Calculate the par swap rate.
(ii) Calculate net payment made by the xedrate side in 18
months if the sixmonth LIBOR interest rate compiled in 12
months is 2.3%.
c 2009. Miguel A. Arcones. All rights reserved. Manual for SOA Exam FM/CAS Exam 2.
38/78
Chapter 7. Derivatives markets. Section 7.10. Swaps.
Solution: (i) The par swap rate is
R =
1P(0,t
n
)
P
n
j =1
P(0,t
j
)
=
10.8413
0.9748+0.9492+0.9227+0.8960+0.8687+0.8413
= 0.02910484714 = 2.910484714%.
(ii) The payment made by the xedrate side is
L(Rr
t
j 1
(t
j 1
, t
j
)) = 10000(0.029104847140.023) = 61.0484714.
c 2009. Miguel A. Arcones. All rights reserved. Manual for SOA Exam FM/CAS Exam 2.
39/78
Chapter 7. Derivatives markets. Section 7.10. Swaps.
Notice that the oating interest payment use the interest rates
compiled one period before the payment. These interest rates are
called realized interest rates.
Since interest rates change daily, we may be interested in the
market value of a swap contract. One of the parties in the swap
contract may sell/buy his position in the contract. The market
value of a swap contract for the xedrate payer is the present
value of the no arbitrage estimation of the payments which he will
receive. The market value of a swap contract for the xedrate
payer immediately after the kthe payment is
n

j =k+1
P(t
k
, t
j
)L(r
t
k
(t
j 1
, t
j
) R).
If this value is positive, the xedrate payer has exposure to
interest rates. Current interest rates are higher than when the swap
was issued. The market value of a swap is the no arbitrage price to
enter this contract. One of the counterparts in the contract may
be interested in selling/buying his position on the contract.
c 2009. Miguel A. Arcones. All rights reserved. Manual for SOA Exam FM/CAS Exam 2.
40/78
Chapter 7. Derivatives markets. Section 7.10. Swaps.
Example 6
Suppose current LIBOR discount factors P(0, t
j
) are given by the
table below. An interest rate swap has 6 payments left. The swap
rate is 3.5% per period. The notional principal is two million
dollars. The oating payments of this swap are the realized LIBOR
interest rates.
LIBOR
discount
rates
P(0, t
j
)
0.9748 0.9492 0.9227 0.8960 0.8687 0.8413
time (months) 6 12 18 24 30 36
Calculate market value of this swap for the xedrate payer.
c 2009. Miguel A. Arcones. All rights reserved. Manual for SOA Exam FM/CAS Exam 2.
41/78
Chapter 7. Derivatives markets. Section 7.10. Swaps.
Solution: The market value of the swap per dollar is
n

j =k+1
P(0, t
j
)(r
t
k
(t
j 1
, t
j
) R)
=0.9748
_
1
0.9748
1 0.035
_
+ 0.9492
_
0.9748
0.9492
1 0.035
_
+ 0.9227
_
0.9492
0.9227
1 0.035
_
+ 0.8960
_
0.9227
0.8960
1 0.035
_
+ 0.8687
_
0.8960
0.8687
1 0.035
_
+ 0.8413
_
0.8687
0.8413
1 0.035
_
=0.9748 (0.02585145671 0.035) + 0.9492 (0.02697008007 0.035)
+ 0.9227 (0.02872006069 0.035) + 0.8960 (0.02979910714 0.035)
+ 0.8687 (0.03142626914 0.035) + 0.8413 (0.03256864377 0.035)
=0.0321445.
The market value of the swap is
(2000000)(0.0321445) = 64289.
c 2009. Miguel A. Arcones. All rights reserved. Manual for SOA Exam FM/CAS Exam 2.
42/78
Chapter 7. Derivatives markets. Section 7.10. Swaps.
A deferred swap is a swap which begins in k periods. The swap
par rate is computed as
R =

n
j =k
P(0, t
j
)r
0
(t
j 1
, t
j
)

n
j =k
P(0, t
j
)
=

n
j =k
P(0, t
j
)
_
P(0,t
j 1
)
P(0,t
j
)
1
_

n
j =k
P(0, t
j
)
=

n
j =k
P(0, t
j 1
)

n
j =k
P(0, t
j
)

n
j =k
P(0, t
j
)
=
P(0, t
k1
) P(0, t
n
)

n
j =k
P(0, t
j
)
.
c 2009. Miguel A. Arcones. All rights reserved. Manual for SOA Exam FM/CAS Exam 2.
43/78
Chapter 7. Derivatives markets. Section 7.10. Swaps.
Example 7
Suppose the current annual nominal interest rates compounded
quarterly from the LIBOR are given by the table below. Two
counterparts enter into a xed against oating swap using the
LIBOR rate compiled a quarter before the payment is made. The
notional principal is $50000. The times of the swap are in 12, 15
and 18 months.
i
(4)
4.5% 4.55% 4.55% 4.6% 4.6% 4.65%
maturation time
in months
3 6 9 12 15 18
(i) Calculate the par swap rate.
(ii) Calculate the payment made by the xedrate party in 18
months if in 15 months the spot annual nominal interest rate
compounded quarterly is 4.65%.
c 2009. Miguel A. Arcones. All rights reserved. Manual for SOA Exam FM/CAS Exam 2.
44/78
Chapter 7. Derivatives markets. Section 7.10. Swaps.
Solution: (i) The par swap rate is
R =
P(0, t
k1
) P(0, t
n
)

n
j =k
P(0, t
j
)
=
(1 + 0.0455/4)
3
(1 + 0.0465/4)
6
(1 + 0.046/4)
4
+ (1 + 0.046/4)
5
+ (1 + 0.0465/4)
6
=0.01187359454 = 1.187359454%.
(ii) The payment made by the xedrate party is
L(Rr
t
j 1
(t
j 1
, t
j
)) = (50000)(0.011873594540.0465/4) = 12.429727.
c 2009. Miguel A. Arcones. All rights reserved. Manual for SOA Exam FM/CAS Exam 2.
45/78
Chapter 7. Derivatives markets. Section 7.10. Swaps.
The par swap rate R =
1P(0,t
n
)
P
n
j =1
P(0,t
j
)
is a weighted average of implied
forward rates. If the current interest rate does depend on the
maturing time, then P(0, t) = (1 + i )
t
, for some constant i > 0.
In this case,
R =
1 P(0, t
n
)

n
j =1
P(0, t
j
)
=
1 (1 + i )
t
n

n
j =1
(1 + i )
t
j
.
If the periods in the swap have the same length, then t
j
= jh,
1 j n, for some h > 0, and
R =
1 (1 + i )
nh

n
j =1
(1 + i )
nj
=
1 (1 + i )
nh
(1+i )
h
(1+i )
(n+1)h
1(1+i )
h
=
1
(1+i )
h
1(1+i )
h
=
1 (1 + i )
h
(1 + i )
h
= (1 + i )
h
1.
(1 + i )
h
1 is the eective rate for a period of length h. Notice
that the assumption P(0, t) = (1 + i )
t
, for some constat i > 0,
almost never happens.
c 2009. Miguel A. Arcones. All rights reserved. Manual for SOA Exam FM/CAS Exam 2.
46/78
Chapter 7. Derivatives markets. Section 7.10. Swaps.
Example 8
Suppose the current annual nominal interest rates compounded
quarterly from the LIBOR is 5.4% independently of the maturity of
the loan. Two counterparts enter into a xed against oating swap
using realized LIBOR rates. The times of the swap are in 3, 6 and
12 months. Calculate the par swap rate.
Solution: The par swap rate is
R =
1 P(0, t
n
)

n
j =1
P(0, t
j
)
=
1 (1 + 0.054/4)
4
(1 + 0.054/4)
1
+ (1 + 0.054/4)
2
+ (1 + 0.054/4)
4
=0.017959328 = 1.7959328%.
c 2009. Miguel A. Arcones. All rights reserved. Manual for SOA Exam FM/CAS Exam 2.
47/78
Chapter 7. Derivatives markets. Section 7.10. Swaps.
Example 8
Suppose the current annual nominal interest rates compounded
quarterly from the LIBOR is 5.4% independently of the maturity of
the loan. Two counterparts enter into a xed against oating swap
using realized LIBOR rates. The times of the swap are in 3, 6 and
12 months. Calculate the par swap rate.
Solution: The par swap rate is
R =
1 P(0, t
n
)

n
j =1
P(0, t
j
)
=
1 (1 + 0.054/4)
4
(1 + 0.054/4)
1
+ (1 + 0.054/4)
2
+ (1 + 0.054/4)
4
=0.017959328 = 1.7959328%.
c 2009. Miguel A. Arcones. All rights reserved. Manual for SOA Exam FM/CAS Exam 2.
48/78
Chapter 7. Derivatives markets. Section 7.10. Swaps.
Notice that in the previous problem the swap periods do not have
the same length. Even if annual interest rate is constant over time,
the interest payments vary over time. If the annual interest rate
remains constant over time, the oatingrate payments are
L ((1 + i )
t
1
1) L ((1 + i )
t
2
t
1
1) L ((1 + i )
t
n
t
n1
1)
t
1
t
2
t
n
Suppose that we take a loan of L at time zero. We make payments
of LR at t
j
, for 1 j n. The par swap rate R is the constant
periodic rate such that the nal outstanding in this loan is L.
Notice that the present value of the payments is LR

n
j =1
P(0, t
j
).
If the nal principal is L =
LLR
P
n
j =1
P(0,t
j
)
P(0,t
n
)
, then, R =
1P(0,t
n
)
P
n
j =1
P(0,t
j
)
.
c 2009. Miguel A. Arcones. All rights reserved. Manual for SOA Exam FM/CAS Exam 2.
49/78
Chapter 7. Derivatives markets. Section 7.10. Swaps.
Commodity swaps.
A commodity swap is a swap where one of the legs is a
commodity and the other one is cash. Hence, there are two
counterparts in a swap: a party with a commodity leg and another
party with a cash leg. Since the spot price of a commodity changes
over time, the commodity leg is oating. Certain amount of a
commodity is delivered at certain times. In some sense is like to
combine several forward contracts. But, swaps are valuated
considering the total deliveries. Hence, changes in interest rates
change the value of a swap. Usually, the swap payment for this
commodity is constant.
The commodity leg party is called the short swap side. The cash
leg party is called the long swap side.
c 2009. Miguel A. Arcones. All rights reserved. Manual for SOA Exam FM/CAS Exam 2.
50/78
Chapter 7. Derivatives markets. Section 7.10. Swaps.
Suppose that a party would like to sell a commodity at times
0 < t
1
< t
2
< < t
n
. Suppose that the nominal amounts of the
commodity are Q
1
, Q
2
, , Q
n
, respectively. The commodity leg is
Amount of delivered commodity Q
1
Q
2
Q
n
Time t
1
t
2
t
n
Usually the cash leg is either
Payments C
0
0 0 0
Time 0 t
1
t
2
t
n
or
Payments C
1
C
2
C
n
Time t
1
t
2
t
n
c 2009. Miguel A. Arcones. All rights reserved. Manual for SOA Exam FM/CAS Exam 2.
51/78
Chapter 7. Derivatives markets. Section 7.10. Swaps.
If two parties enter into a commodity swap, one will be designed
the party with the commodity leg and the other the party with the
cash leg. The party with the commodity leg will deliver commodity
to its counterpart according with the table above. The party with
the cash leg will pay cash payments to its counterpart. From a
practical point of view, the party with the commodity leg is a
seller. The party with the cash leg is a buyer. A commodity swap
contract needs to specify the type and quality of the commodity,
how to settle the contract, etc. A swap can be settled either by
physical settlement or by cash settlement. If a swap is settled
physically, the commodity leg side delivers the stipulated notional
amount to the cash leg side, and the cash leg side pays to the
commodity leg side. If a swap is cash settled, one of the parties
will make a payment to the other party.
c 2009. Miguel A. Arcones. All rights reserved. Manual for SOA Exam FM/CAS Exam 2.
52/78
Chapter 7. Derivatives markets. Section 7.10. Swaps.
Suppose that the current forward price of this commodity with
delivery in T years is F
0,T
. Let P(0, T) be the price of a
zerocoupon with face value $1 and expiration time T. Then, the
present value of the commodity delivered is
n

j =1
P(0, t
j
)Q
j
F
0,t
j
.
In a prepaid swap the buyer makes a unique payment at time
zero. If there exists no arbitrage, the price of a prepaid swap is

n
j =1
P(0, t
j
)Q
j
F
0,t
j
.
c 2009. Miguel A. Arcones. All rights reserved. Manual for SOA Exam FM/CAS Exam 2.
53/78
Chapter 7. Derivatives markets. Section 7.10. Swaps.
Usually the cash leg consists of series of payments made at the
times when the commodity is delivered. Usually each swap
payment per unit of commodity is a xed amount. Hence, the
cashow of payments is
Payment Q
1
R Q
2
R Q
n
R
Time t
1
t
2
t
2
where R is the swap price per unit of commodity. The present
value of the cashow of payments is

n
j =1
P(0, t
j
)Q
j
R. The no
arbitrage price of a swap per unit of commodity is
R =

n
j =1
P(0, t
j
)Q
j
F
0,t
j

n
j =1
P(0, t
j
)Q
j
.
R is a weighted average of forward prices.
c 2009. Miguel A. Arcones. All rights reserved. Manual for SOA Exam FM/CAS Exam 2.
54/78
Chapter 7. Derivatives markets. Section 7.10. Swaps.
Suppose that at time of delivery, the buyer pays a level payment of
R at each of the delivery times. Then, the present value of the
cashow of payments is

n
j =1
P(0, t
j
)R. The no arbitrage level
payment is
R =

n
j =1
P(0, t
j
)Q
j
F
0,t
j

n
j =1
P(0, t
j
)
.
A commodity swap allows to lock the price of a sale. It can be used
by a producer of a commodity to hedge by xing the price that he
will get in the future for this commodity. It also can be used by a
manufacturer to hedge by xing the price that he will pay in the
future for a commodity. A commodity swap can be used instead of
several futures/forwards. Since the price of a swap involves all the
deliveries, a commodity swap involves loaning/lending somehow.
c 2009. Miguel A. Arcones. All rights reserved. Manual for SOA Exam FM/CAS Exam 2.
55/78
Chapter 7. Derivatives markets. Section 7.10. Swaps.
Example 9
Suppose that an airline company must buy 10,000 barrels of oil
every six months, for 2 years, starting 6 months from now. The
company enters into a long oil swap contract to buy this oil. The
cash leg swap consists of four level payments made at the delivery
times. The following table shows the annual nominal rate
convertible semiannually of zerocoupon bonds maturing in
6, 12, 18, 24 months and the forward price of oil with delivery at
those times.
F
0,T
$50 $55 $55 $60
annual nominal rate 4.5% 5% 5% 5.5%
expiration in months 6 12 18 24
c 2009. Miguel A. Arcones. All rights reserved. Manual for SOA Exam FM/CAS Exam 2.
56/78
Chapter 7. Derivatives markets. Section 7.10. Swaps.
(i) Find the no arbitrage price of a prepaid swap with the commodity
leg which the airline needs.
c 2009. Miguel A. Arcones. All rights reserved. Manual for SOA Exam FM/CAS Exam 2.
57/78
Chapter 7. Derivatives markets. Section 7.10. Swaps.
(i) Find the no arbitrage price of a prepaid swap with the commodity
leg which the airline needs.
Solution: (i) The present value of the cost of oil is
n

j =1
Q
j
F
0,t
j
P(0, t
j
)
=(10000)
50
1 +
0.045
2
+ (10000)
55
_
1 +
0.05
2
_
2
+ (10000)
55
_
1 +
0.05
2
_
3
+ (10000)
60
_
1 +
0.055
2
_
4
=488997.555 + 523497.9179 + 510729.676 + 538299.4402 = 2061524.589.
c 2009. Miguel A. Arcones. All rights reserved. Manual for SOA Exam FM/CAS Exam 2.
58/78
Chapter 7. Derivatives markets. Section 7.10. Swaps.
(ii) Suppose that the airline company pays the swap by a level pay-
ment of R at each of the delivery times. Calculate R.
c 2009. Miguel A. Arcones. All rights reserved. Manual for SOA Exam FM/CAS Exam 2.
59/78
Chapter 7. Derivatives markets. Section 7.10. Swaps.
(ii) Suppose that the airline company pays the swap by a level pay-
ment of R at each of the delivery times. Calculate R.
Solution: (ii) We have that

n
j =1
P(0, t
j
)
=
1
1+
0.045
2
+
1
(
1+
0.05
2
)
2
+
1
(
1+
0.05
2
)
3
+
1
(
1+
0.055
2
)
4
= 0.97799511 + 0.9518143962 + 0.9285994109 + 0.8971657337
= 3.755574651
and
R =

n
j =1
Q
j
F
0,t
j
P(0, t
j
)

n
j =1
P(0, t
j
)
=
2061524.589
37555.74651
= 548923.8747.
c 2009. Miguel A. Arcones. All rights reserved. Manual for SOA Exam FM/CAS Exam 2.
60/78
Chapter 7. Derivatives markets. Section 7.10. Swaps.
(iii) Suppose that the airline company pays the swap by unique price
per barrel. Calculate the price per barrel.
c 2009. Miguel A. Arcones. All rights reserved. Manual for SOA Exam FM/CAS Exam 2.
61/78
Chapter 7. Derivatives markets. Section 7.10. Swaps.
(iii) Suppose that the airline company pays the swap by unique price
per barrel. Calculate the price per barrel.
Solution: (iii) The price of the swap per barrel is
548923.8747
10000
=
54.89238747.
c 2009. Miguel A. Arcones. All rights reserved. Manual for SOA Exam FM/CAS Exam 2.
62/78
Chapter 7. Derivatives markets. Section 7.10. Swaps.
Example 10
Suppose that an airline company must buy 10,000 barrels of oil in
two months, 12,000 barrels of oil in four months and 15,000
barrels of oil in six months. The company enters into a long oil
swap. The payment of the swap will be made at the delivery times.
The following table shows the annual nominal rate convertible
monthly of zerocoupon bonds maturing in 6, 12, 18, 24 months
and the forward price of oil with delivery at those times.
Barrels of oil $10000 $12000 $15000
F
0,T
$55 $56 $58
annual nominal rate 4.5% 4.55% 4.65%
expiration in months 2 4 6
c 2009. Miguel A. Arcones. All rights reserved. Manual for SOA Exam FM/CAS Exam 2.
63/78
Chapter 7. Derivatives markets. Section 7.10. Swaps.
(i) Suppose that the cash leg swap consists of a level payment of R
at each of the delivery times. Calculate R.
c 2009. Miguel A. Arcones. All rights reserved. Manual for SOA Exam FM/CAS Exam 2.
64/78
Chapter 7. Derivatives markets. Section 7.10. Swaps.
Solution: (i) We have that
n

j =1
Q
j
F
0,t
j
P(0, t
j
)
=(10000)
55
_
1 +
0.045
12
_
2
+ (12000)
56
_
1 +
0.0455
12
_
4
+ (15000)
58
_
1 +
0.0465
12
_
6
=545898.0877 + 661903.8839 + 850044.0252 = 2057845.997,

n
j =1
P(0, t
j
)
=
1
(
1+
0.045
12
)
2
+
1
(
1+
0.0455
12
)
4
+
1
(
1+
0.0465
12
)
6
= 0.9925419775 + 0.9849760176 + 0.9770620979 = 2.954580093
and
R =

n
j =1
Q
j
F
0,t
j
P(0, t
j
)

n
j =1
P(0, t
j
)
=
2057845.997
2.954580093
= 696493.5564.
c 2009. Miguel A. Arcones. All rights reserved. Manual for SOA Exam FM/CAS Exam 2.
65/78
Chapter 7. Derivatives markets. Section 7.10. Swaps.
(ii) Suppose that the cash leg swap consists of a unique payment per
barrel made at each of the delivery times. Calculate the no arbitrage
swap price per barrel.
c 2009. Miguel A. Arcones. All rights reserved. Manual for SOA Exam FM/CAS Exam 2.
66/78
Chapter 7. Derivatives markets. Section 7.10. Swaps.
(ii) Suppose that the cash leg swap consists of a unique payment per
barrel made at each of the delivery times. Calculate the no arbitrage
swap price per barrel.
Solution: (ii) We have that

n
j =1
P(0, t
j
)Q
j
=
10000
(
1+
0.045
12
)
2
+
12000
(
1+
0.0455
12
)
4
+
15000
(
1+
0.0465
12
)
6
= 9925.419775 + 11819.712212 + 14655.931469 = 36401.06346
and the swap price per barrel is
R =

n
j =1
P(0, t
j
)Q
j
F
0,t
j

n
j =1
P(0, t
j
)Q
j
=
2057845.997
36401.06346
= 56.53257903.
c 2009. Miguel A. Arcones. All rights reserved. Manual for SOA Exam FM/CAS Exam 2.
67/78
Chapter 7. Derivatives markets. Section 7.10. Swaps.
If a swap is cashed settled, then the commodity is valued at the
current spot price. If the current value of the commodity is bigger
than the value of the cash payment, then the (cash leg side) long
swap pays this dierence to the (commodity leg side) short swap.
Reciprocally, if the current value of the commodity is smaller than
the value of the cash payment, the (commodity leg side) short
swap side pays this dierence to the (cash leg side) long swap.
c 2009. Miguel A. Arcones. All rights reserved. Manual for SOA Exam FM/CAS Exam 2.
68/78
Chapter 7. Derivatives markets. Section 7.10. Swaps.
Suppose that the swap involves the sale of a commodity at times
t
1
< t
2
< < t
n
. with notional amounts of Q
1
, Q
2
, , Q
n
,
respectively. Suppose the swap payment is a xed amount per unit
of commodity. We saw that this amount is
R =

n
j =1
P(0, t
j
)Q
j
F
0,t
j

n
j =1
P(0, t
j
)Q
j
.
When a swap is cash settled, the value of the commodity is found
using the current spot price S
t
j
. At time t
j
the long swap pays
Q
j
R Q
j
S
t
j
to its counterpart. Notice that Q
j
R Q
j
S
t
j
could be
a negative real number. If Q
j
R Q
j
S
t
j
< 0, the short swap pays
Q
j
S
t
j
Q
j
R to its counterpart.
c 2009. Miguel A. Arcones. All rights reserved. Manual for SOA Exam FM/CAS Exam 2.
69/78
Chapter 7. Derivatives markets. Section 7.10. Swaps.
Changes in the forward contracts and interest rates alter the value
of the swap. The market value of a swap is found using the present
values of its legs using the current structure of interest rates. The
market value of a long swap immediately after the settlement at
time t
k
is
n

j =k+1
P(t
k
, t
j
)Q
j
(F
t
k
,t
j
R).
This is the price which an investor would pay to enter the swap as
a long swap side. Immediately after the swap is undertaken the
market value of the contract is
n

j =1
Q
j
(

P(0, t
j
)

F
0,t
j
P(0, t
j
)F
0,t
j
),
where

P(0, t
j
) and

F
0,t
j
are the new market values.
c 2009. Miguel A. Arcones. All rights reserved. Manual for SOA Exam FM/CAS Exam 2.
70/78
Chapter 7. Derivatives markets. Section 7.10. Swaps.
Example 11
Suppose that an airline company must buy 1,000 barrels of oil
every six months, for 3 years, starting 6 months from now. Instead
of buying six separate long forward contracts, the company enters
into a long swap contract. According with this swap the company
will pay a level payment per barrel at delivery. The following table
shows the annual nominal rate convertible semiannually of
zerocoupon bonds maturing in 6, 12, 18, 24 months and the
forward price of oil at those times.
F
0,T
$55 $57 $57 $60 $62 $64
annual nominal rate 5.5% 5.6% 5.65% 5.7% 5.7% 5.75%
expiration in months 6 12 18 24 30 36
c 2009. Miguel A. Arcones. All rights reserved. Manual for SOA Exam FM/CAS Exam 2.
71/78
Chapter 7. Derivatives markets. Section 7.10. Swaps.
(i) Find the price per barrel of oil using the swap.
c 2009. Miguel A. Arcones. All rights reserved. Manual for SOA Exam FM/CAS Exam 2.
72/78
Chapter 7. Derivatives markets. Section 7.10. Swaps.
(i) Find the price per barrel of oil using the swap.
Solution: (i) We have that
n

j =1
P(0, t
j
)Q
j
F
0,t
j
=(1000)
55
1 +
0.055
2
+ (1000)
57
_
1 +
0.056
2
_
2
+ (1000)
57
_
1 +
0.0565
2
_
3
+ (1000)
60
_
1 +
0.057
2
_
4
+ (1000)
62
_
1 +
0.057
2
_
5
+ (1000)
64
_
1 +
0.0575
2
_
6
=54254.00740 + 55436.90114 + 54651.28637
+ 56698.44979 + 57765.24340 + 58747.41357 = 337553.3017,
c 2009. Miguel A. Arcones. All rights reserved. Manual for SOA Exam FM/CAS Exam 2.
73/78
Chapter 7. Derivatives markets. Section 7.10. Swaps.
(i) Find the price per barrel of oil using the swap.
Solution: (i)
n

j =1
P(0, t
j
)Q
j
=
1000
1 +
0.055
2
+
1
_
1 +
0.056
2
_
2
+
1000
_
1 +
0.0565
2
_
3
+
1000
_
1 +
0.057
2
_
4
+
1000
_
1 +
0.057
2
_
5
+
1000
_
1 +
0.0575
2
_
6
=986.4364982 + 972.5772129 + 958.7944977
+ 944.9741632 + 931.6974742 + 917.9283370 = 5712.408183.
We have that
R =

n
j =1
P(0, t
j
)Q
j
F
0,t
j

n
j =1
P(0, t
j
)Q
j
=
337553.3017
5712.408183
= 59.09124329.
c 2009. Miguel A. Arcones. All rights reserved. Manual for SOA Exam FM/CAS Exam 2.
74/78
Chapter 7. Derivatives markets. Section 7.10. Swaps.
(ii) Suppose the swap is settled in cash. Assume that the spot rate
for oil in 18 months is $57. Calculate the payment which the airline
receives.
c 2009. Miguel A. Arcones. All rights reserved. Manual for SOA Exam FM/CAS Exam 2.
75/78
Chapter 7. Derivatives markets. Section 7.10. Swaps.
(ii) Suppose the swap is settled in cash. Assume that the spot rate
for oil in 18 months is $57. Calculate the payment which the airline
receives.
Solution: (ii) The airline gets a payment of
Q
j
S
t
j
Q
j
R = (1000)(57) (1000)(59.09124329) = 2091.24329.
c 2009. Miguel A. Arcones. All rights reserved. Manual for SOA Exam FM/CAS Exam 2.
76/78
Chapter 7. Derivatives markets. Section 7.10. Swaps.
(iii) Suppose that immediately after the swap is signed up, the future
prices of oil are given by the following table

F
0,T
$55 $58 $59 $61 $62 $63
expiration in months 6 12 18 24 30 36
Calculate the value of the swap for the cash leg party.
c 2009. Miguel A. Arcones. All rights reserved. Manual for SOA Exam FM/CAS Exam 2.
77/78
Chapter 7. Derivatives markets. Section 7.10. Swaps.
Solution: (iii) The market value of the swap for the long party is
n

j =1
P(0, t
j
)(Q
j

F
t
0
,t
j
Q
j
R)
=
1000
1 +
0.055
2
(55 59.09124329) +
1
_
1 +
0.056
2
_
2
(58 59.09124329)
+
1000
_
1 +
0.0565
2
_
3
(59 59.09124329) +
1000
_
1 +
0.057
2
_
4
(61 59.09124329)
+
1000
_
1 +
0.057
2
_
5
(62 59.09124329) +
1000
_
1 +
0.0575
2
_
6
(63 59.09124329)
=4035.7517041 1061.3183576 87.4835644
+ 1803.7257747 + 2710.0812797 + 3587.9585464 = 2917.211975.
c 2009. Miguel A. Arcones. All rights reserved. Manual for SOA Exam FM/CAS Exam 2.
78/78
Chapter 7. Derivatives markets. Section 7.10. Swaps.
Suppose that immediately after the forward is signed, every future
price increases by K. Then, the market value of the swap is
n

j =1
P(0, t
j
)(Q
j
(F
0,t
j
+ K) Q
j
R) =
n

j =1
P(0, t
j
)Q
j
K.
The swap counterpart is a scalper which hedges the commodity
risk resulting from the swap. The scalper has also interest rate
exposure. The scalper needs to hedge changes in the price of the
commodity and in interest rates.
c 2009. Miguel A. Arcones. All rights reserved. Manual for SOA Exam FM/CAS Exam 2.

Вам также может понравиться